You are on page 1of 371

SUPPORT STUDY

MATERIAL

XII English Core

Support Study Material


and VBQ
SUPPORT MATERIAL

ENGLISH CORE
CLASS XII

1
SUPPORT MATERIAL

2
3
Foreword

The objective is to ensure that the students not only learn well but score well in the
examinations as well. The “Support Material for Class XII - English Core” has been
prepared to realize this objective.

The Support Material (SM) is expected to serve as a supplement to the prescribed text
books, and hence to be used alongside the original text books. The gist of the lesson –
be it prose or poetry – given in the SM is followed by short and long Questions and
Answers, and additional questions for practice. In some of the cases, the value points
are given instead of the complete answer so that the students can get to know the
points relevant to the question. The topics in Advance Writing Skills are dealt with
properly giving the meaning of the skills along with the points to be kept in mind while
answering questions.

Two solved question papers given in the Support Material can be of great help to
students in knowing how to write answers to questions with a view to scoring centum in
the English paper. The unsolved papers should be solved using academic rigour by the
students so that their fear of facing the examination gets reduced to a naught.

There is an article ‘tips to score well in the examination’ given in the Support Material
which shall help the students know the ways and means of facing the examination with
confidence.

4
CONTENTS

Sl.No. Topics Page No.

1 Examinations Specifications 6 to 8

2 Tips to Score better in English 8 to 9

3 Reading Comprehension 10 to 22
4 Advanced Writing Skills 23 to 65

5 Literature:
Flamingo 66 to 84
Vistas 84 to 122
6 Sample Papers(Solved) 122 to 156

7 Practice Papers (Unsolved) 156 to 170

5
EXAMINATION SPECIFICATIONS
CLASS XII
Time- 3 Hours Marks: 100
Unit-wise weightage

Unit/Areas of Learning Marks


A Section A
Reading Skills
Reading unseen prose passages and note making 20

B Section B
Advanced Writing Skills 35

C Section C (Prescribed Texts)


(i) Flamingo 30
(ii) Supplementary Reader - Vistas 15

SECTION-A
Reading unseen Passages and Note-making 20 Marks
Two unseen passages, with a variety of questions including 03 marks for vocabulary
such as word formation and inferring meaning and 05 marks for note-making.
The total length of the two passages will be between 950-1200 words. The passages will
include two of the following:
(a) Factual Passages e.g. instructions, descriptions, reports.
(b) Discursive passage involving opinion e.g. argumentative, persuasive or
interpretative text.
(c) Literary passage e.g. extract from fiction, drama, poetry, essay or biography.
Summary - Class XII

Unseen No. of words Testing Areas Marks allotted


Passages
1 600-700 Short answer type questions 9
to test local, global and
inferential comprehension
vocabulary
3 Total 09+3=12
2 350-500 Note making in an 5
appropriate format
Abstraction 3 Total 05+3=08

6
1. A passage of about 600-700 words carrying 12 marks and another passage of about
350-500 words carrying 08 marks.
2. A passage to test reading comprehension. The passage can be literary, factual or
discursive. The length of the passage should be between 600-700 words. 12 marks
3. A shorter passage of 350-500 words for note-making and abstraction. 08 marks

SECTION B
Advanced Writing Skills 35 Marks.

3. One out of two short compositions of not more than 50 words, 5 marks each e.g.
advertisement and notices, designing or drafting posters, writing formal and informal
invitations and replies.
4. A report or a factual description based on verbal input provided. 10 marks
(one out of two) (100-125 words)
5. Writing one out of two letters based on verbal input. 10 marks
Letter types include:
(a) Business or official letters (for making enquiries, registering complaints, asking for
and giving information, placing orders and sending replies):
(b) Letters to the editor (giving suggestions on an issue)
(c) Application for a job
6. One out of two compositions based on visual and/or verbal input. 10 marks (150-200
words). Output may be descriptive or argumentative in nature such as an article, or a
speech.

SECTION C
Text Books 45 Marks
Prescribed Books: Flamingo 30

7. One out of two extracts based on poetry from the text to test comprehension and
appreciation. 4 marks
8. Three out of four short questions from the poetry section to test local and global
comprehension of text. 6 marks
9. Five short answer questions based on the lessons from 10 prescribed text. (2x5)=10
marks
10. One out of two long answer type questions based on the text to test global
comprehension and extrapolation beyond the set text. 10 marks (Expected word limit
about 125-150 words each)
Vistas 15
11. One out of two long answer type question based on Supplementary Reader to test
comprehension and extrapolation of theme, character and incidents. 7 marks (Expected
word limit about 125-150 words)
12. Four short answer questions from the Supplementary Reader (2x4) 8 marks

Prescribed Books:

7
1. Flamingo : English Reader published by National Council of Education Research and
Training, New Delhi.
2. Vistas : Supplementary Reader published by National Council of Education Research
and Training, New Delhi.

TIPS TO SCORE WELL IN ENGLISH

As a student of Class XII, you must be wondering how to score better in English in the
fast approaching Board Examinations. How to Prepare for Exam is a big question that
you always ask yourself. Here are some tips that may help you. Many of you may be
working hard and getting good marks and some of you may not be working hard but still
manage to get high marks in exams. Others may be wondering how it happens? Also
there will be a few of you who study well but still are not able to score well in exams.
Don’t worry; there are some very simple tips to follow which will help you achieve
success. These tips will tell you 'How to prepare for Exams”.

STEPS

1. READ-Spend about 30 minutes reading at home every day. You will be amazed by
the number of new words you can learn and use these words to improve writing
skills.

2. ASK QUESTIONS! - Nothing pleases an English teacher more than a question about
the topic at hand. If you don’t know, then ask! Do not hesitate.

3. STAY FOR EXTRA HELP AFTER SCHOOL – If you want to improve in English (ex. Prose,
poetry, writing skill), then ask the teacher for extra help. He/she will like to help you
after class.

4. Make sure to practice the reading and comprehension skills at a regular basis. Every
week solve one unseen passage and one passage for note making. After solving
request your teacher to evaluate. The passage can be picked up from the News
papers. Speaking Tree from “Times of India”, Editors column from other News
Papers.

5. Practice minimum two writing skills every week and get it evaluated from your
teacher. Improvise as per the teachers suggestions.

8
6. GO ABOVE AND BEYOND YOUR TEACHER’S EXPECTATIONS – The topics given for the
articles should be written in more words than expected. Write and practice the
issues which attract common mans attention.

7. Talk to your teacher after class about something that was discussed in the class that
you thought was interesting or confusing. This will clarify the issue in your mind as
the teacher would explain again.

8. If the practice of writing on a regular basis is done keeping in mind the marking
criteria, better scores in final exams will be ensured.

9. Participate in the class discussions on the lessons. Support your opinions with quotes
from the lesson.

10. Try to relate the lessons in your text books with current events. (if possible)

11. Read the classic literature, such as The Scarlet Letter, To Kill a Mocking Bird and
Gone with the Wind.

12. Attend your English class with interest. Pay attention and you will definitely learn
something of value and interest in every class.

13. Do miss the valuable study materials, question papers given to be solved by the
teacher. Solve them meticulously and get them evaluated.Unnnnn744566+525

14. While studying make short notes. Notes should be brief so that you can revise the
lesson with these notes. You can refer back to the portion in the text book if needed.

There is no short cut to success. WORK HARD and success will be yours.

If you work hard as a student the remaining life will be comfortable. But if you have
enjoyed life as a student then get ready to face hard life ahead.

---------------------

9
SECTION-WISE Analysis of the Question Paper

SECTION - A

READING COMPREHENSION

Comprehension means understanding or perception. The following points are to be


carefully noted while attempting questions on comprehension.
• Go through the passage carefully and arrive at the general idea of the subject the
passage presents.

• Read a second time to get a better understanding of the passage.

• Arrive at the meaning of difficult words by relating them to the preceding and
following sentences.

• Underline relevant words and phrases of the passages which can help you
deduce the answers.

• Read the questions carefully.

• Answer precisely using simple language.

• Answers should reveal your understanding of the passage.

• If you are asked to provide a suitable title or heading to the passage remember
the title is hidden either in the beginning or ending of the passage.

• Title should relate to the main idea of the passage and should be brief.

Unseen passages for comprehension (Solved)

A. 1. Read the passage given below and answer the questions that follow (12
Marks)

1. First, a warning. The journey is tough, steep and peppered with hairpin bends. If
you are not a hardened traveller, by the time you reach Tawang your head is
likely to be spinning. Add to it the breeze which pierces through all your
protective clothing and you could well be wondering what prompted you to
make this arduous trip to such Himalayan heights.

2. To get acclimatized to high altitude, the recipe is simple and strict – take it easy
on the first day, lest you find yourself out of breath and panting. Tuck yourself in

10
a warm bed and sip some thupka (Tibetan noodle soup) and begin your
adventures the following day.

3. In fact when you get up the next morning, you will scarcely believe what you see.
The picture postcard beauty of the hill station in Arunachal Pradesh will simply
take your breath away. The scenery is pristine and the Himalayan ranges are
lush with pine, oak and rhododendron forests. There is also a rich growth of
bamboo, which is the favorite food of the red panda found in this part of the
Northeast.

4. Located about 10,000 feet above sea level, the Tawang monastery is the second
oldest monastery in Asia, which explains the rush of tourists to this remote
settlement. The magnificent monastery overlooks the valley and surrounded by
mountains that seem to be towering around it like guards. The monastery’s
enormous yellow roof and white walls stand out like a beacon. The place is
completely isolated from the world.

5. One of the biggest attractions of the monastery is the three storey dukhang
(assembly hall) that has a magnificent eight-meter-high gilded image of Buddha.

6. The ancient library, leading onto the parkhang (main courtyard) has an excellent
collection of old scriptures, images and thankas (traditional paintings and
monastery – or gompa in local language – is over 350 years old and is an
important centre of pilgrimage for Buddhists.

7. The sixth Dalai Lama was born here. Also known as the Galden Namgyan Lhatse,
the monastery is a repository of Tibetan Buddhist culture.

8. Tawang does not have an airport or railway station of its own. It is connected
with other town in Arunachal Pradesh and Assam by road. Reaching Tawang
from Kolkata is braving a long but rewarding journey. The drive through the
picturesque mountain country is spectacular and can leave you breathless as you
wind around steep hill-roads and maneuver sharp hairpin bends. In parts, the
road can be rather treacherous. This is definitely not a drive for the faint-
hearted.

9. The journey to Tawang starts from Guwahati on a bus or a taxi to Bomdila. Past
the Dirang valley with its old dzong (fort), the road climbs sharply to Sela Pass at
13,940 feet. This barren, desolate landscape is softened by a serene lake that lies
below Sela Pass.

11
10. Twang has one main street and a warren on alleys to the houses that climb up
the hillside towards the towering monastery. It also has one quaint little bazaar
which sells products ranging from talismans and prayer wheels to garish
sunglasses and transistors. The snack stalls are a plenty offering solja, the yak
butter tea, thupka and hot memos, the delicious steamed meat dumpling with
chilly sauce.

1.1. Answer the following questions briefly. (9 marks)

(i) Why is the journey up Twang so arduous? (2)

(ii) What should a visitor do to acclimatize to high altitude? (1)

(iii) Why do people like to visit Tawang? (2)

(iv) Describe the Tawang monastery. (2)

(v) How can one reach Tawang? (2)

1.2. Pick out words or phrases which convey the same meaning as the
following. (3 marks)

(i) Extremely attractive ( Para 4)

(ii) Holy place (Para 6)

(iii) Huge, in great (Para 3)

Answers
1.1.
(i) The journey to Tawang is difficult because the breeze pierces the protective clothes.
The traveler’s head starts spinning.
(ii) The visitor must get acclimatized to high altitude. He should take it easy on the first
day.
(iii) People are greatly attracted by the beauty of this hill station. The scenery is
wonderful.
(iv) Tawang monastery is the second oldest monastery in Asia. It is located about
10,000 feet about sea level and overlooks the valley and seems isolated from the world.
(v) Tawang does not have any airport. It is connected with other towns in Arunachal
Pradesh and Assam by road. So the visitor should make up his mind to undertake a long
and rewarding journey.
1.2.
(i) Magnificent
(ii) Pilgrimage
(iii) Rhododendron

12
Unseen Passages for Practice

1. Read the following passage.

Light Pollution
1. Every urbanite knows the bright lights of the city make it impossible to see the stars in a
night sky. What most of us do not know is that those lights also may be making us sick.
The cause is light pollution – the unearthly glow of billions of street lamps, security and
porch lights, searchlights, office lights and signs – as people everywhere try to dispel the
darkness of the night.

2. For the first time, light is being investigated seriously as a pollutant and a health hazard
– a possible cause increased incidence of breast cancer, depression and other ailments.

3. In the most heavily urbanized regions, it no longer ever really gets dark. Satellite images
reveal that in large areas of eastern North America, Western Europe, Japan and Korea,
night has become a constant twilight. In a natural night sky, someone looking at the
heavens should be able to see nearly 3500 stars and planets and the glow from the
Milky Way, our galaxy. But in some brightly lit cities, the number of visible stars has
dwindled to about a few dozen.

4. And for many wildlife species, light pollution seems to be as grave as environmental
threat as bulldozed habitats and toxic-chemical dumping.

5. Lighting from office towers confuse migratory birds which fly into buildings lit up at
night. Millions of birds in North America die from these crashes. Researchers have
noticed since the 1980s that artificial lights along ocean beaches confuse millions of
baby turtles. Observers say that the turtles instinctively crawl to the brightest thing on
the horizon – normally the reflection of the moon on the sea. But where beaches are
illuminated, baby turtles often crawl to the lit roads, where they are flattened by cars,
or wander in circles on the beach. Once day breaks, they bake to death in the sun.

6. Sea turtles and birds are clearly in peril because of light at night, but scientists have
begun to study whether human may share something of the same fate. Richard Steven,
a US Epidemiologist, has developed the idea that night light can disrupt critical
hormonal levels that affect human health. Stevens came to this conclusion while trying
to solve the puzzle of why breast cancer risk is five times higher in industrialized
societies than in non-industrialized countries.

7. Stevens turned to literature on circadian rhythms- the 24-hour biological clock that
guides daily body functions – and on melatonin, a hormone most living creatures

13
produce only in darkness. In a study published in 2001, Stevens says that there is
‘mounting evidence to suggest that disruption of the melatonin rhythm may lead to
chronic fatigue, depression, reproduction anomalies and perhaps even cancer’.
Melatonin is produced in the brain’s pineal gland only when the eyes signal it is dark.
Those working under lighting at night could be reducing the amount of melatonin they
produce.

8. Travis Longcore, Science Director at the Urban Wild lands Group (a Los Angeles
conservation group), says that light pollution should be receiving the same attention as
other environmental ills. Canada has set up a conservation reserve north of Toronto, the
first in the world to preserve a pristine night sky. Conservationists are hoping to expand
the area of preserved night sky beyond the park’s boundaries by encouraging local
municipalities to curtail the use of poorly designed night lighting.

9. In the United States, the International Dark-Sky Association (IDA) has been campaigning
for bylaws requiring ‘night-friendly’ lights, with the bulbs recesses into the fixture so
that the light does not glare out horizontally or upwards. Lamps that direct a light beam
at the ground use less electricity and do not uselessly light the sky. While the health and
environmental impacts of night pollution are starting to capture attention, Dave
Crawford, the executive director of IDA, has other concerns too. He too worries that
light may be making people sick and harming wildlife, but he says the fading of the
heavens could also cause a fading of the human imagination – in many ways a greater
long-term threat.

10. Writers and artists have been drawn inspiration from the night sky. Says Crawford, “It’s
the glamour and wonder of the universe we live in. We’ve got to preserve that.”

1.1. Answer the following questions (9 marks)

1. How is light a pollutant? What harm does it cause? (2)

2. What startling revelation does this passage provide about death of turtles? (1)

3. How does lighting affect our daily body functions? (2)

4. What measures have been taken in Canada to reduce light pollution? (2)

5. What are the suggestions given by the IDA to counter light pollution? (2)

1.1. Pick out words or phrases which convey the same meaning as the following: (3 marks)

(i) Threat or danger (Para 6)

14
(ii) Unusual, irregularity ( Para 7)

(iii) To limit ( para8)

2. Read the passage.

1. A scholar is refused admission to a good school primarily because of his weak


interactive skills. It is indeed pathetic that though English is spoken fluently and used
every day; yet none is satisfied with their abilities. A learner feels he cannot express
himself the way he wants to. And he fails miserably to connect with his own hidden
power.
2. The Education system, in its endeavors to make Curriculum relevant and life-
oriented has, as a matter of fact, fallen short of its standards. It is oblivious of the fact
that the most applied and practical subject- English: is devoid of practical afflatus.
English is at cross roads. Both aspects of Education: the cultural that empowers a
learner to grow and the productive aspect that makes him do things, is relegated to the
background curriculum thus stands lopsided. It fails to provide full range of services and
cannot tap teachers’ expertise. No proper moves and strategies have been formulated
to make it unique. English courses are mushrooming and alluring advertisements clip
are often spotted.
3. With the onset of the new millennium, demands of the Educational System to
sensitize itself to changing societal needs has also increased manifold. The new race of
human beings has to be served New Curriculum that caters to the Unity Of Thought,
Action & Deed and help evolve an integrated human personality. A Comprehensive
Curriculum alone can enhance their understanding of four basic skills.
4. Acquiring the skill of English is no Catwalk. And English is no Science where
results are verified; but it means Construction; ingenuity at work. The Architecture that
it builds can never be complete if it is divorced from learning by doing.
5. There are a number of Projects like Phonetics, News-reading, Indian
literature, Poetry- composition, Interview skills, biography- launch, etc. which can help a
pupil to experiment till he finds a medium that helps free flow of thoughts, to think
critically and creatively and emerge as literary competent. As long as there is proper

15
feeding, English will live in their hearts and reign in their minds. Sooner or later, they will
be blessed with a marathon for actionable learning that shall make a multidimensional
impact on them.
“Within the enclosure (pupils) lies the potential for a new beginning
And within this exciting beginning lies an astonishing future for him.”

6. English cannot be conferred so easily and so soon. We need to take control


of English. There is a grave need to designate English, without the slightest hesitation, a
practical status if at all, we want a remarkable future. The lack of efforts in learning
English and the general feeling of not being interested in learning the English language
that are plaguing the system should be urgently addressed. Should not we all battle
against this abuse?
1.1. On the basis of your reading the passage answer the questions given below.
(9marks)
1. What are the interactive situations? (1)
2. What is the ‘matter of grave concern’? (2)
3. Explain unity of thought, action and deed? (2)
4. How can English help evolve an integrated personality? (2)
5. How can you find an astonishing future? (2)

1.2. Pick out one word from the passage that means the same as: (3 marks)

a. Nascent (para1)
b. Pragmatic (para2)
c. Depths (para6)

Note making

Tips to prepare Notes and Summary:

• Prepare notes using phrases only and never use complete sentences.

• The topic sentence of each paragraph is the main point and the ideas affiliated to
it are the sub-points – one or more depending on the concepts in the paragraph.

16
• Each sub-point may or may not have supplementary ideas which become sub-
sub points.

• Proper indentation is essential.

• Provide an appropriate title for the notes and the summary.

• Include a minimum of 4 to 6 distinctly different, recognizable short forms of the


words in the notes.

• Use short forms only for longer words. Underline all short forms. Provide the
key for the short forms at the end of the notes.

• Cover all the important points in the passage while preparing the notes.

• Include all the important points in the notes meaningfully to prepare the
summary in 80 – 90 words.

• Write the summary in complete sentences in a paragraph.

Split up of Marks

Note making (5) Summary (3)


Title: 1, Content: 3 Content: 2
Abbreviations and Key: 1 Expression: 1

Passage for Note making (Solved)

2. Read the passage given below carefully and answer the questions that follow:
(8 marks)

1. The small village of Somnathpur contains an extraordinary temple, built around


1268 A.D. by the Hoyasalas of Karnataka – one of the most prolific temple-
builders. Belur and Helebid are among their better-known works. While these
suffered during the invasions of the 14th century, the Somnathpur temple stands
more or less intact in near-original condition.

2. This small temple captivates with the beauty and vitality of its detailed sculpture,
covering almost every inch of the walls, pillars and even ceilings. It has three
shikharas and stands on a star-shaped, raised platform with 24 edges. The outer
walls have a profusion of detailed carvings: the entire surface run over by carved

17
plaques of stone. There were vertical panels covered by exquisite figures of gods
and goddesses with many incarnations being depicted. There were nymphs too,
some carrying an ear of maize a symbol of plenty and prosperity. The elaborate
ornamentation, very characteristic of Hoyasala sculptures, was a remarkable
feature. On closer look- and it is worth it – the series of friezes on the outer walls
revealed intricately carved caparisoned (covered decorative cloth) elephants,
charging horsemen, stylized flowers, warriors, musicians, crocodiles, and swans.

3. The temple was actually commissioned by Soma Dandanayaka or Somnath (he


named the village after himself), the minister of the Hoyasala king, Narasimha,
the Third. The temple was built to house three versions of Krishna. The inner
center of the temple was the kalyana mandapa. Leading from here were three
corridors each ending in a shrine, one for each kind of Krishna – Venugopala,
Janardana and Prasanna Keshava, though only two remain in their original form.
In the darkness of the sanctum sanctorum, I tried to discern the different images.
The temple’s sculptural perfection is amazing and it includes the doors of the
temple and the three elegantly carved towers.

2.1. On the basis of your reading of the above passage make notes on it using
headings and sub-headings. Use recognizable abbreviations, wherever necessary. Give
the passage a suitable title. (5)

Answer:

Temple of Somnathpur

1. Temple: the beauty and vitality


1.1. Detailed sculpture – covering walls, pillars, ceilings
1.1. a. Series of friezes on outer walls
1.1. b. intricately carved elephants
1.1. c. charging horsemen
1.1. d. stylized flowers
1.1. e. warriors, musicians, crocodile and swans
1.2. three shikharas – stands shaped, raised platform – 24 edges
1.3. the outer walls – detailed carvings
1.4. the entire surface – carved plaques of stone
1.5. vertical panels covered by exq. fig.

2. Representation of Hinduism

18
2.1. incarnations
2.2. many deities
3. Temple in the History
3.1. comm. Soma Dandanayaka or Somnath
3.2. the inner center of the temple – kalyana mandapa
3.3. three corridors ending in a shrine
Key
1. star
2. exq exquisite
3. fig figures
4. comm. commissioned

2.2. Write a summary of the note prepared in not more than 80 words. (3 marks)
The temple of Somnathpur is extraordinary due to the sculptures on the walls, pillars,
and even the ceiling which is covered by exquisite figures of gods and goddesses. It is a
representation of Hinduism with its many incarnations and deities. The temple
commissioned by Somnath has a ‘kalyana mandapa’ with three corridors ending in a
shrine.

Passages for practice

(i) Read the passage given below carefully and answer the questions that follow:
(8 marks)
1. The practice of soil conservation involves methods to reduce soil erosion,
prevent depletion of soil nutrients, and restore nutrients, already lost by erosion and
excessive crop harvesting. Most methods used to control soil erosion involve, keeping
the soil covered with vegetation.
2. In conventional farming, the land is ploughed several times and smoothed to
make a planting surface – a practice that makes it vulnerable to soil erosion. To reduce
erosion, an increasing number of farmers in many countries are using conservation –
tillage farming, also known as minimum – tillage, or no- till farming, depending on the
degree to which the soil is disturbed. Farmers using these methods disturb the soil as
little as possible in planting crops.
3. For the minimum-tillage method, special tillers break up and loosen the
subsurface soil without turning over the topsoil. In no-till farming special planting
machines inject seeds, fertilizers and weed-killers into slits made in the unploughed
soil.

19
4. In addition to reducing soil erosion, conversation – tillage and no-till farming
reduce fuel and tillage costs and water loss from soil. They can also increase the
number of crops that can be grown during a season.
5. Soil erosion can also be reduced by 30-50 percent on gently sloping land by
means of contour farming – ploughing and planting crops in rows across, rather than
up and down, the sloped contours of the land. Each row planted horizontally along the
slope of the land acts as a small dam to help hold and slow the runoff of water.
6. Terracing can be used on steeper slopes. Each terrace retains some of the water
running down the vegetated slope. Terracing provides water for crops at all levels and
decreases soil erosion by reducing the amount and speed of water runoff. In areas of
high rainfall, diversions ditches must be built behind each terrace to permit adequate
drainage.
7. In strip cropping, a series of rows of one crop, such as corn or soybeans, is
planted in a wide strip. Then the next strip is planted with a soil-conserving cover crop,
such as grass or grass-legume mixture, which completely covers the soil and thus
reduces erosion. These alternating rows of cover trap soil that erodes from the other
rows, catch and reduce water runoff, and help prevent the spread of plant diseases and
pests from one strip to another.
8. Windbreaks can reduce erosion caused by exposure of cultivated lands to high
winds or shelter beats. These are long rows of trees planted to partially block the wind.
Windbreaks also provide habitats for birds, pest eating and pollinating insects and
other animals.
2.1. On the basis of your reading of the above passage make notes on it using
headings and sub-headings. Use recognizable abbreviations, wherever necessary. Give
the passage a suitable title. (5 marks)
2.2. Write a summary of the notes prepared in not more than 80 words. (3 marks)

(ii) Read the passage given below carefully and answer the questions that follow:
(8 marks)

20
1. The tests of life are its plus factors. Overcoming illness and suffering is a plus
factor for it moulds character. Steel is iron plus fire, soil is rock plus heat. So let’s
include the plus factor in our lives.
2. Sometimes the plus factor is more readily seen by the simple-hearted. Myers
tells the story of a mother who brought into her home - as a companion to her own son
- a little boy who happened to have a hunchback. She had warned her son to be careful
not to refer to his disability, and to go right on playing with him as if he were like any
other boy.
3. The boys were playing and after a few minutes she overheard her son say to his
companion: “Do you know what you have got on your back?” The little boy was
embarrassed, but before he could reply, his playmate continued: “It is the box in which
your wings are and some day God is going to cut it open and then you will fly away and be an
angel”.
4. Often it takes a third eye or a change in focus, to see the plus factor. Walking along the
corridors of a hospital recently where patients were struggling with fear of pain and tests, I was
perturbed. What gave me fresh perspective were the sayings put up everywhere, intended to
uplift. One saying made me conscious of the beauty of the universe in the midst of pain,
suffering and struggle. The other saying assured me that God was with me when I was in deep
water and that no troubles would overwhelm me.
5. The import of those sayings also made me aware of the nether springs that flow into
people’s lives when they touch rock bottom or lonely or even deserted. The nether springs
make recovery possible, and they bring peace and patience in the midst of pain and distress.
6. The forces of death and destruction are not so much physical as they are psychic and
psychological. When malice, hate and hard-heartedness prevail, they get channeled as forces of
destruction. Where openness, peace and good-heartedness prevail, the forces of life gush forth
to regenerate hope and joy. The life force is triumphant when love overcomes fear. Both fear
and love are deep mysteries, but the effect of love is to build whereas fear tends to destroy.
Love is often the plus factor that helps build character. It helps us to accept and to overcome
suffering. It creates lasting bonds and its reach is infinite.
7. It is true that there is no shortage of destructive elements - forces and people who seek
to destroy others and in the process, destroy themselves - but at the same time there are signs

21
of love and life everywhere that are constantly enabling us to overcome setbacks. So let’s not
look only at gloom and doom - let’s seek out positivity and happiness. For it is when you seek
that you will find what is waiting to be discovered.
2.1. On the basis of your reading of the above passage make notes on it using headings and
sub-headings. Use recognizable abbreviations, wherever necessary. Give the passage a suitable
title. (5 marks)
2.2. Write a summary of the notes prepared in not more than 80 words (3 marks)

SECTION-B

ADVANCED WRITING SKILLS (35 Marks)

QUESTION -3
THIS QUESTION INCLUDES THE FOLLOWING SHORT COMPOSITIONS:
1. NOTICE
2. ADVERTISEMENTS – CLASSIFIED AND COMMERCIAL
3. POSTERS
4. INVITATIONS AND REPLIES
1. Notice (5 Marks / 50 Words)
A Notice is a written or a printed information or news announcement. Notices are
either displayed at prominent places or published in newspapers/magazines. It is meant
only for a select group. Since a notice contains a formal announcement or information,
its tone and style is formal and factual. Its language should be simple and formal. A
notice is always brief and to the point.
Remember, Circulars are also written like notices, but unlike notices, they carry more
than one message, and they are circulated through a messenger.
IMPORTANT TIPS TO BE FOLLOWED WHILE WRITING NOTICES:
• Adhere to the specified word limit of 50 words.
• Write the word NOTICE at the top.
• Name and place of the school, organization or office issuing the notice should be
mentioned.
• Give an appropriate heading.
• Write the date of issuing the notice.

22
• Clearly mention the target group (for whom the notice is to be displayed).
• Purpose of the notice.
• Mention all the relevant details (date, venue, time).
• Mention whom to contact for extra information.
• Signature, name and designation of the person issuing the notice.
• Put the notice in a box.

Format of a notice
A NOTICE IS ALWAYS WRITTEN IN A BOX
NOTICE
NAME OF THE INSTITUTION/ORGANIZATION/OFFICE , PLACE
SUITABLE HEADING
29 JUNE 2011
Content Target group-for whom the notice is. Date, time, venue
and all important details and any extra information
that is needed. (Body of the Notice 50 words )

Signature
(NAME)
Designation

Marking Scheme of Notice


Format : Title (Notice/Heading/Place/Date/Signature with name and
Designation) (1 mark)
Content : (a) where?
(b) When?
(c) Target group
(d) Agenda
(All within 50 words) (2 marks)
Expression : Coherence, spellings and grammatical accuracy. (2 marks)

23
Solved Example-1
Imagine you are a member of RED CROSS Blood Bank Society, Bangalore. You have been
asked by the President of the society, Mr. Manish Nair to organize a Blood Donation
Camp. Write a notice in 50 words urging the people to come in large numbers for this
noble cause. (5 marks)
NOTICE
RED CROSS BLOOD BANK SOCIETY,BANGALORE
DONATE BLOOD AND SAVE LIVES!
29th June 2011
A State Level function to observe voluntary Blood Donation Day is being organized on
5th and 6th July 2011 at Red Cross Blood Bank Society Office, M.G. Road, Bangalore. It
starts at 8 a.m. and will continue till 5 p.m. both days. All are requested to volunteer
and donate blood and save the lives of the needy people.
Manish
(MANISH NAIR)
PRESIDENT,RED CROSS BLOOD BANK SOCIETY

Solved Example-2

Your school (Amar Vidyalaya), Bangalore is organizing a cultural evening to collect funds
for the slum children. The Vice-Chancellor, Bangalore University has consented to be the
Chief Guest on the occasion. Draft a notice in this connection to be displayed on your
school notice board. Imagine you are the Rakesh Tilak ,Cultural Secretary of your school.
(Word limit: 50 words)

24
NOTICE
AMAR VIDYALAYA , BANGALORE
CULTURAL EVENING FOR A CHARITY
1st JULY 2011
The school is organizing a Cultural Evening to collect funds for the slum children in the
neighborhood, on 10th July 2011 from 6 p.m. to 9 p.m. in the school playground. Dr.
S.M.Rao Vice-Chancellor, Bangalore University has consented to be the Chief Guest on
the occasion. All are invited and requested to give wide publicity to the event. For
more details contact the undersigned.
Rakesh Tilak
(RAKESH TILAK)
Cultural Secretary

NOTICES FOR PRACTICE


1- As the Secretary of the Science Club of your school, write a notice in about 50
words informing students of an inter class science quiz.
2- You are Abhinav/Alka. You have planned a 2-week course to be arranged to help the
children of your housing society called “Renaissance Housing Society” at Mysore
acquire Spoken English Skills. As the Secretary of the Society , prepare a notice
for the Society’s notice board stating the objective of the course ,giving
necessary details of the course and requesting the children of the society to join
the course.(Word limit : 50 words)
3- Your school, Adarsh Vidyalaya, Solan, is organising a lecture by the famous writer
Vikram Seth on the occasion of ‘Literacy Week’ being celebrated in your school.
Write a notice informing students about the lecture. Invent necessary details.
4- Draft a suitable notice for your School Notice Board giving details of the
inauguration of the Literary Association activities in your school. You are
Deepak/Deepaika, Secretary, School Literary Association. Your School is called
Modern Senior Secondary School, Belgaum.
5- Your school has just completed 25 years of its meritorious service to society. The
Students’ Council of your school has decided to celebrate its Silver Jubilee. As
President of the Council, write a notice in not more than 50 words, for the

25
students of your school, informing them about the Council’s decision and
seeking their co-operation for the success of the proposed Silver Jubilee
Celebrations.
6- You are Sanjay / Sangeeta. As President of the Excursion Club you have
organized an excursion to Katmandu (Nepal) during the summer vacation for the
senior students of your school. Write a notice in about 50 words informing the
students about the proposed excursion.

QUESTION -3
Advertisements: (5 Marks /50 Words)
An ADVERTISEMENT is a kind of public notice asking for or offering services or buying
and selling property, goods etc., or providing information about missing persons, pets
etc.
There are two kinds of advertisements:
1- Classified
2- Commercial
CLASSIFIED ADVERTISEMENTS
You will come across classified advertisements in the columns of newspapers and
magazines. The important features of a Classified Advertisements are--
* No blocks, no design and language to be factual.
* Simple and formal and to the point
* Comprehensive; yet must leave out no important matter.
* Never be too lengthy (confine to the word limit)
Marking Scheme
Format Suitable Heading/Classification) 1 Mark
Content relevance 2 Marks
Expression (Grammatical Accuracy, Spellings Suitable Style- 2 Marks
IMPORTANT TIPS TO BE FOLLOWED
CLASSIFIED ADVERTISEMENTS
(a) Classified Advertisements
• Clearly state the category at the top-e.g.-For Sale, To Let etc.
• Give all necessary details in points using commas.
• Give contact address, name, and telephone number.
• Put the matter in a box.

26
Kinds of Classified Advertisements
I. Situation Vacant/Wanted
2. Lost and found
3. Sale and purchase
4. Accommodation wanted
5. Educational
6. Placement services
7. Matrimonial
8. To-Let
9. Tuitions
10. Packers and Movers
11. Kennel
12 Travels and Tours

27
28
FURTHER SAMPLES OF CLASSIFIED ADVERTISEMENTS
SAMPLE—1
Question 1-You are Personal Assistant to G.M., ABC Metal Tools Company, Mumbai.
Your General Manager has asked you to draft a Classified advertisement for a local daily
for the post of two engineers. Draft the advertisement in not more than 50 words.

SITUATION VACANT
Required two well qualified Mechanical Engineers with a maximum experience of five
years. Remuneration commensurate with the performance and output of the
candidate. Send the Curriculum Vitae within seven days to the Personnel Manager,
ABC Metal Tools Company, 7, Vashist Complex Sikanderpur , Malad East, Mumbai,
along with necessary certificates and references within a week.
Ph-26090003,26093330
SAMPLE—2
Question 2-You are Anurag/Aparna of 110,Swasthya Vihar , New Delhi .You wish to let
out a portion of your newly built house. Draft an advertisement in not more than 50
words for publication in the “To-Let” column of The Hindustan Times, giving all
necessary details .

TO LET
Available 1500 sq.ft. ground floor of a newly built house in Swasthya Vihar with two
spacious bedrooms, attached bathrooms, drawing-cum-dining room, kitchen and
interiors. Rent expected-Rs. 7000/-. Interested persons may please contact—
Anurag/Aparna ,
110 Swasthya Vihar , New Delhi.Phone-27439543

29
NECESSARY INFORMATION
SITUATION VACANT LOST AND FOUND
• Specify the post and number of vacancies • Begin with LOST/FOUND
• Qualities of the person required • Description of the article
• Name of the company (optional) • When and where the article was
• Age and sex of the candidate Lost/Found
• Pay scale and perks • Reward for finding it
• Mode of applying • Contact address and phone number
• Contact address and phone number
TO-LET KENNEL
• Type of accommodation, No. of • Breed of dog
rooms/floor • Age
• Whether it’s independent or an apartment • Colour
• Rent expected • Training
• Type of tenant required-Bank • Price expected
employee/small family • Contact address and phone number
• Whom and when to contact
• Contact address/phone number
VEHICLES FOR SALE PACKERS AND MOVERS
• Make : Maruti/Hyundai Santro etc. • Specify the services
• Give reason why you should be given
• Model/colour/accessories/year of a chance
manufacture/mileage • Area of work
• Condition • Whom to contact
• Contact address and phone number
• Ownership details
• Price expected
• Contact address/phone number
PROPERTY FOR SALE Travels and Tours
Plot for sale • Name of the agency
• Location-where it is • Destinations and durations
• Area : in sq. meters/yards • Details of the package
• Name of the development authority • Discounts if any
• Price expected • Contact address and phone number
• Contact address/phone number

30
TUTIONS HOUSE/FLAT FOR SALE
Classes and subjects Type of accommodation, No. of rooms/floor
Special qualities of the tutor Whether it’s independent or an apartment
Qualifications and experience of the tutor Price expected/negotiable
Previous results of his/her students Location-where it is
Contact address/phone number Area : in sq. meters/yards
Name of the development authority
Whom and when to contact
Contact address/phone number

COMMERCIAL ADVERTISEMENT
COMMERCIAL OR DISPLAY ADVERTISEMENTS
• They are designed for commercial purposes.
• Require more space, hence are costly. Must be attractive as they with visuals,
catchy phrases and slogans.

MAIN FEATURES:
• Must be attractive-with a catchy caption, heading or sub-headings.
• Figurative language. (alliteration and metaphors especially)
• Proportionate spacing of fonts with different sizes.
• Usually attractive with catchy slogans, punch lines, witty Expressions, pictures or
sketches.
• Special offers or discount, if any.
• Details of the product or event given in a clear, precise way.
• Give name, contact number and address of the advertiser.
• Present the matter in a box.

Commercial Advertisements-Sample 1
Draft attractive Commercial Advertisement inviting customers to a leading resort called
‘FORT RESORT AND FARMS’ to organize weddings, parties and other events.

31
SAMPLE—2

A HEAVENLY PLACE

FOR WEDDINGS, PARTIES AND EVENTS

WELCOME TO FORT RESORT AND FARMS!!!

LARGEST VENUE AROUND CHANDIGARH

SPREAD OVER 12 ACRES


BREATHTAKING BACKDROP! MORE THAN 500 TREES
COMPLEMENTED BY WATERFALLS AND FOUNTAINS!!
*****************************************************

FORT RESORT IS AN IDEAL LOCATION TO CELEBRATE

MARRIAGES—PARTIES—ANNIVERSERIES AND VARIOUS OTHER


EVENTS

ISSUED BY : FORT RESORT HOTELS

i. SHAHEED BHAGATH SINGH MARG

ii. CHANDIGARH --56

CALL US AT – 060-5467834/568/769

32
Draft an advertisement on behalf of Department of West Bengal highlighting the
beauty of Dargeeling, Digha Sunderbans and Dooars known for their green hills, sea

shore, tea-gardens and Royal Bengal Tigers.

WEST BENGAL—YOUR DREAM HOLIDAY PACKAGE


Darjeeling the “Queen of the Hills” Digha , the resort by the sea
Sunderbans-with Royal Bengal Tiger Dooars with its rolling verdant
tea-gardens
West Bengal packs in the richest natural variety within its small periphery. To this
ultimate destination the Tourism Development of West Bengal adds excellent facilities!
Reach out to this splendor today

GOVERNMENT OF WEST BENGAL


For details contact: Tourism Department, Government of West Bengal. @. Brabourne
Road 4th floor,Kolkata-700001 , West Bengal
Phone-091-033-2225/4723/24/25

Note- You can use Match Stick figures and rough sketches for illustration. Do not spend
much time on illustration. Written language and expressions will only help you score
better.
Exercise for Practice (Advertisement)
1- M/s Blue Towers, a readymade jeans company is launching their 2011 range of
jeans for men and women. Prepare a beautiful advertisement.
2- M/s Leather Shoes have brought a new range of formal shoes for men. Prepare a
suitable advertisement for the range.
3- M/s Metro Shirts have introduced their new summer range of men’s shirts and
T-shirts. Prepare an attractive advertisement for the range.

33
4- M/s Pen India has brought out a new gel pen in many colors. Prepare an eye
catching advertisement for the pen, giving all relevant details.
5- Draft an advertisement announcing the launch of special health drink by Health
Care Private Ltd., highlighting its nutritive value.
6- You are working for an advertising agency. Draft an attractive advertisement for
the company called Modern Machines which is launching washing machines.
(Word limit: 50 words)
7- You have recently started a Yoga Centre for school children. Draft an
advertisement to be published in a local daily about it, giving all relevant details.
8- You want to dispose off your car, as you are going abroad. You are Harish of No.
10, Kailash Ganj and Lucknow. Draft an advertisement to be published in the
daily, ‘The Hindustan times’, under classified Columns.
9- You have recently started a Centre for Personality Development for school
children. Draft an advertisement to be published in a local daily about it , giving
all relevant details.
10- You are Manisha. You have started hobby classes for children of 6 to 12 years.
Prepare a suitable advertisement giving all the required details.. (50 words)
11- You are Mohan of Raja ki Mandi, Agra. You want a dance teacher for your
younger sister. Write an advertisement to be published in the classified columns
of a local paper. (50 words)
12- You propose to sell your flat as you are going abroad. Draft an advertisement to
be published in the classified columns of ‘The Times of India’, New Delhi. (50
words)
13- You have a commercial flat suitable for an office/bank. You wish to rent it out.
Draft an advertisement to be published in ‘The Times of India’, New Delhi under
the classified columns.
14- You are Ashoka/Ashita of Sitapur, Lucknow. You have got a foreign assignment
and would be going abroad soon. You have a color television set and a

34
refrigerator to sell. Draft a suitable advertisement to be displayed on the housing
society notice board. (50 words)
15- You have recently constructed a house with all the facilities in a posh area. You
have decided to give it on rent. Draft a suitable advertisement for the ‘To-Let’
column of a local daily. Invent all the necessary details, including a contact
address. (50 words)

Posters: (5 Marks -50 Words)


A POSTER is a large notice announcing or advertising something. It generally creates
social awareness about any problem or needs .It also conveys a social message in an eye
catching way.
It must capture the reader’s attention and motivate him/her to read on.
Main Features of a Poster:
 Layout: A creative title in the form of a slogan or rhyming words for
example :- Speed thrills but kills, Pollution needs a Solution,
Green is Clean
• You can use Match Stick figures for illustration.
• Proper spacing and proportionate headings and illustrations.
• Date, time and venue must be mentioned in case of an event.
• A poster displays a message, or an appeal to create awareness.
• The content should be organized in an appropriate style.
• Use appropriate expressions and correct language.
• Theme should be clear.
• The poster should be creative and related to the content and theme
should not be vague.
• Do not spend much time on illustration. Written language and
expressions will only help you score better.
• Name of the issuing authority or the organizers is a must.

35
Making scheme
Content - 3 Marks, Expression - 2 Marks

SAMPLE POSTER—1

Question-On behalf of Department of Environment, Bangalore Administration, draft a


Poster on the detrimental consequences of burning leaves and garbage upon the
environment and also issuing a warning of the punishment to those indulging in the
same.

SAVE ENVIRONMENT! GENERATE PROSPERITY!


SAVE ENVIRONMENT

DON’T BURN DRY LEAVES OR GARBAGE


Emits toxic and harmful fumes!
It increases suspended particle matter (SPM) in the air and pollutes it!
Produces carbon-di-oxide causing greenhouse effect!
Disturbs the natural climate of the planet.
Leads to cough, eye irritation and respiratory disorders!
Municipal Corporation Bangalore has banned the burning of leaves and garbage.
Any Violation depending on the scale is punishable with imprisonment for a term extending to five
years and fine up to one lakh rupees
ISSUED IN PUBLIC INTEREST
DEPARTMENT OF ENVIRONMENT :
BANGALORE ADMINISTRATION
SAVE ENVIRONMENT! GENERATE PROSPERITY!

36
SAMPLE POSTER-2
The Citizens’ Awareness Forum of your city has decided to launch a campaign against
the use of drugs. You are asked to get posters prepared. Design an attractive and
effective poster against the use of drugs.

Don’t be late
SAY IT JUST TODAY
DRUG ADDICTION
Brings
ENDLESS MISERY AND RUIN

BE FIRM
ACT NOW
IT IS NOW OR NEVER
DRUGS MAY THRILL!
BUT THEY SURELY KILL!
EASY TO GET IN!
DIFFICULT TO GET OUT!

Issued by
CITIZENS AWARENESS FORUM , PUNE

37
POSTERS FOR PRACTICE
1- Recent rains have caused havoc in some parts of our country. You are Surya, a
member of the social service organization, SEVA MANDIR, Bangalore. Draft a
poster requesting people to help the rain and flood affected families physically
and economically.
2- A ‘Check Vehicular Pollution’ campaign is being organized in your city. Draft a
poster to be used in the campaign.
3- Water is precious and each one of us must stop wastage. Prepare a poster in not
more than 50 words, for creating that awareness.
4- Draft a poster announcing a ‘Book Week’ being organized by the Cultural Society,
Ahmedabad.
5- Your school is planning a campaign in support of eye donation to mobilize the
students and society. Design a poster to be displayed in different areas of the
locality surrounding your school highlighting the need for eye donation and eye
banks.
6- Prepare a poster to bring about awareness against the hazards of reckless use of
plastics.

4. Invitations and Replies: (50 Words 5 Marks)


No man is an island. Man is a social animal. We need to be with people. This is the
reason why solitary confinement is the worst form of punishment one can experience.
So, we celebrate many occasions together. To invite someone for an occasion we use
the written form-INVITATION.
Invitations are generally printed cards through which we invite our guests on some
auspicious occasion like wedding, birthday, wedding anniversary, house warming,
inauguration of a shop/factory etc.
Invitations are of two types: (a) Formal (b) Informal. They can be printed on cards or can
be drafted in the form of letters.

38
MAIN CHARACTERISTICS
An invitation is complete information. It answers the questions: who, whom, when,
where, what time and for what. The important components of an invitation therefore
are:-
o The occasion
o Name(s) of the invitee(s)
o Name(s) of the host(s)
o Date, time and venue
o Name(s) of the chief guest or special invitees , in case of an official
invitation

FORMAT OF FORMAL INVITATIONS


In case of formal invitations, each of the following is written in a separate line with fonts
of varying sizes.
• Names of the hosts
• Names of the invitee (in case of a formal letter of invitation))
• Formal phrase of invitation , for example :-
Request the pleasure of your benign presence/company.
Seek your auspicious presence.
Solicit your gracious presence on the auspicious occasion
• The date, time and venue of the event.
• The occasion/reason of the invitation.
A-CHARACTERISTICS OF FORMAL INVITATIONS

1- Meant for a lot of invitees

• These are written in the third person.

• In case a VIP is invited as the chief guest the name of the VIP must appear
prominently.

• Name of the invitee is not to be included. The addressee’s address is to

39
be written only on the envelope.

• Simple Present Tense is to be used.

• The date of writing is not to be given.

• There is no signature of the host.

• The abbreviation RSVP (French : repondez s’il vous plait ) i.e. ‘please
reply’ is written below on the left with name(s) , address and phone
number of the host(s)

• Put the invitation into a box

• Do not exceed 50 words

2- Meant for an individual (a formal letter of invitation)

• Include the name of the invitee

• These are to be written on run-on lines. The sentence is not broken


into different words/phrases.

• Other details are similar to the mass-scale invitations

EXAMPLES OF FORMAL INVITATIONS

1. Your father, Mr. Raj Kumar Gupta , residing at K-18 ,13th Cross Malleswaram,
Bangalore wants to celebrate the success of your brother Rohith’s clearing the IIT-JEE
Entrance Examination and securing admission in IIT Powai ,Mumbai. He wants you to
draft a formal invitation for him on his behalf. Draft the invitation for him in not more
than 50 words.

40
Mrs. And Mr. Raj Kumar Gupta

Seek your auspicious presence at the

Grand Celebration Party

of their son Rohit’s successfully passing the

IIT-JEE ENTRANCE EXAMINATION

at

8.00 p.m.

On

1st July , 2011

at

UTSAV BANQUET HALL

84-Dr. Raj Kumar, Road, Rajaji Nagar

Bangalore-55

Compliments from Friends and Relatives

RSVP

k-18, 13th Cross ,Malleswaram

2712001/9732966964

41
2. Your school, Bal Peeth, Bellary is planning to organize the Annual Sports Day in the
school premises. Draft an invitation in not more than 50 words giving all the essential
details.

The Principal ,Staff and Students of

Bal Peeth , Bellary

request the pleasure of your company on

ANNUAL SPORTS DAY

at 5.00 p.m. on Monday , the 4th July ,2011

at the school grounds

Air Marshal S.K.Sinha , AVSM,

has kindly consented to be the CHIEF GUEST

Smt. Radha Sinha

has graciously agreed to give away the prizes

RSVP

2354679

(You are requested to be seated by 4.30 p.,m.)

42
FORMAL INVITATION-LETTER FORMAT

Kamala Nehru School

Cantonment

Bangalore

July 10, 2011

Mr. K.J. Singh

DCP

Police Lines

Bangalore

Invitation---Annual Day Function

Sir,

It gives us immense pleasure to inform you that we are celebrating our school’s Annual
Day function on July 10, 2011 at 5.30 p.m. in the Activity Hall of the school. We shall
consider it a great honor if you could grace the occasion and preside over the function
as the Chief Guest and give away the prizes.

Yours faithfully

Sd/-

Principal

43
B-WRITING INFORMAL INVITATIONS
Informal:-
* Written in a letter form, in informal format. Such letters are very persuasive in
nature
* Written in the first person
* Salutation is ‘dear + name’
* Complimentary close ‘yours sincerely’
* Date of writing the invitation is given
* The sender’s address appears on the left hand side
* Various tenses used to suit the sense.
Value Points:
Format - 1 Mark
Content Relevance - name of the person(s) who is inviting; the invitee, date, day, chief,
guest place Occasion etc. - 2 marks
Expression - Grammatical accuracy, Language - 2 marks

REPLIES
Replies- Accepting or Declining
Formal - follow a set formula
- formal words — kind invitation’, ‘great pleasure’, ‘regret’ etc.
- Use the third person (‘they’) instead of first person (‘I’, we’)
- The address of the writer and the date to be written.

Informal - Accepting and declining


- Like an ordinary letter
- do not use any formal expressions but use informal words
and expressions
-use the first person (‘I’, ‘We’)

44
SOLVED EXAMPLES

• Look at the following series of invitations (formal & informal) and their replies. The
occasion is the same but the style is different.

1. FORMAL INVITATION

Mrs. Malathi & Mr. Venkatesh

request the pleasure of your company at dinner

On the occasion of their

50th Wedding Anniversary

At 8 p.m. on Tuesday ,12th July 2011

at

Hotel LEELA PALACE, Bangalore

RSVP: 12,Margosa Road, Malleswaram, Bangalore

Phone-080-23564543

45
FORMAL REGRET

7th July 2011

Mrs. And Mr. Naidu thank Mrs. And Mr. Venkatesh for the invitation for dinner on the
occasion of their 50th Wedding Anniversary on 12th July, 2011 at Hotel Leela Palace.
However, they express their inability to be present on the occasion due to a prior
engagement.

Best wishes,
Naidus

FORMAL ACCEPTANCE

7th July 2011

Mrs. and Mr. Naidu were pleased to receive the invitation for dinner on the occasion
of the 50th Wedding Anniversary of Mrs. and Mr. Venkatesh, and confirm their
participation in the function.

Best wishes

Naidus

46
EXERCISES FOR PRACTICE
1. Your school is organizing the prestigious KVS National Level Social Science
Exhibition in its premises. Draft an invitation in not more than 50 words giving all
essential details.
2. You are Dharma raj, son of Ramesh Patnaik, Bannerghatta , Bangalore. Your
father wants you to draft a formal invitation to be sent on the occasion of your
sister, Sheela’s marriage. Prepare the invitation.
3. You are Mohan/Molly. You have been invited by Rotary Club to act as one of the
judges for an Elocution Competition for students of classes XI and XII. But due to
a previous engagement you cannot accept this invitation. Write a formal reply to
the President of the Club regretting your inability to accept the invitation.
4. You are Aakash/Varsha. You have been invited to attend the wedding of your
friend’s sister during the summer vacation. Respond to the invitation informally
accepting it.
5. You have opened a restaurant in Uppal Road, Hyderabad. Draft an invitation for
the inauguration of the same, specifying the chief guest and other important
details like date, time and venue. Do not exceed 50 words.
6. Your school is organizing a Book Fair on its sprawling play-ground. Draft an
invitation giving all the details including the chief guest.

4. FACTUAL DESCRIPTION (10 Marks)

Factual description is a description of place, person, process and object.


• Description of a place / building includes the location, surroundings, interiors,
purpose etc.
• Description of a person includes appearance, mannerisms, dress, gait and any
other particular information to be given may be relevant to the description.
• Description of an object incorporates appearance, its parts, its function or purpose,
advantages and any other detail relevant to the description.

POINTS TO REMEMBER
• Collect details of description and logically write them in a paragraph.
• Provide a suitable title.
• Include external features, finer or internal details, how the object functions and
its operation.
• Use appropriate connectors while describing a process. Use words like ‘first’ and
‘second’.
• Use the passive voice unless the active voice is desirable (e.g. in a piece which
describes how to bake a cake)

47
Split up of marks
Total marks: 10,
Title: 1, Content: 4, Expression: Accuracy – 2 ½ + Fluency – 2 ½ : 5
SOLVED FACTUAL DESCRIPTION
Robots
A robot is like a human being in the actions it carries out. It can perform a chain of
actions. The brain of the robot is a small computer which directs its movements. In the
computer’s memory are instructions for carrying our specific tasks. The robot usually
has an arm that can move in several directions and can do several things. The hand has
a magnet or a gripper attached to it to pick up things.

PROCESS WRITING (SOLVED)


How does a Pressure Cooker work?
Water is poured into the pressure cooker and the food to be cooked is placed in it. The
pressure cooker is then placed over the flame of a stove and the water begins to heat.
When the boiling point, about 100 degrees centigrade, is reached, steam begins to form.
However, the steam is not allowed to escape and pressure builds up within the cooker.
As the pressure increases, the temperature within also increases and food is cooked. To
prevent explosion due to high pressure, the cooker has a weight and a safety valve that
let out excess of steam.

Questions for practice


• You organized a quiz in your school last week. Write a description of how you
went about it. (125 words)
• Write a factual description of a tourist spot you have visited in about 125 words.
• Write a factual description in about 125 words of the bustle at your bus
terminus.
• Write a factual description of the children’s park in your locality in about 125
words.
• Write a factual description of shopping mall in your city in about 125 words.
Include details of layout, special facilities like ATM, restaurants, escalators etc.

REPORT WRITING

A report is a brief account of an event that has already taken place. A Report helps in
recording events of importance that occurs in our day to day life. A report attempts to
present the first hand information of an incident or event. A report of an event presents
a record of events that took place. A report on an event includes one’s ideas, opinions
and impressions on the event.

48
POINTS TO REMEMBER
• Mention the place, date, time and other relevant facts about the event.
• Include information collected from people around or affected by the event.
• Write the name of the reporter.
• Provide a suitable title/heading.
• Write in past tense.
• Write in reported speech and use passive form of expression.
• Develop ideas (causes, reasons, consequences, opinions) logically.
• Write in a less formal and more descriptive manner while writing a report for a
school magazine.
• Present your ideas and impressions to make the report interesting.
Split up of marks

1. Total marks: 10

2. Title + Reporter’s name: 1 mark

3. Content: 4 marks

4. Expression (Accuracy 2 ½ + fluency 2 ½): 5

SOLVED REPORT

You are Roshan /Roshini, Secretary of Cultural club, Hyderabad Public School,
Hyderabad. As a member of N.G.C. (National Green Corps), write a report, in 100 – 125
words for your school magazine. (10 marks)

TOWARDS A GREENER ENVIRONMENT

The cultural club of our school in association with National Green Corps
organized many activities during the year 2010 for the cause of greener
environment in and around the school. A seminar in the month of February was
held for students on the need to save environment. In April, the Vanamahotsav
week was observed, which was inaugurated with a tree plantation drive. 500
saplings were planted in the colonies in and around the school.

Several inter-house competitions including poster-making and drawing were


conducted which witnessed a huge participation. Students volunteered to clean
the surroundings. Pamphlets were distributed among the public to promote
cleanliness and hygiene.

49
A rally was organized in September to create awareness among the public to
stop the use of plastic bags and to replace it with eco-friendly products. The
various purposeful activities conducted during the year were well received and
were to a great extent effective in creating awareness among the people.

Roshni

Secretary, Cultural Club

Questions for Practice

1. You are Brinda/ Bharath. You worked as a volunteer in the Literacy campaign
organized in your city. Write a report for the school magazine in about 125 words on the
effectiveness of the programme highlighting the methods used and the role played by
government and non government agencies.

2. Your school observed Library week. As a head boy/ head girl of your school write a
report on it to be published in your school magazine (125 words)

3. You are Poorva / Partha, Cultural Secretary of your school, D.B. Senior Secondary
School, Ambur. A week-long Music and Dance festival was organised by your school.
Write a report in about 125 words for your school magazine.

4. The Debating Society of your school has recently held a workshop on ‘Continuous
and Comprehensive Evaluation’ (CCE) introduced for the students for class X in all
schools. The students discussed the assessment made by the school on the basis of their
participation in various activities and the system of grading. Write a report in 100 – 125
words for your school magazine. You are Parveen / Payal , Secretary of the Society.

5. You visited a Job Fair organized by Ability Foundation at Chennai recently. You were
impressed to see that nearly 55 companies from various sectors such as Information
Technology, Telecommunication, and Electronics etc. offered jobs to the final year
students of colleges. As a reporter of ‘The Deccan Times’ Chennai prepare a report in
100 -125 words. You are Peeyush / Priya.

6. You are Anand / Anandi, a Times of India correspondent. You attended the inaugural
function of Tamilnad Hospital, Chennai. Mentioning the specialties of the hospital, the
number of beds, all available facilities etc. and details about the ceremony, write a
report in 100 - 125 words for your newspaper.

7. You witnessed a programme performed by differently abled persons on Zee TV. You
were very much impressed by their performance and were emotionally touched.

50
Highlighting their talent, reaction of the judges to their performance etc., prepare a
report in 100 – 125 words for your school magazine. You are Govind/ Gayathri.

8. You are Latha / Lalith of Gurgaon. You attended the inaugural ceremony of the newly
opened vocational stream and hobby classes at the secondary school level in Paras
Public School, Gurgaon. Draft a report in 100 – 125 words to be published in ‘Gurgaon
Times’. Invent other details.

9. The Readers’ Club of your school visited schools for the less privileged to encourage
and promote reading habits. Besides donating books, the members also distributed
books collected from individuals and organizations. Write a report in 100 – 125 words
about your campaign and its success. You are Anush / Anu, the Club Secretary.

10. You are Sushma / Swaraj of L.M. Jain School, Ajmer. As Secretary of Co curricular
Activities club, you visited a slum area in your city where the people suffered a great
loss of life and property in a massive fire. The students of your school rendered their
services and material help to the victims. Write a report in 100 – 125 words for your
school magazine.

WRITING A SPEECH
A SPEECH is a formal talk that a person delivers to an audience. It must have the
capacity to keep the audience spellbound with proper choice of words, expressions and
examples. At the same time care should be taken not to deviate from the main subject.
You must keep in mind that it is a speech and you need to begin the draft with
addressing your audience. The conclusion should be with a sentence thanking your
audience for their patient listening.

Writing a speech needn’t be a nail biting experience!

Before you go further you need to know-

WHO you are writing your speech for (the audience)

WHAT your speech is going to be about (its topic)—the main points in order of
importance with supporting research.

Follow the system of CODER for writing your speech


C- Collect the ideas.
O-Organize them in a logical manner
D-Develop the points into paragraphs
E-Edit the speech
R-Revise and rewrite it

51
Brainstorm to start planning your speech

On a piece of paper write these headings with enough space between them for notes:

• WHO
• WHAT
• HOW
• WHEN
• WHERE
• WHO is this speech for?
• WHAT am I going to tell them that are relevant and interesting?
• HOW long is the speech expected to be?
• WHERE is the speech going to happen? (Hall, outdoors, stadium, morning
assembly…)

NOTE – It is important to note that the written speech should consist of small
sentences having not more than 10 to 12 words in a sentence.

OK. You’ve got all the notes ready; so let’s mix and match and re-write until the outline
is clear. After you’ve completed this part of planning your speech you’ll be ready to do
any RESEARCH required and then you’re on to the task of WRITING your speech.

POINTS TO REMEMBER:

 Greet your audience and introduce the topic you are speaking on.

 The introduction part of the speech is the most important part. It is this
part which decides the fate of the speech.

 Do not give a separate title for the speech. Keep in mind as already stated
the occasion of the speech and the nature of the audience.

 The introduction should not be very lengthy. The main topic should be taken up
as quickly as possible. Use concrete terms and tangible examples. Avoid
abstract phrases which are quite vague. Use simple and familiar language.
 The effects, such as raising one’s voice, giving stress to certain words,
pausing for a short time etc. can be represented in a written speech by
using such techniques as increasing the size of the letters of the sentence
to be stressed, or using all capitals for a particular word to be
emphasized, leaving a few dots (….) after a question to indicate a pause.
 The conclusion plays a very important role in the success of a speech.
Here we should remember Shakespeare’s advice that brevity is the soul
of wit. Give your own conclusion on the topic in a telling manner.
 A powerful quotation or lines from a beautiful poem which can
summarize your view point can also be used.

52
 Marking scheme is the same as for the article.
 Say/Write ‘Thank you’ at the end of your speech.

A SAMPLE SPEECH

You are a student volunteer of National Literacy Mission (NLM) wing of your school
involved in the Adult Literacy Campaign: “Each one, Teach One”. Write out the speech
you would be delivering at each place of your visit. Do not exceed 200 words.

ANSWER:

Dear Friends

Good morning to you all

We have all assembled here for a noble cause—a mission to eradicate illiteracy. It is the
dream of the National Literacy Mission to educate all the Indians. We would work
together for an India where EACH PERSON IS LITERATE. Most of the learners are adults
who have a rich experience of life. Therefore, the learning strategies that will be
adopted in our programme will be exploratory and interactive. We have to teach as well
as learn. We must keep in mind certain important factors……… their experience, wisdom
and maturity while discussing any topic. We should encourage them to talk and express
their views in a frank and fair manner.

We should always keep in mind the OBJECTIVES set for the adult illiterates. We have to
make them literates. Inculcate in them a love for reading, writing and arithmetic. Let
them do simple calculations and exult at the right calculations. Health awareness and
job related problems are the next in order. IT IS ONLY AFTER A STRONG BRIDGE OF
CONFIDENCE IS BUILT that we would be able to create in them awareness about civil
rights, duties, privileges and obligations. When they become confident of their powers,
we must create in them social awareness. Issues like environmental pollution,
population control and employment generation can then be discussed…..In short let’s all
resolve to make sincere efforts to achieve these goals.

Thank you.

53
Questions for practice:

Q.1.-Water is very precious. Some scientists even go to the extent of saying that the
Third World War may be fought on the issue of water. Write a speech in 150-200 words
expressing your views to be delivered in the morning assembly in your school.

Q.2.-“To use the latest technology the right way, is in the hands of the youth today.”
Write a speech 150-200 words discouraging the misuse of technological products like
cell phones, computers etc and highlighting the need to use them to promote harmony
and goodwill in the society.

Q.3.-“Our Good Earth”, an environmental awareness magazine has launched a


marathon 'Clean Your City’ campaign. As an active participant write a speech to be read
out in the morning assembly urging students to participate in the campaign in 150-200
words.

Q.4.-As a concerned citizen, you are worried about the harmful effects of the insanitary
conditions in your town. Write a short speech on the necessity of sanitation, to be given
at the morning assembly in your school.

5. LETTER WRITING (10 MARKS)

The most common form of written communication is the letter. Letters should have a
format that goes with the latest conventions.
Letter types include
• Informal Letters-These are letters written to close associates.
• Formal Letters include-
(a) business or official letters (for making enquiries, registering
complaints, asking for and giving information, placing orders and sending
replies;
(b) Letters to the editor (giving suggestions on an issue);
(c) Application for a job.
MARKING SCHEME OF LETTER WRITING
Format- 2 marks.
Proper Layout – Sender’s address, Date, Receiver’s address, Subject, Salutation,
Complimentary close.

54
Content-4 marks
• Relevant ideas to be presented in a coherent way.
• Use connectors appropriately, indent the paragraphs.
• There should be minimum three paragraphs.

Expression-4 marks.
• Grammatical accuracy, spellings-2 Marks.
• Coherence, relevance of ideas and style- 2 Marks.
Important points to remember while drafting letters:
• Write the letter in the full block format (i.e. to the left)
• State your reason for writing in the first paragraph and stay on track.
• Always include specifics that will help ease the recipient’s task. For instance, if
you are writing to a bank, mention your account number.
• If you refer to other correspondence, quote date of the letter/s reference
numbers, file numbers, order numbers, cheque number with date. Include
copies whenever possible.
• Gently and respectfully direct the recipient’s course of action.
• It may be strongly worded but always be courteous and use polite language and
pleasant tone.
• Self introduction should be avoided.
• Informal expressions, greetings or contracted forms of words should not be used.
• The language should be simple, straight forward and to the point.
• Leave a line/ additional space between paragraphs since no indentation is followed.
• Open punctuation to be followed. ( No punctuation marks to be used)
• Do not mix up the old format and new format.
• In letters to the editor – Do not ask the editor to solve the problems.
• In job application letters Curriculum-vitae (CV) /Bio-data/ Resume/ should be written. It
should not be included in the body of the letter but after the complimentary close as
enclosure.
• In the Letter to the Editor use ‘yours truly’ as the complimentary close.

55
FORMAT OF THE LETTER

Devi Enclave
Sender's Raj Ram Marg

Address. Bangalore-21

Address of the The Director/Mr/Mrs


Addressee  Global Enterprises,
8th cross, Victoria Layout,
Bangalore

Date 
14/April/20XX

Salutation 
Sir,

Subject 
……………………
………..

Content 
Introduction Conclusion
Body

Signatory  Yours faithfully


Rohan Mathur

56
Sample Letters
You are Sidhi/ Sanjay of U-03 G Block Kasturi Nagar Mysore-37.Write a letter to the Director NGO Bird
Links Society, Bangalore expressing your concern about the decrease in the number of the birds in Delhi.
House No- U/03

G Block

Kasturi Nagar

Mysore -37

30 June 2011

The Director

NGO Bird Links Society

Bangalore

Karnataka

Sir

Diminishing number of birds

With deep regret I wish to bring to your kind notice about the decrease in the population of birds in
Mysore. Mysore is known to have more than 500 species of birds but many of them are on the verge of
extinction.

According to the reports from the recent seminar held on Conservation of Birds, there was a time, when a
number of birds would flock to the banks of river Kaveri, Bandipur wild life sanctuary and forested canopy
in Mysore. These numbers have come crashing down in the recent years.

The main reasons are rampant construction of buildings, bridges and flyovers, cutting of forests and of
course environmental pollution. Many species have become endangered. It is time the people of Mysore
were made aware of this grave situation by conducting seminars and workshops.

The locals should be given incentives for preserving the birds. Sanction for the construction of buildings,
malls, bridges ands so on should be given only after proper evaluation of the environmental impact of
those constructions over the life of species there. Let’s not forget that birds are an important link in the
food chain.

I request the government, environmentalists and the society to show their concern before it is too late.

Yours faithfully

(Sidhi Sharma)

57
Sample Letter No-2
You are Suresh/Sarika Jain of No-20 Jawahar Nagar,Jaipur. Two months ago you bought
a desert cooler from M/s Cool Home Coolers Jaipur. Now you discover that it is not
working properly. Write a letter to the Manager Customer Care complaining about the
malfunctioning of the unit and asking them to repair and if needed to replace it against
warranty.

20 Jawahar Nagar
Jaipur

24 June 2011

M/s Cool Home Cooler


Jaipur

Sir
Complaint about the desert cooler
Refer Cash Memo No DC-98765 dated 17.4.2011

I wish to bring it to your notice that I purchased a ‘Cooler Home’ desert cooler from you on 17
April, 2011 (Vide Cash Memo No DC-98765, dated 17.04.2011). The cooler did function well in
the beginning for one and a half months and then it has started giving problems. The problems
are - water leakage, heavy noise while working and high consumption of power.

I, therefore, request you to get the set repaired by a technician. In case it is irreparable, kindly
replace it with a new one as it carries a warranty period of three years.

Yours faithfully
Suresh

58
Sample Letter No- 3
You are Isha/Ishan, the secretary of the Students’ Forum of Sahrudaya Vidyalaya,
Indranagar. You have planned an excursion to Jog falls during the summer vacation.
Write a letter to the General Manager, Southern Railway, for reservation of a bogie for
60 students from Bangalore to Shimoga and back, supplying all necessary information.

Secretary
Sahrudaya Vidyalaya
Indranagar.

01 Mar 2012

The General Manager


Southern Railways
Bangalore

Request for the reservation of a Bogie for 60 students

Sir
A group of 60 students of class XI of this school has planned to visit Jog Falls during the summer vacation.
They will be accompanied by three teachers and two attendants. They plan to leave Bangalore on 15th
May by Jeevan Express and return by the same train from Shimoga on 25th May.

Kindly confirm whether a bogie can be reserved for the above mentioned dates. We would also like to
know the estimated fare so that we can make necessary arrangements at our end.

The List of the members of the tour party is enclosed in the prescribed proforma.
We request you to send the detail at the earliest to enable us to plan in advance.

Yours faithfully
Isha
Secretary

59
Sample letter No. 4

You are Satish/Sonali, the student prefect in charge of the school library. You have been
asked to place an order for children’s story books (10 – 13 years). Write a letter to
M.S.Book Depot Ramnagar, Bikaner placing an order for the books. Invent the necessary
details.

St. Mary’s Convent


Jabalpur

15th July, 20…

M/s M.S. Book Depot


Ramnagar
Bikaner

Sir

Supply of children’s story books

The school has decided to purchase story books for children between the age group of
10 and 13 years. The books mentioned below may please be sent by V.P.P. within 10
days from the date of receipt of this letter.

S.No Name of the book Author Quantity


1 Stories from the Panchatantra A.S. Rawal 5
2 Tales from the Arabian Nights S.E. Paces 10
3 The adventures of Tom Sawyer S.E. Paces 10
4 Tales from Shakespeare Charles & Mary Lamb 10

We request you to give 20 % discount on the printed prices. We assure you to pay the
VPP charges at the time of delivery of books.

Yours faithfully
Satish
(SATISH)
School Prefect
St.Mary’s Convent

60
QUESTIONS FOR PRACTICE
1. You are Manas/ Manasi at the Press Apartments of Sunder Nagar, Bihar. The main road
leading to this colony has three man holes causing frequent accidents at night. There are no
street lights on the main road. Write a letter to the Editor of the Times of India drawing
attention of the government to this problem of the residents.

2. Write a letter to the Editor of a national daily inviting the attention of the authorities towards
the increasing Environmental pollution in your city.

3. You are Vishal / Veena, a student of class XII of 10, Green Park, New Delhi is interested in
learning languages. You come across an advertisement in The Times of India issued by The
Institute of Foreign languages, New Delhi about the certificate course in Japanese offered by
them. You decide to write a letter to the Director of the Institute seeking more information
about the course duration, working hours, fee structure, etc. Write the letter.

4. As Mr. R. Singh, Head of the Department of Chemistry, Cambridge High School, Pune, you had
placed an order with Messrs. Scientific Equipments, Dadar, Mumbai for test tubes and jar for the
lab. When the parcel was received you observed that markings on the test tubes were not clear
and some of the jars were damaged. Write a letter of complaint seeking immediate
replacement.

5. You are the Manager of Fitness Club of Gandhi Road, Ernakulam. Write a letter to Messrs.
Pioneer Sports Co, Kochi, placing an order for a minimum of 4 fitness equipments with details.

6. You are Rama/Raman, a member of Parent-Teacher Association of Little Valley Senior


Secondary School, Shillong. Write a letter to the Principal of the school asking him to introduce
vocational stream in the school providing facility of teaching such subjects as computers,
insurance etc. so that the students may not needlessly continue academic studies.

7. Write a letter to the Corporation authorities of South Chennai to complain about the general
unsustainable growth of building activities in South Chennai with poor parking and other civic
amenities to support it. Urge them to plan with a long-term view.

8. The photocopier of your office which was purchased recently has a problem with the toner
settings. It is still in the guarantee period. As Purchase Manager of Talent Search, 15th Cross, J.P.
Nagar, Mysore, write to the manufacturer seeking replacement of the defective part.
9. Write a letter to the librarian of the District Library in your town asking for details regarding
membership. Also ask about the timings and rules and regulations of the library.

10. You are Apoorva/Asha, Industrial colony, Ambattur, Chennai. You came across an
advertisement for the post of Senior Executive, Accounts in Solace Medical Equipments, Chennai
unit. Write a letter to the advertiser applying for the job. Also give your detailed resume.

61
6. ARTICLE WRITING

Writing an article is an art. The dictionary defines an article as a piece of writing about a
particular subject in a newspaper or magazine. An article is an expression of one’s
thought on an issue or a subject logically and coherently written in meaningful
paragraphs.

POINTS TO REMEMBER
• Give a title that catches the attention of the reader.
• Begin with a striking opening sentence which addresses the readers and gets
them interested in the topic.
• Present a strong argument for your ideas supporting it with evidences or
elaboration.
• Use linking devices (however, therefore, although, even though, in order to…) to
make the composition appear a whole.
• Introduce a new point at the beginning of an each paragraph that follows, to
strengthen your ideas.
• Develop your ideas as much as you can to make them interesting and
substantial.
• Conclude with your strongest point.
• Use passive voice, humor, emotive language, rhetorical questions to provide a
specific effect.

Split up of marks

Total marks: 10

Format (title + writer’s name): 1mark

Content (logical organization, relevance): 4 marks

Expression (Accuracy – 2 ½ + fluency – 2 ½ ): 5 marks

SAMPLE ARTICLE

You are Vikranth / Vinitha, a freelance writer much concerned about the discriminatory
treatment of women in the Indian society. Write an article in 150 – 200 words throwing
light on this problem and giving suggestions for putting an end to it.
(10 marks)

62
DISCRIMINATION OF WOMEN

It is a pity that in a country where women are said to be worshipped, there is a


widespread discrimination of them even before they are born. Certain brutal practices
like the female feticide throw light on our attitude. The girl child is considered to be a
liability and doesn’t enjoy the privileges of a boy. She is denied the advantage of proper
education. The dowry system haunts parents and the harassment she is subjected to at
the in-laws often force her to commit suicide.

Even in enlightened homes, the woman has to live her life under surveillance, if not in
strict ‘purdah’. Working women are physically and verbally abused, denied
opportunities of growth and subjected to discrimination.

Education can empower women to fight for their rights. It is also the responsibility of
the educated society to work for the cause of women. Let she not be worshipped but let
her live a wholesome life.

Vikranth

Questions for practice


1. Write an article in about 200 words for your school magazine on ‘Films and their
influence on school-going children’. You are Manpreeth / Manjula, a student of
class XII A.
2. You are Rohit / Kamal. You attended a seminar on Yoga, a way of life. Write an
article in about 150 – 200 words on the contribution of yoga in leading a healthy
and peaceful life.
3. In today’s world, letter-writing has lost its charm. People and the youth in
particular rely more on mobiles and computers to communicate. Write an article
on the present trend in about 150 to 200 words. You are Kiran / Karthik.
4. Recently you visited your ancestral home in a village in Gujarat. You were elated
and strongly felt the necessity to hold on to the roots of our modern life, which
lie in our villages. Write an article for your school magazine. You are Chetna /
Chirag, Cultural Secretary of your school.
5. With rising number of people in almost all the big cities of the country, the rate
of crime has also increased proportionately. The police needs to be trained in
new methodology of combating the crime besides changing the mindset. Write
an article in 150 – 200 words on the role of police in maintaining law and order
in the metropolitan cities. You are Ravi/ Ravina.

63
6. The invention of mobile phone h it can be a blessing but if misused it can prove
to be a curse. Write an article in about 150 – 200 words on this invention. You
are Karthik / Karishma.

7. Presently the prices of essential commodities are skyrocketing causing much


hardship to the common man. Write an article in 150 -200 words expressing your
views and suggesting measures to curb this problem.

8. It is really a Do or Die struggle when we talk about environmental concerns. As


an active member of the Green Club of your school write an article in about 150
– 200 words on your concerns and the need for change in our attitude and
practices.

9. Computer and video games have become popular with children today. Outdoor
games seem to have no place in their life anymore. You are Sakthi/ Sathish. You
enjoyed playing hide-n-seek with your cousins in a small town. You found it so
refreshing that you decide to write an article on the joys of playing outdoor
games for the school magazine. Write it in about 200 words.

10. Children between 13 and 19 years of age are neither too young nor too mature.
Today these children feel that the pressures of the competitive world they live in
have made their lives less exciting.
11. Write an article in 150-200 words expressing your views. You are Manish /
Manisha.

64
LITERATURE SECTION
FLAMINGO (POETRY)

MY MOTHER AT SIXTY SIX by KAMALA DAS


Summary:
• The poets driving from her parents home to Cochin by car, her mother by her
side—sleeping –open mouthed very pale, colorless and frail-like a dead body
indicating that her end was near.

• The poet looks at her and feels intense pain and agony to realize that soon death
will cast her mother from her.

• Tries to divert her mind, looks outside at the young trees and happy children
bursting out of their homes in a playful mood (a contrasting image)

• After the security check at the airport looked again at her mother’s face—pale
and cold.

• “Familiar ache-My childhood fear” –the poet has always had a very intimate and
close relationship with her mother and she has always felt the fear of being
separated from her mother hence it is familiar.

• The poet reassures her mother that they will meet again

COMPREHENSION QUESTIONS

1. “Driving from my parent’s home to Cochin last Friday

Morning, I saw my mother, beside me, doze,


open mouthed, her face ashen like that
Of a corpse and realized with pain
That she thought away.”

a) Where was the poet coming from? Where was she going?
The poet had gone to her parents’ home to visit them. She was now going to
Cochin airport.

b) How does the poet describe her mother?


The poet describes her mother as old, pale, cold and senile. As she dozed off beside
her, the mother looked almost like a corpse, for her face was colorless and seemed
to have lost the fervor of life.

65
d) Who does ‘she’ refer to in the last line? What thoughts had she driven away?
‘She’ here refers to the poet. The thought of her mother’s approaching death which
she wanted to put it away.
2. “ and looked but soon
put that thought away, and looked out at young
trees sprinting, the merry children spilling
out of their homes”
a) What was the poet ‘looking’ at? What did she notice?
The poet was looking at her mother. She noticed the mother’s ashen and almost
lifeless face distraught with pain.
b) What thought did she try to drive away?
She tried to drive away the thought of her mother’s approaching death.
c) Why did the poet start ’looking out’? What does her gesture suggest?
The poet started looking out of the window because she wanted to drive away the
pain and agony she experienced on seeing her aged mother. She wanted to drive
away her helplessness in the wake of her mother’s ageing and approaching death.
d) What did the poet see from the window of the car?
The poet saw young trees running past her car and merry children sprinting out of
their homes to play.
e) What did the images of ‘young trees’ and ‘merry children’ symbolize?
Trees and children symbolize the spring of life, its strength, vigour and happiness
which contrasts with the lifelessness and helplessness that sets in with age.
3. “ but after the airport’s
security check, standing a few yards
away, I looked again at her, wan, pale
as a late winter’s moon”
a) Where was the poet standing?
The poet was at the Cochin airport waiting to board the plane after the security
check.
b) Who does ‘her’ here to? How did she look like?
’Her’ here refers to the poet’s mother. She was an aged lady and hence looked pale,
cold like a corpse and colourless.
c) Why does the narrator ‘look at her again’?
The narrator looked at her mother once again for the last time before she left to
reassure herself about the well being of her mother. She had tried to drive away the
pain she had felt on seeing her weak and aged mother. One last time she looked at
her to wish her goodbye.
d) Explain: ’wan, pale as a late winter’s moon’.
In this simile, the poet compares the mother’s pale and withered face with the
winter’s moon. The moon seems to lose its brightness in the winter season as it is
veiled behind fog and mist. The mother’s face also seemed to have lost its radiance
which was now misted by age. Winter symbolizes death and the waning moon
symbolizes decay.

66
4. “ and felt that old
familiar ache, my childhood’s fear,
but all I said was, see you soon, Amma,
all I did was smile and smile and smile.”

a) What ‘familiar ache’ did the poet feel?


The ‘familiar ache’ refers to the poet’s fear of losing her mother and the realization
that she has not cared and cannot care for her ageing mother. It is an ache of
helplessness. It is also a fear of separation from the mother or the mother’s death.

b) What could have been the poet’s childhood fears?

I think the poet’s childhood fear was that she would lose her mother or be separated
from her and that death would consume her mother.

c) Did the poet share her thoughts with her mother?

The poet did not share her fears and agony with her mother. She only bid good bye to
her with the hope of seeing her soon.

d) Why do you think, the poet did not share her thoughts with her mother?
I think the poet did not share her thoughts with her mother because they were
caused by her fear of the unknown. Sharing them with the mother would have
worried the frail old woman to death.

e) Why did the poet only ‘smile’?


The poet only smiled to hide her guilt, anxiety and fear of the unknown. Also, she
wanted to bid a cheerful farewell to her mother before boarding the flight.

QUESTION AND ANSWERS

1. What is the kind of pain and ache that the poet feels?
When the poet looks at her mother’s face she found that it had become pale and
withered. She realized that her mother was at the edge of her life and her end was
near. The thought that her mother would be soon separated form her caused
unbearable pain and ache in the poet’s heart.

2 What does the poet do to shrug off the painful thought of her mother’s
approaching end?
To get rid of painful thought her mother‘s nearing the poet looked out to seethe
sprinting tree and the happy children, bursting out of their house.

3. Why does the poet draw the image of sprinting trees and merry children?

67
Sprinting trees and merry children bursting out from the doors suggest fresh life and
warm energy. The poet draws this image to strikes a scene of contrast with the pale,
dull and withered face of the mother. Here the curtain is falling and the life coming to
an end and there the curtain is rising and fresh life is beginning and bubbling with
energy and vitality.

4. Why have the trees been described as sprinting?


The poet was driving in a car along with her mother. Her movement created the
visionary, illusion of the trees outside appeared to be sprinting past.

5. Why has the mother been compared to the late winter’s moon?
The late winter moon lacks luster. The mothers face was pale and withered.
Moreover, the late winter moon suggests the end of season and mother too is
nearing the end of her life, therefore the poet compares her with the late winter’s
moon.
6. What is the ‘familiar ache’?
The fear of losing her mother has tortured the poet from her very childhood because
she had been intimately bound up with her. Therefore this ache is familiar to her; it is
known to her.

7. What do the parting words of the poet and her smile signify?
The parting words of the poet reflect the poet’s pain. But she puts on a smile on her
face to mask her pain and to give hope, happiness and reassurance to her mother.

AN ELEMENTARY SCHOOL CLASSROOM IN A SLUM -BY STEPHEN SPENDER


GIST OF THE POEM
• In this poem the poet focuses on the theme of social injustice and inequalities.
• He presents the pathetic and miserable picture of the elementary classroom in a
slum.
• These children have pale and lifeless faces.
• They are like rootless weeds which are uncared and unwanted with their
disorderly hair torn around their faces.
• They are depressed and oppressed with the burdens of life and keep their heads
down. They have stunted growth.
• They inherit the diseases of their father.
• Some of them do have dreams. A sweet young boy is sitting at the back of the
dim classroom. He is dreaming of a squirrel’s game in the trees and probably
other interesting things.
• The walls are dirty and creamy and on them are hung the donations given by
the rich and also Shakespeare’s portrait.
• A civilized dome found in the cities and Tyrolese valleys with beautiful flowers
are also put up.

68
• The map on the wall shows the children, the beautiful world outside; but for
these children of the slum it is meaningless.
• The children studying in these schools do not have the means to go and explore
the world. For them what they see through their classroom windows, the narrow
street and the lead sky is the world.
• Shakespeare is wicked for them as he has written only about the rich, beautiful
world tempting them to steal.
• The map is of no interest to them because it does not reflect the world they live
in-cramped and dark lanes.
• Their lives start in darkness and ends in utter darkness.
• They are undernourished and their poverty has distorted their vision as they
spend their whole time in foggy slums.
• The poet feels that the map which shows beautiful and exotic places should be
replaced with slums as it is not the world they live in.
• Unless the governor inspector and visitor play a vital role in bringing about a
change, their lives will remain in dark.
• The slum children will be able to peep through the window only when the gap
between the two worlds is bridged.
• They should break the barriers till they come out of the dirty surroundings and
their world should be extended into the green fields, golden sands and bright
world.
• They should have the freedom of expression and their outlook be broadened.
• For, only the educated and learned people can create history whose language
has strength and power.

SOLVED QUESTIONS

1. “Unless, governor, inspector, visitor,


This map becomes their window and these windows
That shut upon their lives like catacombs.”

(a) Why does the poet invoke ‘governor, ‘inspector’ and ‘visitor’?
The poet invokes the ‘governor, ‘inspector’ and ‘visitor’ because they are the
powerful people who can bring about a drastic change in the miserable lives of the
slum children. They can remove the social injustice and class inequalities.
(b) What does ‘this map’ refer to? How can it become ‘their window’?
This map refers to the beautiful world of the rich. Their window refers to holes and
the stinking slums of the unfortunate children of the slum. This can become their
window only when the difference between the two worlds is abridged.
(c) What have ‘these windows’ done to their lives?
These windows have cramped their lives, stunted their physical and mental growth
shutting them inside filthy and dingy holes.
(d) What do you understand by catacombs?

69
Catacombs are long underground graves. Here they stand for the dirty slums which
block their progress.
(e) Which literary device has been used? Explain.
Simile has been used to describe the oppressive effect of the surroundings on their
pathetic lives. The simile is: ‘these windows that shut upon their lives like
catacombs.’

Answer the following in 30-40 words.

1. What is the theme of the poem?


This poem deals with the theme of social injustice and class inequalities. The poet
presents it by talking of two different and incompatible worlds. The world of the rich
and the civilized has nothing to do with the world of the narrow lanes and cramped
holes. This gap can be bridged by the authorities.
2. So blot their maps with slums as big as doom’. What does the poet want to convey?
The poet is angry at the social equalities in the world. There are two worlds – the dirty
slums and the prosperous and the beautiful world of the rich. The poet wants the map
of the world should also have blots of slums as big as the ‘doom’. In reality he wants
the gap to be reduced.
3. ‘History is theirs whose language is the sun’. Explain.
This statement means that those who have the power and confidence in speech to
influence others create history. One can make a mark only if one can outshine
others. Education only can give them power and strength like the sun which will
bring about a change in the lives of the people.

QUESTIONS FOR PRACTICE


B. Read the stanza and answer the questions that follows:
”Surely, Shakespeare is wicked, the map a bad example,
With ships and sun and love tempting them to steal-
For lives that turn in their cramped holes
From fog to endless nights.”
i) Name the poem and the poet
ii) Why has Shakespeare been described as wicked?
iii) Why is the map a bad example?
iv) What tempts them to steal?
v) How do the children continue to live?
vi) Explain: ‘From fog to endless night.’
C. Read the stanza and answer the questions that follows:
“The stunted, unlucky heir
Of twisted bones, reciting a father gnarled disease
His lesson from his desk. At the back of the dim class
One unnoted , sweet and young. His eyes live in a dream
Of squirrels game, in tree room, other than this.”

70
a) Who is being referred to in the first two lines?
b) Explain ‘father’s gnarled disease’.
c) Who sit at the back of the class? How is he different from others?
d) Explain his eyes live in a dream?
e) What is the comparison drawn with squirrels game?

SHORT ANSWER QUESTIONS

1. What is that these children inherit from their parents? What does it signify?
2. How has the poet described the colour of the wall and why?
3. The poet presents two different worlds. What are they?
4. What picture of the slum children does the poet draw?
5. Where does the poet see hope and relief?
6. What does the poet mean by saying, ‘Let their tongue run naked into books’?
7. Describe some poetic devices used in the poem. Give examples.
8. Explain ‘Open handed map, awarding the world its world’.
------------------------------------------------------------------

KEEPING QUIET - BY PABLO NERUDA

GIST OF THE POEM


• The poet talks about the need of silence and quiet introspection and the
importance of quietude and calmness. He also talks about creating a feeling of
mutual understanding among human beings.
• The poet asks us to keep still and count up to twelve. He also asks us to sit still.
For a moment we should not speak any language. We should not move our arms
so much.
• It will be a moment of complete silence without rush or worry. This would be an
exotic moment.
• Then a sudden strangeness will prevail which we will all enjoy. It will be bliss.
• The fisherman would not harm the whales on the cold sea. Even the man
gathering salt would stop working and look at his hurt hands and reflect at the
pain and harm his strenuous task has caused him.
• All kinds of wars must be stopped at once. The green wars against the
environment, wars with poisonous gases, firearms, must be stopped at once.
• People who are all the time preparing for wars leaving no survivors behind ought
to find time to wear clothes and walk around with their brothers strengthening
the message of peace and brotherhood.
• At the same time the poet cautions not to confuse stillness with total inactivity.
Life is an ongoing process and should not be associated with death. It is to be
lived with positive attitude.
• He does not want us to ruminate over death.

71
• But he feels that if for once we do not focus ourselves single-mindedly to keep
our lives moving but do some introspection or spend some time in silence doing
nothing, we can understand ourselves better and escape from the threatening
calls of death.
• The earth can teach us a lesson how everything comes to a dead end and comes
to life again.
• In the same manner a quiet introspection can bring all evil thoughts to an end
and bring in a new life of peace and tranquility.
• Now the poet will count up to twelve and they should keep quiet and he will go.

SOLVED QUESTIONS
1. Read the stanza and answer the questions that follows:
“Fishermen in the cold sea
Would not harm whales
And the m an gathering salt
Would look at his hurt hands.”

a) What is ‘fisherman’ symbolic of?


The fisherman symbolizes man’s indiscriminate exploitation of nature for his vested
interests.
b) What will happen when fishermen do not harm whales?
The whales will be no longer on the verge of extinction.
c) What has happened to the man gathering salt? What must he do?
The man gathering salt has injured his hands. He must take care of his hurt hands.
d) What would happen in this moment of silence?
He will become conscious of the harm causing to others and to themselves.
e) What image does the poet create in the last line?
He creates the image of incessant suffering. In his effort to add comforts to his life he
has paid no heed to the pain that caused him.
2. Read the stanza and answer the questions that follows:
” Perhaps the earth can teach us
As when everything seems dead
And later prove to be alive
Now I’ll count up to twelve
And you keep quiet and I will go.”
a) Who can teach us?
The earth can teach us.
b) What does earth teach us?
The Earth can teach us how new life emerges from the ashes of the dead remains.
Likewise quiet introspection will enable us to live a life of peace and tranquility.
c) Why does the speaker count up to twelve?
It is a part of an exercise in meditation. All distractions and digressions are washed
away and man is in a moment of bliss.
d) Explain-‘you keep quiet and I will go’?
72
The poet wants all to keep quiet and experience the moment of peace. He will go
and pass on the message to another group of people.

Short Answer Questions


1. Why does Pablo Naruda urge us to keep still?
Stillness is necessary for reflection and quiet introspection. We can hear the voice of
our conscience and thus withdraw ourselves from undesirable actions.
2. ’Under the apparent stillness there is life’. Justify.
The poet does not want to equate stillness with total inactivity. Under the apparent
stillness there is life. We can learn it from the earth when everything seems dead, the
earth still remains alive. The life on earth goes on under the apparent stillness.
3. Why do men become sad? How can this sadness be overcome?
Men fail to understand themselves. They are always threatening themselves with
death. When they do not understand themselves they become sad. A long silence
might interrupt this sadness and make them good.

QUESTIONS FOR PRACTICE


C. “What I want should not be confused
With total inactivity
Life is what it is about
I want no truck with death.”
1. Name the poem and the poet?
2. What is the desire of the poet?
3. What does ‘total inactivity’ imply?
4. Why does the poet say that he does not want his wish to be confused with total
inactivity?
5. Explain-‘I want no truck with death’.

SHORT ANSWER QUESTIONS FOR PRACTICE


1. Why shouldn’t we speak any language and move our arms so much?
2. How does the poet distinguish ‘stillness’ from ‘total inactivity’? Explain.
3. “I want no truck with death.” Explain.
4. What are the various wars mentioned? What is the result of these wars?
5. What would be the result of quietude?

A THING OF BEAUTY By JOHN KEATS


GIST OF THE LESSON
• The Poet, John Keats says that beautiful things will never become ‘nothing’ as
they will continue to hold us in their spell and sooth our soul.

• Every beautiful thing is like a band that ties us to this earth as it makes us want
to live and enjoy these things of beauty.

73
• And these things of beauty, according to the poet, are the things that give hope
to human beings and make them want to live, in spite of all the sorrow, ill-health
and unpleasant experiences that we face on earth.

• Some of the beautiful things on this earth that have such an effect on us are the
sun, the moon, trees, streams, flowers, forests, beautiful monuments that we
have erected for the dead, all the lovely tales that we have heard or read.

• Finally he compares all these beautiful things to immortal drink or nectar given
to us by gods or gifts of God. Thus he states his firm belief in the Divine.

Solved Questions
1. “Such the sun, the moon,
Trees old and young, sprouting a shady boon
For simple sheep; and such are daffodils
With the green world they live in; and clear rills
That for themselves a cooling covert make
‘gainst the hot season.”

a. What proves a blessing for the sheep?

Trees, young and old provide a green covering which proves to be a blessing for the
sheep.
(b) Where do daffodils bloom?

Daffodils bloom among the green surroundings.


(c) What prove to be pleasant shelters in summer?

The small streams with clear water prove to be cooling, pleasant shelters in summer.
(d) Name the objects of beauty referred to here.

(e) The sun, the moon, trees, daffodils and streams of clear water are the objects of
beauty mentioned here what according to Keats are the things that cause suffering
and pain?

Answer: The poet says that a scarcity of good-natured people or in other words the
wicked people outnumber the good people. And the source of all our sorrows is either
ill-health or another human being.

(f) What makes human beings love life in spite of all the suffering?

Answer: The poet says that the beautiful things on earth lifts the pall off our spirits and
make life worth living. Each beautiful thing is like a link that forms a chain or wreath that
binds us to this earth.

74
(g) Why does the poet say ‘mighty dead’?

Answer: Monuments are erected in memory of people who were mighty or great when
they lived. Physically mighty as in mighty warriors or mentally might as in great poets,
writers or philosophers. Their tombs provide inspiration for the living through their
beauty just as their works continue to do.

Questions for practice:

1. “A thing of beauty is a joy forever

Its loveliness increases, it will never

Pass into nothingness; but will keep


a bower quiet for us, and a sleep
full of sweet dreams, and health, and quiet breathing
Therefore, on every morrow, are we wreathing
A flowery band to bind us to the earth.”

a. What is the special virtue of a beautiful thing?


b. How does it bless us?
c. Explain the expression “A bower quiet for us”.
d. What do we do everyday?

2. “the mid forest brake,

rich with a sprinkling of fair musk-rose blooms;

and such too is the grandeur of the dooms

who have imagined for the mighty dead;

All lovely tales that we have heard or read;

An endless fountain of immortal drink,

Pouring unto us from the heaven’s brink.”

a. What do you mean by ‘brake’? Where does it grow? What makes it all the
more beautiful?

b. What do you mean by ‘the grandeur of the dooms’?

c. “All lovely tales that we have heard or read” Explain

75
d. What is the source of the beauty of nature? What is its effect on us?

3. “Some shape of beauty moves away the pall

from our dark spirits. Such the sun, the moon,


trees old, and young, sprouting a shady boon
for simple sheep; and such are daffodils
with green world they live in; and clear rills
that for themselves a cooling covert make
‘gainst the hot season; the mid forest brake
Rich with a sprinkling of fair musk-rose blooms”

a. What removes the pall from our dark spirits?

b. What sprouts a shady boon for sheep and how?

c. How do ‘daffodils’ and rills enrich the environment?

d. What makes the mid-forest brake rich?

Short answer Questions

1. How do we wreathe a flowery band?

2. Why do human beings suffer or what depresses the human soul?

3. What does Keats mean by the ‘grandeur of doom’?

4. Mention 4 things of beauty listed in the poem?

5. How do we bind ourselves to the earth every morning?

6. Why and how is ‘grandeur’ associated with the mighty dead?

7. What is the source of the ‘endless fountain’ and what is its effect?

8. What is the message of the poem?

76
A ROADSIDE STAND By ROBERT FROST
Gist of the lesson

• Roadside stand by Robert Frost is concerned with human tragedies and fears. He
focuses on the rural-urban divide and presents the lives of the poor deprived
people with pitiless clarity and with the deepest sympathy and humanity.
• The dwellers of the little house by the roadside put up a little shed in front of their
house as they wanted to earn a little extra-money but not for making their living.
• The rural people wish to feel some real money that supports the commerce of the
cities.
• The shed was painted artlessly and stood out which made the passers-by irritated
at having the beauty of the landscape spoilt.
• The traffic flowed ceaselessly or if ever they stopped, they felt out of sorts on
seeing ‘N’ and ‘S’ written as their mirror images.
• The Stand sold wild berries and golden squash for sale.
• The owners of the shed felt cross when nobody wanted to buy anything
• The poet feels that the implications of the unstated facts are more pathetic.
• The government announces schemes to allure such people and house them in
villages that are near to the theatre and the store, to reap benefits for their own
selfish needs.
• And the social workers and politicians enforce their decisions by alluring them and
destroy their ability to earn their living, thus stripping them of all dignity and their
voice
• The poet is overcome with pain at the thought of the people waiting in vain for the
vehicles. If ever any vehicle stops, it will be to ask for directions, take a u-turn, and
enquire about the price or to ask if they sell gas.
• But the country people have never felt the extra-money in their hands and they
complain about it.
• The poet wonders if it wouldn’t be better if they were put out of their agony at
one stroke but then wonders if someone offers the same solution to his pain, how
he would feel. Killing is not the solution to the problem.

Solved Questions:

1. “It is in the news that all these pitiful kin


Are to be bought out and mercifully gathered in
To live in villages, next to the theatre and the stone,
Where greedy good-doers, beneficent beasts of prey
Swarm over their lives enforcing benefits
That are calculated to soothe them out of their wits,
And by teaching them to sleep all day,
Destroy their sleeping at night the ancient way”.

77
a) What is in the news?
It is in the news that the poor are to be relocated to better surroundings near
the theatre and the shops.

b) Which word in the verse means the same as ‘generous’ in the above lines?
‘Beneficent ‘

c) Who is going to exploit the rural people and how?


The politicians and the Government exploit the poor by offering them benefits
that are supposed to solve their problems but in reality only add or pose
problems of a different nature thereby making them feel cheated.

d) How will the greedy good-doers soothe the rural poor out of their wits?
By offering them free benefits like housing and other facilities, they rob the poor
of their voice to protest and lull them into a feeling of false security.

e) Who is referred to as beasts of prey and why?


The politicians in power and in opposition and they make no difference in the
conditions of the rural poor.

Questions for Practice

1. “The little old house was out with a little new shed
in front at the edge of the road where the traffic sped
A roadside stand that too pathetically pled,
It would not be fair to say for a dole of bread,
But for some of the money, the cash, whose flow supports
the flower of cities from sinking and withering faint,
The polished traffic passed with a mind ahead,
Or if ever aside a moment, then out of sorts”.

a. Where was the shed made?


b. Why did they put up the shed?
c. Why is the cash import for the cities?
d. Why didn’t the polished traffic stop at the roadside stand?
e. What was being sold at the roadside stand?

2. “Of all the thousand selfish cars that pass,


just one to inquire what a farmer’s prices are.
And one did stop, but only to plow up grass
in using the yard to back and turn around;
And another to ask the way to where it was bound;
And another to ask could they sell it a gallon of gas
They couldn’t (this crossly); they had none, didn’t it see?”

78
a. Explain ‘Selfish Cars’.
b. What did the car-owners generally do not bother about?
c. Why do people generally stop there?
d. What made the rural people cross?

Short answer questions

1. Why did the country folk put up the roadside stand?


They put out a stand to earn some extra money
2. Why are the good-doers said to be greedy?
The good-doers work not for the real welfare of the poor but to further their
own greed.
3. List three reasons why cars do stop at the roadside Stand?
The car-owners stop at the roadside stand to inquire about the prices, to turn
the car around, ask for directions or ask for gas.

4. What the attitude of the city folk who passed by the ‘Roadside Stand’?
The city folk were indifferent and callous towards the plight of the rural folk.

Questions for Practice


1. How did the country folk react when they knew why the passers-by had
stopped?
2. What did the people at the roadside stand sell?
3. Why does the poet call their longing as ‘childish’ and why?
4. Why are the country folk always low in spirits?
5. What kind of life do the rural folk lead?
6. How and why do the good-doers soothe the poor out of their wits?
7. What would give great relief to the poet?
8. Is the poet serious about the suggestion he offers to put the people out of their
suffering?
9. How did the people feel when they knew they have been exploited? How and
why?
10. What is the role of the Government in the lives of the rural people?

79
AUNT JENNIFER’S TIGERS By ADRIENNE RICH
SUMMARY OF THE POEM:
• The poet is a feminist and she addresses the difficulties of a married woman.

• She spends good amount of time in embroidering panel of tigers prancing across
the screen.

• The tigers are fearless creatures pacing elegantly and majestically. They
symbolize the spirit of freedom. Aunt is a victim of male chauvinism (male
domination).

• Aunt Jennifer is so oppressed and terrified that she finds it hard to pull the
needle.

• The “weight of Uncle’s wedding band “expresses how victimized and oppressed
she is. It implies that aunt Jennifer has to work hard to meet his expectation.

• She spends her life in fear but she embroiders on the panel the fearless tigers to
express her secret longing for a life of freedom and confidence.

• Even her death does not end the problem and torture which a married woman
experiences..

SOLVED QUESTIONS

1.”Aunt Jennifer’s tigers prance across a screen,


Bright topaz denizens of a world of green.
They do not fear the men beneath the tree:
They pace in sleek chivalric certainty.”
a) What does the expression ‘Aunt Jennifer’s tigers imply?
Aunt Jennifer was embroidering a panel of prancing tigers. The poet refers to the
tigers as Aunt Jennifer’s tigers because they are her creation, her work of art.
b) What does prancing tigers symbolize?
Prancing tigers are a symbol of the spirit of freedom within Aunt Jennifer which
remains subdued. They also symbolize her fear of her male counterpart.
c) Why are they referred to as ‘denizens of a world of green’?
The tigers are the dwellers of the green forest so they are referred to as
denizens.
d) What qualities of the ‘tigers’ are highlighted here?
Fearlessness and ferocity of the tigers are highlighted here. Aunt Jennifer’s
nervousness and timidity are in sharp contrast to wild ferocity of the tigers who are
not afraid of hunting men. Unlike Aunt Jennifer, the tigers fear nothing.
e) Explain; “They pace in sleek chivalric certainty”.

80
The movement of the tigers are sleek, stealthy, sure, majestic and elegant. They are sure
of their purpose. Gallant and confident, they move ahead fearlessly undeterred by any
obstacles or hindrances.

2.”Aunt Jennifer’s fingers fluttering through her wool


Find even the ivory needle hard to pull.
The massive weight of Uncle’s wedding band
Sits heavily upon Aunt Jennifer’s hand.”

a) Why do Aunt Jennifer’s fingers flutter through her wool?


Aunt Jennifer lives in constant fear of her husband. She feels so nervous and terrified
that her hands shake and flutter when she sits down to knit.
b) Why does she find it hard to pull the ivory needle?
Due to constant fear that she confronts has become a nervous wreck. She finds it
difficult to pull the ivory needle through the tapestry more because of mental
suppression than because of physical weakness.
c) Explain: ‘massive weight of Uncle’s wedding band’.
The expression is symbolic of male authority and power. Matrimony binds the woman
physically as well as mentally. Likewise Aunt Jennifer is trapped in gender oppression
and feels herself burdened by the authority of her husband.
d) How is Aunt Jennifer affected by the ‘weight of matrimony’?
Aunt Jennifer cannot do things freely, she tries to come up to the expectation of her
husband, she seems to have lost her identity. The freedom that she dreams of through
her art is itself symbolic of her oppressed self

3.”When Aunt is dead, her terrified hands will lie


Still ringed with ordeals she was mastered by.
The tigers in the panel that she made
Will go on prancing, proud and unafraid.”

a) What is Aunt Jennifer’s death symbolic of?


Aunt Jennifer’s death is symbolic of her complete submission to her suppression.

b) Explain: “terrified hands”.


Aunt Jennifer is terrified by her dominating husband and hence her hands are shivering.

c) What does ‘ringed with ordeals’ imply?


Aunt Jennifer has been so victimized in her life that even after death she remains
trapped in the struggles of the spirit. Though we do not know what terrors Aunt Jennifer
had to live with relatives did, we find her a victim of gender injustice and oppression.

d) Is the society in any way affected by Aunt Jennifer’s death?

81
Since the society is male dominated, it shows no concern for Aunt’s suffering, even her
death. The loss of her freedom is her individual loss. The society is not affected by it and
the state of women still remains the same.

e) Explain: “the tigers in the panel….will go on prancing, proud and unafraid”.


The expression is symbolic of the dispassionate and unconcerned attitude of the male
towards the desire for freedom among women. Even after her death, the social milieu
remains unaffected, arrogant and ferocious.

SHORT ANSWER QUESTION S

1. How do the tigers made by Aunt Jennifer look like?


The tigers, made by Aunt Jennifer on the screen, are jumping and playing about without
any fear of the men beneath the tree. They walk in elegance and style displaying the
spirit of courage, fearlessness, strength and confidence.

2. What do the tigers made by the Aunt symbolize?


The tigers made by Aunt Jennifer symbolize the spirit of courage, strength and
fearlessness Aunt Jennifer, a victim of male oppression, expresses her crushed feelings
in the form of art. So, the tigers are symbolic of the fear of male domination with which
Aunt Jennifer suffers.
3. Why do you think Aunt Jennifer’s hands are fluttering through her wool? Why is she
finding the needle so hard to pull?
Aunt Jennifer is victimized by the overbearing and dominant nature of her husband. Her
life has become a torture due to her suppression by her atrocious husband. The fear of
her authoritative husband has gone so deep into her being that she seems to have lost
all strength and energy. Thus her hands shake and flutter so much that she is not even
able to pull the needle through the tapestry.

4. What do you understand by “massive weight of uncle’s wedding band”?


Generally ‘wedding band’ is a symbol of joy and happiness. But in case of Aunt Jennifer,
it has become a symbol of torture and oppression. Her relationship with her
authoritative husband has become a painful burden to carry. Her ‘wedding band’ has
brought her a world of pain, misery and torture. She has lost her freedom and entered a
world of humiliation and oppression.

5. Explain ’her terrified hands will lie, still ringed with the ordeals she was mastered by’.
These lines convey Aunt’s complete submission to the oppressive authority of her
husband. The fear of her husband has gone so deep into her being that even death
cannot liberate her from the chains of her mental suppression. Memories of her
husband’s tortures and atrocities which bent her into a humiliating slavery will continue
to haunt her even after her death.

6. Explain ‘The tigers in the panel------------proud and unafraid.’

82
Here the tigers symbolize the unquestioned authority of man enjoyed by him over his
woman counterpart. The lines suggest the dispassionate and unconcerned attitude of
the male towards the desire for freedom among women. Here, Aunt Jennifer tries to
find an escape in her art but ends up portraying an image of her own suppression. While
woman can never free herself from the oppressive authority of her male counterpart,
the male, on the other hand will go on enjoying his authoritative arrogance and ferocity
without any fear of regrets.

FLAMINGO (PROSE)

THE LAST LESSON by Alphonse Daudet



GIST OF THE LESSON

• Franz is afraid of going to school as he has not learnt participles.


• He wants to enjoy beauty of nature. The bright sunshine, the birds chirruping in
the woods, Prussian soldiers drilling but resisted.
• Bulletin board: all bad news, lost battles, the drafts and orders of the
commanding officers: wondered what it could be now

• The changes he noticed in the school.


- Instead of noisy classrooms everything was as quiet as Sunday morning
- The teacher does not scold him and told him very kindly to go to his seat
- The teacher dressed in his Sunday best.
- Villagers occupying the last benches- To pay tribute to M. Hamel for his 40
yrs of sincere service and also to express their solidarity with France.

• M. Hamel making the announcement that that would be the last French lesson;
realizes that, that was what was put up on the bulletin board.
• Franz realizes that he does not know his own mother tongue
• Regretted why he had not taken his lessons seriously.
• Also realizes the reason why teacher was dressed in his Sunday best and villagers
sitting at the back.
• M. Hamel realizes that all three, he himself, the children and the parents are to
be blamed for losing respect and regard for the mother tongue.
• Always keep the mother tongue close to your heart as it is the key to the prison
of slavery.
• Atmosphere in class: teacher teaching sincerely and patiently, students and
others studying with utmost sincerity.
• Franz wonders sarcastically if Prussians could force pigeons to coo in German.
• M. Hamel overcome with emotions could not speak and wrote on the black
board “Long Live France”.

83
SOLVED QUESTIONS:

SHORT ANSWER QUESTIONS:


1. What was the narrator’s greatest fear as he moved towards the school?
Franz had started late for school and thus was afraid of being scolded . His fear gripped
him further for he was also unprepared. He had not learnt his lesson regarding the rules
of participles and thus dreaded the teacher’s anger.
2. What was more tempting to Franz rather than going to school?
The weather was pleasant, warm and bright. The chirruping birds were inviting him, the
soldiers drilling in the field were also outdoors and Franz was not prepared with
participles.
3. What was the news which was put up on the bulletin board?
For the last two years all bad news – the lost battles, the orders of the commanding
officer was displayed on the notice board. That day, the news that only German would
be taught in school of Alsace and Lorraine was displayed on the notice-board which
made the crowd gather there to read the news.
4. What was so unusual about the school on that day?
Usually there would be a great bustle of opening and closing of the desk, lesson
repeated loudly and the teacher’s ruler rapping on the table but that day was very calm
and quiet like Sunday morning. The back benches which were usually empty were
occupied by the village people and M. Hamel wore his special dress and was pacing up
and down with a ruler under his arm.
5. Why were the villagers seated on the back benches?
All the village elders were seated on the back benches as a tribute to the teacher who
had put in 40 years of sincere service. It was also their way of expressing regret for not
learning their mother tongue when they had the chance. They were also expressing
their patriotism and solidarity with France
6. Franz didn’t learn French whom did M. Hamel blame?
M. Hamel didn’t blame Franz for not learning but his parents who were not anxious to
have him learn. Instead they wanted him to work on a farm or at the mill to earn
money.
Even M. Hamel was also to be blamed for sending him to water the flowers instead of
learning and when he wanted to go fishing he declared holiday.
7. What did M. Hamel say about French language?
He said that it is the most beautiful language in the world- the clearest, the most logical.
He requested them to guard it so that they can be united and fight back for their
freedom.
8. What happened when the church clock struck 12?
The moment the church clock struck 12 the Prussian army came to take over and M.
Hamel stood up, wanted to tell something but his voice was chocked. He gathered his
strength and wrote on the black board as large as he could – ‘Vive La France’ and
dismissed the school.

84
LONG ANSWER QUESTIONS
1. Justify the title of the story “The Last Lesson”.
Value Points
People always feel there is plenty of time to learn—so also in Alsace—now no time—
parents not keen—preferred children , work in farms, mill—Franz looked opportunity to
escape school—never serious—receive orders from Berlin—people realize importance
of their language—attend the last lesson by M. Hamel.

QUESTIONS FOR PRACTICE


SHORT ANSWER QUESTIONS

1. Why did Franz not want to go to school?


2. Why didn’t M. Hamel punish Franz even though he was late?
3. Mention the three changes that Franz noticed in the school?
4. What announcement did M. Hamel make and what was its impact?
5. What do you think was written on the bulletin board?
6. Why did M. Hamel say about knowing one’s language is a key to prison?
7. Whom did Mr. Hamel blame for not learning the French?
8. What changes have taken place in the school in the last forty years?
9. What did he mean by “Viva La France”?

LONG ANSWER QUESTIONS

1. Write a note on the character of M. Hamel as a teacher?


2. Do you think the story touches upon the brutalities of war? Comment.
3. What thunderbolt did the narrator receive on reaching the school? How was he
affected by it?
4. At the end of the last lesson M. Hamel wrote, ‘Viva La France’ on the board in bold
letters. Why do you think he wrote that and how did he expect the people of Alsace –
Lorraine to keep their identity intact?

LOST SPRING
STORIES OF STOLEN CHILDHOOD By Anees Jung
GIST OF THE LESSON
• The author examines and analyses the impoverished conditions and traditions that
condemn children to a life of exploitation these children are denied an education
and forced into hardships early in their lives.

• The writer encounters Saheb - a rag picker whose parents have left behind the life
of poverty in Dhaka to earn a living in Delhi.

85
• His family like many other families of rag pickers lives in Seemapuri. They do not
have other identification other than a ration card.

• The children do not go to school and they are excited at the prospect of finding a
coin or even a ten rupee note for rummaging in the garbage.

• It is the only way of earning the life they live in impoverished conditions but are
resigned to their fate.

• The writer is pained to see Saheb, a rag picker whose name means the ruler of
earth, lose the spark of childhood and roams barefooted with his friends.

• From morning to noon the author encounters him in a tea stall and is paid Rs. 800
He sadly realizes that he is no longer his own master and this loss of identity
weighs heavily on his tender shoulders.

• The author then tells about another victim, Mukesh who wants to be a motor
mechanic.

• Hailing from Firozabad, the centre of India’s bangle making and glass blowing
industry, he has always worked in the glass making industry.

• His family like the others there do not know that it is illegal for children to work in
such close proximity to furnaces, in such high temperatures.

• They are exposed to various health hazards like losing their eyesight as they work
in abysmal conditions, in dark and dingy cells.

• Mukesh’s father is blind as were his father and grandfather before him.

• They lead a hand to mouth existence as they are caught in the vicious web of the
money lenders, middlemen, police and the traditions

• So burdened are the bangle makers of Firozabad that they have lost their ability to
dream unlike Mukesh who dreams of driving a car.

SOLVED QUESTIONS
SHORT ANSWER QUESTIONS

1. What does Saheb do for living? Why?

86
Saheb is a rag picker. His family has left the life of poverty behind in Dhaka in to pursue
their dream of finding a better life. The children like him have no access to Education
and are forced into rag picking

2. “Saheb is no longer his own master”, says the writer. What does she mean?

The writer means that having accepted the job with the tea-stall, Saheb has lost the
independence that he enjoyed as a rag picker, even though he was poor. Although he
will now be able to supplement the family income, it will be at the cost of his freedom,
which is difficult, binding and unfair for someone so young.

3. Why did people migrate from the village in Dhaka to Delhi ?

Better education, job opportunities and living conditions.

4. What trade does the family of Mukesh follow? Why does the writer feel that it
will be difficult for Mukesh to break away from this tradition?

Engaged in bangle making-difficult to break away from this trade. He belongs to the
caste of bangle makers His family is caught in the web of sohukars, the middlemen,
policemen, politicians and bureaucrats, from which there is no escape.

5. What does garbage symbolize for the adults and children?

Adults –means of earning a livelihood.

Children –wrapped in wonder, magical

LONG ANSWER QUESTION

1. ‘Lost Spring’, is a sad commentary on the political system of our country that
condemns thousands of people to a life of abject poverty. Comment.

Saheb, optimistic and enthusiastic—prospect of finding gold in garbage—likes going to


school but no opportunity—freedom and joy of childhood to burdens of job at tea-stall.

Mukesh, born at Firozabad (bangle maker)—works under inhuman condition—dark


room, hot furnaces—caught in web of poverty—vicious circle of sahukars, policemen,
politicians, bureaucrates and moneylenders—resigned to fate—unaware of child labour
act—stifiled initiation and hope—lose eyesight before becoming adults.

87
QUESTIONS FOR PRACTICE

SHORT ANSWER QUESTIONS


a. Describe Seemapuri.
b. Where has Saheb come from and why?
c. Describe the living conditions of bangle makers?

d. “Promises made to poor children are never kept. “Explain with examples
from the lesson.

e. Mention the hazards of working in the bangle industry.

f. Do you think Mukesh will realize his dream of becoming a car mechanic?

g. ‘His dreams loom like a mirage’. Whose dreams are being referred to and
why are they compared to a mirage?

h. ‘Together they have imposed the baggage on the child that he cannot put
down.’ Who do ‘they’ refer to? What is the ‘baggage’ and why can the child
not get rid of it?

i. What was Saheb’s full name? How is it ironical?

LONG ANSWER QUESTIONS

1. Like all children of his age, Saheb also had many hopes and dreams. Do you
think children like Saheb are able to fulfil their dreams?

2. Politicians exploit all people and situations to their own benefit. Comment,
keeping in views the situation of refugees in Seemapuri.

3. Saheb wants to blossom and bloom but is nipped in the bud. Elaborate.

4. ‘Saheb and Mukesh are brothers in penury and suffering.’ Discuss.

88
DEEP WATER BY WILLIAM DOUGLAS
THEME
In this essay William O. Douglas talks about his fear of water and how he finally
overcomes it by his courage, determination, handwork, strong will power, perseverance
and the desire to learn. If these are practiced we can definitely achieve success in all our
endeavors.
GIST OF THE LESSON-
• William O. Douglas had a desire to learn swimming since childhood.
• At the age of three or four, he was knocked down and buried by a wave at a
beach in California.
• He developed a great aversion to water.
• At the age of ten or eleven he decided to learn to swim with water wings at the
Y.M.C.A pool since it was safe at the shallow end.
• A misadventure:- while sitting alone and waiting for others to come at the
Y.M.C.A pool, a big boy came and threw Douglas into deep end of the pool.
• Douglas swallowed water and went straight down to the bottom of the pool.
• While going down he planned to make a big jump upwards but came up slowly.
• Stark terror seized him.
• Tried to shout but could not……
• As he went down the pool second time, he tried to jump upwards but it was a
waste of energy.
• Terror held him deeper and deeper.
• During the third trial he sucked water instead of air.
• Light was going out and there was no more panic.
• So he ceased all efforts and he became unconscious.
• He crossed to oblivion.
• When revived he found himself vomiting beside the pool.
• He was in grip of fear of water and it deprived him of the joys of canoeing,
boating swimming and fishing.
• Hired an instructor to learn swimming.
• The instructor taught him swimming piece by piece.
• He went to different lakes to swim and found tiny vestiges of fear still gripped
him.
• He challenged the fear and swam.
• Swimming up and down the Warm Lake he finally overcame his fear of water.
• He realized that in death there is peace and there is terror only in fear of death.
• Will to live is stronger than fear of death.

89
SOLVED QUESTIONS:

SHORT ANSWER QUESTIONS

1. Why was the YMCA pool considered safer when compared to the Yakima River?
Yakima River was very deep, treacherous and there were many cases of drowning but
the YMCA pool only two or three feet deep at the shallow end: and while it was nine
feet at the deep end. So YMCA pool was considered safer when compared to the Yakima
River.
2. When did his aversion to water begin?
His aversion to water began when he was 3 -4 years old when his father took him to
California beach. There the waves knocked him down swept over him.
3. What was the misadventure that happened one day?
William Douglas had just learnt swimming. One day, an eighteen year old big bruiser
picked him up and tossed him into the nine feet deep end of the YMCA pool. He hit the
water surface in a sitting position. He swallowed water and went at once to the bottom.
He nearly died in this misadventure.
4. What strategy did he remember as he went down the water?
To hit the bottom and spring/jump upwards, bob to the surface
–like a cork and come out.
5. What effect did the drowning in the YMCA pool have on the Douglas?
-weak and trembling
- haunting fear
- deprived of the joy of canoeing, boating and swimming.
6. What method did he adopt to overcome terror?
- Rigorous training (breathing moving of legs ,etc )
- went to lake Wentworth and swam for two miles.
LONG ANSWER QUESTIONS
1. How did the misadventure in YMCA pool affect Douglas? How did he overcome
it?
-was ten or eleven decided-learn-swim
-an older boy pushed –almost drowned
-haunting fear gripped him
-could not enjoy any water sports
-finally decided to hire an instructor
-seven months –instructor –made a swimmer
-released the instructor
-vestiges remained
-swam in Lake Wentworth
-challenged the terror
-swam across Warm lake
-shouted with joy-conquered the fear of water
-there is terror only in the fear of death and peace in death.
- the will to live became stronger.

90
QUESTIONS FOR PRACTICE

SHORT ANSWER QUESTIONS


1. Why did mother warn Douglas against River Yakima?
2. What impact did the incident at California beach have on him?
3. What made him decide that the instructor’s role in teaching him swimming was
over?
4. Why did Douglas go to Lake Wentworth in New Hampshire? How did he make
his terror flee?
5. What larger meaning did the experience have on him?
6. How did he interpret Roosevelt’s saying?

LONG ANSWER QUESTIONS

1. What is the ‘misadventure’ that William Douglas speaks about? What were the
series of emotions fears experienced when he was thrown into the pool? What plans
did he make to come to the surface?

2. How did the instructor build a swimmer out of Douglas?


3. Why does Douglas as an adult recount a childhood experience of terror and his
conquering of it? What larger meaning does he draw from his experience?

4. Comment on the appropriateness of the title.

INDIGO by Louis Fischer


GIST OF THE LESSON
• Raj Kumar Shukla- A poor sharecropper from Champaran wishing to meet
Gandhiji.
• Raj Kumar Shukla – illiterate but resolute, hence followed Gandhiji to Lucknow,
Cawnpore, Ahmedabad, Calcutta, Patna, Muzzafarpur and then Camparan.
• Servants at Rajendra Prasad’s residence thought Gandhiji to be an untouchable.
• Gandhiji considered as an untouchable because of simple living style and
wearing, due to the company of Raj Kumar Shukla.
• Decided to go to Muzzafarpur first to get detailed information about Champaran
sharecropper.
• Sent telegram to J B Kriplani & stayed in Prof Malkani’s home –a government
servant.
• Indians afraid of showing sympathy to the supporters of home rule.
• The news of Gandhiji’s arrival spread –sharecroppers gathered in large number
to meet their champion.
• Gandhiji chided the Muzzafarpur lawyer for taking high fee.

91
• Champaran district was divided into estate owned by English people, Indians
only tenant farmers.
• Landlords compelled tenants to plant 15% of their land with indigo and
surrender their entire harvest as rent.
• In the meantime Germany had developed synthetic indigo –British landlords
freed the Indian farmers from the 15% arrangement but asked them to pay
compensation.
• Many signed, some resisted engaged lawyers, landlords hired thugs.
• Gandhiji reached Champaran –visited the secretary of the British landlord
association to get the facts but denied as he was an outsider.
• Gandhiji went to the British Official Commissioner who asked him to leave Tirhut
, Gandhiji disobeyed, went to Motihari the capital of Champaran where a vast
multitude greeted him, continued his investigations.
• Visited maltreated villagers, stopped by the police superintendent but disobeyed
the order.
• Motihari black with peasants spontaneous demonstrations, Gandhiji released
without bail Civil Disobedience triumphed.
• Gandhiji agreed to 25% refund by the landowners, it symbolised the surrender of
the prestige.
• Gandhiji worked hard towards social economic reforms, elevated their distress
aided by his wife, Mahadev Desai, Narhari Parikh.
• Gandhiji taught a lesson of self reliance by not seeking help of an English man
Mr. Andrews.

SOLVED QUESTIONS

SHORT ANSWER QUESTIONS

1. Who is Rajkumar Shukla? Give a pen sketch of him.


--Rajkumar is a sharecropper from Champaran. He was poor and emaciated.
2. Describe the efforts made by Shukla for bringing Gandhiji to Champaran.
---He is very resolute. He followed Gandhiji to Cawnpore, Ahmedabad, Calcutta, Patna
and then to Champaran.
3. What strategy Gandhiji followed in removing the problems of sharecroppers?
----Gandhiji discussed the problems with lawyers. He disregarded British order of
eviction. He insisted peasants to removed their fear
4. Why did Gandhiji feel that it was useless for the peasants to go to law courts?
----The peasants are crushed and fear stricken. The lawyers charged high fee.
5. Why did the British landlords free the sharecropper from growing Indigo? What did
they want instead?
----The British came to know that synthetic indigo was developed in Germany and the
15% of land was released and in return the peasants were asked to pay compensation
for release from the agreement.

92
6. Why did Gandhiji agree for the 25% refund by the British landlords?
----Gandhiji agreed for 25% refund because the amount was not important but the
landlord’s prestige was surrendered.
7. Describe the social economic reforms introduced by Gandhiji in Champaran
villages?
------Gandhiji opened primary schools in six villages. He also engaged a doctor.
8. What was the important lesson taught by Gandhiji to his disciples ?
-----Gandhiji taught rules of personal hygiene and cleanliness. He also taught the -
Champarans to win freedom independently without any support of British.

LONG ANSWER QUESTION

1. Why did Gandiji consider freedom from fear more important than legal injustice for
the poor peasants of Champaran?

Value Points
British ruthless exploitation—farmers fight through lawyers—battles were
inconclusive—terror-stricken—Gandhiji’s declaration—no need of law court –overcome
terror—be bold and courageous.

QUESTIONS FOR PRACTICE


SHORT ANSWER QUESTIONS

1. What made Gandhiji urge the departure of the British?


2. Give a brief account of the reception that Gandhiji got in MOthihar.
3. What made Mahatma Gandhi declare ‘the battle of Champaran is won’?
4. How did Gandhiji keep a distant watch on the Ashram?

LONG ANSWER QUESTIONS

1. Why did Gandhiji’s casual visit to Champaran get extended to over a year?
2. How did civil disobedience triumph?
3. What idea do you form about the Britishers from the chapter “Indigo”?
4. How did the peasants learn courage?

The Rat Trap by Selma Lagerlof


GIST OF THE LESSON
• The peddler was a vagabond who sold rattraps with a little thievery on the side
to make both ends meet. Had no worldly possession to call his own, not even a
name.

93
• It amused him to think of the world as a rattrap and all the material possessions
as bait as the world, he felt was never kind to him. Moreover, he prided himself
in the fact that he was out of it.

• Takes shelter at a crofter’s cottage. The crofter welcomed him, gave him diner,
shared his pipe, played mjolis with him also confided in him about his income
and showed him where he put it.

• Next morning, the Peddler steals the money and takes the back roads to keep
away from people and gets lost in the jungle at night. While he wanders in the
forest he realizes that he has also got caught in the rattrap and that the money
was the bait.

• Finally reaches Ramsjo ironworks, where he takes shelter for the night. The
blacksmith and his assistant ignore him but the master mistakes him to be an old
acquaintance and invites him home. Though the Peddler does not correct the
ironmaster, hoping to get some money out of him, he declines his invitation.

• The ironmaster then sends his daughter who persuades him to go home with
her. She notices his uncouth appearance and thinks that either he has stolen
something or he has escaped from jail.

• The Peddler is scrubbed, bathed, given a haircut, a shave and a suit of old clothes
of the ironmaster. In the morning light, the iron master realizes he is mistaken
and that he is not the Captain. He wants to call the Sheriff. The peddler is
agitated and breaks out that the world is rattrap and he too is sure to be caught
in it. The ironmaster is amused but orders him out. The compassionate Edla
convinces her father that he should spend the Christmas day with him.

• The Peddler spends the whole of Christmas Eve eating and sleeping. The next
day at church, Edla and her father come to know that the Peddler is a thief who
stole thirty kroners from the poor crofter.

• -Back home, they found a letter addressed to Edla, signed as Captain Von Stahl
and a rattrap as a gift from the crofter. In the rattrap were the three ten kroner
notes of the crofter.

SOLVED QUESTIONS
SHORT ANSWER QUSTIONS

1. Why did the Peddler choose to go through the forest?

94
After stealing the thirty kroners from the crofter, the Peddler knew that he would be
caught and put in prison if he continued to walk by the man road. So he chose the back
roads that went through the forest.

2. Why did the Peddler not reveal his true identity when the ironmaster mistakes
him to be the Captain?

The Peddler thought that the ironmaster might take pity on him give him some money if
he thought he was an old acquaintance. So he keeps quiet and allows the iron master to
presume he was the captain.

3. Why did it please the tramp to compare the world to a rattrap?

The world was not very kind to the tramp and so it gave him great pleasure to think of it
as a rattrap.

4. Why did the tramp sign the letter as Captain Von Stahl?

The tramp, though illiterate and a thief, found himself raised to a captain through Edla’s
kindness and compassion. He got a chance to redeem himself and hence he signs the
letter as Captain Von Stahl.

LONG ANSWER QUESTIONS

1. Both the Crofter and Edla Willmanson were kind and hospitable to the Tramp. But he
repays crofters Kindness by stealing his money while Edla is able to transform him to a
better human being. Why?
Value Points:
Crofter very hospitable. Welcomes him with a smile - gives him supper and shares his
tobacco. Tells him about income-shows him the money - very trusting and friendly.
Stealing a way of life for the tramp-no twinge of conscience while stealing-But later he
realizes that he who prided himself in not being caught in the rattrap was caught in it by
stealing – feels depressed.
Edla’s kindness and hospitality awakens his conscience-Realises that there is a way out
of the trap- Returns the money through Edla- His redemption –gift to Edla.

QUESTIONS FOR PRACTICE:

1. “The world was a rattrap and the peddler himself became a victim of it”.
Elucidate.

2. The rattrap exemplifies the truth that essential goodness of human can be
awakened through understanding and love. Discuss

95
3. The story focuses on human loneliness and the need to bond with others.
Explain.

POETS AND PANCAKES by ASOKAMITRAN.


GIST OF THE LESSON
• The Gemini studio owned by S.S.Vasan was one of the most influential film
Producing Organizations of India in the early years of Indian film making
industry.
• The make-up department of studios looked like a hair cutting salon.
• Had lights at all angles, half dozen mirrors, incandescent lights
• The artists were subjected to misery while application of make-up.
• The make-up department, consisting of people from different parts of the
country, was a unique example of National Integration.
• A strict hierarchy was maintained in the make-up dept.
• Narrator worked in a cubicle tearing newspapers, thought he was free, people
barged in always.
• Kothamangalam Subbu, no.2 at Gemini studios, was always cheerful, tailor made
for films, endowed with great creativity, charitable yet had enemies
• He was loyal and faithful, very close to boss.
• He could offer various alternatives for how a scene could be invented. Subbu, in
fact, gave direction to Gemini studios during its golden years.
• The story Department of the studios comprising of a lawyer, officially
known as legal adviser but was treated the opposite. Once he brought a sad end
to the career of a brilliant and promising young actress.
• Story dept wound up-lawyer lost job.
• A favorite haunt for poets
• Most people wore Khadi, worshipped Gandhi, knew nothing about politics.
• Against communism, believed that a communist was a godless man
• A warm welcome was accorded to moral Re Armament Army (MRA) by the
Gemini studios. They presented two plays ‘Jothan Valley’ and “The forgotten
Factor” which had a great influence on Tamil drama.
• THE MRA was a strong countermovement against communism.
• The Gemini studios again got an opportunity to welcome an English poet or an
Editor. But the people of Gemini Studio could not comprehend the purpose as
well as the language of the poet or editor, so his visit was a mystery.
• Later on, he came to know that the visitor was the editor of “The Encounter” and
his name was Stephen Spender.
• “The God That Failed” was the collection of six essays by six men of letters
including Spender. These essays described separately their journey into
communism and their disillusioned return.
• Mystery was solved.

96
SOLVED QUESTIONS
SHORT ANSWER QUESTIONS
1. Describe the make-up room of the Gemini studios, as given in the “poets and
pancakes”?
The makeup room of the Gemini studio had incandescent lights. It also had lights at all
angles, large mirrors. Those subjected to makeup had to face bright light and a lot of
heat there. It was on the upper floor of the o a building that was believed to have been
Robert Clive’s stables.
2. How was the make-up room a fine example of national integration?
The makeup room was headed by a Bengali, succeeded by a Maharashtrian, assisted by
a-Dharwar Kannadiga, an Andhra, a Madrasi, Christian and an Anglo Burmese.
3. How did the legal advisor bring a sad end to the brief and brilliant acting career
of an extremely talented in the studios?
The legal advisor (lawyer) quietly switched on the recording equipment when once she
blew over on the sets against the producer. When the actress paused for breath, he
played back the recording. She was struck dumb on hearing her own voice and never
recovered from the shock. That was the end of the brief and brilliant career of the
actress.
4. What does ‘The God That Failed’ refer to?
‘The God That Failed’ refers to a collection of essays by six eminent literary
personalities, about their journey into communism and disillusionment. Stephen
Spender was one of the authors.
LONG ANSWER QUESTIONS
1. The author has used gentle humor to point out human foibles. Pick out instances of
this to show how this serves to make the piece interesting.
-author uses gentle and subtle humor –bring out human foibles
-use of pancake –ostensibly to cover the pores
-The actors look ugly.
-Strict hierarchy.
-people at the studio imagined to be poets yet no idea about contemporary
-Poets-laughingly brings out their ignorance
-wore khadi looked gandhian no idea about politics
-no idea of communism-welcomed MRA, Stephen Spendor
-Description of office boy-Description of Subbu-wonderful insight into character.
-pokes fun at the ignorance of all the people
-at the same time projecting them as real people with human failings
and eccentricities and foibles.

QUESTIONS FOR PRACTICE


SHORT ANSWER QUESTIONS
1. Bring out the gentle humor that the make-up department present?
2. Why did the author want to know more about the periodical ‘The Encounter’?
What did he finally discover?
3. A strict hierarchy was maintained in the makeup department. Explain.

97
3. Why was the narrator praying for crowd shooting all the time?
4. What is meant by pancakes in the chapter? What do you know about the literary
taste of the taste of Gemini as far English poetry is concerned?
5. What was the suspense about ‘another visitor’ Gemini Studio was to welcome?
6. Give a brief description of the lawyer.

LONG ANSWER QUESTIONS


1. Attempt a character sketch on Kothamangalam Subbu.
2. The ‘Office-boy’ in the make-up department of Gemini Studios has the shades of a
typical universal character lured by glitter and glamour and doomed to frustration.
Discuss.
3. Describe the make-up department of Gemini studio.
4. The staff at Gemini Studio enjoyed the MRA while the visit of the English poet
remained an unexplained mystery. Discuss.

THE INTERVIEW by Christopher Sylvester


GIST OF THE LESSON
PART I
• Interview has become a commonplace of journalism. Opinions on the functions,
methods and merits of Interview vary considerably.

• Some claim it to be the highest form, a source of truth and in its practice an art.

• Some despise the interview as an unwarranted intrusion into lives, which


diminishes their personality.

o V.S. Naipaul feels that ‘some people are wounded by interviews and lose
a part of them selves’.

o Lewis Carroll never consented to be interviewed for he believed it to be


‘a just horror of the interviewer’.

o Rudyard Kipling considered it ‘immoral, a crime, an assault that merits


punishment’.

o H.G. Wells referred interviewing to be an ‘ordeal’.

o Saul Bellow describes it ‘like thumbprints on his windpipe’.

• Despite the drawbacks interview is a supremely serviceable medium of


communication. Interviews are the most vivid impression of our contemporaries
and the interviewer holds a position of unprecedented power and influence.

98
PART II
• An extract from an interview of Umberto Eco interviewed by Mukund
Padmanabhan.

• Umberto Eco was a professor with a formidable reputation as a scholar for his
ideas on Semiotics, literary interpretation and medieval aesthetics before he
turned into writing literary fiction. He attained intellectual superstardom with his
publication “The Name of the Rose”.

• In the interview Eco shares his idea of empty spaces in our lives just as they exist
in an atom, which he calls Interstices. He says that he makes use of these empty
spaces to work.

• Eco’s essays were scholarly and narrative. He likes to be identified more as a


university professor who writes novels.

• Eco’s ‘The Name of the Rose”, a serious novel, which delves into metaphysics,
theology and medieval history, enjoyed a mass audience. It dealt with medieval
past. He feels that the novel wouldn’t have been so well received had it been
written ten years earlier or later.

SOLVED QUESTIONS

SHORT ANSWER QUESTIONS

b. Why do most celebrity writers despise being interviewed?


Most celebrity writers despise being interviewed as they consider it as an undesirable
intrusion into their personal lives. Some viewed it an immoral and offensive activity.
Some others feel it would ‘diminish’ them.
a. How is Umberto Eco’s non-fictional writing style different from academic
writing style?

Umberto Eco’s non-fictional writing style has a certain playful, narrative and personal
quality about it whereas his academic writing is depersonalized and often dry and
boring.
LONG ANSWER QUESTION
a. “Interviews an unwarranted intrusion in the lives of others”. Elucidate with
reference to The Interview.

Value points:-
Interviews are common feature in Journalism – Most celebrities consider them as an
unnecessary intrusion in their lives – “a horror of the interview”, “ an ordeal”,
“thumbprints on windpipe”- interview is a supremely serviceable medium of

99
communication- the most vivid impression of our contemporaries -the interviewer holds
a position of unprecedented power and influence.
QUESTIONS FOR PRACTICE
SHORT ANSWER QUESTIONS
1. “Best interviews are considered as an art.” Mention four qualities of such an art.

2. Saul Bellow consented to be interviewed many a times. Did he like being


interviewed? Substantiate.

3. Why does Brian say that interviewer holds a strong position of power and
influence?

4. Why was ‘Name of the Rose’, a success?

5. Mention two remarkable qualities of Umberto Eco’s scholarly writings?

LONG ANSWER QUESTIONS


1. The interview conducted by Mukund Padmanabhan reveals what a good
interview should be like. Do you agree? Give reasons.

2. Consider interview as a medium of communication.

GOING PLACES by A.R.BARTON


GIST OF THE LESSON
• The lesson explores the theme of adolescent fantasies and hero worship.
• Sophie and Jansie are both in the last year of high school and both knew that
they were destined to work in the biscuit factory as they belong to a working
class family.
• Yet, Sophie, always dreams of big and beautiful things, glamour and glory.
• Her ambitions are not rooted in reality i.e., have no relation with the harsh
realities of life.
• In contrast is Jansie, Sophie’s friend, a realistic and practical girl.
• Sophie lives in male-dominated family where her mother was only a shadow.
The men were football fans and the conversations around the dinner table
were about Danny Casey, their Hero.
• Sophie wants some attention from her father and brother and telling them
that she met Casey, was her way of drawing their attention towards her.
• But she carries her fantasies too far when she starts to live them.

SOLVED QUESTIONS
SHORT ANSWER QUESTIONS
1. What does Sophie dream of doing after she passes out of school? Why do you
call it a dream, and not a plan?

100
Sophie dreams of big and beautiful things, glamour and glory. She is not practical
in her thinking and has no concrete plans to make her dreams real.
2. Compare and contrast the characters of Sophie and Jansie. Sophie was a
dreamer. She was not ready to accept the fact that she would be working in a
biscuit factory after her high school. But the tragedy was that she carries her
fantasies too far when she started living in them, whereas her friend Jansie was
practical and down-to-earth. She firmly anchored her friend Sophie to reality
whenever she spoke of her dreams.
3. Sophie is a typical adolescent hero-worshipper who carries her fantasizing too
far .Comment.
Yes, Sophie is a typical adolescent who worships Danny Casey, the football star.
She fantasizes about meeting him and taking his autograph which is quite normal
for an adolescent. But she crosses the border of normalcy when she tell her
family that she actually met him and that he wants to take her on a date. And
then she actually goes to the place and literally waits for him to appear which is
abnormal.
LONG ANSWER QUESTION
1. Sophie has no touch with reality; Janise’s feet are, however, firmly planted on
the ground. Discuss.
Value Points:
Both school going girls and intimate friends – both belong to lower middle class
family – but different from each other. Different approach to life - one a
romantic, habitual dreamer and an escapist. Sophie dreams of things beyond her
reach – Jansie has a practical approach to life - discourages her friend’s wild
dreams – gossipy.

QUESTIONS FOR PRACTICE


SHORT ANSWER QUESTIONS

1. “Sophie’s dreams and disappointments are all in her mind “.justify this
statement.
2. Describe Sophie’s meeting with Danny Casey.
3. What was Sophie’s father’s reaction when Geoff told him about Sophie meeting
Casey?
4. When did Sophie actually see Danny Casey?
5. Why was Sophie jealous of Geoff’s silence?

LONG ANSWER QUESTIONS


1. Why did Sophie like her brother Geoff more than any other person? From her
perspective, what did he symbolize?
2. Give a brief character sketch of Sophie’s father. What kind of a relationship did
they share?
3. The story “Going Places” draws a beautiful contrast between ‘fact’ and ‘fiction’.
Comment.

101
VISTAS

THE THIRD LEVEL by JACK FINNEY


SUMMARY OF THE LESSON:
• ‘-The Third Level’ is a story that weaves together a psychological journey of
the narrator into past, present and moves towards future.
• -Charley- 31 year old, an American narrates his unusual experience of having
been to the third level.
• -He discovers brass spittoons, flickering gas lights, everyone dressed, like
1890’s with old fashioned beards, side burns, fancy moustaches, engine with
funnel shaped stack, newspaper - The World, few ticket windows etc in the
third level.
• -Charley goes to get the ticket for Galesburg - Illinois- Wonderful town, big
houses, huge lawns, plenty of trees, people with lots of leisure time and
surprises when the clerk says that it is not currency and he will be put behind
the bars.
• Wife worried -takes him to his Psychiatrist friend
• -He refuses to believe -says it a waking wish dream fulfillment as he could
not face the modern world which is full of fear, insecurity, war, worry, stamp
collection a refuge from reality.
• Charley desperate to go to Galesburg & so exchanges new for old currency.
But could never find the third level.
• Sam’s disappearance has something to do with Galesburg as he was fond of
the place.
• One day while going through his stamp collection, he finds an envelope,
containing a letter of July 18, 1894 written by Sam, who is living at Galesburg,
assertions the Third level.
• Charley finds that Sam had bought old currency worth eight hundred
dollars.
• Sam was Charley’s psychiatrist.

SHORT QUESTIONS
1. Was the Third level a medium of escape for Charley?
Suggested Answer / Value Points-
Yes, Charlie was engulfed in the trials and tribulations of life so he created the third level
himself to seek refuge in it. It eased his tensions and pressures and provided him a
platform to relax.
2. Why is Grand Central compared to a tree?
Suggested Answer / Value Points
Grand Central was compared to a tree because it was pushing out new corridors and
staircases like roots. There may be a tunnel under the city up to Times Square and
another to Central Park. Nobody knows about them. For many people it has been a exit,

102
a way of escape through the years. So it is possible that it may have the tunnel Charlie
got into.

How did Charlie realize that he had reached the third level?
Suggested Answer / Value Points-
Charlie saw brass spittoons, flickering gas lights, everyone dressed like in the1890’s with
mutton sleeves side burns and moustaches. The engine was with a funnel shaped stack.
The newspaper was dated June 11 1894.There were fewer ticket windows. This made
him realize that he was on the third level.

3. Why did he wish to escape to Galesburg?


Suggested Answer / Value Points-
He wished to escape to Galesburg because it was a wonderful town with big houses,
huge lawns, plenty of trees. The summer evenings were twice as long and people had
lots of leisure time to sit out on the lawns. It was a peaceful world.

4. What is First Day Cover?


Suggested Answer / Value Points-
At the time when a new stamp is issued, stamp collectors buy some of them and use
them in order to mail envelops to themselves and the postmark proves the date. The
envelope is called the First Day Cover.
5. What is referred to as ‘the obvious step’?
Suggested Answer / Value Points-
The obvious step refers for consulting a psychiatrist. As a fact there are only two levels
& it does not seem wise to visit non-existing third level. Therefore it was very apparent
to consult an expert.
QUESTIONS FOR PRACTICE
1. Who was Charley? What was his problem?
2. “That ain’t money , mister.” When did the ticket-clerk utter these words?
3. Who was Sam? What did he tell Charley?
4. How did Charley contradict the psychiatrist’s opinion?
5. What surprised Charley one day?
6. What had Sam written in his letter to Charley?
7. Why couldn’t Sam go back to his old business in Galesburg?

LONG ANSWER QUESTIONS

Do you think that the third level was a medium of escape for Charley? Elaborate.
Value Points:
-Harsh reality—too oppressive at times
-Stress, strain, fierce competition, fear of failure causes insecurity
-Charley declares –ordinary guy-but not able to cope with the modern world
-Wish to visit Galesburg of 1890 –old frame houses, huge lawns, beautiful trees
-People sitting on lawns relaxed-calm and peaceful-

103
-Exchanging new currency for old currency, Roping in his wife and Sam into his third
world, saying that Sam also bought old currency, the letter from Sam in his first day
cover are all his figment of imagination.
-It was a medium of escape – could not face the challenges –modern world.
QUESTIONS FOR PRACTICE
1. Comment on the ending of the story.
2. Do you think the title ‘The Third Level’ is appropriate?
3. Bring out the contrast between the world Charlie lived in and the one that he stray
into.
4. What devices does Jack Finney use to portray Charley’s transition from reality to
fantasy seem probable and plausible?

THE TIGER KING BY KALKI


GIST OF THE LESSON
• The Maharaja Sir Jilani Jung Jung Bhadur was called “Tiger King”.

• When he was just 10 days old he asked intelligent questions to the astrologers
and was told that he would be killed by a tiger. He uttered “Let tigers beware!”

• No other miracle took place, the child grew like any other Royal child drinking
white cow’s milk, taught by an English tutor, looked after by an English nanny
and watched English films.

• When he was 20, he was crowned as king. It was then the prediction of his death
by the tiger reached the Maharaja’s ear and he in turn to safe guard himself
killed a tiger and being thrilled he told the astrologer who replied that he can kill
99 tigers but should be careful with the 100th.

• From then on he started killing tiger and none was allowed to hunt tigers. A high-
ranking British officer visited the state that was fond of hunting tiger and his
wish was declined.

• The officer requested for getting a photograph with a tiger killed by Maharaja
and this request was rejected.

• So to please the officer’s wife he sent 50 diamond rings expecting that she would
take 1 or 2 instead she kept all the rings costing 3 lakh rupees and sent ‘thanks’
to the Maharaja. But his state was secured.

• In 10 years he killed 70 tiger and didn’t find any in Pratibandapuram so he


decided to marry a girl from royal state which had more tigers to complete his
target.

104
• Whenever he visited his in-laws he killed 5-6 tigers. So he killed 99 tigers and was
feverishly anxious to kill the 100th but couldn’t find.

• News about the presence of a tiger near a village proved disappointing.

• Now the Dewan was warned of his danger so he visited ‘People’s Park in Madras’
and brought an old tiger and placed it in the forest and informed the Maharaja.

• The Maharaja took great care and shot the tiger and left the place with great
triumph.

• The bullet did not hit the tiger but out of fear the tiger had collapsed. Now the
staff killed the tiger and brought it in grand procession.

• It was the 3rd birthday of the Maharaja’s son and he wanted to buy a present
from the toyshop. He bought a wooden tiger which was poorly carved.

• While the Maharaja was playing with the prince a tiny sliver of the wooden tiger
pierced his right hand which later on caused his death. Thus the hundredth tiger
takes his final revenge upon the “Tiger King”.

Question and Answer:


1. What was the miracle that took place in the royal palace?
When the Maharaja was a 10 day old infant spoke and asked intelligent questions about
his death. After knowing that he would be killed by a tiger he uttered saying “Let tigers
beware.”
2. How was the Tiger King brought up?
As a child the Tiger King was brought up by an English nanny and tutored in English by
an Englishman. He was given the milk of an English cow. Like many other Indian crown
princes he watched only English movies.

3. What did the State astrologer say he would do ‘if the hundredth tiger were also
killed’?
The State astrologer was so sure of his prediction that he announced that he would cut
off his ceremonial tuft, crop his hair short and become an insurance agent in case the
king was able to kill the 100th tiger, too. He was sure that the Maharaja’s death would
be caused by the 100th tiger.
4. What did the high-ranking British officer wish to do? Was his wish fulfilled?
The high-ranking British officer wanted to kill a tiger. When he was denied the
permission for hunting, he sent a word to the king that he would be happy if he was
allowed to get photographed with the dead body of a tiger killed by the king. However,
his wish remained unfulfilled.

105
5. How did the Maharaja manage to save his throne?
The Maharaja had annoyed the visiting senior British officer over the issue of tiger-
hunting and ‘stood in danger of losing his kingdom itself’. So, the Maharaja and the
Dewan decided to placate and pacify the officer through bribe by sending gifts of
expensive diamond rings to the ‘duraisani”, the wife of the British officer. Thus he
managed to save his throne.
6. How did the ‘duraisani’ behave on receiving the gifts?
Some fifty samples of expensive diamond rings were sent to the duraisani and it was
expected that she would select on or two and return the rest. But the lady proved to be
greedy as she retained all of them and merely sent a letter of thanks.
7. Why did the Maharaja’s tiger killing mission come to a sudden still?
Within ten years Maharaja’s tiger hunting had resulted in the killing of seventy tigers.
However his tiger killing mission came to a sudden standstill because the tiger
population became extinct in the forest of Pratibandapuram.
8. Why did the Maharaja suddenly decide to marry? Whom did he wish to marry?
The Maharaja suddenly decided to marry because firstly, he was of marriageable age
and secondly, he wanted to kill thirty more tigers in his father-in-law’s state in order to
complete the tally of hundred tigers .For this reason he wished to marry a girl in the
royal family of a state with a large tiger population.
9. Why did Maharaja order the dewan to double the tax?
The Maharaja called the dewan and ordered him to immediately double the tax of the
villagers who had informed him of a tiger in the forest because despite his best efforts
he was unable to locate the beast. This infuriated the Maharaja.
10. What did the Maharaja buy as a birthday gift for his son?
The Maharaja wished to give his son a very special gift on his birthday and he bought a
wooden toy tiger as a perfect birthday gift for his son.
11. How did the king’s arm become seriously infected?
The king’s arm had got infected from a prick caused by one of the slivers on the wooden
tiger. In one day, the infection got flared in the Maharaja’s right hand and in four days it
developed into a suppurating sore which spread all over the arm. Though he was
operated yet he died.
Long answer type question
1. How did the Tiger King meet his end? What is ironical about his death?
The wooden toy tiger the king had got as a birthday present for his son had been carved
by an unskilled carpenter. It had a rough surface with tiny slivers of wood standing up
like quills all over it. One of those slivers pierced the Maharaja’s right hand and although
the king pulled it, his arm got infected. In four days, it developed into a suppurating sore
and spread all over the arm. The king died while being operated upon.
The king’s death is ironical but not surprising for the reader who is, in fact, looking
forward to it. Having ‘killed’ the 100th tiger, the king is jubilant for he has fulfilled his
vow and disproved the prediction of the royal astrologer. He is now at ease for he thinks
he cannot die of a tiger’s attack. No wonder, he orders the ‘dead’ tiger to be taken in a
procession through the town and gets a tomb erected over it. All this while he does not
know that the 100th victim was not killed by him but by other hunters. That is indeed

106
quite ironical. Death is lurking around him and the king is unaware of it. Again, it is
ironical that a king who has killed 100 tigers and is bold and fearless dies of a mere
‘sliver’ on the body of a wooden tiger. Thus, ironically death does come to him from a
tiger.
Unsolved Question
1. How did the Maharaja kill the hundredth tiger?
2. The title of the Maharaja reflected grandeur but his death was an anticlimax. Justify.
3. How did the Tiger King meet his end? What is ironical about his death?
4. What idea do you form about the ruling Indian class during the pre-partition days
from the king’s encounter with shopkeeper?
5. What idea do you form about the married life of the king?

JOURNEY TO THE END OF THE EARTH BY TISHANI DOSHI


GIST OF THE LESSON
• The writer Tishani Doshi was part of a programme called ‘Students on Ice’ and
was taken to Antarctica. She shares her experiences and impressions.

• The aim of the expedition was to take high school children to Antarctica, thereby
providing them with a unique educational opportunity to see first hand the
effect of human activities on environment so that these youngsters, when the
time comes to act, will take correct decisions regarding conservation of the
environment.

• Antarctica is the right place to study about the changes in the environment and
its impact on humanity as a whole.

• Antarctica holds the secrets of how earth came to be in its present form. It gives
us an insight into how the present landforms came into existence, why certain
animal species became extinct.

• Complete absence of human beings has preserved the carbon records intact.

• Lack of bio-diversity in Antarctica makes it possible to study the effect of small


changes in the environment on the animal life and environment.

• It was a humbling experience for the author and she feel that we need to
address environmental issues immediately, if mankind is to survive.

SOLVED QUESTIONS

SHORT ANSWER QUESTIONS

107
1. What were Geoff Green’s reasons for including high school students in the
‘Students on Ice’ expedition?

Children are more receptive and ready to absorb, learn and most importantly act. Unlike
the elderly people who are at the end of their productive life, children can giveback to
society substantially and practice what they learn and experience. Most importantly,
they can take correct decisions and act when the time comes and save the environment.

1. How does the writer realize that the threat of global warming is very real?

The writer visits Antarctica and sees for herself the glaciers retreating and ice shelves
collapsing. She sees with naked eyes the effect on the food chain when a singled celled
phytoplankton is removed from food chain. That’s when she realizes that threat of
global warming is real.

3. What revelation did the author have on her visit to Antarctica?

The writer on her visit to Antarctica noticed the beauty of balance in play on our planet
and realizes that everything in the world is inter-connected.

4. What is phytoplankton? What is their importance?

Phytoplanktons are single celled organisms that live in the southern ocean and are also
called the grasses of the sea. They nourish and sustain the entire ocean’s food chain as
they are the first link in the food chain of the ocean. They use sun’s energy to assimilate
carbon and synthesize organic compounds. The depletion of ozone layer affects these
organisms and their diminishing number is affecting other organisms of the ocean and in
turn lives of all organisms on earth will be affected.

LONG ANSWER QUESTIONS

1. “Take care if small things and the big things will take care of themselves”. What is the
relevance of this statement in context of the Antarctic Environment?

Suggested Value Points: The Statement- greatly relevant in the context of Antarctic
Environment – only place not strained by man – remains pristine – carbon records
preserved in its folds – simple eco system – lack of bio-diversity - perfect place to study
how small changes can have big repercussions – example of phytoplankton – how it uses
sun’s energy to synthesize food – process of photosynthesis – Depletion of ozone layer
effects phytoplankton – life of small marine animals – in turn the food of large animals –
Global warming effect all animals and plants on earth – evident in Antarctica.

108
QUESTIONS FOR PRACTICE

SHORT ANSWER QUESTIONS

1. How is Antarctica a crucial element in the debate on climate change?

2. How is global temperature increasing? What are the immediate


repercussions of this increase on the environment?

3. How does Antarctica differ from the rest of the earth?

4. What were the feelings of the author when she set foot on the Antarctic
continent?

5. What was Gondwana? What happened to it?

6. Why did the writer loose all earthly perspective when she set foot on the
continent?

7. Why is Phytoplankton a metaphor for existence?

8. “It can get pretty mind-boggling”. What is ‘it’ referred to? Why does the
writer say so?

LONG ANSWER QUESTIONS

1. How was the journey to Antarctica an incredible experience for the author?

2. “And for humans, the prognosis is not good”. Explain.

3. Why does the author says that ‘a lot can happen in million years, but what a
difference a day makes’.

4. The history of Antarctica reveals the history of humankind. Explain.

THE ENEMY BY PEARL S BUCK


GIST OF THE LESSON:
• Dr. Sadao, a Japanese surgeon finds a wounded American soldier on the
beach near his house.
• He is unable to throw him back though he was his enemy as he was a doctor
and his first duty was to save a life.
• Hana, his wife, though initially reluctant because it was dangerous for all
including the children to keep the enemy in the house, joins her husband in
operating and nursing the enemy soldier back to health, even though the
servants desert the house.

109
• Hana assists Dr. Sadao in operating the soldier in spite of her physical
discomfort and hesitation.
• Though it was war time and all hands were needed at the front, the General
did not send Sadao with the troops as he is an expert surgeon and the
General needed him.
• Sadao tells him about the enemy soldier but he does not take any action as
he is self-absorbed and forgets his promise that he would send his private
assassins to kill the enemy and remove his body.
• Taking advantage of the general’s self-absorption Sadao decides to save the
soldiers life. After the soldier is out of danger Dr. Sadao helps him to escape
from his house to safety.

SOLVED QUESTIONS:
SHORT ANSWER QUESTIONS
1 Why did the General overlook the matter of the enemy soldier?
The General had an attack and according to Dr. Sadao he could not survive the second
attack. So if Dr. Sadao was arrested, no other doctor was capable of performing the
operation. So for furthering his selfish needs he overlooked the matter and promises to
send his assassins. But he was so self-absorbed, he forgot about it.

2. Why was Dr. Sadao not sent with the troops?


The General thought that Dr. Sadao is indispensable to his life and can save anyone as
he is very skilled. He also does not trust anyone except Dr. Sadao. So he was not sent
with troops.
3. How was the plan of the prisoner’s escape executed in the story?
The prisoner was successful in his escape only because of the right guidance and help
from Dr. Sadao. He provided him his boat, gave his food, made him wear Japanese
cloths and also helped him in comfortable sail to a nearby island.
4. Why did the servants leave Dr. Sadao’s house?
They were not in favour of keeping the American prisoner hidden in the house. They
also did not want Dr. Sadao to save his life as he was the enemy. Also, if the police come
to know of it, all their lives would be in danger. So they left the house.
5. Who was the white man whom Dr. Sadao and Hana found?
The white man was an American soldier as evident from his clothes. They guess that
he was a prisoner of war from his cap that said ‘Navy Seals’
6. ‘‘But Sadao searching the spot of black in the twilight sea that night, had
his reward’’. What was the reward?
The “reward” was the escape of the enemy. Dr. Sadao searched the spot of black in the
twilight sea that night to see if the man was still there but there was no light. Obviously
the man had gone. The escape of the prisoner was his reward.
7 What message does “The Enemy” give?
This is a great lesson in humanism. Dr. Sadao by nursing his country’s enemy proves true
to his professional ethics.

110
LONG ANSWER QUESTIONS
1. How did Dr. Sadao rise above narrow prejudices of race & country to human being in
need?
Suggested Answer / Value Points
Dr. Sadao- a renowned Japanese surgeon- believed in professional loyalty & human
kindness- saw an American wounded soldier in a terrible condition on beach in front of
his house-took him his house with the help of his wife Hana-successfully removed the
bullet-nursed him back to his life-thus rose above racialism.
As a patriot, reported the prisoner’s presence at his house to the Army General. The
general decided to have him killed-he grew vestless to see him & finally decided to help
him to escape form his house-gave him boats & instructed him how he could safely
escape.
2. Do you think the doctor’s final solution to the problem was the best possible one in
the circumstances?
Suggested Answer / Value Points
It is the best possible option-general had promised him that he would get the soldier
quietly killed through his private assassins-but he forgot to get rid of- Dr. Sadao could do
nothing-he wanted to get rid of the wounded soldier-as the servants had left the house-
and news could be spread-so he devised his own plan to get the soldier off to the
nearby island-managed his boat for the soldier and instructed him. The white soldier
took leave of him and followed his instruction and managed to escape safely. Thus all
this proves that that was the only way out for Dr. Sadao to the problem.

3. There are moments in life when we have to make hard choice between our role as
private individuals and as citizens with a sense of national loyalty. Discuss with reference
to the Enemy.
Suggested Answer / Value Points-
Dr. Sadao encounters with the dilemma-to live as private individual whose and moral
ethical responsibility is to save the soldier. So as a doctor and as an individual his first
job is to save the man-takes ethical responsibility, he risks his life, fame and social
status- takes him to his house and makes efforts to save him.
But his other side-sense of patriotism and nationalism also involves a report to police,
takes the general in confidence, and plans to get the enemy soldier killed but later on
helps the soldier in escaping. Thus Dr. Sadao’s personality is displayed as patriotic
citizen.

SHOULD WIZARD HIT MOMMY? By JOHN UPDIKE


Gist of the Lesson
• The chapter captures a very sensitive reaction of a small girl to an important
aspect of the story that her father narrates to her.
• The story reveals the worldview of a little child to a difficult moral question that
shows her mental or psychological richness.
• Jo is a little girl of four years. She is engaged in a story session with her father.
• Jack, the father used to tell her a story every evening and especially for Saturday
111
naps.
• Jo feels herself involved with the characters and the happenings.
• The story always had an animal with a problem. The old owl advises him to visit
the wizard who would solve the problem.
• Skunk’s problem- he smelt bad, visited the wizard who changed it to the smell of
roses.
• Skunk’s mother was unhappy with it and took him back to the wizard. She hit the
wizard and asked him to restore the original smell. She wanted her son to keep
his identity of a skunk and wanted his friends to accept him for himself. So the
wizard changes him back to smell like a skunk.
• After hearing the story of Roger Skunk Jo was not happy with the ending.
• She wants her father to change the ending. She wants the wizard to hit the
mother back and let Roger be which her father was not ready to do to establish
his authority. This raises a difficult moral question whether parents possess the
right to impose their will on their children.
• Her father finds it difficult to answer her question.

SOLVED QUESTIONS
SHORT ANSWER QUESTIONS:
1. How did the wizard help Roger Skunk?
The wizard was moved by Roger Skunk’s story. On finding his magic wand - chanted
some magic words & granted that Roger should smell like roses.
2. How did Roger Skunk’s Mommy react when he went home smelling of roses?
Roger Skunk began to smell like roses. Mommy asked about the smell - Roger Skunk
replied that the wizard had made him smell like that mother did not like that and asked
Roger to come with her.
3. How did the Skunk’s mother get him his old smell back?
Mother was furious to learn about the wizard who changed the original smell. She
immediately visited the wizard and hit him on his head and asked him to restore the
original smell.
4. Who is Jo? How has she changed in the past two years? How did Jo behave in
‘reality phase’?
Jo is Jack’s 4 year old daughter. She was no more a patient listener. She did not take
things for granted and tried to see things in her own way.
5. How does Jo want the story to end and why?
Jo understood Roger Skunk’s need to enjoy the company of his friends; therefore
wanted that the wizard should take Roger’s side.

LONG ANSWER QUESTIONS


1. Why an adult’s perspective of life is different from that of a child’s as given in the
story?

112
Suggested Value points
An adult’s perceptive on life is always different - maturity of a person becomes his
barometer to judge right & wrong. For him/her everything that occurs has a message. In
the story, Jack at no level accepts Jo’s worldview that wizard should hit Mommy.
On the other hand - a child’s perceptive is limited to his activities - child’s perceptive
completely different - they love ‘action’ more than thought - so does Jo in the story - she
would delight in hearing the story of Roger Skunk’s Mommy being hit by the wizard.
QUESTIONS FOR PRACTICE
SHORT ANSWER QUESTION
1. What different queries does the Secretary of the Examinations Board make from the
Governor before conducting the examination for Evans and why?
2. Who do you think made a call regarding a correction in the question paper? What did
it really want to convey?
3. Who is Carter? What does the Governor want him to go and why?
4. How did the Governor manage to reach Evans in the hotel?
LONG ANSWER QUESTIONS
1. What is Jack’s way of telling stories? Why is it appealing?
2. What does Jack want to convey through the story of Roger Skunk?
3. How is Jack’s childhood interwoven in the story of the stinky skunk?
4. How does Jack assert his authority as a father over his daughter?
5. What part of the story did Jack himself enjoy the most? Why?

‘ON THE FACE OF IT’ BY SUSAN HILL

GIST OF THE LESSON


• The play depicts beautifully yet grimly the sad world of the physically impaired.
• It is not the actual pain or inconvenience caused by a physical impairment that
trouble a disabled man but the attitude of the people around him.
• Two physically impaired people, Mr. Lamb with a tin leg and Derry with a burnt
face, strike a band of friendship.
• Derry is described as a young boy shy, withdrawn and defiant.
• People tell him inspiring stories to console him, no one will ever kiss him except
his mother that too on the other side of his face
• Mentions about a woman telling that only a mother can love such a face.
• Mr. Lamb revives the almost dead feelings of Derry towards life.
• He motivates him to think positively about life, changes his mind set about
people and things.
• How a man locked himself as he was scared-a picture fell off the wall and got
killed.
• Everything appears to be the same but is different- Ex. of bees. And weeds
• The gate of the garden is always open.
• Derry is inspired and promises to come back.

113
• Derry’s mother stops him but he is adamant saying if he does not go now it
would be never.
• When he comes back he sees lamb lying on the ground
• It is ironical that when he searches a new foothold to live happily, he finds Mr.
Lamb dead.
• In this way the play depicts the heart rendering life of physically disabled people
with their loneliness, aloofness and alienation.
• But at the same time it is almost a true account of the people who don’t let a
person live happily.

SHORT QUESTIONS
Q1. Who is Derry? What self-opinion does he hold?
Suggested Answer / Value Points-
Derek, also called Derry was a young boy of 14. He was a quiet, shy and defiant boy. One
side of his face was totally burnt by acid. He was a victim of inferiority complex.
Q2. How does Lamb try to remove the baseless fears of Derry?
Suggested Answer / Value Points-
Mr. Lamb influences Derry by his optimistic philosophy. He advised him not to give
attention on other’s comments, try to be internally pure and strong and eliminate the
negativity of life.
Q3. What did Derry’s mothers think of Mr. Lamb?
OR
Why did Derry’s mother stop him, going to Mr. Lamb?
Suggested Answer / Value Points-
Derry’s mother does not hold a good opinion about Mr. Lamb. She has heard many
things about the old man, therefore stops Derry to visit Mr. Lamb.
Q5. Why does Derry go back to Mr. Lamb in the end?
Suggested Answer / Value Points-
Actually Mr. Lamb has taught Derry, the most important lesson of life. He advises him
not to care about the comments made by others. He now no longer cares about his
burned face or looks. He is more concerned what he thinks and feels what he wants to
hear and see. He knows if does not go back, he will never go back. Therefore he returns
back.
Q6. Comment on the moral value of the play?
Suggested Answer / Value Points-
The moral of the play is very loud and clear. The physically disabled should focus on the
brighter side of life and not to brood over the shortcomings. The society should accept
them as they are and expand their social interactions .in this way they can fight out the
loneliness, depression and disappointment.
Q7. Mr. Lamb says to Derry; ‘it’s all relative, beauty and the beast’, what essentially
does he mean by that?
Suggested Answer / Value Points-
Mr. Lamb tells Derry that it all depends upon people’s individual perceptions. A thing is
a beauty for one while that beauty may be a beast for others.
114
Extra questions for practice
1. What is the attitude of Mr. Lamb to the small boy who comes to the garden?
2.”I’m not afraid. People are afraid of me,” says Derry. What do people think on seeing
his face? How do they react then?
3. Why does Lamb’s argument fail to console Derry?
4. What makes Derry think that the old man is always alone and miserable?
What does he tell the old man?
5. What argument does Derry give to convince his mother why he wants to go the old
man’s garden?
6. Comment on the ending of the play.
LONG ANSWER QUESTIONS
Q1. The actual pain or inconvenience caused by a physical impairment is often much
less than the sense of alienation felt by the person with disabilities. What is the kind of
behavior that the person expects from others?
Suggested Answer / Value Points-
Actual pain or inconvenience caused by physical impairment is often less than the sense
of alienation felt by the person with disabilities
– Physical disabilities
– caused pain once in life time
– But after it this physical disability – set chain for other actions – caused mental agony

-called Lamely lamb, mothers were afraid of sending the children because of his tin leg.
-Derry -burnt face –everyone pities him-only a mother could love that face
-Both Mr. Lamb and Derry have been the victims of verbal atrocities –
-Mr. Lamb takes comments lightly –
-But Derry does not have the attitude like Mr. Lamb –
-Attitude of the people needs to be changed
-Do not want sympathy but accept them as they are
-Wounds get healed –but bitter comments never forgotten leaves a scar
QUESTIONS FOR PRACTICE
1. What is the theme of the play? How has it been worked out?
2. Compare and contrast the characters of Mr. Lamb and Derry.
3. What is the bond that unites old Mr. Lamb and Derry the young boy? How does the
old man inspire the small boy?
4. Comment on the appropriateness of the title.

EVANS TRIES AN O-LEVEL BY OLIN DEXTER


GIST OF THE LESSON
• Evans a kleptomaniac was imprisoned thrice and all the time escaped from the
prison. Now he was in the prison for the 4th time and all of a sudden developed
curiosity to appear in O-level German Examination which also was an effort to
break the prison.

115
• The Governor takes utmost care to see that he would not be fooled. Every care
was taken to make Evans prepare for the exam.

• He was tutored by a German tutor for 6 months. The day before the exam the
tutor wishes good luck but makes it clear that he had hardly any ‘chance of
getting through.’ But Evans gives an ironical twist to the tutor’s observation by
saying “I may surprise everybody.”

• On the day of the exam Jackson and Stephens visited Evans cell and took away
everything that may help him injure himself. Evans was insisted to take away the
hat but he refused saying that it was lucky charm.

• Evans cell was bugged so that the Governor could himself listen to each and
every conversation in the cell. The invigilator Rev. S. McLeery too was searched
and left him to complete the task. Stephen sitting outside the cell every now and
then peeped into the cell.

• The exam went on smoothly. Stephen escorted the invigilator to the main gate
and took a look into Evans cell and found the invigilator (actually Evans)
wounded, informed the Governor. The latter was to be hospitalized but
informed that he was alright and asked them to follow Evans. Thus he escaped
the prison.

• When the invigilator was not found in the hospital they went to the residence of
Rev. S. McLeery only to find him ’bound and gagged in his study in Broad Street”.
He has been there, since 8.15 a.m. Now everything was clear to the Governor.

• Evan escaped the prison the 4th time. But by taking the hint from the question
paper the Governor reached the hotel where Evans was and captured him and
came to know how he planned his escape and said that his game was over. Evans
surrenders himself to the Governor.

• The Governor tells Evan they would meet soon.

• The moment they are rid of the Governor, the so called prison officer-a friend of
Evans-unlocks the handcuffs and asks the driver to move fast and Evans tells him
to turn to Newbury. Evans, thus, has the last laugh.

SOLVED QUESTIONS
SHORT ANSWER QUESTIONS
1. What kind of a person was Evans?

116
Evans was a ‘Kleptomaniac’ and had broken jail thrice. He was a master planner and was
very sociable. He knew how to keep intimate contacts with people. In the words of the
Governor, he was a pleasant sort of chap with no record of violence.
2. Do you think Evans’ statement, ‘I may surprise everybody,” has some special
significance?
Evans seems to be telling his teacher that he may surprise everybody by doing well in
the exam, but in reality it is a forewarning that he is going to jolt everybody by his
master-minded perfect escape-plan.
3. Who were the two visitors Evans received in the morning of the day of his exam?
The two visitors --Mr. Jackson, the senior prison officer of the prison’s D Wing-- man
called Stephens, who had been only recently recruited.
4. What made Evans clip his hair short?
Evans’ escape prison-- duplicate McLeery (invigilate during the O-level German exam)
had short hair. In order to give a practical shape to their plan Evans’ hair had to look like
McLeery’s, hence Evans clipped them short.
5. Why did the Governor instruct Jackson to search McLeery?
The Governor asked Jackson to search McLeery, the invigilator, just in case he has
brought something unwittingly which might prove to be a weapon that Evans could use
and try escaping from prison.
6 .Why did Evans drape a blanket round his shoulder? What did Stephens think about it?
In between intervals of Stephens’ peeping into the cell, Evans was changing into the
Parson’s dress to look like McLeery. So, in order to conceal his effort to keep them in
place, Evans draped a blanket round his shoulder. Stephens was misled into believing
that Evans was feeling cold.
7. In spite of strict vigilance, how did Evans’ friend manage to give the material for
disguise in the cell?
Despite all vigilance, Evans’ friend disguised as McLeery, the invigilator, managed to
smuggle the disguised material into the cell. He came wearing two parson’s dresses
with black fronts and collars. Apart from it he also brought an extra pair of spectacles.
All this was passed on to Evans when Stephens’ vigilant eyes were away from the peep-
hole.
LONG ANSWER QUESTION
1. What were the precautions taken for the smooth conduct of the examination?
Since Evans had already escaped from the jail on three earlier occasions, there was
always a lurking fear that he might make another attempt to escape. Therefore all
possible precautions were taken to see that the O-level German examination arranged
in the prison did not provide him with any means of escape. The Governor personally
monitored all security arrangements and heavily guarded the Recreation Block from
where he expected the prisoner to make another break. Evans cell was thoroughly
checked by Jackson to ward off the possibility of the presence of an incriminating
material which might hamper the smooth conduct of the examination. His nail-scissors,
nail-file and razor were taken away; and to keep a strict watch on the activities of the
cell during the examination, the Governor got it bugged. A police officer Stephens was

117
posted to keep a constant vigil on his activities. The invigilator, too was frisked to make
sure that he carried no objectionable material with him.
QUESTIONS FOR PRACTICE
SHORT ANSWER QUESTIONS
1. What different queries does the Secretary of the Examinations Board make from the
Governor before conducting the examination for Evans and why?
2. Who do you think made a call regarding a correction in the question paper? What did
it really want to convey?
3. Who is Carter? What does the Governor want him to go and why?
4. How did the Governor manage to reach Evans in the hotel?
LONG ANSWER QUESTIONS
1. What impression do you form of ‘Evans the Break’?
2. Comment on the ending of the play ‘Evan Tries An O-Level.
3. How far do you agree with the observation: “He was just another good-for-a-giggle,
gullible governor that was all”?
4. Do you think the title ‘Evans Tries an O-Level’ is appropriate? Give reasons in support
of your answer.

MEMORIES OF CHILDHOOD by ZITKALA-SA AND BAMA


GIST OF THE LESSON
PART –I
- The first part deals with the account of Simmons, An American Indian, who
fought against the prejudices of the society against American Indians.

- She describes her experiences on her first day at the Carlisle Indian School.

- The customs and rules of the place were strange and new to her.

- She was forced to wear clothes that were considered undignified in her
culture

- At breakfast, she was embarrassed as she did not know the routine of the
place.

- When she comes to know that they were planning to cut her hair, she
protests by hiding under the bed, even though she knew it was futile. In her
culture, it was the cowards whose hair was shingled.

- She felt like an animal driven by a herder.

PART – II
- The second part is an excerpt from the autobiography ‘Karukku’ by Bama – a
Tamil Dalit.

118
- She was in her third grade when she becomes aware of the indignities that
the lower caste people face.

- She happens to see an elderly person from her community abase himself in
front of a higher caste person as he was not supposed to touch the food that
he was ordered to fetch for the landlord.

- Later, her brother explains to her that the incident was not at all funny as she
initially thought, but very pathetic. The people from the lower caste were
treated as untouchables.

- She was deeply saddened and decided to study hard to overcome


discrimination.

Short Answer Questions with Sample Answers


1. What were the indignities that the new girls were subjected to at Carlisle Indian
School?

The girls were scrutinized thoroughly and supervised by a grey-haired woman. They
were made to wear tight fitting immodest clothes and stiff shoes. During breakfast a
systematic and regimental discipline was observed. The girls with long hair had to get
them shingled and they had to submit to the authorities who were strong, unfeeling and
cruel.

2. On learning that her long hair would be cut the author decided to struggle first.
What does this tell us about the author?

The author knows that she could never prevail against the authorities, yet she struggles
against the injustice. Her mother had told her that only cowards had their hair shingled
and she firmly believed that she was not one. To prove her point as well as raise her
voice against the indignity, she struggles.

3. Why did Bama take half hour to an hour to cover the distance to her home that
would normally take only ten minutes?

Bama would dawdle along, watching all the entertaining novelties and oddities in the
streets. She would gaze at the shops and the bazaar enjoying the street scenes and so
she would take at least an hour to reach home.

4. What was the incident that made Bama laugh as well as feel so provoked and
angry?

119
Bama saw an elderly man of her street carrying a packet of ‘Vadais’ by the strings and
walking gingerly, holding the parcel away from his body. Bama found his manner of
carrying the parcel very funny. But her brother explains to her the higher caste people
believed that if the lower caste people touched the parcel it would be polluted. That’s
why the elder was carrying it in that manner. This provokes and angers Bama.

Long Answer Questions:


1. Had Bama not been guided properly by her elder brother regarding
untouchability, she would have grown up into a complex-torn woman. Do you
agree? Justify.

Value points:
Annan an understanding and considerate elder brother – guides her properly – explains
the social stigma of untouchability – Elder carrying Vadai not comical but pathetic –
victim of social prejudice – Bama angry and provoked – Frustration might have lead to
open and futile revolt – Timely advice of Annan guides her in right direction – He believe
that people of their community should study and outshine others to earn respect of
society. Bama follows his timely advice and grows up to be a balanced and well
respected individual of the society.
Short Answer Questions for Practice:
1. Zitkala-Sa’s friend Judewin tells her that it is better to submit to authority. What
kind of a person do you think Judewin was?

2. Why did Zitkala-Sa start crying in the dining hall?

3. Why does the author feel ‘Spirit tore itself in struggling for its lost freedom, all
was useless’?

4. ‘Now I was only one of many little animals driven by a herder!’ Explain.

5. When and how did Bama come to know of the discrimination faced by the
marginalized people?

Long Answer questions for Practice:


1. Bama’s brother’s right advice at the right time helped her progress in academics
thereby throwing away the chain of untouchability of which dalits like her are
victims of. Justify with reference to ‘Memories of Childhood’

2. Both Bama and Zitkala-Sa are victims of discrimination that is practiced in the
society. What kind of experience did both of them go through?

3. What are the similarities in the lives of Bama and Zitkal though they belong to
different countries?

120
4. Describe how Zitkala tried in vain to save her hair from being cut. Why did she
want to save her hair?

SAMPLE PAPER (SOLVED)


BLUE PRINT
TOTAL MARKS – 100
Q. No. Section Type of Marks Total Testing
Question marks objectives
Reading 20
Q. 1 1.1 (i)V SA 1M 9 Comprehension
(ii) VSA 1M Comprehension
(iii) SA 2M Comprehension
(iv) SA 2M Comprehension
(v) VSA 1M Comprehension
(vi)SA 2M Comprehension
1.2 (a) VSA 1M 3 Vocabulary testing
(b) VSA 1M Vocabulary testing
(c) VSA 1M Vocabulary testing
Q. 2 2.1 Note Making 5M 5 Study Skills (Comprehensing
and Note Making)
2.2 Summary 3M 3 Summary skills
Writing
Writing 35
Q.3. 1st option Short writing 5M 5 Details, organization,
or skill Or fluency and coherence
2nd option Or or
Short writing Details,organization,fluency and
skill coherence
5M
Q.4. 1st option Long Writing 10M 10 Presenting factual details,
Or Task formal,fluency and coherence
2nd option (Newspaper Or
Report) Presenting factual details,
Or informal, fluency and coherence
Long Writing 10M
Task
(Magazine
Report)
Q.5. 1st option Letter 10M 10 Formatting , Organization,
Or Writing coherence, fluency,
2nd option (Long Writing Resume
Task) Or
Or 10M Or

121
Letter Formatting , Organization,
Writing coherence, fluency,
(Long Writing
Task)

Q6. 1st Option Article (Long 10M 10 Organization, coherence,


Or Writing Task) fluency
nd
2 Option Or Or Or
Speech (Long 10M
Writing Task) Organization, coherence,
fluency

TEXT-BOOKS 45

Q7 1st option (a) VSA 1M 4 Poetry appreciation


(b) VSA 1M Interpretation
Or (c) VSA 1M Interpretation
2nd option (d) VSA 1M Or Or
OR OR Poetry appreciation
(a) VSA 1M 4 Interpretation
(b) VSA 1M Interpretation
(c) VSA 1M
(d) VSA 1M

Q8 Any three (a) SA 2M 6 Comprehension , interpretation,


(b) SA 2M understanding
(c) SA 2M
(d) SA 2M
Q9 All five (a) SA 2M 10 Comprehension , interpretation,
(b) SA 2M understanding
(c) SA 2M
(d) SA 2M
(e) Sa 2M
Q10 1st option Long Answer 10M 10 Content, Organisation, fluency,
Or Or Or coherence, understanding
2nd Option Long Answer 10M
Q11 1st option Long Answer 7M 7 Content, Organisation, fluency,
Or Or Or coherence, understanding
2nd option Long Answer 7M
Q12 All four (a) SA 2M 8 Understanding , interpretation

122
(b) SA 2M
(c) SA 2M
(d) SA 2M

SAMPLE QUESTION PAPER (SOLVED)


SECTION A – READING (20 MARKS)
1. Read the passage given below and answer the questions that follow:
(12 marks)
1. Rabindranath Tagore raised the stature of our country in the eyes of the world. A
versatile genius, a literary artist, an educator, a composer, a singer, an actor, Tagore had
all gifts of Nature and fortune in his favors. Born in a renowned Hindu family, Tagore
pleaded not only for concord with the past but also for freedom from the past. All
healthy growth needs continuity and change. We are not the past but also for freedom
from the past. All healthy growth needs continuity and change. We are not free unless
our minds are liberated from dead forms, tyrannical restrictions and crippling social
habits. Tagore condemned the corruption of many of our social practices. He believed
that the essence of life was perpetual renewal and rededication to self development.
2. Tagore did not live in an ivory tower. He led a procession in 1905 through the streets
of Calcutta singing his song, “Are you so mighty as to cut as under the bond forged by
Providence?’’ Millions of voices have sung the National Anthem ‘Jana Gana Mana,’
calling upon us to nourish the unity of our country and be devoted to it. He was not only
a playwright but a novelist and a story teller, a nationalist and an internationalist. As if
these activities were not enough, he turned towards painting in the end of life. He
rejected traditional canons and experimented with new forms and color compositions.
We honor him not only for this many sided genius but also for the guidance of his life
and work in this troubled world. Tagore’s mission was one of reconciliation between
East and West in a spirit of understanding and mutual enlightenment. For India unity is
truth and division is evil. The poet’s name is symbolic of the light of the day. The sun
which dispels the mist of darkness and the clouds of suspicion restores health to the
human system.
3. Though his work was rooted in Indian soil, his mind ranged over the wide world and
hence had a universal appeal. Tagore’s writings have been translated into many
languages, but even the best translations do not bring out the music and the melody or
the force of the original. As Tagore was born at a difficult stage when India was in a
revolutionary mood, he participated in the movement revolting against social, political
and religious institutions. He was all the time convinced of the validity and vitality of the
fundamental ideals set forth by the seers and saints of India. Tagore’s philosophy was
one of wholeness and unity. For Tagore, God, Man and Nature are bound together in
single unity. He was not a dreamer or a visionary. He kept constant vigil over the world.
He was a great sentinel as Gandhi called him. The moral health of a nation depended on
the inspiration the people derived from their poets and artists. Asceticism for Tagore
meant self control and not abstention from world activities. Very early in his life when

123
he was seventeen, he had the need to control his emotions. He was not an unworldly
saint. He had tough earthly quality. The ideals of social life, economic pursuits, and the
enjoyment of beauty should be cultivated equally. Water surrounds the lotus flower but
does not wet its petals. Even so, human beings should work in this world without being
affected by it. For Tagore, as for Gandhi, the measure of man’s greatness in whom there
was a happy blend of contemplation and action. For Tagore, as for Gandhi, the measure
of man’s greatness is not his material passions but the truth in him which is universal.
His voice was the conscience of our age. He bequeathed to the country and the world a
life which had no littleness about it.
a) On the basis of your reading of the passage, answer the following questions:
i) Tagore was a versatile genius. How? (1)
ii) What is the message given in the National Anthem composed by Tagore? (1)
iii) What was Tagore’s mission in life? (2)
iv) What does the poet’s name symbolize? (2)
v) Give an example to prove that his works had a universal appeal.(1)
vi) How should human beings work in this world? (2)
b) Pick out words from the passage that mean:
i) Never ceasing (Para 1)
ii) Living a life of austerity (Para 3)
iii) Left behind (Para 3)

2. Read the passage given below and answer the questions that follow: (8
marks)
Salt, a miraculous gift of nature, is on of the most useful and amazing minerals on Earth
derived from the sea and rocks. Do you know that it is the only rock the humans can
eat?
Salt has seasoned our history, language and food, besides making nutritious foods more
palatable. Used in all bakery products, prepared foods, sauces, soups, spices, cereals,
dairy foods, meats, poultry. It is also an extraordinary effective food preservative,
retarding the growth of spoilage by micro-organisms and making food storage possible
long before refrigeration.
How much salt is necessary for human consumption? Medical experts agree that
everyone should practice some reasonable ‘moderation’ in salt consumption. For the
average person, a moderate amount might run from 4 to 10 gm a day, or roughly half to
one and one-third teaspoons. The equivalent of one to two gm of this salt allowance
would come from the natural sodium in food. The rest would be added in processing,
preparation or at the table.
Common salt, a chloride of sodium, is chemically represented by the symbol NaCl. The
human body has a continual need for salt. Sodium chloride or the common salt is 39
percent sodium and 61 percent chloride. Forming a solution in the body, these two
components separate into sodium and chloride ions, each with a different task. Chloride
maintains the balance of water between the living cell and its environment, plays part
indigestion, and pairs with sodium to maintain the blood’s acid-base balance, critical for
life. Sodium assists in regulating the volume of blood and blood pressure. It facilitates

124
the transmission of nerve impulses and is necessary for heart and muscle contractions.
Although the popular conception is that salt is a flavor enhancer, a recent American
study suggests that it functions as a flavor filter on food, selectively enhancing and
suppressing various tastes. Other studies showed that the use of salt suppresses the
bitter taste of dark green vegetables like bitter gaurs.
Salt’s functions in the body are already elucidated. Deficiency signs include lethargy,
dizziness, cramps and palpitation. In women excessive salt intake promotes fluid
retardation and can cause breast pain
But what the good salt can do, in the right dose, is unequalled. Snorers should try
spraying their nose with a salt water to moisten mucous membrane and make it easier
to breathe. To invigorate the body when tired or to remove dead skin cells, rub a
handful of salt all over your body before having a bath. Salt baths encourage
detoxification and greatly help muscle and joint pains. Add a pound of salt to a
comfortably hot bath and lie in it for 20 minutes, add hot water as it cools. Wrap up in
cotton towels and get into a warm bed. You should perspire freely, sleep well and feel
much better in the morning. Remove all the congestion in your throat by saline gargles.
The recommendation that no one should exclude salt totally from the diet is awfully
wrong. Scientists are of the view that salt is an invisible killer and, therefore, a health
hazard. As new evidence piles up; alarm bells have started ringing. And the next battle
might just be against salt!
Specialists are convinced that a diet high in salt causes high blood pressure, a disorder
that afflicts one-third of people above the age of 60. Moreover, this is a risk factor for
two big killers—coronary heart disease and stroke.

a) On the basis of your reading of the passage carefully, make notes on it, in points only
using headings and sub-headings. Use recognizable abbreviations and a format you
consider suitable. Also supply an appropriate title to it.

b) Make a summary of the above passage in about 80 words:

SECTION B – WRITING (35 MARKS)

3. You are Sumesh of No. 24 Ashoknagar Road, Kolkata. You desire to sell your car. Draft
an advertisement for the newspaper. (5) Marks)
Or
Your sister is getting married and you have been asked to draft a formal invitation. Draft
an invitation on behalf of your parents.

4. You are L. Kullajit. You have participated in CBSE Intel Science Quiz for 2011 at
Ambani Auditorium. Write a report of the same to be published in the newspaper.
(10 marks)
Or

125
You are Oisha / Omesh. Your school has organized an Exhibition-cum-Sale of the items
made by the students in their work-experience classes. You had an excellent and
overwhelming response from the parents and the visitors. The proceeds of the sale have
been donated by your school in a function to ‘Helpline India; an organization for
supporting the orphans. Make a report of the same in 125 words to be published in your
school bulletin/magazine.

5. You are an educationist and feel strongly about the flaws in the existing system of
education particularly examination. You feel that the system neglects the talents and
skills of students and is restricted to classroom teaching. Write a letter to the editor of
an English newspaper making a case for education beyond classroom teaching. Sign
yourself as Dr. Manoj Lamba. (10)
Or
Read the newspaper clipping and apply for the post considering you to be Ragini/Raju.

D.A.V. PUBLIC SCHOOL, CHENNAI

Requires a competent PGT in English E.Q. M.A from a recognized University.


Minimum 5 years experience of teaching + 2 classes, fluency in spoken English is a
pre-requisite. Pay scale as per Govt. Grade. Apply within seven days with complete
bio-data to Richard Crashaw Principal

6. In monsoon season, outbreak of malaria and dengue is quite common. Your science
teacher has asked you to write an article for the school magazine on the topic “The Killer
Disease-Dengue”. Write the article in about 200 words.
(10 marks)
Or
31st of May is being celebrated by your school as “World No Tobacco Day”. Your teacher
has asked you to prepare a speech on the hazards of smoking. Write your speech.

SECTION C – LITERATURE (45 MARKS)

7. Read the extracts given below and answer the questions that follow: (4
marks)
“Those who prepare green wars,
wars with gas, wars with fire,
victory with no survivors, would put on clean clothes
and walk about with their brothers
in the shade, doing nothing.”
i) Whom does ‘those’ refer to here? (1)
ii) What does the poet mean by ‘victory with no survivors’? (1)

126
iii) What lesson will man learn when he just walks about with his brothers doing
nothing? (1)
iv) Name the poem and the poet. (1)
Or
“We have imagined for the mighty dead,
All lovely tales that we have heard or read;
An endless fountain of immortal drink,
Pouring unto us from the heaven’s brink.”

i) Who are the ‘mighty dead’? (1)


ii) Explain ‘lovely tales have we heard or read? (1)
iii) Explain: ‘Pouring unto us from the heaven’s brink’? (1)
iv) Name the poem and the poet. (1)

8. Answer any three of the following questions in about 30-40 words each:
(3x2=6)

i) What is the significance of the images, ‘sprinting trees’ and merry children spilling out
of their homes’ in the poem “My Mother at Sixty-six”?
ii) What does the poet mean by saying, ‘let their tongues run naked into books’ in the
poem “An Elementary School Classroom in a Slum”?
iii) What are the different reasons for which the cars halt at the roadside stand in the
poem “A Roadside Stand”?
iv) Explain: ‘They pace in sleek chivalric certainty’ in the poem “Aunt Jennifer’s Tiger?
9. Answer the following questions in about 30-40 words each: (5x2=10)
i) How did the Champaran episode change the plight of the peasants?
ii) How does Mukesh’s grandmother view the family occupation of bangle making and
its poverty?
iii) Why didn’t M.Hamel scold Franz for not learning his lesson?
iv) What advantage did the office boy think Subbu had?
v) What are the drawbacks of an interview?
10. Answer any one of the following in about 125-150 words: (10 Marks)
What made the peddler finally change his ways? What is the message conveyed?
Or
How did the instructor ‘build a swimmer’ out of Douglas?
11. Answer any one of the following in about 125-150 words: (7 Marks)
What were the precautions taken for the smooth conduct of the examination with
reference to the lesson Evan tries an O-Level?
Or
Justify that Bama has a keen sense of observation and is capable of giving the minutest
of details. (Memories of Childhood)

12. Answer the following questions in about 30-40 words each: (4x2=8)
a) Why did Maharaja order the dewan to double the land tax?

127
b) In what way can the further depletion of ozone layer disrupt the entire food chain of
the South Seas?
c) Give two reasons why Dr. Sadao was not sent abroad with the Japanese troops?
d) What was Roger Skunk’s problem? How did he get rid of it?

Answer Key

1. i) Tagore was a great literary artist, an educator, a music composer, a singer, an


actor hence versatile. (Any two) ½+½
ii) It calls upon us to nourish unity of our country and be devoted to it. (1)
iii His mission was to reconcile East and West in a spirit of understanding and mutual
enlightenment. (2)
iv) His name symbolizes the light of the day which dispels the most of darkness, the
cloud of suspicion and restores health to the human system. (2)
v) Translation of his works into many languages proves the universal appeal of his work.
(1)
vi) Humans should work without being affected by ideals of social life, economic
pursuits and beauties of nature. (2)
b) i) perpetual ii) asterism iii) bequeathed. (3x1)
2.a)
SALT: A MIRACULOUS GIFT OF NATURE
1. Salt: Useful & Amazing mineral
a) Derived from sea & rock (only edible rock)
b) Used in various foods
(i) bakery products
(ii) prepared foods/diary foods
(iii) sauces, soups, cereals, spices
(iv) meats & poultry
c) Extraordinary food preservative
(i) retards growth of spoilage
(ii) makes food storage possible
2. Chemistry of Salt
a) Symbol : NaCl (39% Sodium,61% Clorine)
b) Chloride of Sodium
c) Imp. Of Cl
(i) maintains water balance in cells
(ii) helps in digestion
(iii) maintains blood’s acid-base balance (critical for life)
d) Imp. Of Na
(i) regulates blood vol.& pressure
(ii) facilitates transmission of nerve impulses
(iii) necessary for heart & muscle contractions
3. Uses of Salt
a) Flavour enhancer/filter

128
b) Facilitates breathing in snorers
c) Invigorates skin by removing dead skin
d) Salt baths detoxify body, relieve muscle/joint pains
e) Saline gargles: remove throat congestion
4. Harmful Effects of Salt
a) ideal consumption : 4 to 10 gm a day
b) def. causes
(i) lethargy
(ii) dizziness
(iii) cramps
(iv) palpitation
c) Excessive Salt intake causes
(i) Retardation of fluids and breast pain (in women)
(ii) high blood pressure
(iii) coronary heart diseases
(iv) stroke

Key to Abbreviations
Imp. - importance
& - and
vol. - volume
def. - deficiency
Marking scheme:
Title (1)
Abbreviations +Key (1)
Content (3)
b) Salt, a miraculous gift of nature is an essential ingredient in food. It is an excellent
preservative, flavor enhancer and detoxificant. It is used in making and preserving a
majority of foods and food products. Common Salt (NaCl) is 39% sodium and 61%
chloride. It maintains water balance, helps in digestion, relieves muscle and joint pain,
removes throat congestion, and facilitates nerve impulses and muscle contradictions. It
should be taken in a balanced way (4 to-10gms everyday) because both its deficiency
and excess can cause a number of hazardous diseases including heart disease and
stroke.
Marking Scheme:
Content (2) Expression (1)

129
3. CAR FOR SALE
2004 Honda Accord, white colour,
2.2 Lt automatic transmission,
equipped with leather upholstery,
CD changer, alloy wheels, sunroof,
in good condition. Please contact : 98782-66340

Marking Scheme:
Format (1)
Content (2)
Expression (2)
Or

Mrs. & Mr. S.S. Ahluwalia


(No 273 Block IV Quality Salt Lake, Kolkata)
solicit your gracious presence on the auspicious occasion
of the marriage of their daughter
MEHER
With
MUKESH
(son of Mrs. and Mr. L. B. Kashyap)
at 10.00 A.M
on 2nd August 2011
at INVITATION BANQUET HALL
Model Town, Kolkata.

RSVP With compliments from


27149456, 9876543210 Sameer Ahluwalia

Marking Scheme:
Format (1)
Content (2)
Expression (2)

4. CBSE INTEL SCIENCE QUIZ COMPETITION ORGANISED


By L. Kullajit
The CBSE Intel Science quiz for 2011 was held at the Ambani Auditorium on 15th July
2011. It is organized every year by the Central Board of Secondary Education to promote

130
interest in science among students. Eight teams competed in the National semi final
round and four teams made it to the National finals. The four teams selected for the
final rounds were KV AFS Yelahanka, Bangalore; St. George’s School Kolkata, Sophia
school Kota, DAV model School, Chennai.

In the final rounds of the quiz, in a highly charged atmosphere and in nail biting
competition, the four selected teams battled it out for the National Champion and
finally it was St. George’s School, Kolkata who were declared National Champions of
CBSE Intel Science Quiz 2011.

The winning team was awarded a cash prize of Rs. 20,000. The second and third prizes,
which were won by Sophia school Kota, DAV model school Chennai respectively, were
awarded Rs.12,000 and Rs.7500.

The event ended with Vineet Joshi, Secretary CBSE, giving away the prizes. In his
address, Josh lauded the efforts of the participants and urged the young ones to be
good citizens. Around 70 students from CBSE affiliated school, from across the country
participated in the competition. (10)
Marking Scheme:
Title + Name of the Reporter (1)
Content (4)
Expression: Fluency 2 ½ , Accuracy 2 ½
Or

Exhibition-cum-Sale
( A report by Oisha)
Bangalore: Dec. 20

Yesterday the school had organized an Exhibition-cum-Sale of items made by the


students during their work-experience classes. It was inaugurated by Mr. Bhoop Singh,
the Chairman of the Vidyalaya at 9.30 a.m. He was overwhelmed with joy and was
impressed to see the creativity of the students. He also appreciated the work-
experience teacher who had guided the students so well and for tapping and grooming
their hidden talents.
By 10.00 a.m. visitors flocked the Vidyalaya. Within no time the exhibition hall was
flooded with people. The response of the parents and visitors was amazing.
It was an eye-opener for the visitors to acknowledge that so much was being done in
school and their children were exposed to various activities which was otherwise not
possible for them to learn. The sale too was satisfactory and the feeling of enthusiasm
prevailed, inspiring the students to do better in future also.

The children’s expertise was talked about by one and all and every one witnessed the
appreciation. The parents left with a positive note wishing the school the very best.
Marking Scheme:

131
Title + Name of the Reporter (1)
Content (4)
Expression: Fluency 2 ½ , Accuracy 2 ½

5.
27 University Enclave
Bhiwani
7th December, 2011
The Editor
The Hindustan Times
New Delhi
Sir
Education beyond Classroom Teaching
Education has been reduced to preparing the students for examinations. Rote memory
and recall are the skills required for passing the school examination whereas in life many
more skills are required for meaningful living.

Most schools seem to believe that classroom education is the only stepping stone to
success. In the process, they overlook other talents and skills of students like music,
painting and sports and qualities of head and heart such as courage, chivalry,
personality, wit and friendship. What are these schools doing to inculcate these qualities
in children? Instead of pushing and prodding the children with burden and leaving them
dazed and flabbergasted, they should aim for the overall development of children.

The main problem is our inability to live peacefully with others and our grab mentality.
The schools have to teach the students the age-old values of brotherhood, tolerance,
team spirit, satisfaction and sacrifice. Extra-curricular activities have a vital role to play
in this regard.

Yours truly
Dr Manoj Lamba

Marking Scheme:
Format (2)
Content (4)
Accuracy (2)
Fluency (2) (10)
Or
B-8 Green Field
Chennai
7th July 2011
The Principal
D.A.V. Public School
Chennai

132
Application for the post of P.G.T. English

Sir
In response to your advertisement in “The Tribune” dated 5th July 2011, I wish to offer
my services for the post of P.G.T. in English in your prestigious institution.

I am a hardworking and honest person, passionate about the noble vocation of teaching.
I wish to make a difference in the lives of people through education.

I am enclosing my bio-data herewith for your kind perusal; I shall be available for an
interview on any day of your convenience.

If selected, I assure you that I shall render my services with utmost devotion and
sincerity to your satisfaction.

Yours faithfully
Ragini
(Ragini)
Encl.: Bio-Data and Testimonials

BIO-DATA

Name : Ragini Sharma


Husband’s Name : Raj Sharma
Address : 8,Green Field, Chennai
Age : 32 years
Marital Status : Married
Nationality
Academic Qualification :
i) B.A from Annamalai University, Annamalai (68% marks)
ii) M.A. (English) from Annamalai University, Annamalainagar (62%
marks)
iii) B.Ed from Annamalai University, Annamalainagar (70% marks)
Experience :
i) Worked in St Johns School, Chennai for 7 Years.
ii) Currently working as a P.G.T. English in Tagore Public School, Chennai.

Scholarships/Prizes won :
i) Won Gold Medal for highest score Linguistics
ii) Have been a scholarship holder from Class X to Class XII.
Hobbies : Reading, Traveling
Travels : U.S.A for 6 months, Course in Spoken English
Salary Drawn : 13,000/- p.m.
Names and Address of References : i) Mr. C.K. Mehta, Principal

133
Sanjay College for Women, Chennai
Phone : 2724874
ii)Dr. Sunil Kumar
12, M.G, Road, Chennai
Phone : 2462359
Salary Expected : Govt. grades with minimum 2 additional Increments
Marking Scheme:
Format (2) Content (4) Accuracy (2) Fluency (2)

6. THE KILLER DISEASE—DENGUE

Dengue fever spreads due to the mosquitoes. Mosquitoes breed in the stagnant water
found in places like water coolers and roadsides. When these mosquitoes bite people,
the fever may lead to dengue. It is highly infectious.

If a person is down with dengue fever he should immediately contact a doctor so that
the fever can be prevented from spreading at the right time. Dengue fever if not treated
properly and at the right time can even be fatal. We can prevent the outbreak of dengue
by taking some measures.
Water in the coolers should not be allowed to stagnate for long and must be changed
regularly.

One or two spoons of Kerosene can be put in stagnant water so that mosquitoes cannot
breed in it. Water should not be allowed to stagnate outside the house. One should
wear clothes that cover the arms and legs both and insecticides should be sprayed
regularly. To prevent mosquito bites, mosquito repellents and mosquito nets at night
should be used.

If a person finds any mosquito-breeding place, in his locality or in nearby places, he


should immediately contact the municipal corporation so that they will take appropriate
measures to eliminate them.

Even after taking these precautions, if one catches the disease, he should be taken to
the doctor immediately and should be treated properly. Remember “Prevention is
better than cure.” Prevention of Dengue is as much in the hands of people as in the
hands of the municipality.

(Aishwarya Singh)
Marking Scheme:
Format (2)
Content (4)
Accuracy (2)
Fluency (2)
Or

134
QUIT SMOKING NOT LIFE

Respected Principal, worthy teachers and my dear friends, it is a matter of great


privilege for me that I have got an opportunity to speak on the occasion of “World No
Tobacco Day”. I am going to speak on the harmful effects of smoking.

Do you know that 8 to 10 lakh people die every year due to tobacco related diseases
that is one person every 10 seconds? 50% of all smokers die at the age of 20-25 years
mainly due to lung cancer and heart diseases. All products containing tobacco cause
disease and death. If you think passive smoking is less harmful, you are wrong. Even
passive smokers are prone to bronchitis, pneumonia, asthma and reduced rat of lung
growth. Regular smokers run 20-25 times higher risk of developing lung cancer and 2-3
times higher risk of heart attack and sudden death.

Yet, people continue to smoke. In fact thousands take to the habit every day. Why? Well
the major reason is the nicotine that has an addictive effect and tends to relax or
energies nervous system. Another reason is the smokers consider smoking a macho or
manly thing to do. Cigarette companies through their alluring advertising promote
smoking as something heroic and adventurous.

But medical research has conclusively established the hazards of smoking. It adversely
affects our brain, ENT systems, heart, chest, liver, stomach, kidney, bladder and
reproductive system. Smoking ultimately leads to death.

Fortunately, people can kick this menace from their lives. All they need is strong
determination and counseling.

Friends, let’s take a pledge that we will never smoke in our life.

Thank you.
Marking Scheme:
Format (2)
Content (4)
Accuracy (2)
Fluency (2)

7. i) ‘Those’ refers to the politician, the statesmen and the scientists who are involved in
initiating and aggravating wars. (1)
ii) Man’s activities are indeed heading him towards his doom. The war which man is
waging against nature will one day convert this ‘living planet’ into a dead one and
human beings will become an extinct species. The war may be a victory of man’s
scientific knowledge, but there would be no survivors to celebrate this victory.
(1)

135
iii) Man will realize the strength of humanity and become conscious of universal
brotherhood. His destructive activities would come to an end. (1)
iv) The name of the poem is ‘Keeping Quiet’ and the poet is Pablo Neruda. (1)
Or
i) The ‘mighty dead’ refers to the great men and women who are remembered for their
glorious deeds. (1)
ii) It refers to the tales or stories of great personalities who have inspired and motivated
the reader and the hearer. (1)
ii) Beauty is the greatest gift of God to man which has been showered upon us from the
heavens above. This beauty is eternal and everlasting, in whose glory men on earth bask
and derive their perennial source of joy and happiness.
(1)
iv) The name of the poem is ‘A Thing of Beauty’ and the poet is John Keats.
(1)
8. i) Both sprinting trees and merry children are happy and young. They present a
contrast to the mother’s pain and old age and the poet’s worry and fear. They represent
youth, vigour and spring whereas the mother is old, decaying and frail. (any two)
(2)
ii) It is in education that the poet sees hope for the slum children. He strongly feels that
the lives of these children will remain dark, narrow and unfulfilled unless they are
educated. (2)
iii) The city dwellers halt their cars at the roadside stand at times to plough the grass
and use the yard to back and turn around. At other times, they stop to enquire for the
way and sometimes ironically to ask if they could sell them a gallon of gas.
(2)
iv)The movement of the tigers is sleek, stealthy, sure, majestic and elegant. They are
sure of their purpose. (2)
9.a) The Champaran episode released the peasants from the mortal fear of British
landlords and made them aware of their rights. Apart from getting back 25% of the
compensation money, they also learnt ‘courage’. Within a few years, the British planters
abandoned their estates, which reverted to the peasants.
(2)
b) Mukesh’s grandmother views bangle making as the destiny of her family. Her
husband’s blindness their misfortune and impoverished condition, she feels, are
ordained by destiny. Years of suffering makes her accept everything in the name of
karma or fate. (2)
c) M. Hamel did not scold Franz because he wanted him to realize his mistake of always
putting off his lessons for tomorrow. Such was the nature of all Alsacians and now the
worst had hit them. They could no longer learn their own language. Though they were
Frenchmen, yet they could neither speak nor write their language.
(2)
d) Subbu, in no way was superior to the office boy in talent or education. It was by the
dint of his birth in a Brahmin family that he got a lead over him. His ‘birth” helped him

136
get better openings, opportunities and exposure. Thus he had an edge over the office
boy. (2)
e) Interview is an encroachment upon an individual’s privacy. Most of the times, it is an
ordeal for the celebrity. At times it is condemned for its ‘diminishing effect’; at times it is
dreaded because it ‘lionizes’ the subject. Sometimes it wounds and at other times it
robs one of a part of oneself.
(2)
10. The peddler had been living a despicable life of poverty, despair and frustration
without ever coming across any soul to understand, sympathize, love and guide him.
Neither the crofter’s hospitality nor the ironmaster’s invitation to the manor house
made any impact on him. In fact he repaid the crofter by stealing his earnings and the
ironmaster by giving a piece of his mind when the latter talked of taking the matter to
the sheriff.

However Edla’s warmth, understanding and genuineness touched him. When the girl
treated him like a Captain, he spontaneously behaved like a real Captain. He left a
rattrap as a Christmas gift for Edla and enclosed a letter of thanks and confession in it.
Leaving behind the stolen money to be restored to the owner, he redeemed himself
from his dishonest ways and emerged an altogether transformed person.

The message is that man is basically good. Goodness even in a criminal can be
awakened through love and understanding. (10)
Marking Scheme:
Content (5)
Expression: Accuracy + Fluency (2 ½ + 2 ½ )
Or
Douglas was determined to learn swimming to get over his fear of water. Hence he
engaged a professional instructor who well understood the gravity of Douglas problem
and the intensity of the terror that gripped his heart. Thus rather than teaching him
swimming in one go, the instructor built a swimmer out of him bit by bit.

First of all, to make Douglas get over the terror of drowning, the instructor put a belt
around his waist and attached it securely to a pulley that ran on an overhead cable. For
almost three months, he was repeatedly made to go back and forth across the pool.
Next he was taught to put his face under water and exhale through his nose and inhale.
He repeated the exercise hundreds of times.

Later, for weeks together he was made to kick with his legs at the side of pool. Initially
his legs refused but they gradually relaxed and finally he could command them. Thus
inch by inch a swimmer was built.
Marking Scheme:
Content (5)
Expression: Accuracy + Fluency (2 ½ + 2 ½ )

137
11. Since Evans had already escaped from the jail on three earlier occasions, there was
always a lurking fear that he might make another attempt to escape. Therefore all
possible precautions were taken to see that the O-level German examination arranged
in the prison did not provide him with any means of escape.

The Governor personally monitored all security arrangements and heavily guarded the
Recreation Block from where he expected the prisoner to make another break. Evans
cell was thoroughly checked by Jackson to ward off the possibility of the presence of an
incriminating material which might hamper the smooth conduct of the examination. His
nail-scissors, nail-file and razor were taken away; and to keep a strict watch on the
activities of the cell during the examination, the Governor got it bugged.

A police officer, Stephens was posted to keep a constant vigil on his activities. The
invigilator too was frisked to make sure that he carried no objectionable material with
him. But in spite of all these elaborate preparation Evans escaped.
(7)

Marking Scheme:
Content (4)
Expression : Accuracy + Fluency (1 ½ + 1 ½ )
Or
Bama, a student of class three has a very keen and observant eye and has a flair for
giving minute details. In her own words, for a distance needing ten minutes of walking,
“It would take me half an hour to an hour to dawdle along”. She enjoyed all the fun and
games, entertaining novelties and oddities in the streets. The performing monkey, the
snake displayed by the charmer, the indefatigable cyclist, the pongal offerings being
cooked. Nothing escaped her eagle eyes. She also gives minute details of the coffee
clubs and the process of cooling coffee by the waiters. Various seasonal fruits and
vegetables displayed too, did not escape her notice. In sum, she makes use of her pen to
give the minutest details, as a painter would give with his brush.

12.a) The Maharaja called the dewan and ordered him to immediately double the tax of
the villagers who had informed him of a tiger in the forest because despite his best
efforts he was unable to locate the beast.
(2)
b) Depletion of ozone layer, which protects us from the sun’s harmful rays, will
adversely affect the activities of the phytoplanktons. These single-celled plants nourish
and sustain the food chain of entire South Ocean and use the sun’s energy to assimilate
carbon and synthesis compounds. Any further depletion in the ozone layer will hamper
their activity, which in turn is going to stand in the way of the growth of marine animals
and birds, and the global carbon cycle. (2)

c) Dr Sadao was not sent abroad with the Japanese troops firstly because he was the
General’s doctor the General was ailing and might need an operation any time secondly

138
Sadao was perfecting a discovery which was likely to “render wounds entirely clean.” So
his presence in Japan was indispensable.
(2)
d) Roger Skunk was a delightful child. His only problem was that he smelled awful. As a
result nobody likes to befriend him and play with him. He got rid of his bad smell with
the help of wizard who cast a magic spell to change the smell into that of roses.
(2)
BLUE PRINT
TOTAL MARKS – 100

Q. No. Section Type of Marks Total Testing


Question marks objectives
Reading 20
Q. 1 1.1 (i) SA 2M 9 Comprehension
(ii) VSA 1M Comprehension
(iii) SA 2M Comprehension
(iv) SA 2M Comprehension
(v) SA 2M Comprehension
1.2 (a) VSA 1M 3 Vocabulary testing
(b) VSA 1M Vocabulary testing
(c) VSA 1M Vocabulary testing
Q. 2 2.1 Note Making 5M 5 Study Skills (Comprehension
and Note Making)
2.2 Summary 3M 3 Summary skills
Writing
Writing 35
Q.3. 1st option Short writing 5M 5 Providing factual details,
or skill Or organization,
2nd option Or fluency and coherence
Short writing or
skill Details, organization, fluency
5M and coherence
Q.4. 1st option Long Writing 10M 10 Presenting factual details.
Or Task formal, fluency and coherence
2nd option (Newspaper Or
Report) Presenting factual details,
Or informal, fluency and coherence
Long Writing 10M
Task
(Magazine
Report)
Q.5. 1st option Letter 10M 10 Formatting , Organization,
Or Writing coherence, fluency,

139
2nd option (Long Writing Resume
Task) Or
Or 10M Or
Letter Formatting , Organization,
Writing coherence, fluency,
(Long Writing
Task)

Q6. 1st Option Article (Long 10M 10 Organization, coherence,


Or Writing Task) fluency
nd
2 Option Or Or Or
Article (Long 10M
Writing Task) Organization, coherence,
fluency

TEXT-BOOKS 45

Q7(a) 1st option (e) SA 2M 4 Poetry appreciation


(f) VSA 1M Interpretation
Or (g) VSA 1M Interpretation
2nd option OR OR Or Or
(e) SA 2M Poetry appreciation
(f) VSA 1M 4 Interpretation
(g) VSA 1M Interpretation

Q7(b) Any three (e) SA 2M 6 Comprehension , interpretation,


(f) SA 2M understanding
(g) SA 2M
(h) SA 2M
Q8 All five (f) SA 2M 10 Comprehension , interpretation,
(g) SA 2M understanding
(h) SA 2M
(i) SA 2M
(j) Sa 2M
Q9 1st option Long Answer 10M 10 Content, Organisation, fluency,
Or Or Or coherence, understanding
2nd Option Long Answer 10M
Q10 1st option Long Answer 7M 7 Content, Organisation, fluency,

140
Or Or Or coherence, understanding
2nd option Long Answer 7M
Q11 All four (e) SA 2M 8 Understanding , interpretation
(f) SA 2M
(g) SA 2M
(h) SA 2M

SAMPLE PAPER (SOLVED)

Time Allowed: 3 hours] [Maximum Marks-100]

General Instructions:

i) The paper is divided into three Sections: A, B and C. All the sections are
compulsory.
ii) Separate instructions are given with each section and question, wherever
necessary. Read these instructions very carefully and follow them faithfully.
iii) Do not exceed the prescribed word limit while answering the questions.

SECTION-A; READING

1. Read the passage given below and answer the questions that follow:
(12 marks)

1 ---- I was in Hyderabad giving a lecture, when a 14 year old girl asked me for my
autograph. I asked her what her goal in life is: She replied: I want to live in a developed
India. For her, you and I will have to build this developed India. You must proclaim: India
is not an under-developed nation; it is a highly developed nation.

2 ---- Allow me to come back with vengeance. Got 10 minutes for your country? YOU say
that our government is inefficient. YOU say that our laws are too old. YOU say that the
municipality does not pick up the garbage. YOU say that the phones don't work, the
railways are a joke, the airline is the worst in the world and mails never reach their
destination. YOU say that our country has been fed to the dogs and is the absolute pits.
YOU say, say and say.

3 ---- What do YOU do about it? Take a person on his way to Singapore. Give him a name
- YOURS. Give him a face - YOURS. YOU walk out of the airport and you are at your
International best. In Singapore you don't throw cigarette butts on the roads or eat in
the stores. YOU are as proud of their Underground Links as they are. You pay $5

141
(approx. Rs. 60) to drive through Orchard Road (equivalent of Mahim Causeway or
Pedder Road) between 5 PM and 8 PM.

4 ---- YOU comeback to the parking lot to punch your parking ticket if you have over
stayed in a restaurant or a shopping mall irrespective of your status identity. In
Singapore you don't say anything, DO YOU? YOU wouldn't dare to eat in public during
Ramadan, in Dubai. YOU would not dare to go out without your head covered in Jeddah.
YOU would not dare to buy an employee of the telephone exchange in London at 10
pounds (Rs. 650) a month to, "see to it that my STD and ISD calls are billed to someone
else." YOU would not dare to speed beyond 55 mph (88 kph) in Washington and then
tell the traffic cop, "Jaanta hai sala main kaun hoon (Do you know who I am?). I am so
and so's son. Take your two bucks and get lost." YOU wouldn't chuck an empty coconut
shell anywhere other than the garbage pail on the beaches in Australia and New
Zealand. Why don't YOU spit Paan on the streets of Tokyo? Why don't YOU use
examination jockeys or buy fake certificates in Boston? We are still talking of the same
YOU.

5 ---- YOU who can respect and conform to a foreign system in other countries but
cannot in your own. You who will throw papers and cigarettes on the road the moment
you touch Indian ground. If you can be an involved and appreciative citizen in an alien
country why cannot you be the same here in India. Once in an interview, the famous Ex-
municipal commissioner of Bombay Mr. Tinaikar had a point to make. "Rich people's
dogs are walked on the streets to leave their affluent droppings all over the place," he
said. "And then the same people turn around to criticize and blame the authorities for
inefficiency and dirty pavements. What do they expect the officers to do? Go down with
a broom every time their dog feels the pressure in his bowels? In America every dog
owner has to clean up after his pet has done the job. Same is in Japan. Will the Indian
citizen do that here?" He's right.

6 ---- We go to the polls to choose a government and after that forfeit all responsibility.
We sit back wanting to be pampered and expect the government to do everything for us
whilst our contribution is totally negative. We expect the government to clean up but
we are not going to stop chucking garbage all over the place nor are we going to stop to
pick a up a stray piece of paper and throw it in the bin. We expect the railways to
provide clean bathrooms but we are not going to learn the proper use of bathrooms.
We want Indian Airlines and Air India to provide the best of food and toiletries but we
are not going to stop pilfering at the least opportunity. This applies even to the staff
who is known not to pass on the service to the public. When it comes to burning social
issues like those related to women, dowry, girl child and others, we make loud drawing
room protestations and continue to do the reverse at home. Our excuse? "It's the whole
system which has to change, how will it matter if I alone forego my sons' rights to a
dowry." So who's going to change the system? What does a system consist of? Very
conveniently for us it consists of our neighbors, other households, other cities, other
communities and the government. But definitely not me and YOU.

142
7 ---- When it comes to us actually making a positive contribution to the system we lock
ourselves along with our families into a safe cocoon and look into the distance at
countries far away and wait for a Mr. Clean to come along & work miracles for us with a
majestic sweep of his hand. Or we leave the country and run away. Like lazy cowards
hounded by our fears we run to America to bask in their glory and praise their system.
When New York becomes insecure we run to England. When England experiences
unemployment, we take the next flight out to the Gulf. When the Gulf is war struck, we
demand to be rescued and brought home by the Indian government. Everybody is out to
abuse and rape the country. Nobody thinks of feeding the system. Our conscience is
mortgaged to money.

8 ---- Dear Indians, The article is highly thought inductive, calls for a great deal of
introspection and pricks one's conscience too....I am echoing J.F. Kennedy's words to his
fellow Americans to relate to Indians.....

9 ---- "ASK WHAT WE CAN DO FOR INDIA AND DO WHAT HAS TO BE DONE TO MAKE
INDIA WHAT AMERICA AND OTHER WESTERN COUNTRIES ARE TODAY"

10- ---- Let’s do what India needs from us.

A.1.1. Answer the following questions briefly: (9 marks)


i) What are the negative remarks we SAY about our country? (2 M )
ii) How does an Indian behave in Singapore? (1 M)
iii) List the deeds an Indian would not dare to do while traveling abroad. (2 M)
iv) What is our attitude towards elections and social issues? ( 2 M )
v) Dr. Kalam says , “ Our conscience is mortgaged to money.” What does he mean by
this statement ?
A.1.2. Find words from the passage which mean the same as the following.
(3 marks)
(a) not genuine, imitation ( para 4)
(b) rich (para 5)
(c) surrender, give up (para 6 )

2. Read the following passage and answer the questions that follow: (8 marks)

It is worth saying something about the social position of beggars, for when one has
consorted with them, and found that they are ordinary human beings, one cannot help
being struck by the curious attitude that society takes towards them. People seem to
feel that there is some essential difference between beggars and ordinary "working"
men. They are a race apart--outcasts, like criminals and prostitutes. Working men
"work," beggars do not "work"; they are parasites, worthless in their very nature. It is
taken for granted that a beggar does not "earn" his living, as a bricklayer or a literary
critic "earns" his. He is a mere social excrescence, tolerated because we live in a humane
age, but essentially despicable.

143
Yet if one looks closely one sees that there is no essential difference between a beggar's
livelihood and that of numberless respectable people. Beggars do not work, it is said;
but, then, what is work? An accountant works by adding up figures. A beggar works by
standing out of doors in all weathers and getting varicose veins, chronic bronchitis, etc.
It is a trade like any other; quite useless, of course--but, then, many reputable trades are
quite useless. And as a social type a beggar compares well with scores of others. He is
honest compared with the sellers of most patent medicines, high-minded compared
with a Sunday newspaper proprietor, amiable compared with a hire-purchase tout--in
short, a parasite, but a fairly harmless parasite. He seldom extracts more than a bare
living from the community, and, what should justify him according to our ethical ideas,
he pays for it over and over in suffering. I do not think there is anything about a beggar
that sets him in a different class from other people, or gives most modern men the right
to despise him.

Then the question arises, Why are beggars despised?--for they are despised, universally.
I believe it is for the simple reason that they fail to earn a decent living. In practice
nobody cares whether work is useful or useless, productive or parasitic; the sole thing
demanded is that it shall be profitable. In all the modem talk about energy, efficiency,
social service and the rest of it, what meaning is there except "Get money, get it legally,
and get a lot of it"? Money has become the grand test of virtue. By this test beggars fail,
and for this they are despised. If one could earn even ten pounds a week at begging, it
would become a respectable profession immediately. A beggar, looked at realistically, is
simply a businessman, getting his living, like other businessmen, in the way that comes
to hand. He has not, more than most modern people, sold his honor; he has merely
made the mistake of choosing a trade at which it is impossible to grow rich.

by George Orwell (1933)

A.2.1—On the basis of your reading of the passage make notes on it, uses recognizable
abbreviations wherever necessary. Use a suitable format. Supply an appropriate title.
(5 marks)

A.2.2.-Write a summary of the passage in not more than 80 words. (3 marks)

SECTION-B, ADVANCED WRITING SKILLS


3. You are Sharon/Shan. You have been invited to attend the wedding of your friend’s
sister during summer Vacation. Respond to the invitation regretting your inability to
attend it. (5 marks)

OR
Design an attractive Poster on “Say No to Plastic” issued by Sarva Suraksha Samiti an
NGO stationed in Bangalore. (5 marks)

144
4. You are Mudit/Manasi working as a newspaper Reporter for The Times of India, New
Delhi. Yesterday, you were invited to attend a press conference convened by the Union
Minister for Human Resources Development on the proposed changes in the
examination pattern for the present class X, to bring about a stress free education
system in the country. It is called CCE i.e. Continuous and Comprehensive Evaluation.
Write a Report for publication in your paper covering the government’s proposals in
100-125 words. (10 marks)
OR
An “Each One Plant One” camp was organized by your school to celebrate
Vanamahotsava Day on 10th November. As Secretary of ‘Go-Green Club’ of your school,
Armada Public School, Bhopal, write a Report in 100-125 words for your school
magazine. You are Ankit/Ankita of class XII. (10 marks)

5. Sheena, C-4, Postal Colony, Thrissur, bought an Usha Lexus automatic iron from M/s.
Nandillath, Round North, Thrissur. Within a week it started giving problems. She writes a
letter to the dealer, complaining about the defects and asks them to replace the piece.
Write the letter for her in 125 words. (10 marks)
OR
You are Anuj/Anita living at #9/2, 5th Cross, Malleshwaram, Bangalore. You are very
much pained to know about the current state of affairs with regard to corruption and
scandals in the Indian political Scene. Write a letter to the Editor of the Hindu, showing
your concern about the falling standards. (10 marks)

6. Write an article in not more than 200 words on the topic “The Power Crisis and its
Impact” with regard to the frequent power break down and load shedding operations in
your city and many other cities in India.
(10 marks)
OR
A recent survey showed that there are still many communities in India which do not
welcome the birth of a girl child. Can a country which does not give equal rights to all its
citizens’ even dream of becoming great? Write an article in 175-200 words giving your
views on the above issue and the steps we should take to solve this problem.
(10 marks)

SECTION-D (LITERATURE)

7 A. Read the lines given below and answer the questions that follow:
(4 marks)

“They do not fear the men beneath the tree ;


They pace in sleek chivalric certainty .”
(a) Are Aunt Jennifer’s tigers real ? Give reasons for your answer.
(2 marks)
(b) Why do the tigers not fear the men beneath the tree? (1 mark)

145
(c) What do you understand by ‘chivalric certainty’? (1 mark)
OR
“A thing of beauty is a joy forever
Its loveliness increases, it will never
Pass into nothingness; but will keep
A bower quiet for us.”

(a) ‘ A thing of beauty is a joy for ever’. Explain. (2 marks)


(b) Why does a beautiful thing ‘never pass into nothingness’? (1)
(c) What does the poet mean by ‘a bower quiet for us’? ( 1 mark )

7 B. Answer any three of the questions briefly in 30-40 words. 3 x 2 = 6


(a) What does Stephen Spender want for the children of the slums? How can their
lives change?
(b) How will counting up to twelve and keeping still help us?
(c) What is the childish longing that Robert Frost refers to? Why is it in vain?
(d) What was the significance of the parting words and the smile of the poet?
8. Answer the following questions in about 30-40 words. (2 x 5 = 10)
(a) What was the mood in the classroom when M. Hamel gave his last French lesson?
(b) What does the writer mean when she says, “Sahib is no longer his own master.”?
(c) Why did Gandhiji agree to the British Landlords’ offer of just 25% refund of the
compensation to the farmers of Champaran ?
(d) What were the positive qualities of Subbu that Asokamitran admired?
(e) How did the incident at the Y.M.C.A. pool affect Douglas?
9. Answer any one in about 150 words. (10 marks)
The bangle-makers of Ferozabad make beautiful bangles and make everyone happy but
they live and die in squalor. Elaborate.
OR
How did the Champaran episode end the sufferings of the share-croppers?
10. Answer any one in about 125-150 words. (7 marks)
When did the Tiger King stand in danger of losing his kingdom? How was he able to
avert the danger?
OR
How did the question paper and the correction slip help the prisoner and the Governor?
11. Answer the following briefly in just 30-40 words each. (4 x 2 = 8)
(a) Why did Charley say that Grand Central Station is growing like a tree in the story
‘The Third Level’?
(b) ‘The world’s geological history is trapped in Antarctica’-What does Tishani Doshi
mean by this statement?
(c) Did Jo approve of the mother’s action? What did she want the story to be like
and why?
(d) What does Derry know about the fairy tale
‘Beauty and the Beast’? Why is he not convinced by its moral?

146
ANSWER –KEY

Section –A (READING)

1—No marks to be deducted for mistakes in grammar, spelling or word limit. Full marks
awarded if the student identifies the core ideas. No marks deducted for lifting portion of
the given passage.

Q.1. A
(a) Govt. is inefficient; laws too old; municipality does not pick up garbage; phones
don’t work; railways a joke; airline worst in the world; mails don’t reach their
destination. Any four points (½ x 4 )
(2 marks)
(b) In Singapore Indian is at his International best; doesn’t throw cigarette butts on
roads; eat in stores; pays 5 dollars t drive through Orchard Rd. Any two
( ½ + ½ = 1 mark)

(c) Indians don’t dare to eat in public during Ramadan in Dubai; not dare to go out
without head covered in Jeddah; not dare to buy an employee of telephone exchange in
London; not dare to speed beyond 55 mph in Washington; won’t chuck empty coconut
shell anywhere; won’t spit PAAN on streets. Any four points (½ x 4 ) (2 marks)

(d) We go to polls to choose a government and after that forfeit all responsibilities
expecting govt. to do everything for us; as for social issues make loud protestations and
continue to do the reverse at home.
( 1+ 1 = 2 marks)

(e) He means we leave the country and run away to make more money; we don’t do
anything to improve our country. (2 marks)

Q.1. B
a) fake [1 mark
b) affluent [1 mark
c) forfeit [1 mark

2. If a student has attempted only summary or only notes due credit should be given

A title given anywhere 1 mark to be allotted


Minimum 3 main headings and maximum 6
The notes provided below are just guidelines. Any other title, heading or subheading
can be accepted if they are indicative of the student’s understanding of the passage.
Notes must include main points and abbreviations. Complete sentences should not
be accepted as notes.
NOTE MAKING

147
Distribution of marks
Abbreviations/symbols with /without key-any four [1 mark
Title [1 mark
Content [3 marks
Suggested Notes
Title: Why despise Beggars ? (or any other title)
1.—Pub. Opinion on beggars
1.1—very diff. from ‘working’ men
1.2—likened to criminals & prostitutes
1.3—do not ‘work’
1.4—Social Excrescence, despicable
2-- Diff. b/w beggar & others
2.1—Accountant Works with figures
2.2—beggars stand in bad weather
2.3—Beggar exposed to varicose veins, chronic bronchitis
2.4—more honest, harmless
3—Why despise ?
3.1—B’coz. fail to earn decent living
3.2—don’t earn lot of money
3.3--Never sells his honour
3.4—Only mist. Chose a trade where can’t grow rich
Summary
The summary should include all the points given in the notes.
Content [ 2 marks
Expression [ 1 mark
( above 90 words deduct ½ mark)
SECTION—B (Advanced Writing Skills)
Here expression and content both must be tested

QUESTION-3
If the answers exceed 60 words deduct ½ mark
Reply to informal Invitation
Format 1M
(Including Senders Address, date, Salutation and Complimentary close)
Content 2M
Expression 2M
Suggested value points
Suggested value points
a. Thank for inviting followed by details of event
b. Express inability with reason
c. Convey Good wishes
OR

POSTER

148
Format [1 mark
Content [2 marks
Expression [2 marks
Suggested value points
• A Catchy Heading on the Topic
• Message of Appeal to avoid Plastics in catchy slogans and phrases
• Sketch to add attraction
• Name of the organization issuing it

QUESTION 4
REPORT WRITING
Format
title, reporter’s name [1 mark
Content [4 marks
Expression [5 marks
Grammatical accuracy, appropriate words and spelling [2 ½
Coherence and relevance of ideas and style [2 ½
Suggested value points
(MINISTER ELABORATES ON CCE)
(or any other relevant title)
---what
---where
---when
--- Main speakers at the meeting
--- Minister’s explanation of CCE ; reason for its implementation; expected success
--- questions asked by the media; Minister’s replies
OR

Suitable Title
Suggested value points
--- guests and dignitaries
---planting of saplings
---- placards/march on the road
--- cultural programme; quiz etc
--- seminars
---- any other relevant details
(150 t0 175 words deduct ½ mark)
(above 175 words deduct 1 mark)
QUESTION 5
LETTER WRITING
No marks are to be awarded if only the format is given. Credit should be given to
creativity in presentation of ideas.
Format

149
1. sender’s address, 2. date, 3. receiver’s address, 4. subject heading, 5. salutation,
6. complimentary close [2 ]
Content [4 marks]
Expression [4 marks]
Grammatical accuracy appropriate words and spelling (2)
Coherence and relevance of ideas (2)
COMPLAINT LETTER
Suggested value points
---- Details of the purchase( Where, When , how much, Bill no. etc.,)
---- State the problem with it.
---- Request for replacement
---- Complimentary close
OR
CORRUPTION AT HIGH LEVEL
----introducing self
----frustration over various scams
----detrimental effects on the future of India
----appeal to take the path of honesty
----request for a pledge to raise the nation’s reputation

QUESTION 6
ARTICLE WRITING
225 to 250 words deduct ½ mark
Above 250 deduct 1 mark
Format (Title and writer’s name) [1 mark
Content [4 marks
Expression [5 marks
(Grammatical accuracy, appropriate words and spelling) 2 ½
(Coherence and relevance of idea and style) 2½
Suggested value points
THE POWER CRISIS AND ITS IMPACT
---Power a great boon to mankind
---indiscriminate usage causing shortage
---leading to power cuts
---problems at various levels at home
---problem in the industry
----need to save power
----suggestions on saving electricity at home and public places
----how to have a power bright future
---- a suitable ending
---- (any relevant suggestions)
OR

Suggested value points

150
THE GIRL CHILD (or any other relevant title)

-----a thought provoking introduction


----the hateful attitude towards a girl child shocking
----girl child has equal part in the family/society
----some incentives given and today the scenario is changing
----people must awake to the fact girls can also shine in various fields
--- Examples of Indian women who have excelled
----- Suggestion to erase these outdated ideas

SECTION –C (LITERATURE)
QUESTION 7 A
This question has been designed to test only the student’s understanding of the text.
Therefore there is no penalisation for language errors.
a)
(a)—no; not real; on screen embroidered on a panel [ 2 marks
(b)---do not fear the men beneath the tree; symbol of chivalry; powerful [ 1 mark
(c)---sure of their power and strength [1 mark
OR
b)
(a)--- a thing of beauty has a lasting impact can never move into emptiness. we even
think of them in our dream [ 2 marks
(b)---it has long standing impression, not subject to time [ 1 mark
(c)--- shady place where one can sit and ponder

1—QUESTION 7B
(On exceeding 60 words deduct ½ mark)
Short answer type questions (Poetry)
Distribution of marks:
Content [1 mark
Expression [1 mark
(deduct ½ marks for two or more grammatical / spelling mistakes
Value points|
a) –wants true education; their lives can change if given opportunities; authorities
should work for upliftment
b) – counting up-to twelve will help us think about ourselves; about world around
us; chaos we humans have created; introspect the world around
c) – the poor farmer keeps waiting all days for some car to stop at his stand; hardly
any stops there; longing is in vain as no one comes to buy the things he has put
up for sale
d) – Parting word “See you soon Amma” and the smile was meant to give moral
support/ strength/reassurance. The smile was also meant to hide her own fear
and emotional turmoil

151
QUESTION 8
Short answer type questions (Prose)
Questions to be answered in 30-40- words.
If answer exceeds 60 words deduct ½ marks
Distribution of marks
Content [1 mark
Expression [1 mark
(deduct ½ mark if more than two grammatical or spelling mistakes)
a)---school strange and silent; most of the village people sitting on the back bench; all
quiet and sad; showed sympathy and respect to teacher; thanked M Hamel. Learnt the
lesson as in they wanted to learn everything
b)---poor rag picker
---Saheb-E-Alam
---means Lord of the Universe
---irony is he is very poor and doesn’t even have shoes
c)---Gandhiji explained that the amount of refund was less important than the fact that
landlords were obliged to surrender part of the money and with it part of their prestige
d)----he gave direction and definition to Gemini Studios; separate identity as a poet;
tailor made for films; charitable; readiness to say nice things to everyone
e)---could not go for swimming; gripped with the fear of water; could not enjoy boating,
fishing , canoeing etc.
QUESTION 9
Distribution of marks:
Content [ 5 marks
Expression [ 5 marks
(Grammatical accuracy, appropriate words and spelling-2 ½)
(Coherence and relevance of ideas and style—2 ½ )
175 To 200 Words deduct ½ mark
Above 200 words deduct 1 mark
Value points
--- utter poverty generation after generation
-----believe they are destined to work in bangle factories
----make beautiful bangles but live in dark
-----bright furnaces to do welding
----they loose their eyesight
----victims of vicious circle of middlemen
---law enforcing authorities prey upon them
--- bleak future
OR
Value points
--- Share croppers forced to grow Indigo-15% of land
--- Submit produce to British landlord as rent
--- Germany – invented artificial Indigo – Peasants unaware of it
--- Fall in price of indigo

152
--- British demanded compensation to release farmers from agreement
--- Raj Shukla approaches Gandhiji
--- Gandhiji fights for the cause and agrees to 25% refund by the British.
----He helps farmer realize British not above law
--- Gandhi also works to remove illiteracy and improve health and hygiene.

QUESTION 10
Distribution of marks
Content : { 4 marks
Expression { 3 marks
(Grammatical accuracy appropriate words and spelling -1 ½)
(Coherence and relevance of ideas and style – 1 ½ )
175 to 200 words deduct ½ mark
Above 200 words deduct 1 mark
Value points:
---after growing up Tiger King started killing tigers due to astrologer’s prediction
---A British high ranking officer wanted to hunt tigers
---British Officer’s secretary suggested to take a photograph with a killed tiger
--- Maharajah refused and was in danger of losing his kingdom
----sent 50 rings worth to the Lady of the officer to select some as appeasement
===The Lady kept all and no longer was the Maharajah troubled
OR
---Evans wrote the exam in the cell, wrote his name in the question paper
---The Asst. Secretary informed the Governor that they had forgotten to place a
correction slip in the exam package and dictated it.
---The clue for Evans was in the paper itself, Index No was 313 and centre No 271
----Correction slip helped Evans to escape and helped Governor also to trace him
---Evans was able to escape again as he had many friends to help him

QUESTION 11
Marking Scheme same as Question 8
a)--- because he gets lost there many times; he was always discovering new corridors
;pushing out new tunnels; new doorways like roots of a huge tree
b)—there was a giant super continent 650 million years ago called Gondwana.%00
million years ago it disintegrated into countries as they exist today. Antartica was then
at the centre of Gondwana. The study helps us know of the past of cordilleran folds, pre-
Cambrian granite shields, ozone and carbon, evolution and extinction
c)---Jo did not approve of the mother’s action; she believed instead of agreeing to give
back the skunk his smell the Wizard should have hit the mother back; she believed the
mother was silly and overbearing
d)---Derry had heard the tale; Beauty liked the monstrous Beast; when she kissed he
turned into a handsome prince; moral was not how you look outside but how you look
inside is important; Derry was not convinced as even if someone kissed he would never
change and his mother always kissed him only on the other cheek

153
SAMPLE QUESTION PAPER (UNSOLVED)

SECTION-A

Read the following passage carefully and answer the questions that follow:

1. Dussehra and Diwali leave us in a mess-physically speaking. After all the lavish
indulgences, our body literally sends signals that it is time we slowed down. And
there is a way out-remove the toxins.
2. Most people believe that going on a detox trip is like suffering a crash diet and a
method to lose weight, which is wrong notion. Nutrition head of Gold’s Gym
Ashwini Sukumar explains that detoxification has its roots in traditional Chinese
medicines that is also been used in ancient India, Asia and Egypt. It is the body’s
natural, ongoing process if neutralizing or eliminating toxins from the body. “A
detox diet post Diwali blast is for those who want to bring their bodies back to its
original condition following festival fun. You may decide to use a detox plan as a
regular thing when you begin to feel the benefits and can use the principles
again, depending on the extent of detox you are looking to achieve,” she
explains.
3. She also explains that it is necessary and important to go on a detox diet after
binging on your favourite food. “Many chemicals we ingest daily, through food,
water and air, are deposited as fat cells in our bodies. A diet that lacks certain
nutrients may also impair our natural ability to detoxify chemicals, which further
leads to their build-up in the body. So, it is necessary to go on a detox diet to
cleanse all that. “States Ashwini. The benefits include improved digestion, and
increased concentration and clarity.
4. Many people also go on such diets to lose weight, is it a good thing? We ask. “If
you fast for several days, you will shed weight, no question. But typically, most of
the weight loss is water loss and some of it may be muscle tissue. Fasting or
detoxing for longer periods can also slow down your metabolism, making it
harder to keep the weight off or lose weight later.
5. While, director of wellness chi-health, Sheela Krishnaswamy says, “modern
medicine has no scientific-belief in detoxs diets because the liver, kidneys, colon
and skin remove waste products from our body on a daily basis, irrespective of
what we may eat. But alternate therapists endorse detox diets very strongly,
especially after a season of dietary excesses.”
6. She also points out that working on such diets is a new concept. “Earlier women
used to prepare sweets and snacks at home. They always knew what ingredients
they would use and the preparation would also involve tedious work. Even if
they binge the physical work that was involved during the preparation of these
festivals would help them to cut down on the fat. But now everything is available

154
on the shelf and of late, people prefer buying sweets at stores, where the calorie
content is high,” she states.
7. So what does a typical detox process entail? “cut back on sugary stuff. Have lot
of vegetables, fruit juices, coconut water and exercise if you have gained weight.
But, remember to detox only for day or two. “Don’t go on a continuous crash, it
will harm your body.” She warns.

Now answer the questions that follow, based on your reading of the above extract:
1. What is the wrong notion harboured by most people about detox diets?
2. Is it a good sign to go on an extended diet to lose weight?
3. Why is it necessary to go on a detox diet?
4. What comprises of a detox diet?
5. Why does modern science not believe in detox diet?

Find words in the passage that mean the same as


1. Intake {para3}
2. Remove harmful or poisonous substance
3. Slow or dull {para6}
2. Read the following passage carefully and answer the questions that follow.
The origin of water on Earth is linked to the formation of Earth. According to some
currently accepted theories Earth began as a waterless mass of rock surrounded by
cloud of gas. Radioactive materials in the rock and increasing pressure in the Earth’s
interior gradually produced enough heat to melt the interior of the Earth. The heavy
materials, such as iron ores, then sank. The light silicates (rocks made up of silicon and
oxygen) rose to the Earth’s surface and formed the earliest crust. Many silicate rocks
have water molecules integrated into their atomic arrangement – water can be driven
out of such rocks by the action of heat. Thus the heating of the Earth’s interior caused
release of water contained in such rocks to the surface. Over millions of years, water
thus released collected slowly in low places of the crust and formed the oceans.
Whatsoever might have been the origin of water, Earth’s original supply of water is still
in use and very little, if any, has been added during the past billion years or so. The same
water has been pumped time and again from the oceans into the air, dropped down
upon the lands and transferred back to sea. A single drop of water spends 8 to 10 days
passing through air, 2 to 3 weeks in a river, as long as 100 years in a Himalayan Glacier
or from 100 to 40,000 year underground.
As chemical, water is unique and rather odd. All its oddities can be traced to its
molecular structure. It is a rather sturdy molecule. Until some 180 years ago water was
believed to be an invisible element rather than a chemical compound. Today students of
science knows that each of its molecules is made up of two atoms of Hydrogen and one
atom of oxygen the bond between the oxygen and the hydrogen atom is polar, that is, it
has positive and negative charged ends because of an unequal distribution of electrons.
The oxygen atom has a denser distribution of electrons around it and hence a net
negative charge. The hydrogen atoms in a water molecule, on the other hand, are
positively charged. This leads to a lopsided (molecule with electrical charges

155
concentrated on opposite sides). Water molecules, are therefore, attracted to each
other as well as to other molecules having a similar charge distribution. And many of the
characteristic features of water can be traced to the so-called hydrogen bond between
its molecules.
When a substance dissolves in another substance, the resulting distribution of the
molecules of the two substances has lesser number of molecules of either substance
surrounded by its own kind. This necessitates disrupting prevailing intermolecular forces
in each of them. The molecules of most organic compounds (e.g., oily substances are
non-polar). As a consequence the intermolecular forces between organic molecules are
much weaker than in water. If such a substance is to mix with water – the resulting
distribution of molecules must lead to lowering of energy content. If more energy is
required to separate water molecules from each other (by breaking hydrogen bonds)
than is gained when water molecules get closer to organic molecules, the two
substances will not mix together. It is for this reason that water and oil do not mix and
many organic compounds do not dissolve in water.
2.A On the basis of your reading of the above passage, make notes using headings and
sub-headings, use recognizable abbreviations wherever necessary.
2.B Make a summary of the above passage in not more than 80 words using the notes
made and also suggest a suitable title.

SECTION-B

3. You are Gokul/Gopika, the Secretary of Integrity Club. Draft a notice inviting the
students of your school to attend a Skit to be presented by the members of your club on
the occasion of the vigilance day being observed in your school on Nov 1. Write the
notice in not more than 30 to 50 words.
5M
OR
You are Rani and have been accepted at NIT, Suratkal as you have scored a high rank in
AIEEE. You decided to host a party for your close friends and relatives before you leave
for Suratkal. As Rani draft an informal invitation in not more than 50 words.

4. The social service league of your school celebrated Diwali in a novel way by collecting
money and discarded clothes and distributing them along with sweets and crackers to
the children at a local orphanage. Write a report for your school magazine as Praneet /
Preeti, Secretary of your club. 10
M
OR
You are Winnie/William. The Book Fair which lasted for a month in your city was the
topic of all book lovers. Write a report on it in about 100-125 words.

4. Your sister has just passed the all India Secondary School Examination of CBSE.
She has seen an advertisement in a newspaper regarding Air Hostess – as a career. She is
keen to join the training course provided by ABC Service, Connaught place, New Delhi.

156
Write a letter to the Director of the Centre seeking necessary information about her
admission. You are Rohit. 10 M
OR
You are Chetna Sharma of Siddarthnagar, Mysore. You came across an article in the
news Paper about the effects of Endosulfan on human beings and the Governments
decision not to ban the use of Endosulfan. You decide to write a letter to the Editor
expressing your deep shock and outrage at the callous attitude of the Government. As
Chetna write the letter in about 200 words.

5. The other day the value Education Club of your school organized a visit to the “The
Home for the Aged” in your town. There you got a chance to know the feelings and
problems of the aged from close quarters. On your return from the trip, you decide to
write an article for the Bangalore Times on the problems of the Aged and governments
expected role in solving them. Write the article in about 150-200 words.

OR
Love for one’s country is a great Virtue. We must inculcate this virtue in our students
and modern youth. In order to stress the need of ‘Patriotism’ Remo, a student of class
XII, writes an article for the school magazine. Write the article in about 150-200 words.
10 M

SECTION-C
5. Read the following extract and answer the questions that follow:

“It would be an exotic moment


Without rush, without engines,
We would all be together
In a sudden strangeness.”
1. When would it be an exotic moment for the poet? 1M
2. What does the poet wish to achieve by this silence? 2M
3. Why does the poet describe the togetherness as ‘Strange’? 1M

OR
“Standing a few yards
Away, I looked again at her, wan,
Pale
As a late winter’s moon and felt that
Old
Familiar ache, my childhood’s fear,
But all I said was see you soon,
Ammo, all I did was smile and smile and smile…….”
1. Why has the poet compared her mother to a late winter’s moon?1M
2. What kind of an ache is the poet taking about in the above lines?
1M

157
3. How did the poet give herself and her mother a reassurance of meeting again?
1M
4. What is the significance of the word ‘Smile’ that is used more than once?
1M

6. Answer any three of the following questions 30-40 words.


3x3=6 M
1. In the poem ‘A Thing of Beauty’ how has the poet brought out his immense faith
in the divine?
2. What kind of things have the slum children received as a legacy from their
elders?
3. What kind of a lesson can we learn from Mother Earth.?
4. What makes we human beings able to love life in spite of innumerable suffering?

7. Answer the following question in 30-40 words. 5x2=10

a. Why did the peddler not respect the confidence reposed in him by the crofter?
b. How is Mukesh different from the other member of his family?
c. In ‘Poets and Pancakes’, why is the legal adviser referred to as the opposite by
the other?
d. What had been put on the bulletin board that attracted everybody’s attention
the day Franz was late to school?
e. What were some of the fears/beliefs about being interviewed?

8. Bring out vividly with the help of examples, the multi-faceted personality of
kothamangalm Subbu.

OR
“Freedom from fear is a prerequisite for justice”. How did Gandhi
prove this through the Champaran episode? 10 M

9. Bring out the irony in the story “Tiger King”.


OR
How did Dr. Sadao rise above narrow prejudices of race and
country to help a human being in need? 07M

10. Answer the following in 30-40 words: 4x2=8

a. What was the objective of the ‘Students on Ice programme’?


b. What clues did the answer sheet of Evans provide to the Governor?
c. How does Lamb transform Derek?
d. Why was Roger skunk not accepted by his peer group?

158
SAMPLE QUSTION PAPER (UNSOLVED)
SECTION A – READING COMPREHENSION
1. Read the passage given below and answer the questions that follow:
• Today, India looks like it is on course to join the league of developed nations. It is
beginning to establish a reputation not just as the technology nerve centre and
back-office to the world, but also as its production centre. India’s secularism and
democracy serve as a role model to other developing countries. There is great
pride in an India that easily integrates with a global economy, yet maintains a
unique cultural identity.

• But what is breathtaking is India’s youth. For despite being an ancient civilization
that traces itself to the very dawn of human habitation, India is among the
youngest countries in the world. More than half the country is under 25 years of
age and more than a third is under 15 years of age.
• Brought up in the shadow of the rise of India’s service industry boom, this group
feels it can be at least as good, if not better, than anyone else in the world. This
confidence has them demonstrating a great propensity to consume, throwing
away ageing ideas of asceticism and thrift. Even those who do not have enough
to consume today feel that they have the capability and opportunity to do so.
• The economic activity created by this combination of a growing labour pool and
rising consumption demand is enough to propel India to double-digit economic
growth for decades. One just has to look at the impact that the baby boomers in
the US had over decades of economic activity, as measured by equity and
housing prices. This opportunity also represents the greatest threat to India’s
future. If the youth of India are not properly educated and if there are not
enough jobs created, India will have forever lost its opportunity. There are
danger signs in abundance.
• Fifty-three per cent of students in primary schools drop out, one-third of children
in class V cannot read; three-quarters of schools do not have a functioning toilet,
female literacy is only 54 per cent and 80 million children in the age group of 6-
14 do not even attend school.
• India’s IT and BPO industries are engines of job creation, but they still account
for only 0.2 per cent of India’s employment. The country has no choice but to
dramatically industrialize and inflate it domestic economy. According to forecast
by the Boston Consulting Group, more than half of India’s unemployed within
the next decade could be its educated youth. We cannot allow that to happen.
• India is stuck in a quagmire of labour laws that hinder employment growth,
particularly in the manufacturing sector. Inflexible labour laws inhibit
entrepreneurship, so it is quite ironic that laws ostensibly designed to protect
labour actually discourage employment.

• Employment creation needs an abundant supply of capital. Controls on foreign


investment have resulted in China getting five times the foreign direct

159
investment, or an advantage of $200 billion over the past five years. The growing
interest in India by global private equity firms augurs well as they represent
pools of patient and smart capital, but they too face many bureaucratic hurdles.
• When it comes to domestic capital availability, budget deficits adding up to 10
per cent of the national GDP impede capital availability for investment and
infrastructure.
• Raising infrastructure spending, coupled with rapid privatization, may not only
create employment but also address the growing gaps in infrastructure. China
has eight times the highway miles and has increased roads significantly in the
past few years while India has only inched along. Freight costs at Indian ports are
almost double the worldwide average, just to give two examples.
• Moreover, like the Lilliputians that kept the giant Gulliver tied down there are
some 30,000 statutes in India, of which only a portion are even operational, and
these keep the employment creation engine tied down. Since there are no
sunset provisions in any laws, the regulatory morass only grows every year.
• In the meantime, we as citizens of the world and descendants of India have to
make a difference. We have to ensure that India and its youth attain that
potential, both through our business pursuits and the support of educational
charities, on-the-ground proponents of participative democracy as well as other
deserving organizations and initiatives.
• I believe that hope can triumph and that this can be India’s century – not one
that will happen as surely as the sun will rise each day, but one that many willing
hands will need to create together.
a) Read the passage and answer the given questions
1. What makes the author think India is on the verge of joining the select band of
developed nation? (2)
2. Despite the fact that India is one of the oldest civilizations why does the author
say it is young?(2)
3. The author feels that if certain problems are not arrested, India would lose its
opportunity. Why would India lose this opportunity?(2)
4. What hinders employment growth?(1)
5. Who/what in the passage is referred to as the ‘Lilliputians’?(1)
6. How can we ensure that India and its youth attain their full potential?(2)
b) Find words in the passage which mean the same as (2)
i) One of its kind ii) a great success
Q2. Read the passage given below: 8
1. The art of listening has become one of the most important skills in modern life –
more important even than the ability to read. Increasingly, communication is by
the spoken word in personal conversation, group addresses, in communication
by telephone, in reception of news and announcements over the radio and
through the cinema or television. The liveliness and activeness of response is a
matter of habit born of proper training.

160
2. Great though the differences between them, many people do not discriminate
between hearing and listening. The former is merely the exercise of one of the
senses while the intellect remains passive. Certain sounds strike the ears, and we
may or may not attach meaning to them. In any case, we do not exert ourselves
in the matter. Pupils in our schools ‘hear’ what popularly passes for ‘English’,
and continue speaking a jargon of their own – usually a mispronounced amalgam
of shoddy Americanism sentence patterns based on prevailing language of the
region.

3. Listening can go a long way towards correcting this situation. In listening, we


hear with a purpose, with a consciously directed intellect. In listening
comprehension as applied to English, our aim should be to train the pupil to
understand the language, the type of speech that Professor Lloyd James
suggests, ‘can be heard anywhere without causing discontent”. Such English is
not ‘elocution’, Oxford’, or even ‘B.B.C.’ English. The last type is ‘Standard
English’ – the kind that can be understood wherever the language is spoken.
Most of our pupils will never attain to that type in their own speech. They may,
even after all the training we can give them, retain regional peculiarities of
cadence and stress – a sort of Modified Standard English.

4. Assuming such English in the teachers at our schools, we suggest the following:

5. We ought to distinguish between ‘listening for comprehension of content’ and


‘listening to the sounds of English with a view to imitation and reproduction, i.e.,
learning to speak well’.

6. Both kinds of listening must be cultivated, but with more attention to the latter
in the earlier stages and with more attention to the former as pupils progress
towards the senior classes.

7. ‘Listening, pen in hand’ may be instituted, to be completed with instruction and


practice in the proper method of intelligent note-taking that testifies to
intelligent listening.

a) On the basis of your reading of the passage, make notes on it in a suitable format. Use
recognizable abbreviations, wherever necessary. Give a title to your notes.
5
b) Write the summary of the above passage. 3

SECTION-B

161
3. You are the team leader of the Everest Summiteers’ from Nepal to participate in the
march past at Copenhagen, Denmark during the Global Climate Change Conference. You
have to draft a poster to take it during the march. Draft the poster.
5
OR
You are Mohan/Molly Manager SBI Mysore. You have been invited by the Lions Club to
act as one of the judges for a fancy dress competition for children. But due to previous
engagement you cannot accept this invitation. Write a formal reply regretting your
inability to attend.

4. You are Rahul , the School Pupil Leader of your school. Citizen’s Health Council
recently organised a unique Health Workshop in your school. Write a report on the
workshop for a newspaper magazine. (125-150 words.)
10 M
OR
The International Book Fair was inaugurated by the Chairman of Children’s Book Trust,
Dr.Kumar. The theme this year was ‘Illustrated Works for Children’. You are Akshay/
Anandi and had visited the exhibition and were impressed. Write a factual description in
125-150 words.

5. You are Satish /Sonali the school librarian. You have been asked to place an order for
children’s story books (Age-10-13 years.). Write a letter to M.S. Book Depot, Ramnagar,
Bangalore placing an order for
the books. Invent all the necessary details. 10 M
OR
You are Mr. M.L. Sareen. You have seen an advertisement in the Hindustan Times for
the post of marketing manager. Write an application with complete bio-data.

6. You have been asked to participate in the Debate Competition on the topic ‘Machines
have enslaved Man’ .Write the debate in 200 words either for or against the motion.
10
OR
You are Suhas/Suhasini. Environmental Week is being celebrated in your school. You
have been asked to present a paper on ‘Global Warming’. Write the article in 200 words.

SECTION-C

7. Read the extract and answer the questions that follow.


“ For once on the face of face of the earth
let,s not speak any language ,
let’s stop for one second ,
and not move our arms so much.”
a) If no language is spoken then what will happen? 1x4=4

162
b) What does he mean by one second?
c) What was the instruction the poet had given at the beginning of the poem? How
is it different at the end?
d) Why does the poet not want us to move our arms so much?
OR
“ On their slag heap, these children
Wear skins peeped through by bones and spectacles of
steel
With mended glass, like bottle bits on stone.
All of their time and space are foggy slum.
So blot their maps with slums as big as doom.”
a) What does the poet mean by slag heap?
b) How has the poet brought out the pathetic condition of the children?
c) Explain ‘So blot-----------doom’.
d) Who can bring about a change in their lives? How?

8. Answer any three of the following in 30-40 words. 2x3=6


1) What does the line, ‘Therefore are we wreathing a flowery band to bind us to the
earth’ suggest to you in the poem ‘A Thing of Beauty’.
2) Explain ‘soothe them out of their wits’ with reference to ‘An Elementary School
classroom in a slum’
3) Interpret the symbols in the poem ‘Aunt Jennifer’s Tigers’.
4) What is the kind of pain and ache that the poet feels in the poem ‘A
Roadside Stand.
9. Answer the following in 30-40 words. 2x5-10
1. ‘Together they have imposed the baggage on the child that he cannot put down.’
Explain with reference to ‘Lost Spring’.
2. What were the series of emotions and that fears that Douglas experienced when he
was thrown into the water? What were his plans?
3. Why did Gandhi agree to a settlement of 25 per cent refund to the farmers?
4. What was Umberto’s secret in being successful?
5. What was the peddler’s explanation for withholding his true identity? How did he
convince the latter not to call the law?
10. Answer in 150 words. 10
The writer of Poets and Pancakes uses humour to highlight human foibles. Discuss.
OR
Jansie is as old as Sophie yet very different. Bring out the contrast between them.
11. Answer in 150 words. 7

Why is Antartica the place to go to, to understand the earth’s present past and
future?
OR
It may take a long time for oppression to be resisted, but the seeds of rebellion are
sowed early in life. Discuss with reference to the lesson ‘Memories of Childhood.’

163
12. Answer the following in 30-40 words. 2x4=8
1. Why does Charlie say that grand station is growing like a tree in the story ‘ The Third
Level’?
2. Why was there a delay in starting the examination for Evans’?
3. How did the Maharaja overcome the unforeseen hurdle that had brought his mission
to a standstill?
4. Why did Jo disapprove of Jack’s ending of the story? How did she want the story to
end?
UNSOLVED QUESTION PAPER
SECTION-A (Reading) 20 Marks

1. Read the following passage and answer the questions that follow: (12)
1.Thousands of years ago, when man was in the process of evolving, he looked around
for ways of expressing himself. He etched on stone and wood. He began to craft objects
thrilled with the fact that stone and clay took shape at his will. He found that certain
types of clay could be made into exciting shapes. Terracotta is the use of clay in its
simplest form, dried out by the sun or fired at a low temperature. There are many
theories as to how man began to create beautiful objects from clay, like this legend…
2. A pot was needed for the marriage of Shiva and Parvati. But none of those assembled
knew how to make one. Finally a Brahmin, Kulalak offered his services. He borrowed
the Sudarshan Chakra from Vishnu, and used the Mandar hillock as a pivot. From Shiva
he took his pestle for turning the wheel, his langota for mopping spills, his kamandalu
(water jar) for holding water and his sacred thread for detaching the pot from the
wheel,Brahma offered the Adi-Kurma or tortoise using it as a scraper to smoothen the
product. Kulalak made the pots and the wedding took place. The descendants of Kulalak
came to be known as Kumbhars.
3. In 3000 BC excavations in Baluchistan showed evidence of clay objects. Most of the
time, the clay objects were figures of worship, but in the latter half of the third
millennium BC, better developed products were discovered in Harappa. Bricks were
fired for construction, and a range of animal figures like monkeys, goats, pigs, elephants
and lions were crafted. In the beginning, the figures were solid, later they were built
around straw, and when they were fired, the straw burnt out leaving the figures intact.
The objects had movable heads and holes in the lower part so that they could be
mounted on wheels. Bird toys and whistles were made. It was in 2000BC that the
Egyptians first discovered the wheel.
4. In the Neolithic Age, man discovered that food could be cooked. He became the
provider and the woman became the householder. Vessels were required and so came
the invention of utilitarian pottery. With Aryan invasions, iron came into use and the
terracotta craft of Harappa was laid to rest. It was only during the Mauryan period from
322 BC to 185 BC that terracotta art was revived.
5. The terracottas of North are different in texture and the pottery of better quality,
because large quantities of clay are found in the rich, alluvial soil of the Ganga-Yamuna
valleys. The black polished pottery of the North evolved during the Maurya period. In

164
the Sunga period, (from 2nd century onwards) the themes drifted to ones connected
with occupation.
6. In India, during Ganesh Chaturthi, clay figures of Ganapati are made. Durga is
elaborately craft in Bengal during the Puja festival. In the cort of Aiyanar in South India,
offerings in the shape of huge terracotta horses are made. Bihar, Orissa and
Maharashtra are other parts of India where clay effigies are made by devotees. You will
often see terracotta horses standing outside temples in villages. The villagers believe
that at night the protector of the village mounts these horses. Today, terracotta
products are crafted all over India.
7. Modern technology and the brush with ceramics have slowly replaced the craftsman’s
traditional functions in daily life as the vessel makers of rural India. The village potter
has been banished and he suffers without a livelihood. But to give him back his status,
considering clay art is so vital in the life of its maker, we have to recognize his worth, his
contribution, and his craft, and restore his dignity, by encouraging him, and taking his
crafts into our homes, giving clay its position of pride, something that it richly deserves.
(a) Read the passage and answer the given questions:
i) Mention two ways in which early man used clay.
(2)
ii)How is Terracotta created?
(2)
iii) How did Kumbhars come to be called so?
(2)
iv) Name two festivals of modern India where clay figures are used. (2)
(b) Find words in the passage which mean the same as (3)
i) to carve (para 1)
ii) future progeny (para 2)
iii) useful (para 4)

2. Read the given passage and answer the questions that follow: (8)

Much before medical science discovered it, reader’s Digest came out with the
prescription-Laughter is the Best Medicine. Newspapers and magazines which regularly
run humour columns are, therefore, doing their bit to keep the readers in good health.
Reading light articles, whether they are satirical, comic or just humorous, relieves the
tedium of work-a-day world. Some pieces may even tickle one’s grey matter.

It is said that if you laugh for ten minutes you will be in a better position to put up with
pain for two hours. According to US researchers, laughter is a good antidote to stress
that tones up the system. Facial laugh muscles instruct the brain to “feel good”
regardless of how you feel.

According to a French doctor, laughter deepens breathing, improves blood circulation,


speeds up the process of tissue healing and stabilizes many body functions. In short, it
acts as a power drug with no side effects. Researchers state that laughter stimulates

165
production of beta-endorphins, natural pain-killers in body and improves digestion.
Those who laugh are less prone to digestive problems and ulcers.

Some people in France have made it a career. You can hire a ‘Jovialist’ who cracks jokes
and laughs and promises to make you dissolve your worries in helpless laughter.
A word of caution. Although laughing is a good exercise for toning up the facial muscles,
laughing at others expense, particularly at their disabilities, is in a bad taste and is to be
avoided. Secondly, laughing with food in the mouth is dangerous as the foodstuff can
get into the windpipe and may choke the digestive system.
Eating, anyway, is a serious business not to be trifled with by any jocular diversion.
Laughter comes best when it is free of encumbrances, whether it is constricting food or
the need to humour the boss.

a) On the basis of your reading of the above passage, make notes using headings and
sub-headings. Use recognizable abbreviations where necessary.
(5)

b) Make a summary of the above passage in not more than 80 words using the notes
made.
(3)
Section B (Writing)

35 Marks

3. You have been asked to draft an advertisement for a new model of solar cooker. Draft
the advertisement.
(5)
Or
You are Sneha. You have been invited by the youth club of your locality to act as one of
the judges for an inter-school group song compition organized by the club. Write to the
secretary of the club accepting the invitation.

4. Your school has celebrated ‘Library Week’. As the Head Boy/Head Girl of your school
write a report on it to be published in your school newsletter.
(10)
Or
You are Anjali/Amitabh of Jai Bharat Sr. Secondary School, Agra. Your school had the
unique privilege of having been invited to witness the dance drama, ‘Bharat Jaag Utha’
in the Red Fort, Delhi on the occasion of the 60th anniversary of India’s independence.
Write a report to be read out at the morning assembly of your school. The report should
not exceed 100-125 words

166
5. You are Manish/Manisha No.3 Banjara Hills Hyderbad. Write a letter to the
Superintendent of Nampalli Railway station Hyderabad, complaining about the lack of
proper facilities at the station.
(10)
Or
Write a letter to the Commissioner of Police requesting him to be the chief guest at
your school sports day function to be held at your school play ground. You are Neha,
school pupil leader, Crescent Secondary School, Civil Lines, Bhopal

6.You are a member of the environment club of your school. After visiting many places
you have realized that it is the need of the hour to protect our environment. Write an
article on this topic to create awareness among the people. (150-200words)
(10)

Or
Write an article on the Evils of Dowry System for Indian Express, Delhi. Write your article
in about 150-200 words.

Section C (Literature)

45 Marks
7..Read the extract from the poem and answer the questions that follow:
(4)
“A thing of beauty is a joy forever
Its loveliness increases, it will never
Pass into nothingness; but will keep
A bower quiet for us, and a sleep
Full of sweet dreams, and health, and quiet breathing.”

a) How is a thing of beauty a joy forever? (1)

b) What kind of sleep does it provide? (1)


c) Who is the poet of these lines and what is the name of the poem? (1)

d) Name the poem and the poet. (1)

Or
“When Aunt is dead, her terrified hands will lie
Still ringed with ordeals she was mastered by.
The tigers in the panel that she made
Will go on prancing, proud and unafraid.”

i)Name the poem and the poet. (1)


ii) What are the ordeals that Aunt Jennifer faced in her life? (1)

167
iii) How are the tigers different from her? Why did she create them? (2)
8.Answer any three of the following questions in 30-40 words each:
(2x3=6)
i) What will counting up to twelve and keeping quiet help us to achieve?
ii) How did the travellers on the highways react to the roadside stand?
iii)Why has Kamala Das described the trees as sprinting?
iv) How do the pictures and maps on the wall contrast with the world the slum children
live in?
9.Answer the following questions: (5x2=10)
a) “ And she was jealous of his silence.” Why was Sophie jealous of Geoff’s silence?
b)In the story ‘The Last Lesson’ the people of the village suddenly realize the importance
of their languages. Why?
c)What are the hazards faced by those working in the glass industry?
d) Douglas mother warned him against River Yakima.Why?
e)How can we say that Rajkumar Shukla was a determined and resolute person?
10.Answer the following question in about 125-150 words: (10)
Though Umberto Eco believed himself to be from the academia, he had a taste for
narration. Justify with reference to ‘The Interview’.
Or
‘The peddler receives unexpected and unusual behavior from various people’. Do you
agree with the statement? Justify with reference to the story “The Rattrap”.
11. Answer the following in about 125-150 words: (7)
Pearl S Buck through the story “The Enemy” deals with basic humaneness, which is an
essential quality of all human beings, even in times of war.
Or
Jack, the father, insists that it was the wizard that should be hit and not the mommy.
Why?
12. Answer the following questions briefly: (4x2=8)
a) Though Charlie reached the third level once, why could he not reach there again?
b) In the present times do you think a story like the ‘Tiger King” is relevant?
c) In what way are the phytoplanktons useful to the ecosystem?
d) Why do you think Derek remains to himself and not mingle with anyone?

******************************

168
CONTENTS

S. No. Chapter Page

1. Comprehension Passage 3 – 15

2. Note Making 16 – 53

3. Report or Factual Description 54 – 56

4. Report Writing 57 – 60

5. Letter-Writing 61 – 70

6. Article Speech 71 – 79

7. Text-Books 80 – 139

Sample Question Papers 1 & 2 140 – 162

Board Exam Papers - 2012 163 – 169

4 XII – English
PASSAGE 1
Read the passage given below and answer the questions that follows:
1. New Delhi : It seems the common house sparrow has disappeared from
the city. But ornithologists maintain that while sparrow numbers are dwindling,
the bird has not disappeared entirely from the city and only shifted to more
inhabitable parts. Conservationists are now trying to understand why some
areas have managed to hold back sparrows and what has driven them
away from others.
2. A recent countrywide survey initiative called ‘Citizen Sparrow’ is now roping
in residents who want to report about their experience with sparrows. So far
this unique sparrow survey organized by the Bombay National History Society
(BNHS) and ministry of environment and forests (MOEF) has received close
to 410 responses from Delhi. Of these, 86 have claimed they have not
seen any sparrows at all. But the majority seems to have sighted the bird,
which gives new hope to conservationists.
3. “I have seen lots of sparrows in Ghaziabad, in Sheikh Sarai where I live but
hardly any in say the Greater Kailash area. What is different in GK and
other parts is still a mystery. But certain factors drawing the birds have
become clearer, such as they nest more around old buildings, houses or
may be in houses where there are old electricity meters, kitchen gardens,
shrubs,” says co-in-vestigator, BNHS Citizen Sparrow Project, Koustubh
Sharma.
4. Another conservationist and birder, Ananda Banerjee says she has seen
lots of sparrows in parts of Lutyens’ Delhi, Mayur Vihar, parts of old Delhi
and parts of Noida. He cites urban landscape to be the reason behind the
decline in the sparrow population.
5. "Urban architecture, tall glass buildings that lack nesting spaces for the
sparrow, pesticides used in farming that kill the worms that sparrows feed
on are some of the reasons. Even our markets have changed. There are
not many open markets where they can get grains. But you can see lots
of sparrows in Khari Baoli open grain market," he says.
6. But there is no doubt among conservationists that sparrows are fast
disappearing. Declining number of sparrows and their complete absence
from some parts of the city isn't just about missing the tiny bird. It is an
indicator of something much graver.

5 XII – English
7. Ecologist and forestry expert, Neeraj Khera, who has been studying the
sparrow population in Delhi, feels that sparrows are an important indicator
species. "There is always a threshold level. Big changes like an epidemic
outbreak for instance will not happen overnight, but when we cross the
buffer line then changes take place in our ecosystem. Sparrows as an
indicator species is very sensitive to change. So it's obvious that a lot must
have changed in our ecology to have driven them away," says Khera.
8. Some of the important factors responsible are air and water pollution, loss
of native herbs and shrubs.
9. Another trend being noticed by experts is the increase of rock pigeons in
most parts of Delhi. They seem to have almost replaced the sparrows that
used to nest in the same places.
10. "Rock pigeons have almost grown out of proportion and taken up the space
of sparrows. They can be seen nesting in houses and other buildings.
Studies have shown that it is not a welcome change as the excreta of rock
pigeon carries a lot bacterial pathogens," said Khera.
11. She says that sparrow population is moderate in places where there are old
government buildings, water bodies or green spaces. Saving the sparrow,
she says is not a lost cause yet. It is the right time to intervene and check
further decline. It will take 'Citizen Sparrow' survey a couple of more months
to come up with the preliminary analysis of the results from their - survey
and may offer a stronger argument on why the sparrows are disappearing
from the city
Source : The Times of india
A. (a) What is ‘Citizen’s Sparrow’ and who has initiated it? 2
(b) What has increased the hope of conservationists? 2
(c) According to survey study, which type of house is sparrow’s first choice
of nesting? 2
(d) How use of pesticides in farming responsible for decrease in sparrow’s
population? 2
(e) What did the ecologist Neeraj Khera observe? 1
B. Find out words from the passage which means the same as the following:
(1x3) = 3
(a) becoming less or smaller (Para 1)
(b) secret/observe/impossible to understand (Para 3)
(c) picture of inland scenery (Para 4)

6 XII – English
PASSAGE 2
1. Read the following passage carefully and answer the questions that
follow :
1. Republic is essentially a nation-state in which supreme political power vests
in the people and in elected representatives given a mandate to govern,
by those people. Most importantly–and this is the principal point of difference
from a monarchy–republics have an elected or nominated head of state,
usually a president, not a hereditary monarch. In effect, all sovereignty,
power and authority in a republic are vested in the people.
2. ‘Republic’ is derived from the Latin phrase res publica—“a public thing”.
Ancient Romans used this to describe the wellspring of their governance
system for their city-state by about 500 BC. Inspired by notions of Athenian
democracy, Rome’s republic was a noble experiment.
The inscription ‘SPQR’, emblazoned on all Roman standards and public
buildings, expanded to ‘The Senate and People of Rome’. It touted to the
world that Roman political power was vested in a great many, not
concentrated in one ruler or family.
Rome’s republican tryst, sustained by public elections and classical debate,
lasted until Julius Caesar seized control in 44 BC. Being succeeded by his
wily nephew, Augustus–who founded a famous empire that lasted a while
longer–300 consigned the republican ideal to the dustbin of the world.
3. Rome took much of its republican template from Greece. In particular, from
Athens, most luminous of ancient Greece’s many city-states. The notion of
moving political power away from an individual to the masses sparang from
the need to safeguard the then ‘new’ notion of personal and individual
freedom. It meant citizens would willingly join any battle to safeguard this
freedom from any aggressor. But it was a troubled ideal. Athens ran on
slave labour, democracy became limited to narrower sections as time went
by. Tyranny and mobrule reared their ugly heads; Athenian imperialism
overstretched the city-state so much so that even Plato and Aristotle, in
effect, argued for enlightened oligarchies in their political philosophy.
4. Aristotle’s star pupil, Alexander of Macedon, soon put paid to all notions of
republicanism by conquering large parts of Eurasia to establish an empire
so large that it would only truly be eclipsed by Rome’s later rise.

7 XII – English
5. Besides the many obvious fruits of Renaissance and Reformation–Europe’s
two most epochal events in the second millennium–the republican ideal
owes much to Niccolo Machiavellie and John Locke. Machiavelli, a 15th
century Italian statesman-writer, located sovereignty in a collective exercise
of power. The governed would guide actions of their ideal governor, he
argued forcefully. Little wonder that Rousseau later referred to Machiavelli’s
‘The Prince’ as “a handbook for Republicans’.
6. Locks, 17th century England’s most notable philosopher, cut through
mythological mumbo-jumbo to argue that true power must formally lie with
the people. A ‘contract’ existed between rulers and people, that bound both
to establish “directed to no other end but the peace, safety, and public
good of the people”.
7. The rise of England’s parliament soon after injected a strong republican
element into its bodhpolitic Modern liberalism–which sprang from Locke’s
work–did the same in most of the western world.
8. Two revolutions, one decade and two continents apart, brought forth two
republican models the world still looks to. The American, in 1776, and
French in 1789. The first saw England lose its earliest colony. Monarchy
was sternly repudiated and the ideas behind the Declaration of Independence
exploded onto the western world as a serious alternative whose time had
come.
9. The declaration laid the basis for much republican-democratic ideation.
The US’s new constitution firmly located power with the people by stating
that governments derived “their just powers from the consent of the
governed”.
10. The French Revolution brought French monarchy, and all its attendant
power structures, to a violent end, sending shockwaves through European
kingdoms. The new republic’s bloody convulsions and military campaigns–
for liberty, equality, and national self-aggrandizement–spread the spirit of
revolution. Even under Napoleon Bonaparte. France would flirt with monarchy
again but remained firmly democratic and republican is spirit ever after.
11. Nationalism soon proved a potent new force, redrawing Europe’s map several
times over. Old power structures were found severely wanting. All big
European monarchies made room for democratic representation.
12. Given India’s long history of fractious monarchies and mighty empires, the
freedom movement set itself in democratic tradition. That wish came to final
culmination of January 26th, 1950, when the nation was declared a republic
and given the world’s most comprehensive Constitution to abide by. But
India may not be a stranger to this ‘western-inspired’ system. Historical
research has shown, but not proven, that some city-states in north India

8 XII – English
between 500 BC and 400 AD might have actually been ancient republics
of a sort. The Licchavi state, a Buddhist Kingdom with ganas and sang-
has–normally translated as republics, but best referred to as ‘selfgovernment
multitudes’–was the most prominent. A good to hold as India basks in 60
years of republican glory. After all, that’s no mean achievement.
A. Questions
(a) How is a republican state different from Monarchy? 2
(b) How did Julius Caesar change the face of a republican state? 2
(c) What are the major gifts of French Revolution? 2
(d) Mention the different forms of Republics started. 2
(e) How did India get its first Republican state. 1
B. Find out the words which mean the same as the following : (1×3) = 3
(a) ruler (para 1) (b) student (para 4)
(c) powerful (para 11)

9 XII – English
PASSAGE 3
Rarely have homemakers found a place in India's policy. But an innovative
scheme in Goa's budget introducing a grant for them, thereby officially
acknowledging their invaluable contribution, could change all that.
The scheme, likely to be rolled out by July, proposes to give Rs 1,000 per
month to all homemakers with annual household income of below Rs 3 lakh,
benefiting some 1.25 lakh families. "Homemakers are finding it difficult to manage
households in these times of inflation. This is meant to empower them," says an
official of the women and child development ministry.
Bicholim-based Nutan Sakalkar, a homemaker, is overjoyed at the prospect
of getting some money of her own. "We get scant respect in our society," she
says. "Working women are often absolved of household responsibilities, but no
one gives a thought for the work we put in 24X7." She feels the grant will bring
back her sense of independence. "Though he never refuses, I feel guilty asking
my husband for money."
The role of homemakers has been a contentious issue since the '70s.
Can their contribution be regarded as work? Does it warrant monetary
compensation? While this is perhaps the first time a government scheme has
directly targeted homemakers, the struggle to bring them true dignity is a long
way off.
"The profits of society today are subsidized by the unpaid work of women
all over the country. Not only do they form the care economy but they produce
the workers of tomorrow," says economist Vibhuti Patel of Mumbai's SNDT
Women's University. A grant, she believes, would merely help some of them
move from starvation to subsistence. But what they should be given is a direct
stake in their husbands' pay cheques, she feels.
Madhu Kishwar, editor of women's journal 'Manushi' , too, doesn't believe
doles can help empowerment. "Why should the government pay homemakers?
I would consider it an insult. Women are the gruha lakshmis and should be
treated as equals and given the charge of household finances," she says.
The struggle for gender equality is reflected in the National Family Health
Survey III (2005-06 ). Not only are fewer women counted in the workforce as
compared to men (some 43% of married women in the 15-49 age group were
employed as compared to 99% men), but one in four didn't receive any payment
as compared to 1 in 20 men.

10 XII – English
The crisis of dignity isn't restricted to India alone. In April, there was an
outcry against Democratic strategist Hillary Rosen's comments on Republican
candidate Mitt Romney's wife when she questioned the latter's status as an
ambassador on women's issues because she had "never worked a day in her
life" . Romney is a stay-at-home mum of five children. Rosen's comments went
viral with many on Twitter protesting, forcing an apology from her. Even Michelle
Obama tweeted, "Every mother works hard, and every woman deserves to be
respected."
The Goa model is one among many social experiments underway globally.
Venezuela recognizes housework as 'work' under its Constitution and pays
homemakers 80% of minimum wages.
Whether such initiatives will bring respect to housework waits to be seen.
Source : The Times of India

A. Questions
(a) Which innovative scheme did Goa’s budget introduce recently? 2
(b) What does the scheme intend to? 2
(c) What do the housewives like Bicholim think of working women? 2
(d) What does Vibhuti Patel of SNDT women’s university advocate for
home makers? 2
(e) What does Madhu Kishwar suggest? 1
B. Find out the words from the passage which means same as the following:
(1x3) = 3
(a) new/changed (para 1)
(b) declared from (Para 3)
(c) money distributed in small amounts (Para 5)

11 XII – English
PASSAGE 4
1. This year marks a great milestone in the human saga, similar in magnitude
to the agricultural era and industrial revolution. For the first time in history,
a majority of human beings will be living in vast urban areas, according to
the United Nations-many in mega cities and suburban extension with
populations of 10 million people more. We have become Home Urbans.
2. Millions of people huddled together and stacked on top of each other in
gigantic urban centres is a new phenomenon. Recall that 200 years ago,
the average person on earth might have met 200 to 300 people in a
lifetime. Today, a resident of New York City can live and work amongst
2,20,000 people within a 10 minute radius of his home or office in midtown
Manhattan. Only one city in all of history-ancient Rome-boasted a population
of more than a million inhabitants before the 19th century. London became
the first modern city with a population of over one million people in 1820.
Today, 414 cities boast populations of a million or more people and there’s
no end in sight to the urbanisation process because our species is growing
at an alarming rate. Around 3,76,000 people are born every day on earth.
The human population is expected to increase to nine billion by 2042, most
living in dense urban areas.
3. No one is really sure whether this profound turning point on human living
arrangements ought to be celebrated, lamented, or merely acknowledged
for the record. That’s because our burgeoning population and urban way
of life has been purchased at the expense of the demise of the earth’s vast
ecosystems and habitats. Cultural historian Elias Canetti once remarked
that each of us is a king in a field of corpses. If we were to stop for a
moment and reflect on the number of creatures and earth’s resources and
materials we have expropriated and consumed in our lifetime.
4. Large populations living in mega cities consume massive amount of the
earth’s energy. To put this in perspective, the Sears Tower alone, one of
the tallest skyscrapers in the world, uses more electricity in a single day
than a town of 35000 inhabitants. Even more amazing, our species now
consume nearly 40 per cent of the net primary production on earth-net
amount of solar energy converted to plant organic matter through
photosynthesis-even though we only make up one half of 1 per cent of the
animal biomass of the planet.
5. It’s no accident that as we celebrate the urbanisation of the world, we are
quickly approaching another historic watershed, the disappearance of the
12 XII – English
wild. Rising population, growing consumption of food, water and building
materials, expanding road and transport, and urban sprawl continue to
encroach on the remaining wild, pushing it to extinction. Our scientists tell
us that within the lifetime of today’s children, the wild will disappear from the
face of the earth after millions of years of existence, The Trans-Amazon
Highway is hastening the obliteration of the last great wild habitat. Other
remaining wild regions from Borneo to the Congo Basin are fast diminishing
with each passing day. It’s no wonder that according to Harvard Biologist
EQ Wilson, we are experiencing the greatest wave of mass extinction of
animal species in 65 million years. By 2100, two-thirds of the earth’s remaining
species are likely to become extinct.
6. Where does this leave us? Try to imagine 1,000 cities of nearly one million
or more inhabitants in 35 years from now. It boggles the mind and is
unsustainable for the earth. Perhaps the commemoration of the urbanisation
of the human race in 2007 might be an opportunity to rethink the way we
live on this planet. Certainly there is much to applaud about urban life : its
rich cultural diversity and social inter course and dense commercial activity
come readily to mind. But the question is one of magnitude and scale. We
need to ponder how best to lower our population and develop sustainable
urban environments that use energy and resources more efficiently, are
less polluting, as well as better designed.
7. In short, in the great era of urbanisation we have increasingly shut off the
human race from the rest of the natural world in the belief that we could
conquer, colonise and utilise the rich largesse of the planet to ensure our
complete autonomy and without dire consequences to us and future
generations. In the next phase of human history, we will need to find a way
to reintegrate ourselves back into the rest of the living earth if we are to
preserve our own species and conserve the planet for our fellow creatures.
A. Questions
(a) What is the significance of the present year in human history? 2
(b) How is life different today from the life in Past? 2
(c) Why should we lament at the growth of population and not celebrate
the progress? 2
(d) How is the urbanization harmful for the natural resources? 2
(e) What is the warning given by our scientists for future? 1
B. Find out the words which mean just the opposite of the following: (1×3) = 3
(i) small (para 1) (ii) shrinking (para 5)
(iii) thin/scanty (para 6)

13 XII – English
PASSAGE 5
1. The first written public examinations were introduced over 2,000 years ago
with the establishment of the imperial examination system in 606 AD in
China. By the middle of the 19th century, competitive examinations had
been introduced in Britain and India to select government officials. Public
examination in schools have a shorter; but still considerable, history.
2. At present, the examination system in India in characterised by heterogeneity.
They differ in their vintage, organisational design, financial stability, autonomy,
organisational culture and credibility. For instance, the National Institute of
Open Schooling is unique in conducting exams through the distance mode;
the Indian School Certificate Examination confines itself to only private
schools; and the West Bengal Board of Madrasa Education is for Islamic
educational institutions only. In a situation where public funds are involved,
the equity, efficiency and transparency of this expenditure need to be
assessed to ensure that the taxpayers’ money is being spent with due care
and concern. The Amrik Singh report (1997) on reorganising boards
wondered, “Can there also be a system whereby it becomes possible to
grade and categorise the boards in respect of how efficiently and honestly
examination are organised?”
3. To do so, a set of parameters were chosen to measure effectiveness, quit,
transparency and economic efficiency of the functioning of these
organisations. These include indicators like cost incurred per students, fee
charged per student, number of examinees per employee and number of
affiliated schools per employee. An analysis was made of the boards chosen
on the basis of these parameters. The functioning of a total of 20 boards
was analysed in terms of 18 performance indicators in 2005.
4. The data collected from the states shows that most boards have an operating
surplus. However, the boards seem to be more sensitive to the needs of the
disabled. All of them have some special provisions for such candidates.
While these provisions vary widely in scope, most provide for a longer
duration of examination, concessions in the examination of languages. The
CBSE, along with the Maharashtra board, seem to fare the best.
5. Another important measure of effectiveness of the functioning of the boards
is the time taken for declaration of examination results. It was found that
this ranged from a mere 26 days in the case of Karnataka Secondary
Education Examination Board in 2005 to 48 days for class X in neighbouring

14 XII – English
Maharashtra. The Supreme Court has recently issued directions that results
should be declared within 45 days from the end of examinations. In 2005,
10 of the 20 boards, which provided this information, could not declare
results within 45 days.
6. The reason for such a wide variation in performance in such a vital parameter
was the processes utilised. Boards, which have adopted a centralised system
of assessment in comparison to the earlier system of sending answer sheets
to examiners, have an obvious edge. Similarly many boards have outsourced
the work of compilation of results faster and used technology extensively.
These boards are also able to declare results. In addition, agencies, which
have consciously designed question papers with a high proportion of multiple-
choice responses, do not need much time for evaluation. In fact, the
Karnataka board has 60 percent of all the questions as objective type.
7. The quality of assessments is measured by the reliability of results. Even
minor errors serve to destroy confidence in exams. Considerable variation
was found between boards on this measure. The criteria was the number
of applications made for scrutiny of marks after results had been declared,
and the number of corrections made subsequently by the board. Of the 18
boards for which such figures were available, over 40 per cent had more
than 10 per cent such corrections. The Council for the Indian School
Certificate Examinations reported the lowest number; the highest number of
mistakes found in the results declared was by the Bihar Intermediate
Education Council at a shocking 60.36 percent. Surprisingly, the Bihar
School Examination Board fared much better. It would be interesting to
explore the reasons for such as vast difference between the functioning of
two similar organisation in the same state.
8. A similar discrepancy was noticed in Assam where the Higher Secondary
Education Council reported an error level of 19.14 per cent while the Board
of Secondary Education had only 7.27 per cent errors. The third such case
was in West Bengal where the West Bengal Board of madrasa Education
had an error rate of 8.81 per cent while the West Bengal Board of Secondary
Education reported a much higher rate of corrections made in the results
declared of only 24.92 per cent.
9. The Karnataka Secondary Education Examination Board exhibited interest
in results. It has the lowest cost per examinee and also manages to declare
results the fastest.
A. On the basis of your reading of the above passage answer the following:
(a) How have the Chinese played an important role in enhancing the
education system? 2
(b) Mention any four distinguishing features of examination in India? 2

15 XII – English
(c) What are the basic parameters to measure effectiveness in functioning
of an organization? 2
(d) What is the time limit fixed by Supreme Court for declaring the results? 1
(e) What efforts are being made by different Boards to declare the results
at the earliest? 1
(f) What factors make Karnataka secondary school examination Board
superior to the other Boards? 1
B. Find out the words which mean the same as the following : (1×3) = 3
(a) keep on hold/restrict (para 2)
(b) easily seen / clear (para 6)
(c) thorough and detailed examination (para 7)

16 XII – English
NOTE MAKING
Characteristics of Notes
❑ Short
❑ Include the important information in brief
❑ Logically presented in sequence
❑ Organised appropriately under Heading and Sub Heading.
Mechanics of Note-Making
(a) Use of Abbreviations :
(i) Capitalized first letter of words : UNO, CBSE, NCERT, etc.)
(ii) arithmatic symbols :
(><, ,↑↓, ∴ kg., % etc.)
(iii) Commonly used : (in newspapers, magazines etc.)
(sc., govt., Eng., Sans.)
(iv) Invented : First and last few letters of the words with a dot at the
end (edul, poln., popn. mfg.).
(b) Proper Indentation
Heading (Title)
1. Sub Heading .................
a. — (Point)
b. — (Point)
c. — (Point)
c.i. — (Sub-point)
c.ii. — (Sub-point)
2. Sub-Heading
a. —

17 XII – English
(c) Make use of words and phrases only. Avoid full length sentences.
(d) Give appropriate Title. The title may be given at the starting of notes
or before summary which is of 3 marks and should not be more
than 1/3 of the passage’s length.
Marking Scheme / Pattern for Note Making and Summarising
1. Notes — Heading / Title 1 Mark
2. Abbreviation / Contraction 1 Mark
(Minimum four)
3. Sub-Headings (Two-three as per the requirement) alongwith 2-3
points. (1½×2) = 3
4. Summary — Content – 02
Expression – 01
(Standard word limit for Sumary is 80 words or around 1/3 of the
passage)
Note : In case a student uses his/her invented contraction, he/she is
required to provide key for the same as —
prblm. = problem
trp’nt. = transparent
rqd. = required
dprsd. = depressed
* Notes / Points of students may vary from one another but one should
ensure that the main ideas/views are covered.

18 XII – English
PASSAGE 1
Q. 2. Read the passage given below :
In a very short period of time the internet has had a profound impact on
the way we live. Since the Internet was made operational in 1983, it has lowered
both the costs of communication and the barriers to creative expression. It has
challenged old business models and enabled new ones. It has provided access
to information on a scale never before achievable.
It succeeded because we designed it to be flexible and open. These two
features have allowed it to accommodate innovation without massive changes to
its infrastructure. An open, borderless and standardized platform means that
barriers to entry are low, competition is high, interoperability is assured and
innovation is rapid.
The beauty of an open platform is that there are no gatekeepers. For
centuries, access to and creation of information was controlled by the few. The
internet has changed that --and is rapidly becoming the platform for everyone,
by everyone.
Of course, it still has a way to go. Today there are only about 2.3 billion
internet users, representing roughly 30% of the world's population. Much of the
information that they can access online is in English, but this is changing rapidly.
The technological progress of the internet has also set social change in
motion. As with other enabling inventions before it, from the telegraph to television,
some will worry about the effects of broader access to information -- the printing
press and the rise in literacy that it effected were, after all, long seen as
destabilising. Similar concerns about the internet are occasionally raised, but if
we take a long view, I’m confident that its benefits far outweigh the discomforts
of learning to integrate it into our lives. The internet and the world wide web are
what they are because literally millions of people have made it so. It is a grand
collaboration.
It would be foolish not to acknowledge that the openness of the internet has
had a price. Security is an increasingly important issue and cannot be ignored.
If there is an area of vital research and development for the internet, this is one
of them. I am increasingly confident, however, that techniques and practices
exist to make the internet safer and more secure while retaining its essentially
open quality.

19 XII – English
After working on the internet and its predecessors for over four decades,
I'm more optimistic about its promise than I have ever been. We are all free to
innovate on the net every day. The internet is a tool of the people, built by the
people for the people and it must stay that way.
(a) On the basis of your reading of the above passage make notes on it
using recognisable abbreviations (minimum four) wherever necesary.
Use a format you consider appropriate. Supply a suitable title.
(5 marks)
(b) Write a summary of the passage in about 80 words. (3 marks)

PASSAGE 2
A. Read the following passage for note-making.
For many of us, our lives are composed of millions of meaningless moments,
all strung together–perhaps with a sprinkling of sacred moments mixed in. I am
sure you can think of a few sacred moments in your own life.
Maybe your marriage, the birth of your child or perhaps a heartfelt moment
of connection with good friends. These are the moments when we are consumed
with joy and awe. At these moments, we are fully present in the moment. We
are’t worrying about tomorrow, or trying to rush through he experience to get to
the next. We are in the now, and the now is amazing.
Why do we wait for major events to honour these sacred moments? Why
can’t every day be sacred? Every moment? Each moment is sacred, if we
decide to make it that way. I stated above that our lives are filled with meaningless
moments, but hose moments are meaningless only because we don’t honour
them. We are hurrying along, focused on other things, not stopping to notice
them at all. They come and go without any acknowledgment from us.
Honoring the sacred means simply choosing to make each moment count.
It is a tiny shift in perception that assigns meaning and importance to each
moment. It means stopping to notice the beauty and love around us, and within
us. It means being fully present in each moment, because only the present
moment truly exists.
So many of us worry about the future, hold regrets about the past, and
completely ignore the present. When we give the now our full attention even
mundane tasks can take on an aura of holiness. Then our lives become millions
of meaningful moments. Nothing has changed except our perception, but it
makes all the difference.

20 XII – English
How do we change our perception to one of sacredness? What does it
mean to stay in the present moment? Let’s use an example : washing dishes.
When most of us wash dishes, we hurry through it, often thinking about the
million other things we need to get done that day (or the next day). Our minds
are scattered all over the place, focusing on everything but what we are doing.
Who can blame us? Washing dishes is not the most exciting experience. However,
if we choose to make it a sacred experience, we will want to focus on it.
We want to take our time and really pay attention to how the water feels
on our hands, how the soap cleanses away the grease and grime, and the
sense of satisfaction we get as each dish moves from the dirty pile to the
strainer, now clean and shiny.
Maybe that still doesn’t sound very thrilling. Let’s look at what else is
happening. As we give our full attention to washing the dishes, we are not only
doing, we are being. Ah, that is the key. It doesn’t matter what we’re doing or
not doing in each moment of our lives, It matters only that we become aware of
our own state of Being. In those moments, we are fully alive and conscious. We
are connected to everything, and we are one with everything. We are.
Have you ever embarked on a walking meditation? This can be great
practice to help you honour the sacred in your own life. Many of us walk for
health or recreation, but we can also walk for spiritual awareness. A walking
meditation is a slow. Focused walk where you give your full attention to each
step you take, the feel of your muscles working, your breathing, and even your
surroundings.
Especially if you walk someplace in nature, a place with lots of trees and
wildlife. The trick is to not let your mind wander, but stay focused on the now.
It can take practice, but once you accomplish it, you will return from your walks
fully refresh and revitalised.
(a) On the basis of your reading of the above passage make notes on it using
recognizable abbreviations (minimum 4) wherever necessary. Use a format
you consider appropriate Supply a suitable Title. (5 Marks)
(b) Write a summary of the passage in about 80 words. (3 Marks)

21 XII – English
PASSAGE 3
Excessive use of plastic bags and their unregulated disposal has been
choking lakes, ponds and urban sewage systems, the Supreme Court said on
Monday while warning that it posed a threat more serious than the atom bomb
for the next generation.
This observation from a bench of Justices GS Singhvi and S J Mukhopadhaya
came on a PIL filed by two Andhra Pradesh-based NGOs drawing the court's
attention to 30-60 kg of plastic bags recovered from the stomachs of cows
because of irresponsible disposal of plastic bags and defunct municipal waste
collection system.
The court issued notice to the Centre and state governments on the PIL
seeking ban on use of plastic bags in municipal areas which did not have a
prompt garbage collection, segregation and disposal system. The NGOs said
absence of a proper system allowed cows to rummage through garbage bins
and eat foodstuff disposed of in plastic bags, which get stuck in their stomach.
"Apart from the plastic choking the digestive system of the plastic residues enter
the human food chain through dairy and animal products," he added.
But the bench wanted to address the larger questions arising from
indiscriminate use of plastic bags, which not only posed a grave threat to nature
and environment but also to the human race itself. It suggested that the petitioner
make the manufacturers and a television channel, which has been running a
campaign against use of plastic, parties to the PIL for a wider scrutiny of the
important issue.
“We want to expand the scope of this petition. Unless we examine a total
ban on plastic bags or put in place a system for manufactuers mandating them
to collect back all plastic bags, the next generation will be threatened with
something more serious than the atom bomb,” the bench said. The court also
drew the petitioner’s attention to large quantities of water packed in plastic
pouches, which were thrown around in undisciplined manner across the country
every day, “A rough estimate shows more than 100 million water pouches are
thrownaway,” the bench said.
(a) On the basis of your reading of the above passage make notes on it using
recognizable abbreviations (Minimum four) wherever necessary. Use a format
you consider appropriate. Supply a suitable title. (5 marks)
(b) Write a summary of the passage in about 80 words (3 marks)

22 XII – English
PASSAGE 4
A. Read the passage given below for Note making :
The Trail of smoke in most cases inevitably leads to school. And college
days-those adolescent times when mere lighting up gave them the thrill of
indulging in the prohibited. And yes, they weren‘t mama’s boy (or girls) anymore;
they were macho and grown up beyond listening to nannies. Or so they thought,
till they got addicted.
There is a clear link between the youth and tobacco addiction. Statistics
reveal that many children are initiated into the habit of smoking at the tender
age of 10 years, according to Dr. Srinath Reddy, a researcher-activist.
That’s why it makes sense to stop them young, when they are vulnerable
to peer pressure. Smoking, which often starts as an experiment in the company
of friends often transforms into addiction.
According to the Non-Smoker’s Health Protection Act 1997 nobody is allowed
to store, sell or distribute cigarettes, beedies or any other tobacco product
within an area of 100 m around colleges, schools and other educational
institutions. The Indian Parliament passed another comprehensive legislation,
the Cigarette and Other Tobacco Products Act, a few years ago banning the
sale of tobacco products to minors.
Has that made any difference? “You have to go to north campus to see if
any law is followed.’
Many students even argue that it is a matter of personal choice. “Once in
college, a student is old enough to take decisions. It is clearly mentioned on
cigarette packs that ‘smoking is injurious to health’ and after that if someone
smokes, it is completely the individual’s choice,” says Manu Singh, a student at
JNU.
Sad reality is that nothing has been able to stop students from smoking in
schools and colleges. It’s fashion.
The law enforcement agencies takes refuge behind pleas like they have
largeareas to cover with a small term. Their strength, they say, is not adequate
to enforce laws. “Sometimes people pay the fine (Rs 200), which is hardly a big
amount. It universities and colleges, raids are not possible. With just one person
from the police, it becomes impossible to control them,” said Dr. M.D. Thapa,
Chief District Medical Officer, Northwest district.

23 XII – English
Advocate Ashok Agarwal does not buy the argument According to him,
there is a clear lack of interest on the part of the lawmakers. “The police and
the administration have their own priorities hence they have little time to look
into these sensitive issues. The situation in this case is that of accepted and
agreed violation where just nobody is bothered.” he says.
The one answer the experts agree on is; the government. “There is no
effort to implement the laws,” complains advocate Ashok Agarwal. When society
does not care, the government becomes duty-bound to make them care. “There
is a direct link between the youth and tobacco addiction. They are the key
targets for the MNCs,” says Bijon Mishra, a social activist involved with the NGO,
Voice. While society and its institution ignore those mischievous puffs in school
canteens, the biggest beneficiaries are the tobacco companies.
(a) On the basis of your reading of the above passage make notes on it using
recognizable abbreviations (minimum 4) wherever necessary. Use a format
you consider appropriate Supply a suitable Title. (5 Marks)
(b) Write a summary of the passage in about 80 words. (3 Marks)

PASSAGE 5
A. Read the passage given below carefully for Note-making :
There’s a part of India where the tiger may still have a fighting chance ; the
Western Ghats. The big cat roams free here and in goodly numbers, from the
southern tip right up to Maharashtra, Eight tiger reserves–in Karnataka, Tamil
Nadu and Kerala–have been rated ‘good’ to ‘satisfactory’ by the Centre’s 2009
preliminary status report on the tiger. Experts say this is because of good
governances, constant surveillance and monitoring, pro-active local tribes a
zealous scientific community, habitat quality and contiguity and an excellent
‘prey base’, which means plentiful supplies of deer.
In Mudumalal, for instance, tiger numbers are believed nearly to have
doubled in recent times. Field director Rajiv K Srivastava says anti-poaching
watchers patrol the deep deciduous forests round-the-clock. “The wireless network
helps rush them to vulnerable areas when they receive information about
movement of suspected poachers,” he adds. Each watcher, mostly from a local
tribe, covers 15-20 km daily.
The tiger has also returned to Sathyamangalam sanctuary–erstwhile
Veerappan country–after two decades. Some say this is because the guns have
fallen silent, along with rising tiger numbers in adjoining Mudumalal and Bandipur;
which sends the animals looking for more area to roam. Scientists working in the

24 XII – English
field spotted two tigresses with five cubs at two different locations last year.
Forest officers estimate that there are at least 10 tigers in the division.
The 2008 status report on tigers by the National Tiger Conservation Authority
and Wildlife Institute of India estimates tiger numbers in the Western Ghats of
Karnataka, Tamil Nadu and Kerala at 402, with a lower limit of 336 and upper
limit of 487. The Bandipur and Nagar-hole tiger reserves are almost full
“Highquality research on tigers and their prey base has resulted in a pool of
scientific data which facilitates reliable monitoring” says Ravi Chellam, country
director, Wildlife Conservation Society (WCS), India programme, WCS staff range
across 22,00 sq km of forest in Karnataka, tracking tigers to gather data from
the field. Every quarter, the WCS shares data with the Karnataka forest
department. “Strict protection of the forests by using science is the hallmark of
tiger conservation in Karnataka,” says Chellam.
Recently, WCS scientists led by Ullas Karanth used high-tech fecal sampling
to tally and assess numbers. Tiger scat is thought to provide a unique DNA
signature allowing researchers to accurately identify individual animals.
Another encouraging sign are tiger sightings in non-contiguous areas. This
indicates the presence of a “meta-population,” i.e., tigers who move from one
reserve to another, thereby improving the gene pool. This gives conservationists
reason to hope that another not waiting to happen in the south.
In the Eastern Ghats, the Nagarjunasagar-Srisilam Tiger Reserve in Andhra
Pradesh is back from the brink. The centre’s report damned the reserve as
‘poor’. The naxalite presence threatened the tiger’s core habitat for more than
a decade and foresters could not enter the area. But the tiger population inched
up to 53 in 2008 form just 34 in the nineties. “The Naxal presence is still there.
But the forest field staff have started going inside for habitat improvement, a
vast change from the time when no kind of administration existed there,” says
AK Nayak, the field director.
But there are reasons to worry as well. At a recent seminar in Chennai, the
chief wildlife wardens of the southern states admitted they did not have enough
trained staff to take on poachers. In the rainforest habitats of Kalakad-Periyar
and Anaimalal-Parambikulam, low tiger density can be reversed only if the prey
base is protected. “The time has come for the foresters to go back to old-
fashioned conservation, that is physical protection of forests, leaving development
to other departments.”
(a) On the basis of your reading of the above passage, make notes on it using
recognizable abbreviations (minimum 4) wherever necessary. Use a format
you consider appropriate. Supply a suitable Title. (5 Marks)
(b) Write a summary of the passage in about 80 words. (3 Marks)

25 XII – English
Section ‘B’ : ADVANCED WRITING SKILLS
(Coverage 35 Marks)

Advertisement
POINTS TO REMEMBER : Advertisements are of two types —
Personal purpose
(i) Classified Categorised in separate columns,
occupy less space, more
economical. Written in short
phrases and words, Language-
simple, concise and compact, give
title and contact address.
(ii) Non Classified or Display Commercial purpose, more space,
visually attractive, with catchy
slogans, Varying font size or
shape, (language-colourful), lucid
and appealing overall
arrangement.
(b) Classified Advertisements are covered under different headings:
– Situation vacant/situation wanted
– Sale and Purchase of Property/vehicles/Household Goods etc.
– Missing persons/Pet animals.
– Lost and found
– Travels and Tours/Packers and Movers.
– Educational Institution/Language course/Hobby classes.
– Matrimonials / Alliance.
– Change in name
Points to Remember :
❑ Keep Heading — Situation Vacant / Wanted
❑ Begin with ‘Wanted’ or ‘Required’.

26 XII – English
❑ Mention name of the employee with address.
❑ Mention the no. of posts lying vacant/name of the post.
❑ Mention required age limit and qualification (educational and
professional). Skills, personality and minimum experience required.
❑ Pay scale and perks of the post concerned
❑ Mode of applying and whom to apply
❑ Time limit for submission of application
❑ Contact address and Ph. No.
Example :
1. You are the General Manager of a leading industrial concern. You need
a Chartered Accountant for your office. Draft an advertisement in not
more than 50 words to be published in The Times of India, New Delhi,
under the classified columns.

Situation Vacant
Required a Chartered Accountant for a leading concern “Brakes and
Brakes India Ltd.”. An individual with CA degree, having two years
experience of an MNC and not more than 35 years can apply to the
General Manager with complete resume within seven days. Attractive
salary and promising career. General Manager, Brakes and Brakes Pvt.
Ltd., 116 Sect. 62, Noida, Ghaziabad.
2. You are the manager, Excel Pharma Ltd. Draft an advertisement for your
company for the post of Sales Executive (two) mentioning all details as
per your requirement.

Advertisements
3. Delhi Public School Rohtak, Haryana needs four post graduate teachers
(2 English, 1 Physics, 1 Maths) for the school. As the chairman of the
school draft a classified advertisement for a leading newspaper inviting
applications from suitable candidates. Include details like essential
qualification, experience, salary, working condition etc. the mailing
address of the school is P.O. Box-516, Civil Lines, Rohtak, Haryana.
4. Vasant International school requires a receptionist. The administrative
office drafts an advertisement for publication in the situation vacant
column of a newspaper. Draft an advertisement inviting application for
walk-in interview along with the testimonials.
27 XII – English
Situation Wanted
1. Arun Gupta of M-3, Neel Kamal Appt. ‘Shimla, has a very good knowledge
of tourists places in Shimla. He wants to work as a Tourist Guide. Draft
a suitable advertisement for publication in the ‘Tours and Travels Section’
of a local newspapers in about 50 words.

Situation Wanted
A Tourist Guide widely travelled, well acquainted with hill and historical
places around Shimla. Well-versed in English, Hindi, Punjabi and Local
Language seeks a job in the Local Tours and Travels. Desirous Party
may contact Arun Gupta. M-3, Neel Kamal Appartment Shimla or Call
at 9868....(M).
1. You are A.L. Bhagat of C-108 Shakti Nagar, Delhi. You are Chartered
Accountant and you are capable of handling accounts and Managing
Finances, having six years of experience. Draft an advertisement for the
Times of India seeking a job (full time/part time)
2. You are Mohan Garg. You intend to start hobby classes during the coming
summer vaction at your residence. Write an advertisement to be
published in Hindustan Times. (50 words)
Hints - (Name hobby classes, time, date to start, qualified faculty, mode
of admission contact No etc.)
(a) PROPERTY
❑ Begin with for Sale/Purchase or Available/Wanted.
❑ Type of house (Flat, independent floor/house/office etc.)
❑ Size-floor. no. of rooms etc.
❑ Fixtures & fittings
❑ Surroundings-centrally located, facing park, nearby market, school,
hospital, bank, bus stand etc.
❑ Use adjectives like airy, well ventilated, fully furnished, brand new,
newly constructed.
❑ Company or Bank lease preferred.
❑ Expected price : fixed/negotiable.
❑ Contact address and Ph. No.

28 XII – English
1. You propose to sell your flat as you are going abroad. Draft an
advertisement for it to be published in Daily Times under classified
columns.

FOR SALE
Available DDA Flat, MIG, First floor, two Bedrooms with attached Baths,
Drawing and Dining with two balconies. Well furnished, airy, East and
park facing. Dilshad Garden, nearby market, school, hospital, bank,
Metro and bus stand. Price 65,00000/- but negotiable. Contact Details
: Om Properties, H Block, Dilshad Garden, 9810......... (M)
2. You own an independent house in West Delhi and want to sell it. Draft
on advertisement for a local daily giving all necessary details.

FOR PURCHASE
3. Suppose you are Amitabh Ambani, Director of Study plus, Australia. You
wish to purchase land for setting up a school in India. Write a suitable
advertisement inviting offers from land owners. Give necessary datails like
plot size, area or location and offer of outsight purchase. Do include your
address in India.
(b) VEHICLE FOR SALE
❑ Model No., Year of Manufacturing
❑ Colour, accessories (New seat cover, mats, music system, A/C etc.)
❑ Single hand driven, good condition, new tyres, non-accidental,
scratchless, milage, authorised CNG kit/diesel version.
❑ Insurance, Road Tax, and Pollution Certificate
❑ Expected price 2,00000/- (fixed/negotiable)
❑ Contact address and telephone No.
1. You want to sell your Maruti WagonR as you are going abroad. Draft an
advertisement for the classified column of a local daily giving particulars
of model, milage, etc.

29 XII – English
FOR SALE
Beginning (Model and Year of Manufacture) : Available Maruti
Wagon RLX 2006 model,
Colour and Accessories : Metallic grey, new seat covers, new tyres,
A/C and stero fitted.
Other Attraction : Single hand driven, scratchless, sparingly used,
beautiful interior. Insurance, Road Tax and PUC.
Selling Price : Price 90,000/- but negotiable
Contact Details : Mahindra Automobiles, Ph. No. 011-2329...........

2. You plan to sell your two wheeler. Draft a suitable advertisement in not
more than 50 words under the classified columns of a local daily, giving
all necessary details.
(c) HOUSEHOLD / OFFICE GOODS
❑ Item, brand and year of manufacturing.
❑ General condition – excellent working condition, looks brand new
etc.
❑ Specify material in case of furniture.
❑ Price offered/expected
❑ Contact address and Ph. No.
1. Your family is moving out of Delhi since your father has been transferred.
You want to sell off some household items at a reasonable rate. Draft
on advertisement for local daily giving necessary details.

HOUSEHOLD GOODS
Beginning : Available
Item Name and Brand : LG fully automatic Washing Machine, model
splash 2006. and Samsumg Colour TV. 21” with remote control;
General Condition : Excellent working condition.
Specific Details : Brand new furniture of Teak Wood five seater
sofaset, six seater dining table and double bed with box.
Selling Price : Reasonable price.
Contact Details : A.B. Singh 43/1, B-Block, Dilshad Garden /
9212734510 (M)

30 XII – English
2. You want to sell off some office furniture such as Table with chair,
Cupboards etc. as you are moving out of Delhi. Draft an advertisement
for a local daily.
TO LET
❑ Begin with ‘Wanted’ or ‘Available’.
❑ Type of accommodation – Size, floor, No. of rooms etc.
❑ Location and surroundings
❑ Rent expected, Bank and company Lease preferred.
❑ Contact Add, email address or telephone No.
1. You want to let out a portion of your newly constructed independent
house. Write an advertisement to be published in the ‘To Let’ classified
columns of the Hindustan Times. (Word Limit : 50)

TO LET
Available on rent a newly constructed house in B-Block, Ashok Vihar,
Delhi. Two bedrooms with attached baths, ground floor, well ventilated
and spacious with wooden work. Walking distance from market, school,
hospital and Metro Station. Expected rent 25,000/- per month
(negotiable). Contact : Lala Ji at 9868.............. (M)

2. Rahul Dev of 28, Dream Land Apartments, Circular Road, Delhi wants
to rent out 2nd floor of his house to students only. Write an
advertisement to be published in the ‘To Let’ column of a local daily.
ACCOMMODATION WANTED
❑ Accommodation required : No. of rooms, size, floor
❑ Location and surrounding : area, colony, facilities
❑ Contact Add. (Ph. No.)
1. You want to purchase an independent house in a posh area. Draft a
suitable advertisement under the classified column, “Accommodation
Wanted”, giving your requirements and capacity to pay.

31 XII – English
ACCOMMODATION WANTED
Required a newly built independent house in a posh area of East Delhi.
Having three bed rooms with attached baths, drawing and dining
alongwith modular kitchen, well ventilated. Fully furnished, 24 hrs.
Electricity and drinking water, parking. East and Park facing, nearby
school, Bank and shopping complex. Rent offered Rs. 15000 per
month. Contact : Mr. R.V. Singh, 90122..........(M)

2. You want to purchase a newly constructed office in a prime location of


Preet Vihar, Vikas Marg. Draft an advertisement as per your requirements
about location and price etc.
MISSING PERSON/PET ANIMALS
❑ Begin with ‘Missing’
❑ Physical description
(a) Person
______ Name, age, sex, height, complexion, built _______ clothes
and other identifying features.
(b) Pet animal ________ name, breed, colour of fur or skin.
❑ Since when and from where missing
❑ reward/message if any
❑ Contact address and Ph. No.
1. You are the sister of a boy who has been missing from his home for the
last two days. Draft an advertisement under the caption ‘Missing’ for a
local newspaper provide necessary details and also offer a reward.

MISSING
Suresh Kumar, 14 years, 5”3’. Wheatish complexion, average built,
wearing white T-shirt and blue jeans, specks, injury mark over the right
eye brow. Missing since 4.8.2012 from Central Market Lajpat Nagar.
Informer will be duly rewarded. Inform Lajpat Nagar Police Station or
Call at : 901032...(M)

2. Your grand mother aged 80 years, mentally upset has been missing for
three days. Draft an advertisement for “Missing” Column mentioning all
details. (Word Limit : 50).

32 XII – English
3. Your Cat named Tessa is missing since 5th Aug. 2012. Draft an
advertisement under the “Missing” Column of a local daily newspaper
providing all details in about 50 words.

MISSING PET
Lovely cat Tessa, White, brown fur, 1 year old, with a red ribbon around
its neck.
Missing : Since 5th Aug, 2012 from the Central Park, Dilshad Garden,
Delhi.
Contact : A.B. Singh if found, finder/informer to be rewarded. Call at
90123......(M)

4. You are Mona of New Friends Colony. Your pet dog named Scooby is
missing since 3rd Aug., 2011. Draft an advertisement for the ‘Pet Missing’
column of a local daily in about 50 words.
6. LOST AND FOUND
❑ Begin with ‘Lost’ and ‘Found’.
❑ Brief physical description ______ Name of the object/article, Brand,
Colour, size, condition.
❑ When/where lost or found.
❑ Reward if any
❑ Contact address and Ph. No.
❑ For ‘Lost’ give all details whereas for ‘Found’ the details need not
be given completely.
1. You have lost your bag in Metro between Uttam Nagar and Dwarka while
going for an interview. It contains a file having important certificates. Draft
an advertisement for a local daily under column ‘Lost and Found’ in not
more than 50 words.

LOST AND FOUND


Lost a black VIP bag, 16” × 20”, containing Certificates and important
papers. Lost in Metro, between Uttam nagar and Dwarka on 4th Aug.
2012 between 9 to 9.40 am. If found, kindly contact Mr. Rajeev Kumar,
or call at 9998...........(M). Finder will be duly rewarded.

33 XII – English
2. You found a wrist watch in the school canteen. Draft a notice for School
Notice Board in not more that 50 words.

NOTICE
Found a ladies’ wrist watch in school canteen on 5th Aug. 2012 at
around 11.30 a.m. The owner can take it from the school office after
providing details about the wristwatch. Contact Mr. Sharma after school
time.

1. You are Puran Singh, a Property Dealer. You lost a bag containing
valuable documents, while travelling by bus between Model Town and
Rohini on Route No. 982, Draft an advertisement in not more than 50
words.
2. You found a Chemistry Text Book in the Lab. Write a notice for your
school notice board in not more than 50 words.
7. MATRIMONIALS
• Begin with ‘Wanted/Alliance’ or Proposal invited
• Physical description : age, height, complexion built (Slim/Tall).
• Academic, Professional Qualifications, job status, salary etc.
• Caste/sub-caste, religion etc.
• Contact / Post Box No. ...............
1. After completing his M.B.A. from reputed University, your son is working
as a manager in an M.N.C. in Noida. He wants to marry a beautiful,
educated, well cultured girl, Draft an advertisement for the “Matrimonial”.
Column of a national newspaper.

WANTED BRIDE
For 26 years, 5’8” tall, handsome, fair complexioned MBA, MNC
Executive, settled in Noida. The girl, 5’5”, 24 years, Science graduate,
computer savy will be suitable match. Caste no bar. Contact at Post
Box No. 678, Times of India, New Delhi-110001.

2. A retired army officer is looking for a suitable match for his smart, convent
educated daughter. Write a matrimonial advertisement for a local
newspaper.

34 XII – English
GROOM WANTED
Alliance invited from a tall handsome, educated, well settled business/
professional Delhite boy of Jain family for a smart, beautiful, fair, slim,
23/5’3”/ 50 kg. Convent educated M.B.A. girl working in M.N.C. Contact
Mr. S.K. Jain, at 90123......(M)

3. A highly placed I.T. Professional settled in America seeks alliance with a


fair slim beautiful, educated, cultured Brahmin girl. Write a suitable
advertisement.
4. Draft an advertisement for the matrimonial columns of a newspapers for
your sister who is working in an MNC as a manager.

Non Classified/Display Advertisement


Q. No. You are working for an advertising agency. Draft an attractive
advertisement for a company which is launching a new toothpaste.

35 XII – English
2. M/s Kirori Mal and Sons, Sarita Vihar, New Delhi have manufactured a
new pressure cooker, draft a very attractive advertisement for the
promotion of the product on behalf of the company.
3. Your company has manufactured a new washing machine. Draft a suitable
advertisement to be published in the local daily for the promotion of your
washing machine. Mention its warranty or guarantee too always with other
details and contact No.

Notice (5 Marks)
Format : The word ‘NOTICE’, date, Heading, Writer ’s name with
designation. name of the issuing agency (school). 1
Content : What : Event/Activity/Heading.
When : Date and Time
Where : Venue
Who : Participants/organisers, Chief Guest if any
How : Process and contact (to whom). 2
Expression : Overall organisation, accuracy and fluency 2

TYPES OF NOTICE
1. Notices at Public Places, No Parking
No use of Mobiles, No Smoking etc.
2. Notice for General Public

PUBLIC NOTICE

36 XII – English
4. Notices issued by ‘Residents Welfare Association, Mayur Vihar,
Phase-I, Delhi
NOTICE
Power Cut
Aug. 7, 2012
This is to inform all the residents of the society that there will be a
power cut for four hours on 8th Aug. 2012 from 1.00 p.m. to 5.00 p.m.
the reason is the installation of electronic meters. In convenience is
negretted
M.L. Verma, President), RWA

NOTICE
Water Shortage
Aug. 8, 2012,
Considering the acute water shortage in the colony this summer, all the
residents of Vasant Kunj, A-Block are requested not to waste potable
water by cleaning their cars, verandahs, drive ways etc. or watering
the plants with a hosepipe. A fine of Rs. 500/- will be imposed on the
defaulters. Kindly save water for better tomorrow.
President
Resident Welfare Society
Vasant Kunj, A Block., Delhi

5. NOTICE ABOUT TOURS/CAMPS/PICNICS


1. S.K.V. Dilshad Garden is organising an educational tour to Goa for
the class XI and XII. Write a notice for your school notice board
inviting students to join the tour. Invent other necessary details.

S.K.V. DILSHAD GARDEN


NOTICE
Excursion Tour
Aug. 7, 2012
The students of class XI and XII are hereby informed that our school
is organizing an educational tour to Goa as per the details.
Destination : Goa

37 XII – English
Duration : From 13 Sept. to 18 Sept. 2012. Expenses to be born by
the Department of Education.
Interested students may give their names to the undersigned alongwith
the NOC from their parents by 30 Aug., 2012
Mrs. S. Krishna (P.E.T.) 1/C Excursions.

6. SCHOOL ACTIVITIES OR EVENTS


1. You are Rajan/Rachna, the Head boy/ girl of the school, G.G.S.S.S.
No. 1, Rohini. Write a notice about the debate competition in the
school on the topic “Utility of Mega Events” for common man.

G.G.S.S.S. NO. 1. DILSHAD GARDEN


NOTICE
Debate Competition
Feb. 17, 2012
All the students are hereby informed that English Debate Competition
of Zonal Level will be held in our school as per the details given below:
Date : 21 Feb. 2012
Time : 8.30 a.m.
Venue : School auditorium
Topic : Utility of Mega Events for Common Man.
Time limit : 3-4 min. One each in favour and against. Interested
students may give their names in pair to the undesigned latest by 19th
Feb. For further details/enquiry, contact the undersigned.
Rachna
(Head Girl)

2. You are the Editor of your School Magazine. Draft a notice for your School
Notice Board inviting articles, poems ,jokes, sketches etc. from the
students for your school magazine. Sign as Neha/Neeraj, I/C Cultural
Activities, SKV, Vikas Puri.
NOTICES
3. The cultural club of DAV Public school Noida is organising A Talent Hunt
Evening. Pt. Ravi Shankar, the eminent vocalist will be the guest of
honour. Mridul/Mridula the secretary of the cultural club drafts a notice

38 XII – English
to inform and invite the names of the students about the same. Draft his/
her notice in not more than 50 words.
4. Head boy/Head girl of your school Mother Merry Public School, Dilshad
Garden, Delhi you are organising a career counselling session for XI &
XII Std. Students of your school Write a notice giving details of it to be
displayed on your school notice board.
Note : (Name the dignitary who will throw light on various vistas open for
youth and mention about the exhibition corner in the school)
5. Your school AVB bal Vidyalaya Sirsa has completed 50 years of
meritorious service to the society. As president of the students council
of your school write a notice informing and inviting the names of the
students to participate in the Golden Jubilee Celebration of the school
in not more than 50 words. You are Ram/Radhika.
Note : (Mention the name of the chief guest along with other detais)
3. CIRCULARS
Circular is for wide circulation sent by a school or office or club to its
members/parents/employees.
Points to Remember :
❑ Name of the institution on Top
❑ Date on left hand
❑ Subject
❑ Salutation such as Dear Parents / Members.
❑ Include all relevant information
❑ Signature with designation.

RPVV Rajniwas Marg, Delhi


CIRCULAR
Aug. 12, 2012
Welfare Scheme
(Cash Disbursal)
Dear Parents,
The school is going to distribute Rs. 1200/- to all students towards free
stationary and uniform for the year 2012-13. Kindly make yourself
available between 9.30 to 11.30 a.m. on 16th Aug., 2012 to collect the
said amount.
S.D. Sharma
(Principal)

39 XII – English
1. You are Ashu/Aakash the Head girl/boy of D.A.V. Public School,
Anand Vihar. Write a circular to inform the parents about P.T.A.
meeting on second Saturday.
2. Write a circular to inform the students of Class VI, VII and VIII about
the Exam. of Engligh, Hindi and Maths based on Minimum level
learning programme on 31st July, 2012. Sign as Nakul/Neha of
Saraswati Public School, Rohini.
3. Frequent Cases of theft, burgalry, eveteasing and electricity break
down in the society are being reported to the Resident Welfare
Association of Ashok Nagar. As the President of the society write
a circular inviting for general body meeting of the members to
discuss the issue and find the solution Agenda of the meeting must
be mentioned. You are surya.
4. Your school has been selected for the venue of state level science
exhibition therefore the principal of your school has decided to hold
a meeting with the staff to plan for the same. Draft a circular on
behalf of the principal. Give agenda of meetings (50 words)
Difference between Notice and Circular
❑ Notice is more formal than circular
❑ Salutation is given in circular such as Dear Parents and Dear
Members
❑ Purpose of the circular is to convey the message to the definite
target group whereas a Notice may have a set group as well as
general public as its target (as per the need).
4. INVITATIONS
Formal Invitations/Replies to formal invitations.
Points to Remember
❑ To be written in third person
❑ No abbreviation to be used
❑ Simple present tense is used
❑ Each entry to be mentioned in a separate line, e.g., (a) the name
of the person(s) who is/are inviting (b) formal expressions like
‘request the pleasure of your company (c) time and date of event
(d) purpose and occasion of invitation.
❑ In RSVP - address and telephone no. is given at which the invitee
may contact for any queries.
❑ In official invitations, time by which the guests are to be seated,
name of the chief guest, programme may be given.

40 XII – English
Formal Invitations
I. Marriages/Auspicious Occasions
(i)
Mrs. and Mr. Satish Sharma
Solicit your gracious presence on the auspicious occasion
of the marriage of their grand daughter.
(Daughter of Mrs. and Mr. Sunil Sharma)
with
(Son of Mrs. and Mr. Lokesh Sharma
on
17th September, 2011
at 7:30 p.m.
at
Red Carpet, Party Lawn, Preet Vihar, Vikas Marg, Delhi-110031.
R.S.V.P. With best compliments From
Vinod Kr. Sharma All Relatives
B-36 Rajdhani Encl., Vikas Marg, Delhi

(ii) You are the member of Punjab Academy, Delhi. Draft a formal
Invitation to invite all for celebrating. “Freedom of Nation” with Kavi
Darbar.
A Specimen of Formal (Printed) Invitation

41 XII – English
(iii) You are organising an Exhibition of Painting “Nayika Series’ on 31th
Aug., 2011 at Azad Bhavan Art Gallary at 5:45 p.m. Draft an
invitation card to invite the General Public.

(iv) You are a student of Laxmi Public School, Model Town, Delhi. The
school is holding its Annual Function at 5:30 p.m. on 21st March,
2011. The Education Minister has consented to be the Chief Guest.
Design an invitation card to be sent to the Parents and other
invitees (word limit 50).
II. Founders Day/Annual Day/Sports Day
THE PRINCIPAL, STAFF AND STUDENTS OF
Laxmi Public School, Model Town, Delhi, request the pleasure of
your company on
ANNUAL DAY CELEBRATION
on Saturday, the 28th Dec., 2011 at 5:30 p.m. in the school auditorium
Sh. Arvinder Singh Lovely, Hon’ble Education Minister
has very kindly consented to be the Chief Guest.
R.S.V.P.
Admn. Officer
23456788
Special Instruction : (This card admits only two. You are requested
to be seated by 4:30 p.m.)

42 XII – English
FORMAL REPLIES
Acceptance/Refusal
Points to Remember
❑ Acknowledge the invitation
❑ Express thanks in third person
❑ Not to be signed at the end
❑ When accepting, confirm dates and time.
❑ If declining, give reasons, convey your best wishes.
❑ Give date and address at top left hand

1. ACCEPTANCE

Aug. 9, 2012
7, Park Avenue
Mrs. and Mr. Ramesh Kumar thank Mrs. and Mr. Satish Sharma
for their kind invitation which they are delighted to accept.
It’s my great pleasure to attend the ceremony.

GRACEFUL REFUSAL
Aug. 9, 2012
7, Park Avenue
Mrs. and Mr. Ramesh Kumar thank Mrs. and Mr. Satish Sharma
for their kind invitation but regret their inability to accept the same
due to some urgent and unavoidable assignment abroad.

2. INFORMAL INVITATIONS
Points to Remember
❑ Use first person ‘I’, ‘We’ and ‘You’ and avoid using ‘he’, ‘she’ and ‘they’.
❑ Do not write ‘subject’ and ‘receiver’s address’.
❑ Write in warm and personalised style.

43 XII – English
❑ Avoid writing unnecessary details.
❑ Begin with ‘Dear’ ________ (name).

Q. You are Anuj/Anuja Goel. Write a letter of invitation to invite all your
friends for the party you are giving to celebrate your selection in B-Tech
in DTU.

B-5
Yojna Vihar, Delhi.
Feb. 9, 2011
My dear Rahul,
I have much pleasure in inviting you to the post selection party after
my admission in DTU. Reach my residence at 7.30 am on 14th Feb.,
2011 to join my family in its moments of joy.
Yours truly

Anuj/Anuja

INFORMAL ACCEPTANCE / REFUSAL


You are Rahul and have been invited to the post selection party of your
friend Anuj. Write a reply accepting the invitation you got.

ACCEPTANCE
7-C, Yojna Vihar, Delhi.
March 10, 2011
Dear Anuj,
Many thanks for inviting me to attend your post selection party which
will be held on 14th March at your home.
I shall be highly delighted to attend the same and enjoy the party
with great fun.
Yours truly

Rahul Gupta

44 XII – English
REFUSAL
7-C Yojna Vihar, Delhi.
March 10, 2011
Dear Anuj
Many thanks for inviting me to attend your post selection party but
I am sorry to say that I shall not be able to attend the same as I
shall be out of station next week. I shall miss this joyous occasion.
I wish you all the best for the future. God bless you.
Yours truly
Rahul Gupta

SAMPLE INVITATION
You, Neha/Nakul, the President of the English Literary and Cultural
Society of Government Model Sr. Sec. School, Sect. 19, Chandigarh, has
to organise an Inter Zonal Declamation competition on the topic
“Communication skills are very important in modern world” at the +2 level.
You wish to invite Dr. Shailesh Gupta, an eminent educationist to preside
over the function to be held on 21st Aug., 2011 at 9:30 a.m.

Govt. Model Sr. Sec. School, Sector 19, Chandigarh


Dr. Shailesh Gupta
73, Sec. 11A, Chandigarh.
9th Aug., 2011
Sub. : Invitation to preside over Inter Zonal Declamation Competition.
Sir,
The English Literary and Cultural Society of our school is organising
an English Declamation competition on the topic ‘Communication
skills are very important in modern world’, on 21st Aug., 2011 at 9:30
a.m. in the school auditorium.
Kindly consent to preside over the above said programme.
Yours sincerely
Neha.

45 XII – English
Dr. Shailesh Gupta Writes a letter of acceptance of the invitation.

ACCEPTANCE
Dr. Shailesh Gupta.
73. Sec. 11-A Chandigarh
Aug., 16, 2011
Dear Neha
Many thanks for inviting me to preside over the Inter Zonal
Declamation Competition to be held on 21st Aug., 2011 at 9:30 a.m.
I shall be highly delighted to attend the function and enjoy listening
to the views of students.
It will be kind of you, if you could provide me the official transport.
Yours truly
Shailesh Gupta.

REFUSAL
Dr. Shailesh Gupta.
73. Sec. 11-A Chandigarh
Aug., 16, 2011
Dear Neha
Many thanks for inviting me to preside over the Inter Zonal
Declamation competition to be held on 21 Aug., 2011 at 9:30 a.m.
but I am sorry to say that I shall not be able to attend the same
due to some prior appointment with my doctor.
With warm regards
Yours sincerely,
Shailesh
POSTER DESIGNING
Purpose : to create social awareness, to advertise current general
problems.

46 XII – English
Layout : 1
❑ Eye catching and visually attractive
❑ A catchy slogan
❑ Simple drawings / sketching
❑ Letters of different size and shape
❑ Proportionate spacing.
Content : 2
❑ Highlight the main topic
❑ Include all the important details like time, venue and date, in case
of an event.
❑ Name of the issuing authority, organisers etc.
Expression : 2
❑ Appropriate and accurate language
❑ Creativity
❑ Overall organisation
Common Topics for Posters
1. Clean city campaign/Green Delhi, Clean Delhi.
2. World environment Day/Save Environment
3. Book fair/Exhibition/Career Mela etc.
4. Save water/Water harvesting
5. Prevention against Malaria/Dengu
6. Sarva Shiksha Abhiyaan/Each one Teach one etc.
7. Prevention against Terrorists attacks / Be an Alert citizen
8. Say ‘No’ to crackers/Polybags/Plastics.
9. Road Safely/Car-pooling/Air pollution etc.
10. Save girl child.
11. Child labour.
12. Harmful effects of smoking/Drugs/Drinking.
13. Save Power/Save Energy
14. Donate Blood
15. Help Old People
16. Save Earth – Save Humanity

47 XII – English
48 XII – English
1. Draft a Poster on ‘Save Girl Child, Save Nation’ (Awareness)

GIRLS ARE AN ASSET TO A NATION


THEY ARE AN INDISPENSABLE HUMAN RESOURCE
FOR GENERATIONS TO COME
SAVE GIRL CHILD
SAVE HUMANITY
DONOT KILL THEM IN WOMB
EARTH WILL BE BARREN AND COLOURLESS
IF FEMALE FOETICIDE CONTINUES
Girls are :
❑ Productive members of the society
❑ More intelligent, wiser and compassionate
❑ Contributing in all fields
BE AWARE
FEMALE FOETFECIDE IS A CRIMINAL OFFENCE
Issued by :
Delhi Commission for Women
Govt. of NCT, Delhi

2. A week long International Book Fair is being held of Pragati Maidan. Draft
a poster informing general public about it (informative)

COME ONE COME ALL


TO
15th INTERNATIONAL BOOK FAIR-2012
Date : 23rd October to 28th October, 2012
Time : 10.00 a.m. to 7.00 p.m.
Venue : Pragati Maidan (Stall 7 to 12)
Man can Deceive but Books can Never
These are your true friends
Value them
Features :
❑ Books for all age groups & on all subjects
❑ Computer Software and Educaitonal CD’s
❑ Audio Visual Teaching aids
❑ Heavy discount
❑ Entry Free

Organised by : International Trade Authority

49 XII – English
SAVE WATER SAVE EARTH
NO WATER NO LIFE
NO GREENERY NO VEGETATION
SAVE EVERY DROP OF WATER
IT IS GETTING SHORT WATER
LEVEL IS REDUCING
IT REQUIRES WATER
HARVESTING
Therefore follow these:
Dos Don’ts
❑ Close tap after use Don’t allow water to flow uselessly
❑ Use waste water in Washing Don’t use hose pipe to wash
cars and watering Plants your cars
❑ Use buckets to bath and Don’t keep the tap running
cleaning floors, clothes etc. while brushing your teeth
Collect Rain water and allow it to seep in the Earth
issued in interest of public by Delhi Jal Board

50 XII – English
APPEALS !
Appeals are made to help the people in need, to raise funds for helping others.
Such as earthquake victims, Slum children welfare etc.
❑ Use word ‘Appeal’.
❑ Explain who is needy person.
❑ Reason for Appeal.
❑ Who are appealed.
❑ Name Add. and contact no. of Appeal maker.
❑ Put in a box.
1. You are Anita/Anil, residing at Hari Nagar, New Delhi. Your friend has to
undergo a major operation for which he needs rupees two lakhs. Some
generous people have donated some money but that is not sufficient.
Draft an appeal which will be published in a local daily to generate money
for open heart surgery.

Appeal !
A promising badminton player, a national Talent has to undergo an
open heart surgery in Escorts Hospital, New Delhi for which he needs
Rs. two lakh rupees that he cannot afford. All kind hearted and
generous people are requested to help the needy player so that a
precious life may be saved.
Anita/Anil.
116, B-4 Hari Nagar.

2. Write an appeal inviting suitable Donors to offer one Kidney of theirs to


save the life of a little girl of four years old. As her both kidneys are
damaged. You are Ashu, the sister of victim, residing at B-5, Anand Vihar,
Delhi.

Appeal !
A very little girl of 4 years is lying on death bed in AIMS, Delhi. Both
her Kidneys are totally damaged beyond repairs. Your benevolent
act of donating one Kidney can save a precious and innocent life.
Kindly come forward for this noble cause. The blood group of the
patient is B+. The donors may please contact : The Medical
Superintendent AIIMS, New Delhi.
Mob. : 6212731742

51 XII – English
APPEALS
3. You are building a free charitable hospital for the poor and the needy.
Draft a suitable “Appeal” for the public requesting donation for the hospital.
(Note : Write the cause (why you are doing so), where, and attraction to
the donors, name the charitable society (Regd.))
4. The Prime Minister of India has made an appeal to the nation to donate
liberally to the Prime Minister’s Relief Fund to help the victims of Flood in
Bihar. You are Manish/Manisha pupil leader of Ramjas Public School,
Rohtak. Draft an appeal to be inserted in the newspaper.

52 XII – English
REPORT OR FACTUAL DESCRIPTION
Points to Remember Division of marks

Weightage : 10 marks Format : 1 marks

Word limit : 125–150 Content : 4 marks

Time available : 15-20 minutes Expression : 5 marks

Format

❑ Heading/Title

❑ Name of the Reporter/Writer

❑ Date and Place

Content

❑ Para 1 – Introductory para giving day, date, place, what, timings.

❑ Para II – Brief detail of the place/cause/consequence/effects/guests/

❑ Para III – Main steps/activities/people involved

❑ Para IV – Concluding Para-Reactions/Promises by authorities/statement


of people.

Expression

1. Grammatical accuracy

❑ Past tense for report

❑ Present or past tense–factual description

❑ Appropriate words and Spellings 2½

2. Coherence and relevance of ideas and style 2½

53 XII – English
Common Subjects of a Report
(i) Day Organised/Celebrated at School

❑ Sports Day

❑ Grand parents’ Day

❑ Environment Day

❑ Childrens’ Day

❑ Teachers’ Day.

(ii) Workshops/Seminars/Talks

❑ Science seminar

❑ Talks on Adolescent Problems and solutions

❑ Workshop on Examination stress and solution.

❑ Seminar on consumer awareness.

❑ Disaster management

❑ Right to Education

(iii) Competitions

❑ Science Quiz

❑ Zonal Level Drawing Competition

❑ District level essay writing

❑ Zonal level Junior Hockey

❑ District Level Teachers’ Competition

(iv) Campaign/Awareness March/Rally

❑ Plantation Day/week

❑ Water harvesting

❑ Save girl child

❑ Anti Tobacco

54 XII – English
❑ Save Monuments

❑ Pulse Polio

(v) Camp/Visit/Exhibition/Fair
❑ Blood Donation
❑ National Park/Museum
❑ Craft and Art Exhibition
❑ Science Fair
❑ Hospital Visit

(vi) Robbery/Snatching
❑ Day light
❑ On a morning Train/Bus
❑ Chain Snatching

(vii) Accident/Calamity
❑ Railway
❑ Road/falling in manhole
❑ Drowning in swimming pool/river
❑ Cloud burst and mud slide
❑ Flood / Fire
❑ Earthquake

(viii) Civic Problem


❑ Bursting of water pipe
❑ Road rage case
❑ Digging on roads.

(ix) Public Event


❑ Inauguration of Metro station
❑ Inauguration of new stadium
❑ Inauguration of flyover/underpass.

55 XII – English
REPORT WRITING
1. When you were driving back home from work, you were witness to an
accident between B.M.W. and on Indigo Car. The driver of BMW can was
seriously injured where as out of 5 inmates of Indigo car three had died on
the spot and two were critical in the hospital. There was confusion and
chaos prevailing on the road, describe the scene in about 100 to 125
words. You are Sameer/Samiksha.
2. You are Sanjay/Sanjana a reporter with the Indian Express. You witnessed
a fire accident in a crowded market of Delhi. Write a report to be published
in the news paper. Invent details. (100 – 125 words)
3. You are Sahitya/Sahida, a senior consultant (Child education) in Jaideep
school management and trust. You organised a seminar on Right to
education act for teachers and students. You invited educationists and
sociologists to talk on the topic of right to free and compulsory education
to children in age brackets of 6-14 years, problems and implications. Write
a report in about 125 words on it for trust’s magazine.
4. You are Neha/Narayan, head of the history club of ASN Public School
Janakpuri, Delhi. Your School organised Heritage awareness programme in
your school to create awareness regarding our monuments. Write a report
in about 125 words for your school magazine, giving details of the
programme.
5. You are Shweta/Sahitya head of the health and wellness club of GTB
school Shalimar Bagh. Your club organised a seminar to make students
aware about alarming use of chemicals in vegetables and fruits. Experts
from medical field and consumer forum were invited to answer the queries
of the audience. Write a report in about 125 words for your school magazine.
6. You are Geet/Geeta, President of cultural Committee of JPS school
Kalyanpuri. Your school organised a cultural evening in the school for
charity. Prominent personalities of the city attended the programme. Write
a report on it in about 125 words for publishing it in your school magazine.

FACTUAL DESCRIPTION

1. Recently the rooms in your school were renovated. As a result many new
features like two doors, firealarm systems, better blackboard etc. have

56 XII – English
become part of it. Write a factual description of your new classroom in
about 125 words. Assume your self as Shweta/Sahitya a student of XII C.
2. Recently you travelled to Gurgaon in metro train. You enjoyed every aspect
of your journey very much. Write a factual description of the metro ride you
undertook in about 125 words. Assume Yourself as Jai/Jaya, CEO, ASN Ltd.
3. You are Neena/Nitin incharge of science club of RPVV school Mayur Vihar.
Your school was the venue of Zonal Science fair held last month. You
looked after each and every step of the preparations. Write a factual
description of the process and procedure you had to undertake to make
your school a successful host in about 125 words for publishing it in your
school magazine.
4. You are Kritika/Krishan, head of Cultural club of Sarvodaya School, Mayur
Vihar. You met famous Sarod Player Ustad Amjad Ali Khan. Give a factual
description of the maestro and the impression you had of him on you.
5. You are Gagan/Geeta, Archaeologist in ASI India. During your visit to Great
Pyramid you were mesmerised by an old statue of an Egyptian queen. Give
a factual description of the statue.

Solution of Some Questions


1. When you were driving back home from work you were witness to an
accident between B.M.W. and an Indigo. The driver of B.M.W. car was
serioulsy injured where as out of 5 inmates of Indigo card three had died
on the spot and two were critical in the hosptal. There was confusion and
chaos prevailing an the road. Describe the scene in about 100 to 125
words. You are Sameer/Samiksha
Note : (Don’t write report in I person. (Reporter reports it in III person)
Value points :
1. Description of the accident site
2. The cause of collision
3. Condition of the occupants & their number.
4. Condition of the vehicle.
5. Role of witness, crowd
6. Presence of Ambulance
7. Role of Police
Expression – Take care of coherence, relevance grammar and spellings.

57 XII – English
2. You are Sanjay/Sanjana a reporter with the Indian Express. You witnessed
a fire accident in a crowded market of Delhi. Write a report to be published
in the news paper. Invent details. (100 – 125 words)
Hints –
Central Market gulled by fire by _______________ (name of the reporter
__________ date
__________ place
Para-I :Yesterday was a fateful day for those who went for shopping to the
crowded name market
Points :
1. how the fire started
2. how the fire spread
Para-II : There was Panic and stampede as people rushed out _______
1. Write reaction of the crowd
2. No. of people injured/got burned.
Para-III : 1. action taken __ rescue operation when & how 2. Total estimated
loss, possible cause of fire, any other details.

4. REPORT
Heritage Awareness Programme
By : Neha/Narayan
ASN Public School;
10th Aug., 2012
Heritage awareness Programme was organised by history club of the school
on 9th August, 2012 from 10 am to 2 pm in the school premises. The school
was well decorated with posters of various historically important monuments.
Mrs. Deepanshi Dayal, Dean History Department of JNU was the chief
guest. Other prominent guests included Mr. Balachandra Assistant Director ASI
of India, Mrs. Smriti Pant head of Tourism Department of Government of Delhi
Around 2000 students and parents attended the programme.
The Programme began by lighting of inaugural lamp by Chief guest. After
that a skit was presented by students of XI classes showing importance of our
heritage and monuments. The audience enjoyed a engrossing documentary
prepared by history department of the school. The documentary showed a brief
detail of 100 monuments from different parts of India.

58 XII – English
Chief guest gave a very motivating speech on the role of Youth in keeping
the monuments safe. Mrs. Smriti Pant told about the importance of monuments
in encouraging tourism of the concerned place. Principal presented vote of
thanks. There was an arrangement for light refreshment also.

3. FACTUAL DESCRIPTION
PREPARATION FOR HOSTING SCIENCE FAIR
By Neena/Nitin
10th Aug., 2012
As the Zonal Level Science fair was to be held on 10th August, 2012 at our
school, we started the preparations for hosting it immediately after getting the
email from head office at 8 am. on 8th August.
Principal Mrs. R. Bala called the meeting of science teachers and formed
a four member hosting committee with myself as its head. We in turn took stock
of all the tasks which we had to undertake and all the materials and space
available at our hand. Mr. Raghav and Mrs. Neena were given the duty of
cleanliness and decoration of the venue. They started their work with the help
of sweepers and senior students of science sections. They put charts and
posters on walls.
Mr. Jaiwardan and Myself started doing other preparations. We called meeting
of the lab assistants and help staff and distributed various duties to them.
On 8th August, most of the decoration was completed. Sheela, the water
woman put 20 Mayur Jugs in different places for water. Suresh Chand and Khyal
Dev put 50 tables in different rooms. Satish, the electrician checked all
connections. By 2 PM the venue was fully ready to host the science fair. We felt
great relief after the principal had expressed her satisfaction.

59 XII – English
LETTER-WRITING
Points to Remember Division of marks
Weightage : 10 marks Format : 2 marks
Word limit : 200 Content : 4 marks
Time available : 15-20 minutes Expression : 4 marks

Format of the Letter


❑ Left aligned i.e., Start each new line from left hand margin.
❑ Sequence of writing information in a letter
(i) Sender’s address
(ii) Date
(iii) Receiver’s address
(iv) Salutation : Sir/Madam
(v) Subject

Content of the Letter


❑ 3 – 4 paras
❑ Para I – Introductory
❑ Para II and III – Main ideas
❑ Para IV – Concluding

Complimentary Close
❑ To the Principal : Yours obediently
❑ To the employer/editor : Yours sincerely
❑ To the dealer/business vendor : Yours truly.

60 XII – English
Expression
❑ Coherence and organisation of ideas
❑ Think and analyse the subject/topic
❑ Recall all the ideas and put them in a rough sheet.
❑ Organise the ideas
❑ While dealing with social or civic problems in ‘Letter to Editor’, always
provide suggestions to curb or control it.

(A) BUSINESS LETTERS

(i) Giving Information


1. You are Satish/Sarita, Manager of Newage Sports equipments Co. Ltd.
Rohini. Your firm has manufactured sports equipments especially
designed for school students in the age group of 10–15 years, in
collaboration with USA sports research unit. Write a letter to the Director,
Bhartiya Vidya Bhawan Schools, Hyderabad informing about your latest
products which are ready for sale.
2. You are Bharti/Bharat manager of Smart Link School firm Chennai,
T.N. Your firm has designed easily installable smart black boards. Write
a letter to the chairman BBPS managing trust giving information about
your products and services.
(ii) Asking for Information
3. You are Sudha/Sudhir, President of Residents’ welfare association of
Neelamber Apartments, Jaipur. You have decided to get the apartments
whitewashed with plastic paints. Write a letter to the manager New light
Painters and Decorators, Jaipur asking about their services and charges
for whitewashing the apartments.
4. You are Aditi/Aditya from 7-S Najafgarh. You are interested in joining
Yoga and Meditation centre of your locality as a student. Write a letter
to the chairman of the centre asking about various particulars of the
admission and other related information.
(iii) Placing Order/Cancellation of Order
5. You are incharge of Junior Science Laboratory of ASN Public School,
Shalimar Garden. Place an order to SUV laboratory works; Karol Bagh
for various apparatus/equipment used in your laboratory.

61 XII – English
6. You are Neetu/Neel, Eco Club incharge of S.V.C. school Motibagh,
Delhi. Place an order with Green World Nursery, Dayabasti for Non-
flowering Ornamental plants.
7. You are Shivani/Shivam, Incharge of Cultural, committee of Jai Deep
School, Nehru Vihar. You had placed an order for the supply of several
musical instruments for organising annual day, at your school to M/s
Gagan Musical Store, Karolbagh, As the dealer did not supply the
instruments before the due date you were forced to arrange for the
instruments from some other store. Write a letter to the dealer for
cancellation of the order placed by you and returning the advance
money deposited by you.

(iv) Sending Replies in Response to Enquiries


8. You are Zeenat/Zeeshan, Manager of VGC firm Sahibabad, Dealing
with readymade clothes. You have got letters of enquiry from Vision
Costume and Accessories, Vivek Vihar about your products. Write a
letter providing necessary information in response to the enquiry.
9. Sheetal restaurant, Sector-8, Rohini wants to know about various types
of ice creams supplied by Jai Deep Ice Creams, an outlet of Amul Ice
Creams, Pant Nagar. You being the manager of the outlet provide
necessary information in response to the enquiry, assuming yourself
as Rajni/Rajan.

(v) Making Enquiries


10. IGNOU, Delhi has started 6-months certificate course in Disaster
Management. You are Deepika/Deepak head of Disaster management
cell of SDS Public School Bangalore. Write a letter to the Vice-Chancellor
IGNOU enquiring about various aspects of the course like fee, criteria
of admission etc. as you want your colleagues to join it.
11. You are Ameena/Aman from NSCB hostel, Dehradun. You and your
friends are planning a holiday. You come across the following
advertisement in a newspaper. Read the advertisement and on the
basis of your choice of destination write a letter to the tour operator
enquiring about various aspects of the tour.

62 XII – English
Perfect holiday with friends and family; Unlimited Enjoyment
Goa 3d/2n Rs. 7000 for two
Shimla 4d/3n Rs. 8000 for two
Kerala 4d/3n Rs. 15000 for two.
Free breakfast; No hidden charges, concession available on bulk
booking. Contact Sai Tour and Travel, Vivek Vihar-1

12. You are Meenakshi/Mohan from A-8 Rohini. You have been reading
about an advertisement in various newspaper about Dr. Bakson’s
Homeopathic hospital, assuring hundred percent treatment for various
types of allergies. Write a letter to the head of the hospital enquiring
about various aspects of the treatment for skin allergy.
(vi) Complaint regarding a defective household item/items of general
use you purchased
13. You are Sheetal/Satish staying at B-43, Kailash Colony, Delhi. You
purchased a computer from M/s City Hightech computers, Laxmi Nagar.
After using it for one month you found its working was getting faulty.
Write a letter to the dealer asking him to replace it immediately under
the terms and conditions of the deal.
14. You are Jeet/Jyoti resident of R-7, Pitampura, Delhi. You purchased
teak furniture from S.K. Decorators Lajpat Nagar. After three months
of use many items started getting defective. Write a letter to the dealer,
complaining about the problem. Also request him to change the
defective items as per terms and conditions specified in warranty card.

(B) LETTERS HIGHLIGHTING


THE CIVIC PROBLEMS/ISSUE

(i) Letter Regarding a Civic Problem / Issue to the Concerned


Authorities Directly
15. You are Yasmee/ Yashwant from Q-6, Mangolpuri, Delhi. There are
many uncovered manholes in your locality which are constant cause
of fatal accidents. Write a letter to the Commissioner Sewage Disposal
Board of Delhi complaining about the problem and requesting for its
early solution.

63 XII – English
16. You are Jasveer Kaur/Jaideep Singh from C-9, Tilak nagar, You feel
hurt and helpless to read about the accidents, caused due to illegal
occupation of roads by vendors. Write a letter to the Police
Commissioner, Delhi about the problem of road encroachment by
hawkers.
(ii) Writing Letters in Newspaper to highlight the Civic Problems /issues
17. You are Jaya/Jaideep, head of Eco Club of Sai Ram Public School, Jor
Bagh, Delhi. As a team leader of Eco club, you have come to know that
Delhi is the top garbage producer of the country and it is facing a
tough task of disposing it off. Write a letter to the minister of health,
Government of Delhi on this problem suggesting ways to tackle the
problem.
OR

Letter to the Editor


Points to Remember / Important Points
❑ Draw the attention of the concerned authorities/general public towards the
problem NOT of the Editor
❑ Request the concerned authorities to take the action NOT to the editor
❑ Raise the issue by citing some latest news items / Survey report / personal
experience etc.
❑ Analyse the issue in terms of its cause and consequences
❑ Offer suggestions
18. You are Samita/Sunit, resident of C-9, Vasant Kunj, Delhi. You find,
participation of children in various reality shows on T.V. a form of child
exploitation. Write a letter to the editor of a national daily showing your
concern about various forms of child exploitation prevalent in educated,
urban society giving some suggestions to curtail it.
19. You are Kavita/Kailash staying at B-101, Yamuna Vihar, Delhi. You find it
disturbing that despite a ban on the use of polythene bags its use is
rampant in city. Write a letter to the editor of a national daily expressing
your concern about apathy of people towards environmental degradation.
Also suggest ways to mobilise city dwellers for the cause of safe environment
with the help of school children.

64 XII – English
C. APPLICATION FOR JOB

Points to Remember
❑ Start body of the letter giving source of information about the job
(newspaper) day, date, advertisement number etc.)
❑ Close the letter giving note that Bio-data/resume is enclosed.
❑ Write 10-12 points in Bio-data
❑ Bio-data/resume curriculum vitae is integral part of the job applications
❑ Bio-data should include :
– Name
– Date of Birth
– Address
– Education qualifications : Year, marks, and subjects of the
(i) – X
(ii) – XII
(iii) – Graduation
(iv) – Diploma/higher qualifications
– Professional qualifications :
(i)
(ii)
– Experience
(a)
(b)
– Hobbies
– Salary expected
– Reference
20. You are Sudha/Sudhir resident of A-7, Shanti Park. You read the following
advertisement in a newspaper.

65 XII – English
Situation Vacant
Wanted a young and experienced graduate with fluency in English and Hindi to
work as receptionist at customer care booths of NCR group of hotels, Delhi.
Contact : Manager alongwith your complete C.V. within 7 days of this
advertisement.
Draft an application in response to the advertisement giving your detailed resume.
21. You are Krishna/Krishan from F-9, Mayur Vihar, Delhi. You have come
across an advertisement in a national daily for recruitment of Radio Jockey
by Radio one, Noida. Apply in response to this advertisement giving your
detailed bio-data.

LETTER WRITING

Solution of Some Questions


5.
ASN Public School,
Shalimar Garden

10th August, 2012

The Manager
SUV Laboratory Works
Karol Bagh

Sir,
Subject : Order for supply of laboratory apparatus.
After going through your latest catalogue of laboratory equipments. I am
placing order for following apparatus for our school laboratory.
S.No. Name of the Apparatus Specification Quantity
1. Beaker 500 ml 45
2. Test tubes 2” × 5” 200
3. Tripod stand 4 × 10 cm 150
4. Bunsen Burner 2 × 7 cm 50

66 XII – English
The quality of the material should be plus one grade with other specifications
as mentioned in the catalogue.
Substandard items will be returned. The items should be delivered in the
school within one week before 12.30 p.m. on any working day.
Please send the bill after deducting maximum discount as is applicable for
educational institutes.
Thank You
Yours Truly
XYZ
17.
C-9, Vasant Kunj
Delhi

10th Aug., 2012

The Editor
The Times of India
Delhi
Subject : Exploitation of children in Urban Society
Sir,
I want to draw the attention of the general Public and the concerned
authorities towards the above cited problem through this letter of mine in
your newspaper.
In Urban societies parents are increasingly using their children as means
of achieving their own unfulfilled dreams and aspirations, resulting in physical
and mental exploitation of the children. Forcing children to participate in
reality shows or cajoling them to play roles in TV serials by shunning games
are some examples of child exploitation. Even parents among audience of
such programmes openly compare their wards with child actors on TV. In
school arena parents put pressure on their wards, to excel in the field of
academics, sports and other activities alike without considering their field of
interest and abilities.
The implication of exploitation of children in the guise of making their
future bright are evident now. The diseases like hypertension, obesity,
diabetes which were known to be adult diseases a decade back are common

67 XII – English
among children. Psychotic problems among children are also rising with
cases of drug abuse, depression or even suicide are reported frequently.
Authorities in the child’s rights department and educational institutes
should come out with bolder steps to curb the problem. There should be
ban on all types of advertisements and programmes on TV with child actors
below 14 years of age. There should be regular counselling for parents in
schools to make them understand the consequences of stressing the
children.
Thank You
Yours sincerely
Smita/Sumit
18.
A-7, Shanti Park
Delhi

10th Aug., 2012

The Manager
NCR Group of Hotels
Delhi
Subject : Application for the post of receptionist
Sir,
In response to your advertisement in Hindustan Times dated 7th August,
2012 for the post of receptionist, I hereby offer my candidature for the
same.
I possess requisite qualifications and experience. I want to join your
hotels to fully utilize my potential.
You may call me for an interview on any date as per your convenience.
I shall be able to join my duties at one month’s notice if appointed. I am
enclosing my detailed resume for your perusal
Thank you
Yours sincerely
Sudha/Sudhir
Enclosure : Detailed resume.

68 XII – English
LETTER

Job Application
Two Parts
1. Covering Letter
2. Bio-data in Block

Resume/Bio-data/Curriculum Vitae
A. Personal details :
1. Name
2. Fathers Name
3. Date of Birth/Age
4. Address
5. Hobbies
6. Language Known
7. Nationality

B. Educational & Professional Qualifications and Experience :

In educational professional qualifications.

Write down year marks & subjects, university/ Board of X, XII, Graduation,
Post Graduation or Professional Qualification according to post.

Experience – Where worked and for how many years.

Expected Salary – (if required)

C. References :

1. Dr. P.S.K. Marth


Sr. Consultant G..T.B. Hospital, Phone- 9891............

2. Mrs. Aruna Dev


Director Spastic Society, Delhi
Contact No. 9818..........................

(You can name any body with new address)

69 XII – English
ARTICLES / SPEECH

Descriptive or Agrumentative
Points to Remember Division of marks
Weightage : 10 marks Format : 1 marks
Word limit : 150 - 200 Content : 4 marks
Time available : 15-20 minutes Expression : 5 marks

Format
❑ Title
❑ Writer’s Name

Content (Value points related to the topic) 3-4 Paragraph


❑ Para 1 : Brief Introduction of the status of the issue
❑ Para 2 : Analysis of the topic in terms of
❑ Types
❑ Consequences
❑ Causes
❑ Good/bad aspects
❑ related information
❑ Para 3
Implications – social / environmental/Psychological/health related.
❑ Para 4
Concluding Para-Suggestions / reminders.
Expression
❑ Grammatical accuracy, spellings, coherence, relevance of ideas and
style.

70 XII – English
Steps taken to write an article/speech
❑ Thinking about the topic and ideas associated with it
❑ Collecting ideas involved through brain storming / discussion / sources
❑ Organising : the ideas in logical order.
❑ Revising critically

Debate/Speech Guidelines
Points to remember
1. Begin with – “Honourable Judges and my dear friends, I stand before you
to express my views for/against the motion “Topic”.
2. Total agreement or disagreement with the lopic should be expressed
forecefully and clearly.
3. Use argumentative style and logical reasoning.
4. Bank up arguments with relevant information.
5. At the end write ‘Thank you’ at extreme end on the left.
6. Always make a rough draft, edit and time yourself (20 minutes)
Use powerful expressions like :
– I’d like to argue
– In my opinion
– May I ask? etc. etc.
– Refer to your opponent’s view/views
7. Stick to your view point either in favour or against

Speech
1. Give the title at the top
2. Begin with “Good morning to all of you, today I am here to express my views
on the topic_______________”
3. Define the topic; give its causes, effects, the present state and remedial
measures
4. Should be coherent piece clearly stating a particular point of view. Divide
the speech into 3 to 4 paragraphs.
5. Conclude by giving suggestion for improvement

71 XII – English
6. Always make a rough draft first
7. Time yourself (20 minutes)
1. You are Satish/Shweta. Working in an NGO which is working for the uplift
of socio-economic conditions of child labourers by counselling their parents
and helping children to go to school. You find it appalling that many people
in educated society want the practice of child labour to continue. Write an
article on the “Role of Educated Society in Curbing Child Labour”. (150–
200 words.)
2. You are Mamta/Mohan. You find corruption as the biggest impediment in
the development of a nation. You strongly believe that youth can play a
very important role in fighting the menace of corruption. Write an article on
the “Role of Youth in Fighting Corruption”. (150–200 words).
3. You are Jeetender/Jeetu. You have been working on a project related to
effect of modern life on youth. You interviewed school and college students
for the project. Write an article on the subject in about 150–200 words.
4. Write an article in 150 to 200 words on vocational training – as part of the
school curriculum’ expressing your views on its need in the present scenario
and suggesting steps to make it successful.
5. You are Vidya/Vijay, a student of Class XII of Sarvodaya School Dilshad
Colony. You feel disturbed to read news about increasing cases of “honour
killing” in northern India. You feel that such attitude of some elders in the
society deprives children of their free will and pose hindrance in choosing
life partner and career of their choice. Write an article in about 150–200
words on the topic, “Honour Killing a Stigma on Modern Society.”
6. You are Sudha/Sudhir, a counsellor in BBP school, Palam. You come across
cases of Domestic Violence frequently. You found through your interaction
with the victims that children are affected most by this. Write an article in
150–200 words on the “Impact of Domestic Violence on Young Minds”.
7. You are Veer/Veena, a student of Class XII of SPS school Maidan Garhi.
While watching many reality shows on T.V. you felt that they are harmful for
children. Write an article in 150–200 words on Negative impact of reality
shows on children.
8. You are Sona/Sandeep, a worker in NGO – ‘Awareness India’ You feel that
media which has reached every part of the country can play an important
role in spreading awareness about “Rights and Responsibilities”. Write an
article in about 150–200 words on the subject.
9. You are Mohan/Meeta. You are worried abort the hikes in the prices of
essential commodities like LPG, pulses, vegetables etc. Write a speech on

72 XII – English
this in about 150-200 words for the morning assembly suggesting certain
steps to curb inflation.
10. You are Jyoti/Jayant, a class XII Student of RS School, Sagarpur, Recently
you read a survey report in a newspaper on the use of chemicals in
vegetables. You have come across such reports frequently. You find the
trend shocking. Write an article in about 150–200 words on the topic
‘Adulteration a Monstrous Evil’.
11. You are Zeenia/Zeeshan a class XII student of RSV school, Badli. You
interacted with your friends for knowing their views on shopping Malls which
have come up in every corner of the city. You found that around half the
total number of your friends love to go to Malls, while the other half hate
them. Write a debate in 150–200. Words in favour of or against the topic
“Mall Culture in Cities – Positive or Negative Aspect on Teenagers”.
12. You are Sadhna/Siddharth a student of ASN school, R.K. Puram. You have
observed that Tuition centres have come up in different parts of the city.
They charge very high fee and assure the students better marks and seats
in professional colleges. Almost all the students in class X and XII join tuition
centres or coaching centres. They strongly believe that one gets quality
teaching in them. Write a debate in around 150–200 words either in favour
of or against the topic “Tuition and Coaching Centres Necessity of Students”.
13. You are Raksha/Rakesh a student of Class XII of MGH school Geeta colony.
You feel that mobile phones have become integral part of today’s life. You
think that every good thing has bad sides also. Write a debate in 150–250
words either in favour of or against the topic “Mobile Phone and Social
Development of the Children”.
14. You are Shakeela/Shaukeen a student of class XII of BVB school Mehta
Road. You have observed that students of your school discuss too much
about video games like call of the duty etc. Many a times they get so
engrossed in discussion that they neglect their studies but at the same time
they learn so many new things also. You keep reading about reports of
surveys on impact of Video games on students which give varied views.
Write a debate in 150–200 words on the topic “Video Games – their Impact
on Students”, putting your views either in favour of the topic or against it.
15. You are Rajan/Rajni the Head Boy/Girl of your school. You are asked to
address the students of a neighbouring school on the World Environment
Day as a part of students interaction programme. Write a speech in about
150-200 words emphasizing the “Role of Students in Protecting the
Environment”.
16. You are shocked to read a report on murder of a senior citizen in Vasant
Vihar Colony. You being the President of RWA of the colony feel that

73 XII – English
adequate steps in the field of safety and social awareness is needed. Write
a speech on the topic “Crimes against Senior Citizens–Measures to Curb
It” in about 150-200 words to be delivered at RWA meeting. Assume yourself
as Radhika/Rajesh.
17. You are Mamta/Mohan a student of Class XII of Sarvodaya school Nangloi.
You feel that there is a wide gap between civic facilities in urban and rural
India. All the progress in the fields of technology and economy is used up
in improving the life of city dwellers, Whereas rural people remain neglected.
Write an article in about 150–200 words on the topic “How to bring the light
of modernity to rural India.

Solution of Some Questions


4. Write an article in 150-200 words on : Vocational Training - as part of the
school curriculum’ expressing your views on its need in the present scenario
and suggesting steps to make it successful.
Suggested value points :
I. – Present system of education and its flaws.
– Problems faced by students in the competitive world
– Increasing unemployment – various seasons
II. Passage
– Vocational training to be assential part of curriculum
– Knowledge of professional courses to the students through career
counselling programmes and experts
– aptitude tests to choose a professional course
– Practical training – to work in the work shops or institutes.
– Stipends / scholarships to the students on meritorious students
– Selection through campus interviews
– Job opportunities / self employment
– easy loan by the government
III. Conclusion – Your views – concluding the article.
5. Honour Killing – A Social Evil
by Vidya/Vijay
Honour Killing, now popularly and ironically called by media as Horror killing
is a social evil that has existed in our society for a very long time. It might have

74 XII – English
emerged with the onset of civilization. The root cause of this social evil lies in
exercise of parental authority, over their children as their matter of right. Especially
in the patriarchal societies, the thinking that whatever parents think is right; that
children can’t take good decisions for their life partners, their future or career
goals. Majority of Indian parents don’t trust their children when it comes to
decide their life partner or career. Even if society had not been divided along
the lives of caste and creed, problems would have persisted in other forms. But
the evil of Honour killing began to raise its ugliest head when structure of society
became more and more complex. It is more frequently found in rural areas
where the light of education is yet to reach.
These things are very shocking in context of Honour killing. First of that the
perpetrators do not regret after killing their daughters, sisters and their boy
friends. Rather the killings are glorified. Secondly, ways of killing are very heinous
and brutal going beyond the limits of humanity. For example– In Delhi the couple
was electrocuted after being closed and tied in an iron box.
Third thing is that not only older generation i.e., parents but the youths who
are expected to bring out the social change and revolution in society are also
killing their sisters, cousins in the name of family honours.
Whatever the reason and whoever is the perpetrator, Honour Killing is
totally wrong and a punishable crime. It is against the pattern of society and
civilization. Each older generation must give way to the forthcoming generation
the freedom to take their decisions about their life priorities. We must have trust
in our children and respect their decisions. Even it is not right, no law allows us
to kill our children. The practice of Honour killing must be stopped immediately
and all legal agencies. Govt. NGOs, Social activists should come forward to
rescue the young boys or girls who have decided their life partners on their own.
7. Negative Impact of Reality Shows on Children
–Veer/Veena
There has been a lot of discussion on the relative merits of reality shows.
When it comes to different people in society. There are a number of people who
will argue that these shows are very good for children and there are many
reasons to prove their point of views. But I think that these reality shows are not
good for young minds and have a negative impact too. Firstly they watch television
in excess without breaking their concentration regularly. Secondly as children
they tend to ignore their other important activities such as reading, writing,
social and some type of thinking skills. Today children have no interest in reading
story books, to solve puzzles and other brain storming activities. They want to
watch only and only television. Consequently they have poor eyesight, bad
posture and other physical disorder due to lack of outdoor games.

75 XII – English
There are many reality shows which are being telecast on the silver screen
such as Dance India Dance, Little Champs, Chhote Ustad, Boogy woogy, laughter
Challenges. Talent Hunt etc. These shows are being viewed by the children
interestingly and they want to be the part of such shows at the cost of their
studies. I agree that these shows give a big platform to perform and draw out
the talents of young children. Undoubtedly they do that but what about their
childhood. They have to spend or live with the organisers for many months
without family, without love and care. They lose their innocence, their childhood
and to act as per the orders of the organisers just like the puppets. They
perform under great stress and to the last extent of their abilities and physical
capabilities. They face a big challenge to prove their best to compete with other
participants. They face great stress and tension at the moment of Judges’
Remarks, public votes consequently their elimination. The children have become
the earning tools of their parents to earn name, fame and money. They act in
place of learning. They are involved in many shows, serials, modelling and
advertisements. This is child exploitation and should be censored to save their
childhood. The innocent children do the stunts at home inspired by the shows
and untimely entangle with death. They also have deficit attention disorders or
behavioural problems.
8. The Role of Media in Spreading Awareness in Society
–Sona/Sandeep
Media today encompasses Print Media such as newspaper, magazines,
journals, periodicals etc. Electronic Media – radio, television, telephone and the
internet. Entertainment media : Films and Music.
In the world of today, media has become almost as necessary as food,
clothing and other requirement. It is true that media is playing an outstanding
role in strengthening the society, it’s a mirror of the society. It is the duty of
media to inform, educate and entertain the people as it is the fourth pillar of our
democratic country. They help us to know what’s going on around the world.
They put their lives in danger during attacks or a natural disaster, just to inform
us of situation. It is partly because of them that awareness is spreading in the
society. It is the media which shapes our lives i.e., we cannot think our morning
without the newspaper. It is just like morning tea with biscuit. Our lives would be
incomplete without the print and Electronic Media.
The role played by media in developing countries such as India, is key to
realise the dream of inclusive development. Awareness regarding immunization
programmes institutional deliveries, balanced diet, healthy lifestyle and family
planning are spread through media to remote parts of our country. Today All
India Radio covers approx 98% of the population of India, Farmers in distant
villages are able to find the true price of their produce through radio and
telephone services. Even the spread of Education through Distance learning

76 XII – English
has made the dream of 100% Literacy a reality in the years to come.
Media is the watch dog of the political democracy. If it plays its role honestly,
It will be a great force in building the nation but nowadays, media has become
a commercialised sector eyeing only for news that is hot and sells. Instead of
giving important information and educative programmes, all that one gets on
television is sensational depiction of all new stories, their only goal being gaining
television rating points (TRPs).
Media is an integral part of our society, but that’s also a fact that its too
much intervention in every thing is a matter of concern. People have to judge
on their own by looking and listening to different channels for the same news
and then form a conclusion.

Topic the ever rising prices


Q. No. 9
Good morning to all of you. Today I am here to express my views on ‘Price Rise
a constant problem.
Suggested value points
Para-I
– start with rising price index in India
– world wide phenomena with hike in petrol price, natural gas etc.
Para-II
– Write problems related to it to the low income group people, salaried
people, pensioners, labour class etc.
Causes
– increases in population
– more demand less production (supply)
– hoarding, false short supply
– black marketing
– rise and fall of equity shares in stock exchange
– defective planning and distribution system
– Corruption by beurocrates
– increase in standard of living
– M.N.C groups – high pay scale.

77 XII – English
Any other valid point
Para-III
– Suggestions – strict stops by government to encrease the fair prices
– strict action against corrupt officials
– awareness of general public
– use right to information develop public distribution system
– any other relevant point
11. Mall Culture in Cities – Positive Aspects
–Zeenia/Zeeshan
Honble judges and my dear friends, I stand before you to express my views
in favour of the motion Mall culture in cities. Shopping malls are becoming
integral part of cities. Every corner of a city has at least one shopping mall in
it. They are fast emerging as new hallmark of development.
Teenagers love to visit malls with their friends and family. Airconditioned
atmosphere provides a big relief from sweltering heat one has to face while
shopping in markets in summers. The neat and clean, safe and secure building
helps teenagers to forget the stress and pressure of schools and colleges and
enjoy free time in a relaxed way.
Shopping malls help teenagers to shop for any thing under the sky under
one roof. All their favourite things like Junk Jewellery, latest DVD, newly released
books all are available there. They can take electronic item of any brand without
caring about bargaining as the articles sold in malls are of quality brands with
fixed price tags. Window shopping, the best way to learn about latest products
in market is best done in malls. Teenagers with their peer are seen window
shopping at showrooms of famous brands there.
Shopping malls also provide perfect place to give and enjoy parties
Mcdonalds, Pizzahuts, KFC and many such eating joints have their outlets in
malls. It is becoming a new trend among teenagers to celebrate birthdays,
friendship days and many such occasion in the company of whole gang of their
friends in shopping malls.
Shopping malls with their movie theaters provide another way of enjoying
holidays with friends and family. One can watch latest movie in cool and relaxed
atmosphere of such theaters.
Thus shopping malls are emerging as the most favourite place for teenagers
to hang around in the company of their loved ones without worrying about heat
or hunger.

78 XII – English
Debate – Against
Mushrooming of shopping malls in every nook and corner of cities is
becoming a nuisances for common people. Teenagers suffer most because of
increasing mall culture in cities.
Everything about malls is detestable. Their imposing structure and lucrative
exterior symbolise the superficiality of city life with increasing materialistic outlook
of its people. They seem to be blot on social growth of people. Youngsters,
especially teenagers spend their valuable time and hard earned money of their
parents in malls.
Shopping malls are harmful for health of the visitors. The air inside the mall
is stale due to its being airconditioned without proper ventilation. The air inside
it contains fumes from its eating joints besides breathed out air of the crowd
there.
Shopping malls are heavy on the purse also. Most of the showroom in malls
charge higher prices in comparison to those in open markets as they have to
pay higher rents. It costs many times more to watch movies or buy things here.
Shopping malls encourage western culture among teenagers. They get
lured by the glossy advertisements about the eating joints there and want to
give parties in them instead of at home. They thus, harm teenagers as they get
away from Indian values like partying at home with parents.
To conclude, one can say it emphatically that todays’ teenagers under the
bad influence of western culture, encouraged by malls, spend their valuable
time and hard earned money by roaming around aimlessly and many a times
anonymously in malls in the name of window shopping. They want to show off
to their friends their spending powers and western outlook at the cost of time
and money.
17. How to bring the Light of Modernity to Rural Area
–Mamta/Mohan
India is still an agriculture based economy where by virtue of having 70%
share of the agriculture/the ways of the economic development pass through
the streets of our villages. But it would not be wrong to say that still these rural
streets are without the lights of progress in social and economical terms.
The physical conditions of villages are very pathetic, roads are still in
deplorable condition. Most of the houses are still made up of mud and straw and
even if they are constructed in bricks, they are not plastered in cement. Electricity
is still a day dream for many of the villages, or those having electricity it is
provided for few hours.
In Social terms also, many of the social evils are still in practice such as

79 XII – English
child marriage, Parda system. Adult illiteracy is also widely found in these villages.
Superstition is the by product of many of the social evils and practices. These
are the outcome of poor economic growth. All types of unemployment is found
here. Most of the villages are poverty stricken. They are not aware of the
modern gadgets. We are witnessing the tragic outcomes of this poor growth and
lack of substantial support system in wake of failing monsoon or some other
natural calamity. Our farmers are committing suicides and in some parts of the
country their anger is being expressed in terms of maoist movements. The worst
part of the story is that Governments have since the time of mughals reported
to be means of crushing these movements with Coercion. In words of Chetan
Bhagat “We are trying to kill symptoms rather than going into the roots of the
disease. We are giving crocin to the problem whereas it needs a strong antibiotic.
Apart from Govt. it is the duty of NGOs and all big industrialists to adopt
these villages and to make the life of village prosper and delightful. There must
be a movement to set up educational institution in rural areas as well as industries
to provide employment and better life prospects to rural people. Once they are
educated and economically sustained, social evils automatically will disappear
and our villages will be enlightened with the ray of modernity.

80 XII – English
LITERATURE
(35 MARKS)

TEXT BOOK
FLAMINGO
Poems

1. MY MOTHER AT SIXTY SIX

–Kamala Das
Read the following extracts and answer the questions that follow each :
1. Driving from my parent’s
home to Cochin last Friday
Morning, I saw my mother
beside me,
doze, open mouthed, her face
ashen like that
of a corpse and realized with pain
that she was as old as she looked but soon
put that thought away........
Questions
(a) Where was the poet coming from and Who was sitting beside her? 1
(b) What did the poet notice about her mother? 1
(c) With what does the poet compare her mother’s face and why? 2
2. ....... and looked out at young
Trees sprinting, the merry children spilling
out of their homes, but after the airport’s
security check, standing a few yards
away,.......

81 XII – English
Questions
(a) Where is the speaker in these lines?
(b) What thoughts did occupy her mind before she looked outside?
(c) Does lookingout help her, how?

3. I looked again at her, wan, pale


as a late winter’s moon and felt that old
familiar ache, my childhood’s fear,
but all I said was, see you soon, Amma,
all I did was smile and smile and smile.......
Questions
(a) Who looked wan and pale and why? 1
(b) What childhood tear did the speaker have?
(c) What do the parting words suggest? 1
(d) Explain the comparison : “as a late winter’s moon”. 1

Answer the following questions in 30–40 words each : (2 marks each)


1. What does the poet’s mother look like? What kind of image has the
poet used to signify her aging decay?
2. How has the poet compared the scene inside the car with the activities
going on outside?
3. Why is the poet’s mother compared to late winter’s moon?
4. What is the poet’s familiar ache and why does it return?
5. Why does the poet smile and what does she say while bidding good-
by to her mother?
6. What poetic devices have been used in the poem ‘My Mother at Sixty
Six’?
7. What different images does the poet use to convey the idea of her
mother’s old age?

82 XII – English
2. AN ELEMENTARY SCHOOL CLASSROOM IN A SLUM

–Stephen Spender
Read the following extracts and answer the questions that follow each :
1. Far far from gusty waves these children’s faces.
Like rootless weeds, the hair torn round their pallor:
The tall girl with her weighted-down head. The paper seeming boy with rat’s
eyes.
Questions
(a) Who are ‘these’ children? Where are they sitting? 1
(b) Why is the head of the tall girl “weighed down”? 1
(c) What do you understand by “The paper-seeming boy, with rat’s eyes”?1
(d) What do the images “rootless weeds” and hair torn round their pallor”
suggest? 1
2. The stunted, unlucky heir
Of twisted bones, reciting a father’s gnarled disease,
His lesson, from his desk. At back of the dim class
One unnoted, sweet and young, His eyes live in dream;
Of squirrel’s game, in tree room, other than this.
Questions
(a) What has the unlucky heir’ inherited? 1
(b) What is the stunted boy reciting? 1
(c) Who is sitting at the back of a dim class? 1
(d) “His eyes live in a dream”–What dream does he have? 1
3. On sour cream walls, donations. Shakespeare’s head.
Cloudless at dawn, civilized dome riding all cities.
Belled, flowery, Tyrolese Valley. Open-handed map
Awarding the world its world.

83 XII – English
Questions
(a) What does the colour of the classroom walls suggest? 1
(b) Which pictures do the classroom walls have? 1
(c) Explain : ‘Civilised dome riding all cities.’ 1
Awarding the world its world. 1
(d) Why does the poet refer to the ‘Tyrolese Valley’ in these lines? 1

4. ......And yet, for these


children, these Windows, not this map, their world,
Where all their future’s painted with a fog,
A narrow street sealed in with a lead sky
Far far from rivers,
capes, and stars of words.
Questions
(a) What do ‘these windows’ ‘this map’ represent? 1
(b) What is the future of these children? 1
(c) What are the ‘narrow street’ and ‘lead sky’ indicative of? 1
(d) Where would these children prefer to be? 1

5. Surely, Shakespeare is wicked, the map a bad example,


with ships and sun and love tempting them to steal–
for lives that slyly turn in their cramped holes
From fog to endless night?
Questions
(a) Why is the Shakespeare wicked? Why is the map a bad example? 1
(b) How does the poet describe the present condition of these children in
these lines? 1
(c) Explain “From fog to endless night”? 1
(d) What are the things that tempt the slum children to steal? 1

84 XII – English
6. On their slag heap, these children
Wear skins peeped through by bones and spectacles of Steel,
with mended glass, like bottles bits on stones
All of their time and space are foggy slum
So blot their maps with slums as big as doom.
Questions
(a) What does slag heap refer to. 1
(b) How are the children described in these lines? 1
(c) Explain : ‘So blot their maps with slums as big as doom’. 2

7. Unless governor, inspector, visitor.


This map becomes their window and these windows
That shut upon their lives as catacombs.
Break o break open till they break the town.
Questions
(a) What is expected of the governor, inspector and visitor and why? 2
(b) How can ‘this map’ become their window? 1
(c) ‘Break o break open’––What should they break? 1

8. And show the children to green fields, and make their world.
Run azure on gold stands, and let their tongue
Run naked into books the white and green leaves open
History is theirs whose language is the sun.
Questions
(a) Where will the children’s world extend up to? 1
(b) What other freedom should the children enjoy? 1
(c) Who can create history? 1
(d) Explain : “Run azure on gold sands”. 1

85 XII – English
Answer the following questions in 30–40 words each. (2 marks each)
1. What is the social issue that the poet raises in the poem?
2. How does the poem portray/describe the slum children?
3. How can the person in power contribute to improve the condition of
children in slum areas?
4. Which world do the slum children belong to? Which world is inaccessible
to them?
5. Stephen Spender’s poem begins with despair and ends with hope.
How?
6. What does the poet mean by’History is theirs whose language is the
sun’

3. KEEPING QUIET

–Pablo Neruda
Read the following extracts and answer the questions that follow each:
1. Now we will count to twelve
and we will all keep still.
For once the face of the Earth
let’s not speak in any language,
let’s stop for one second,
and not move our arms so much.
Questions
(a) Why does the poet ask his listeners to “Keep still” 1
(b) What changes does the peot expect on the face of the Earth? 1
(c) How would we all be together? 1
(d) Explain “not move our arms so much” 1

86 XII – English
2. Those who prepare green wars,
wars with gas, wars with fire,
victory with no survivors,
would put on clean clothes
and walk about with their brothers
in the shade, doing nothing.
Questions
(a) Who are the people who wage green wars? 1
(b) How will the few moments of introspection affect the people? 1
(c) Explain the irony in ‘Victory with no survivors”. 2

3. What I want should not be confused


With total inactivity.
Life is what it is about.
I want no truck with death
If we were not so single-minded
about keeping our lives moving
perhaps a huge silence
might interrupt this sadness
of never understanding ourselves
and of threatening ourselves with death.
Questions
(a) What does the speaker mean by truck with death? 1
(b) What is our single-mindedness? 1
(c) How has it affected us? 1
(d) What are the two things that have caused ‘this sadness’? 1

4. Perhaps the Earth can teach us


As when everything seems dead
and later proves to be alive.
Now I’ll count up to twelve and
you keep quiet and I will go.

87 XII – English
Questions
(a) What should man learn from the Earth? 2
(b) When does nature seem to come alive? 1
(c) What feelings does the poet express? 1

Answer the following questions in 30–40 words each. (2 marks each)


1. Why does the poet want everyone to keep quiet as he counts up to
twelve?
2. How does the poet suggest that there is life in nature under apparent
stillness?
3. Does the poet suggest total inactivity or death or something else?
Give reasons.
4. How do few moments of introspection affect our lives?
5. What is the ‘exotic moment’? Why?
6. What is the poet’s concept of doing nothing?

4. A THING OF BEAUTY

–John Keats
Read the following extracts and answer the questions that follow each:
1. A thing of beauty is a joy forever
Its loveliness increases, it will never
Pass into nothingness; but will keep
A bower quiet for us, and a sleep
Full of sweet dreams, and health, and quiet breathing.
Questions
(a) How is a thing of beauty a joy for ever? 1
(b) Explain–“It will never pass into nothingness.” 1
(c) How does the poet attribute beauty for sound physical and mental
health? 2

2. Therefore, on every morrow, are we wreathing


A flowery band to bind us to the earth,

88 XII – English
Spite of despondence, of the inhuman dearth
Of noble natures, of the gloomy days,
Of all the unhealthy and o’er-darkened ways
Made for our searching :
Questions
(a) Who are we wreathing a flowery band and why? 2
(b) Why is the poet not happy with human beings? 1
(c) Explain “O’er darkened ways”. 1

3. Yes, in spite of all


Some shape of beauty moves away the pall
From our dark spirits-Such the sun, the moon,
Trees old, and young, sprouting a shady boon
For simple sheep, and such are daffodils
With the green world they live in; and
Clear rills. That for themselves a cooling covert make
‘Gainst the hot season.
Questions
(a) What ‘moves away’ the pain and suffering from the human life? 1
(b) Mention some of the things from Nature that move away the pall from
our dark spirits? 2
(c) What do the clear streams do? 1
(d) Explain “dark spirits” 1

4. Rich with sprinkling of fair musk rose blooms;


And such too is the grandeur of the dooms
We have imagined for the mighty dead;
All lovely-tales that we have heard or read;
An endless fountain of Immortal drink,
Pouring unto us from the heaven’s brink.
Questions
(a) What is the beautiful scene that one sees in the middle of the forest?1
(b) Why does the poet call the dooms to be full of grandeur? 1
(c) Whom does the word ‘mighty dead’ refer to? 1
(d) Explain ‘Immortal drink’. 1
89 XII – English
Answer the following questions in 30-40 words each. (2 marks each)
(a) Why does the poet call a thing of beauty as a joy forever?
(b) How does beauty enrich the human beings?
(c) How do we try to face the gloomy days?
(d) How is man responsible for his own despondence?
(e) How do the daffodils and clear rills adjust to the hot season?
(f) How does the poet express his love for nature?

5. A ROADSIDE STAND

–Robert Frost
Read the following extracts and answer the questions that follow each:
1. The little old house was out with a little new shed
In front at the edge of the road where the traffic sped,
A roadside stand that too pathetically pled,
It would not be fair to say for a dole of bread,
But for some of the money, the cash, whose flow supports
The flower of cities from sinking and withering faint.
Questions
(a) Why has the road side stand been set up? 1
(b) What is the importance of money for cities? 1
(c) What quality of the people who setup the roadside stand is shown
through these lines? 1
(d) Explain–(i) “Pathetically pled”, (ii) dole of bread. 1

2. The polished traffic passed with a mind ahead,


Or if ever aside a moment, then out of sorts
At having the landscape marred with the artless paint
Of signs that with N turned wrong and S turned wrong
Offered for sale wild berries in wooden quarts,
Or crook-necked golden squash with silver warts,
or beauty rest in a beautiful mountain scene,

90 XII – English
Questions
(a) What do these lines tell us about the attitude of city dewellers? 1
(b) In what manner does the traffic pass that way? 1
(c) Explain–”Out of sorts”. 1
(d) What do the road side people sell to the “City folks”. 1

3. Here far from city we make our roadside stand


And ask for some city money to feel in hand
To try if it will not make our being expand,
And give us the life of the moving pictures promise
That the party in power is said to be keeping from us.
Questions
(a) Who are ‘we’ and what do they want to feel in their hands? 1
(b) How do ‘We’ plan to earn money? 1
(c) What type of life do they desire? 1
(d) Write an example of political interface in their lives? 1

4. Sometimes I feel myself I can hardly bear,


The thought of so much childish longing in vain,
The sadness that lurks near the open window there,
That waits all the day in almost open prayer,
for the squeal of brakes.
Questions
(a) What hope does the squeal of brakes kindle in the road side people?
(b) Who has been longing and for what? 1
(c) Why is the longing termed as childish? 1
(d) What do the expressions “open window” and “open prayer” mean? 1

91 XII – English
Answer the following questions in 30-40 words each. (2 marks each)
Q1. Why and where was the roadside stand set up?
Q2. How does flow of money help in the development of cities?
Q3. What things are for sale at the roadside stand?
Q4. In what way are the villagers befooled by the people in power?
Q5. Why do the people who are running the roadside stand wait for the
squeal of brakes so eagerly?
Q6. With what purpose do cars stop at the road side stand? Mention any
two?
Q7. According to the Poet, how can the problem of rural people be solved?

6. AUNT JENIFFER’S TIGERS

–Adrienne Rich
Read the following extracts and answer the questions that follow each:
1. Aunt Jennifer’s tigers prance across a screen,
Bright topaz denizens of a world of green.
They do not fear the men beneath the tree;
They pace in sleek chivalric certainty...
Questions
(a) Where do the tigers appear? What are they doing? 1
(b) What is the attitude of the tigers towards men? 1
(c) How does the poet portray the outer and inner qualities of the tigers?2

2. Aunt Jennifer’s fingers fluttering through her wool


Find even the ivory needle hard to pull.
The massive weight of Uncle’s wedding band
Sits heavily upon Aunt Jennifer’s hand.
Questions
(a) Why are aunt Jennifer’s fingers fluttering? 1
(b) What is the result of the fluttering? 1

92 XII – English
(c) Explain–‘The massive weight of Uncle’s Wedding band.’ 1
(d) What is Aunt Jennifer’s state of mind?

3. When Aunt is dead her terrified hands will lie


Still ringed with ordeals she was mastered by
The tigers in the panel that she made
Will go on prancing, proud and unafraid.
Questions
(a) Why are aunt’s hands said to be terrified? 1
(b) Explain–‘still ringed with ordeals? 1
(c) How will Aunt’s death affect the tigers? 2

Short Answer Type Questions


Answer the following questions in 30-40 words each. (2 marks each)
1. Where do the tigers appear? Write two qualities of the tigers as depicted
by the poet.
2. Why is the ivory needle hard to pull?
3. What is the significance of wedding ring in Aunt Jennifer’s life?
4. “The tigers created by Aunt Jennifer are symbolic of her aspirations.” Explain.

1. THE LAST LESSON

–Asphonse Dandet
Notice on Bulletin Board
❑ Crowd Gathered
❑ School quiet
– No sound of desks, lessons, teacher’s ruler.
M. Hamel
– didn’t scold/spoke kindly
– wearing green coat, frilled shirt, black silk cap.
– announced last lesson in French, German to be taught

93 XII – English
❑ Villagers, Hauser, postmaster, others – On back benches.
Franz’s reaction to last lesson in French
❑ Shocked, realized deficiency in language, Wished-not wasted time.
❑ felt sorry, repented for wasting time.
❑ appreciated M.Hamel, accepted him with his faults.
❑ wanted to impress M.Hamel with knowledge of French.
❑ Understands presence of villagers
– to thank master
– to show solidarity for country
– repenting for not attending school
M. Hamel made the class realize
❑ reluctance to learn, postponing learning
❑ parents sending them to work
❑ himself sending students to water his plants
❑ all to blame for situation
❑ too late to make amends
❑ no time to make up for lost time
❑ Germans will laugh at inability to speak French.

Main Characters
1. M. Hamel
❑ Sincere French teacher
– knows subject well
❑ Passionate about French
– considers French-clearest, most beautiful, logical language
– feels language as a key to person’s sense of freedom
– advises to hold on to French
❑ Proud of being French
– upset by occupation of Alsace by Germans
– attached to town, school, people.
94 XII – English
❑ Hard Task Master
– particular about discipline and learning
– students scared
– last day - exercises on all aspects of language
❑ Sensitive, honest
– blames himself for selfishness
– emotional by sound of Prussian soldiers
2. Franz
❑ Sensitive, Honest
– Blames himself for ignoring lessons
– Wonders if pigeons will coo in German
– Understands feelings of Hauser
❑ Loves outdoors
– Sunshine, birds, butterflies, collecting bird’s eggs.
❑ As a student
– repents, good observer, notices changes.
❑ Empathizes with M. Hamel
– Understands emotions, accepts him with faults

Short Answer Type Questions


Answer the following in 30-40 words each. (2 marks each)
1. Why was Franz afraid to go to school that morning?
2. What temptations did Franz overcome to proceed to school?
3. What was the significance of the bulletin-board in the town hall?
4. Why was Franz blushing and feeling frightened when he entered the class
room?
5. Why did the writer want to run away and spend the day outdoors? Did he
overcome his temptation?
6. How and why was M. Hamel dressed differently that day?
7. Why were the old men of the village present in the class room?

95 XII – English
8. How was the order from Berlin going to make a difference to the lives of
the students?
9. How did Franz’s attitude towards his books change after he heard about
the order from Berlin?
10. What universality of human nature does the teacher comment upon?
11. Whom does the teacher blame for ignoring the learning at school and why?
12. How does M. Hamel held parents and himself responsible for students
neglecting learning? (having not studied?)
13. How does M. Hamel praise the French language?
14. How is the mother tongue important for the enslaved people?
15. Why was Franz able to understand everything?
16. “I never saw him look so tall”. Who is the person mentioned? What does
the speaker mean by the above statement?
17. What did M. Hamel write on the black board at the end of the class? How
is it significant?
18. Why did M. Hamel make a gesture to the students to leave?

Long Answer Questions


Answer the following in about 125-150 words each (10 marks each)
1. Justify the title “The Last Lesson.”
2. In the story M. Hamel emerges not only as a dedicated teacher but also as
a sensitive and understanding human being”. Elaborate the character of M.
Hamel in light of the above statement.
3. “When a people are enslaved, as long as they hold fast to their language
it is as if they have the keys to their prison.” Discuss the significance of this
statement in light of the lesson.
4. Franz’s attitude towards school as well as towards M. Hamel changes when
he comes to know about the take over of his village by the Prussians. Do
you agree? Discuss with reference to “The Last Lesson.”

96 XII – English
2. THE LOST SPRING :
STORIES OF STOLEN CHILDHOOD

–By Anees Jung


Sahib-e-Alam
– Name means – lord of the universe, migrated with his family from Dhaka
in 1971, lives in Seemapuri, survives by ragpicking, doesn’t go to school, works
barefooted.
Living Conditions : Lack of basic amenities, structures of mud, tin and
tarpaulin, no sewage, no drainage, no water, loses freedom when he starts
working at a tea-stall.
Mukesh : Banglemaker of Firozabad, works in high temperatures, workplace
small and dirty, working conditions hazardous. Dreams of being a motor mechanic.
Similarities between Saheb-e-Alam and Mukesh
– have aspirations and desires like other children
– suffer from immense poverty and scarcity of food
– caught in the web of child labour
– leading a life of exploitation
– undergo physical, mental and emotional sufferings
Living Conditions in Seemapuri
– on the outskirts of Delhi yet miles away from it, home to 10,000
ragpickers, make a living by picking garbage, mastered the art of rag
picking, food and survival more important for their identity, live in a
state of poverty. Garbage to them is gold because it is a means of
survival, sometimes they find a rupee or even a ten rupee note and
for children garbage is wrapped in wonder.
Hazards of Working in Glass Bangle Factory
– high temperatures, lack of proper ventilation and light,
– long working hours in front of hot furnace
– boys and girls assist parents in the dim light of flickering oil lamps.
– eyes are more adjusted to dark than light
– dust from polishing bangles affects their eyes.
– exploited by moneylenders, police, bureaucrats, politicians.
97 XII – English
– fear of being ill treated by police
– lack of a leader who can organize them
– live in state of intense poverty
– sleep on empty stomachs
– live in stinky lanes
– overcrowded with humans and animals.
Short Answer Type Questions (30-40 words). (2 marks each)
1. Why is the author’s advice to Sahab-e-alam to go to school hollow?
2. Is Sahab-e-alam hurt by the author’s failure to keep her promise of starting
a school? Give a reasoned answer.
3. Bring out the irony in the name Sahab-e-alam?
4. How is Seemapuri on the outskirts of the city but still miles away from it?
5. How do children of rag pickers become equal partners in survival?
6. How has rag picking acquired the proportions of a fine art?
7. How did Saheb get tennis shoes? Why does he explain the author how he
had got them?
8. Describe the working condition of bangle making of Firozabad.
9. “Garbage to them is gold”. Bring out the significance of this statement.
10. How does an adult and a child rag picker look at garbage differently?
11. Why is Saheb not happy with his new job?
12. How is the bangle industry of Firozabad a curse for the bangle makers?
13. What is the significance of bangles for an Indian woman?
14. Why don’t the bangle workers organise themselves into a co-operative?
15. Why hasn’t Firozabad changed with time?
16. What does Mukesh proudly say that his house is being rebuild?
17. Why is Mukesh’s dream of becoming a motor mechanic significant?

98 XII – English
Long Answer Type Questions
Answer the following in 125-150 words (10 marks each)
1. “Survival in Seemapuri means rag picking”. Give a detailed account of the
life and activities of the refugees from Bangladesh settled in Seemapuri.
2. The life of bangle makers of Firozabad was full of obstacles which forced
them to lead a life of poverty and deprivation. Elaborate
3. The lesson “Lost Spring” is a realistic portrayal of the lives of street children.
Discuss.
4. The beauty of the bangles made in Firozabad is in direct contrast to the
life of the people who make them. Elaborate.
5. What are the hazards of working in bangle factories?
6. “Poverty is a vicious cycle for refugees, slum dwellers and bangle makers.”
Discuss in the light of the lesson.

3. DEEP WATER
–William Douglas
❑ William Douglas
– Feared Water
– 3/4 years old, went to beach with father knocked down, swept over by
waves suffocated, frightened.
❑ As 10/11 year old
– Decided to learn swimming, Y.M.C.A. pool, Yakimo
– pool safe 2/3 feet at shallow end, 9 feet at deep end
– got water wings
– fear back, beginning to feel comfortable
– feeling short lived, another incident.
❑ Misadventure
– big boy threw him into deep end
– frightened, lungs ready to burst
– decided to hit feet on bottom and return as a cork
– came up slowly, opened eyes, water every where
– swallowed water and choked

99 XII – English
– terrified, tried to scream, no sound
– legs paralyzed, rigid
– went down, second time
– hit bottom, felt dizzy, paralyzed, rigid
– groped for support, called for help, no result
– went down third time
– stopped struggling, legs limp
– blackness swept his brain
– was quiet, peaceful, drowsy
❑ On being conscious, he
– found himself, lying on stomach, vomiting
– heard voice of same boy
– reached home-felt weak, trembled
– wept, couldn’t eat, frightened, avoided water
❑ Years later, Douglas
– felt terrified near water
– feared water, spoiled holidays
– one October, decided to overcome fear
– hired swimming instructor
– practised swimming, five days a weak, one hour daily
– used belt that went through pulley on overhead cable
– took three months to relax
– learnt to put face in water, exhale, raise nose and inhale
– learnt to kick with legs
– instructor, taught for six months
– made swimmer, no longer feared water
– Douglas not confident
– Went to lake Wentworth, dived of a dock
– swam two miles, scared only once, kept swimming

100 XII – English


– went to Warm Lake, swam across back
– no longer afraid, managed to conquer fear
❑ What we learn from the story : Douglas
– felt who have known fear know feeling to be free.
– recalled Roosevelt’s words
– understood his will to survive and live fully
– confident for facing challenges in future

Short Answer Type Questions


Answer the following in about 30-40 words each. (2 marks each)
1. How and when did Dougles develop an aversion to water?
2. What strategy of escaping drowning did Douglas have in mind?
3. Why couldn’t Douglas implement his strategy of escaping drowning?
4. What was the immediate effect of the drowning incident on Douglas?
5. How did Douglas feel after receiving training from the swimming instructor?
6. Mention any two long term consequences of the drowning incident on
Douglas.
7. Why did the drowning experience deprive Douglas from the pleasure of
water sports?
8. What did Douglas do to overcome his fear of water?
9. Why did the experience of conquering his fear of water have a deeper
meaning for Douglas?
10. “I screamed, but only the water heard me” why did Douglas scream and
what does he mean?
11. Mention two qualities of Douglas that helped him conquer his fear.
12. Even after receiving training from the instructor why did Douglas not feel
confident to swim?

101 XII – English


Long Answer Type Questions
Answer the following in 125-150 words. (10 marks each)
1. “Piece by piece, he built a swimmer”. How did the instructor help the author
to become a swimmer and overcome his fear of water?
2. What emotions did the author experience while he was drowning in the
YMCA pool? How did he face the near-death experience?
3. “If we surrender to our fears, they overpower us; if we face them, they fade
away. Do you agree? Why/Why not? Discuss with reference to the lesson
“Deep Water.”

4. THE RATTRAP

–Selma Lagerlof
❑ Rat Trap Man
– made living by selling rattraps
– business not profitable
– had to beg and steal
– even then poor
❑ His Philosophy
– whole world was a rat trap
– if offered riches, joys, shelter, food etc., as bait.
– if tempted to bait, everything came to end
❑ Simplicity of old man who gave shelter to Rat trap man
– one night rat trap man stayed at old man’s house
– old man happy to have someone
– simple, generous, hospitable
– offered big slice from tobacco roll, played cards
– told about past, was a crofter, prosperous
– now, his cow supported, earned thirty kronors
– showed the rat trap man pouch having Kroner
– next day, rat trap man stole money.

102 XII – English


❑ Rat trap man loses his way
– does not continue on public highway
– goes into woods, loses way
– walks endlessly, tired, realizes
– realizes he himself has been caught in rat trap
– hears sound of hammer strokes
– walks in that direction, reaches iron mill
– finds master smith, doesn’t notice rat trap man
– later, blacksmith grants permission to sleep.
❑ Iron master and his daughter Edla
– owner of Ramsjo Iron Mill
– mistakes rat trap man as old acquaintance called von Stahle
– invites him to home to spend Christmas
– rat trap man doesn’t agree
– agrees when Edla comes and requests
– overwhelmed by her compassionate and friendly manner
❑ Secret revealed
– rat trap man well groomed
– iron man realizes mistake
– rat trap man makes no attempt to hide
– ready to leave wearing old rags
– Edla requests father to allow him to stay
– they had promised Christmas cheer
❑ The rat trap man turns a new leaf
– quietly eats food, sleeps
– next morning, father and daughter go to church
– hear that crofter robbed by a rat trap man
– daughter sad, informed rat trap man left gift
– small rat trap with thirty kronors and a note

103 XII – English


– wishes money to be returned to crofter
– confesses that he had made a mistake
– got caught in his own rat trap
– thanks for treating him like a real captain
❑ What we learn from the story
– Life is one big rat trap
– one gets trapped by own deeds
– everyone should get a second chance to improve oneself

Short Answer Type Questions


Answer the following in about 30-40 words each. (2 marks each)
1. How did the peddler earn his livelihood?
2. What strange idea about the world struck the peddler?
3. How did the crofter treat the pedder and why?
4. Why was the crofter so talkative and friendly with the peddler though he
was a stranger?
5. How did the peddler abuse the trust the crofter had reposed in him?
6. What thoughts came to the peddler’s mind when he realised he had lost
his way in the jungle?
7. How was the iron master’s reaction to the peddler different from that of the
blacksmith?
8. Why did the peddler decline the iron master’s invitation?
9. What made the peddler accept Edla’s invitation?
10. What impression did Edla form about the peddler?
11. What did the iron master realise after the peddler had been groomed by
his servants?
12. Pick two instances from the story ‘The Rattrap’ to show that the peddler
realized he himself was trapped?
13. How did the peddler feel when he realised he could not getout of the
forest?

104 XII – English


14. Why did the iron master decide not to hand over the peddler to the sheriff?
15. Why did Edla insist on entertaining the peddler even after he was exposed?
16. What had the peddler left behind as a Christmas gift for Edla Willmanson?
17. What did the peddler think on the way to the iron master’s house.
18. What did the gift of the rat trap signify?
19. Why was Edla happy to see the gift left by the peddler?
20. Why were Edla and her father surprised on reaching home after the church
service?
21. Why did the peddler sign himself as Captain Van Stahle?

Long Answer Type Questions


Answer the following in 125-150 words each. (10 marks each)
1. What rattrap was the peddler trapped in? How did he come out of it?
2. The story “The Rat Trap” is not only entertaining but also philosophical.
Bring out the entertaining and philosophical elements of the story.
3. “A simple act of mercy and kindness can bring about a change of heart”.
Discuss the significance of love and kindness with reference to the story
“The Rattrap”.
4. “Man is a social animal. He can survive but not live in isolation”. Do you
agree? Justify your answer.

5. INDIGO

–Louis Fischer
❑ Gandhiji’s meeting with Shukla
– Gandhiji at Indian National Congress Annual Convention
– Shukla, a poor, indigo peasant requested him to come to Champaran
– Gandhiji agreed after completing task
– Visited Rajendra Prasad’s place in his absence on way to Champaran
– Peasant came to meet him at Muzaffarpur
– Met lawyers and scolded them for charging fee from poor peasants.

105 XII – English


❑ Problems of Peasants
– Landlords forced sharecroppers to grow indigo in 15% of the land
– Germany developed synthetic indigo
– Landlords demanded compensation for freeing the peasants from 15%
arrangement
– Some agreed but later demanded money back
❑ Gandhiji arrived at Champaran
– wanted to meet secretary of British Landlord’s Association.
– was refused
– tried to meet Commissioner of Tirhut
– was bullied and ordered to leave Champaran
– Defied order
– Prevented from meeting peasant
– Disobeyed notice to leave Champaran
– was summoned at court
– worked whole night to get the support
– wired report to the Viceroy
– peasants gathered in large number to show support to him.
– Gandhiji proved that British power was no longer unchangeable
– authorities got afraid and postponed the case
– Gandhiji released on bail
– lawyers decided to follow Gandhiji
❑ First Triumph of Civil Disobedience
– Case dropped against Gandhiji
– he planned Civil Disobedience
– commission of enquiry appointed by governor
– evidence against landlords found.
– Gandhi agreed for 25% refund as was agreed by landlords
– indigo share cropping abandoned and
– land given to peasants

106 XII – English


❑ Gandhiji’s forsight – beyond Political & Economical solution
– aimed to improve social and cultural status of Champaran
– Aimed at improving health services
– took help of volunteers
– tought Indians to be self reliant and independent
❑ Freedom from fear more important than freedom from legal justice
– Real relief for peasants was to be free from fear
– Courts were useless for fear stricken
– Self-reliance, strong will and courage to win battles.
– Must protest against injustice.
Short Answer Type Questions. (30-40 words each) – (2 marks each)
1. Who was Rajkumar Shukla? Why did he want to meet Gandhiji?
2. How did Shukla persuade Gandhiji to visit Champaran?
3. Why was Gandhiji not allowed to draw water from the well of Dr. Rajender
Prasad’s house?
4. What did the peasants of Champaran pay to the British landlords as rent?
5. Why did Gandhiji go to a near by village in Champaran? Why did he
comeback without visiting the place?
6. What was the outcome of protracted interviews that Gandhiji had with the
Lt. governor?
7. Why did the landlords want to be released from the 15% arrangement?
8. Which incident in the lesson is the beginning of the peasants liberation
from the fear of British?
9. Why did Gandhiji agree to a settlement of 25% refund to the farmers?
10. What was the conflict of duties that Gandhiji underwent? How did he resolve
it?
11. Why did Gandhiji receive summons to appear in court?
12. “Civil Disobedience had triumphed the first time in India.” How did it happen?
13. When and why did Gandhiji say “The battle of Champaran is won”?
14. How was Gandhiji’s style of politics different from the usual politicians?

107 XII – English


15. How was the Champaran episode a turning point in Gandhiji’s life?
16. How did Gandhiji teach his followers a lesson in self-reliance?
17. What steps did Gandhiji take to remove the social and cultural backwardness
of the Champaran villagers?
18. What was done to improve and look after the health of the people of
Champaran?

Long Answer Type Questions


Answer the following in 125-150 words. (10 marks each)
1. Dialogue not violence can resolve situations of conflict and injustice. Do
you agree? Answer with instances from the lesson Indigo.
2. “The visit undertaken casually on the entreaty of an unlettered peasant
occupied almost a year of Gandhiji’s life”. What events unfolded and with
what results?
3. How was Gandhiji’s stay and work at Champaran a great turning point to
the people of Champaran, to the freedom strugg le and to Gandhiji himself?
4. Raj Kumar Shukla played as a significant a role as Gandhiji did in the
success of the Champaran episode. Do you agree? Give reasons in support
of your answer.

6. POETS AND PANCAKES

–By Asokamitran
Key Points
❑ Pancake – What is it? — the make-up material.
❑ The Make-up Department
– The make-up room was like hair cutting salon with bright lights and
mirrors
– headed by a Bengali first, later by Maharashtrian, helped by an
Andhdraite, a Madras Christian and Anglo-Burmese and a local Tamil
– Example of national integration
– Hierarchy was followed
– Chief actors and actresses

108 XII – English


– Assistant the second hero
– The office day of the crowd
❑ The Office Boy
– was in early forties
– did make up of the crowd
– joined to become an actor, or a screen writer, director or lyric writer,
was a bit poet.
– Hopes shattered, seemed frustrated
❑ Inside the Gemini Studios
– Duty of the writer to cut out newspaper clippings
– everyone thought him as free
– office boy expressed his anger against Subbu, the No. 2 at studios.
❑ Subbu
– Kothamangalam Subbu – No. 2 at Gemini Studios
– always accompanied the boss
– was in story department
– story department had lawyer, writers and poets
– was cheerful and loyal
– habit of being nice
– was charitable
– could suggest many ways of filming a scene
– was a poet, an amazing actor
– had many enemies, office boy being the main.
❑ The Lawyer
– was in story department
– was not very popular
– recorded outbursts of an actress and played the recording
– she left acting after this

109 XII – English


– he dressed in pant, coat and tie differently than others who were
Gandhiates
– story deptt was closed and he lost the job
❑ Gemini Studios – The Favourite Meeting Place of Poets
– Mr. Vasan the boss, was editor of Tamil weekly – Ananda Vikatan
– Main prominent poets visited studio
– They wore khadi, were against communism 1952 Frank Buchman’s
moral Re-Armament army visited studio
– Tamil plays copied their sets for years
– Hosting two hundred people of twenty nationalities was great change
❑ Visit of the English Poet
– A tall Englishman, a poet visited the studios
– The boss welcomed the poet
– The poet had English accent, he spoke without being understood
– Years after retirement, Asokamitran saw a pile of books, with a book
titled the God that failed.
– It was collection of essays by six eminent writers
– The name of the editor was Stephen Spender
– The same poet who had visited the studio
– It cleared the mystery of Spender’s visit to the studios

Short Answer Type Questions


Answer the following questions in about 30-40 words. (2 marks each)
1. What is the significance of the words “Poets & Pancakes?”
2. Why did the make up room of Gemini Studio look like a hair cutting salon?
3. How was Gemini studio an example of national integration?
4. How was hierarchy maintained in the make-up department?
5. What work did the office boy do at Gemini Studios?
6. With what dreams had the office boy joined the Gemini studios?
7. Why did the author appear to be doing nothing at Gemini Studio?

110 XII – English


8. What make the lawyer lose his job? What does the writer find so funny
about in situation?
9. Why was the office boy frustrated? Whom did he vent his anger upon?
10. What was the dress code at Gemini studios? How did the lawyer stand
apart?
11. How did the plays staged by MRA influence Tamil plays deeply?
12. Why is the Englishman’s visit referred to as unexplained mystery?
13. How did the magazine ‘The Encounter” ring a bell in Asokmitran’s’ heart?
14. According to Asokamitran what qualities should an aspiring prose writer
possess?
15. Why did Stephen Spender’s visit to the Gemini Studio remain an” unexplained
mystery?
16. Why did Stephen Spender visit Gemini Studios?
17. What made Kothamanglam Subbu the number two at the Gemini Studio?

Long Answer Type Questions


Answer in about 125-150 words each. (10 marks each)
1. Subbu was the man who gave directions and definition to Gemini studio?
In the light of the above statement discuss Subbu’s character.
2. How did the lawyer bring an end to an acting career? What brought an end
to his own career at Gemini Studio?
3. Whether he writes about Pancakes or about poets Asokamitran sprinkles
humour throughout the lesson. Explain with instances from the lesson.
4. Explain the significance of the title of the essay.

7. THE INTERVIEW

–Christopher Silvester
Part - I : Interview
– Common feature of journalism
– Varying views of celebrities about interview
– Lewis Carroll had horror of interviews

111 XII – English


– H.G. Wells calls it an ordreal
– V.S. Naipul that people are wounded and lose part of themselves
– Denis Brian opines it holds a greater power over an interviewee.
Part II : Interview of Umberto Eco
– Mukund Padmanabham interviewed Umberto Eco a professor at the
University of Bologna in Italy.
– Umberto was asked how he managed to do so many things.
– Eco said he believed in working in the spare time.
– he called them empty spaces.
– Eco’s non-fictional writings had a certain light hearted and personal
quality about it.
– At 22, Eco managed to know how to write thesis in the form of story
of a research.
– He enjoyed being Professor more than a novelist.
– Eco’s novel ‘The Name of Rose’
– He did not know the reason behind the success of his novel.

Short Answer Type Questions


Answer the following in 30-40 words each. (2 marks each)
1. Why has the interview become a widely known term today?
2. Mention two extravagent claims made in support of the interview.
3. Why do some celebrities despise being interviewed?
4. What is the belief in some primitive cultures regarding people being
photographed?
5. Why did Lewis Carroll never agree to be interviewed?
6. How did Rudyard Kipling contradict his own views on interviewing?
7. Why does Saul Bellow refer to interviewing as thumbprints on his windpipe?
8. What secret does Eco reveal about his time management?
9. Why according to Eco did the ‘Name of the Rose’ become such a huge
success?

112 XII – English


10. What did the publisher think of ‘The Name of the Rose”?
11. What did Eco’s professor have to say about his doctoral dissertation?
12. How did Eco respond to David Lodge’s remark in the interview of Mukand?
13. Despite the drawbacks of the interview, why is it a supremely serviceable
medium of communication?

Long Answer Type Questions


Answer in about 125-150 words each. (10 marks each)
1. “I can’t understand how a man can do all the things he does”. Discuss this
statement with reference to Umberto Eco.
2. What is Umberto Eco’s book ‘The Name of the Rose’ about? Why did it
become an unprecedented success?
3. “The interview is both a much maligned and a supremely serviceable medium
of communication”. Discuss with reference to the lesson “The Interview”.
4. “Mukund Padmanabhan has done his home work on Umberto Eco and
knows what he is talking. However, he links with Eco spontaneously and this
makes for a very smooth reading.” Do you agree. Why?
5. “The interviewer holds a position of unprecedented power and influence”
What arguments does christopher silvester give in support of this statement?

8. GOING PLACES

–A.R. Barton
❑ Sophie’s Ambition and Unrealistic Dreams
– wants to start a boutique
– to earn money by becoming a manager, an actress or even a fashion
designer
– she belongs to weak socio-economic background
– doesn’t have decent house.
❑ Her Friend Jansie
– simple and nosey
– wants sophie to be happy and realistic

113 XII – English


– cann’t keep information or secrets to herself
❑ Sophie’s Dad
– is strong, sweaty and hardworking
– Loves to watch foot boll matches
– Does not believe in Sophie’s imaginary stories.
❑ Sophie’s brother - Geoff
– is an apprentice mechanic
– speaks less
– sophie trusts him and shares her secrets with him
❑ Sophie lives in a world of her own/her fantasizing
– Sophie is fond of hero worship and fantasizing
– Sophie’s hero is an Irish player Danny Casey
– Dreams of meeting him
– Is so much obssessed with meeting him
– Starts believing her imaginations as real
– No body believes her
– Feels disappointed
– She still believes that she has met him.

Short Answer Type Questions


Answer the following in 30-40 words each. (2 marks each)
1. What does Sophie dream of doing after leaving school?
2. How are Jansie and Sophie poles apart though they are friends?
3. What does the description of Sophie’s home reveal about her family’s
socio-economic status?
4. What work is Geoff engaged in? How does he differ from his sister Sophie?
5. Why did Sophie feel jealous of Geoff’s silence?
6. How did Sophie want to be a part of Geoff’s world?
7. Who is Danny Casey? How do Sophie’s brother and father react to her
claim of meeting Danny Casey?

114 XII – English


8. Do you think Sophie had actually met Danny Casey? Why? Why not?
9. What range of emotions did Sophie undergo as she waited for Danny
Casey?
10. ‘The family was a great fan of Danny Casey and his game’ What incident
in the story suggest this.
11. Who is responsible for Sophie’s tears? Why do you think so?

Long Answer Type Questions


Answer in about 125-150 words each. (10 marks each)
1. Bring out the difference in the character of Sohie and Jansie.
2. “Going Places” is a story that borders on fantasy and reality. Discuss.
3. The meeting of Sophie with Danny Casey is a figment of her imagination.
Do you agree? Why? Why not?
4. Write a character sketch of Geoff. How is it different from Sophie?
5. “Sophie’s dream world clashes with the world of her family and friends” Do
you think Sophie is able to balance her dream world with her reality”
Comment.

115 XII – English


VISTAS

1. THE THIRD LEVEL

–By Jack Finney


Charlie : 31 year old, office goer, claims visiting the third level
❑ Description of the Third level
– Small room, few ticket windows and train gates, wooden and old looking
information booth.
– Men had beards, side burns, fancy moustaches
– Women wore skirts, high buttoned shoes and leg of muttons sleeves.
– A man looking at a pocket watch
– Old style locomotive with funnel shaped stack
– Open gaslights being used
– Brass spittoons on floor
– Wants to visit his home town, Galesburg
– Past is quiet and peaceful
– Tries to buy two tickets to Galesburg (one ticket for his wife Louisa)
– Clerk grows suspicious as Charlie doesn’t have old style currency.
– Back to present day world
❑ People’s reaction to Charlie’s experience
– Presidents of NY rail roads swear on the existence of two levels.
– Psychiatrist friend Sam refuses to believe
– Interprets it as an escape from insecurity fear, war and worry of the
modern world.
– Louisa too disbelieves Charlie
❑ Charlie’s determination to find the Third Level
– Withdraws money, buys old currency worth 300 dollars.
– Fails to find the Third Level
– Louisa and Psychiatrist worried.
116 XII – English
❑ Unexpected Ending
– Sam disappears
– Charlie finds a first day cover, never seen before
– Note from Sam dated 18th July 1894 from Galesburg
– Sam asks Charlie and Loulsa to come to Gelesburg and enjoy quiet
and peaceful life.
– Charlie discovers Sam had bought old currency worth 800 dollars.
– Enough to help him start hay and grain business in 1894 at Galesburg.

Short Answer Type Questions


Answer the following questions in about 30-40 words each. (2 Marks
each)
1. How does the psychiatrist friend interpret Charley’s visit to the Third Level?
2. Why did the psychiatrist’s interpretation upset Charley’s wife? How did the
psychiatrist pacify her?
3. How did Charley defend himself from being accused as an escapist?
4. Why did Charley decide to take the subway from Grand Central?
5. Why did Charley make a quick escape from the platform on the third level
after the clerk at the ticket counter refused to accept the money?
6. Why does charley say “it is easy to get lost at the grand central station”?
7. Why did Charley want to visit Galesburg of 1894?
8. What preparations did Charley make to go to Galesburg?
9. Why was Charley not worried even though he got two hundred dollars in
exchange of three hundred dollars?
10. What do you know about a first-day cover?
11. How did Louisa react to Charley’s quest for the Third Level? How did her
attitude change later?
12. How were Charley and Louisa convinced about the existence of the third
level?
13. Do you think the third level was there? Why/Why not?

117 XII – English


Long Answer Type Questions
Answer the following in about 125-150 words. (7 Marks each)
1. What differences did Charley observe at the third level of the Grand Central?
2. Hobbies provide a refuge from reality from our otherwise monotonous lives.
Elaborate the statement in the light of the lesson.
3. Charley is so engrossed with fiction that he loses touch with reality. Do you
agree. Give reason.
4. How does one cope with the stress and tension of the modern world?
Discuss with special reference to the chapter, “The Third Level.”

2. THE TIGER KING

–By Kalki
❑ Maharaja’s Childhood and Prophecy
– Jung Bahadur born, astrologers predict death due to a tiger
– Royal upbringing, everything had an English stamp-nanny, food, milk,
tutor and entertainment
❑ Maharaja’s vow to kill tigers
– vowed to kill 100 tigers
– faced risks in tiger hunting
– heavy fine if anybody except the king hunted tigers
– risked his throne by not allowing British officer to hunt tigers.
❑ Obstacles in fulfilling his Vow
– Tiger population depleted
– Married a princess whose state had large tiger population
– killed ninety nine tigers, hundredth tiger refused to show up
– Feels frustrated at his inability to kill the 100th tiger.
– Officers lose jobs, revenue of a village increased as punishment for
not finding a tiger.
❑ The Resourceful Diwan
– Arranged old tiger from Madras

118 XII – English


– King shoots the tiger, misses the shot, tiger faints, taken for dead.
– Afraid to reveal this fact to the king
– 100th tiger actually killed by a hunter.
❑ Ironical Ending
– Having killed 100 tigers king becomes complacent and careless
– Buys wooden tiger on son’s birthday
– Wooden sliver pierces King’s right hand
– Infection spreads, operated by best surgeons, doesn’t survive.
Irony – King killed ninety nine tigers but death comes due to a toy tiger.
Answer the following questions in about 30-40 words each. (2 Markseach)
1. What secret did the astrologer reveal at the birth of the prince?
2. What was incredible about the infant’s reaction to the astrologer’s prophecy?
3. How did the tiger pose a threat to the king?
4. What is the significance of the prince’s utterance – “Let tigers beware”?
5. Bring out the humour in the upbringing of the prince.
6. Why was the Maharaja in danger of losing his throne?
7. How did the Maharaja manage to retain his throne?
8. What was the hurdle that came in the way of the Maharaja’s ambition to kill
hundred tigers?
9. Why and on what condition did the Maharaja want to marry?
10. What did the astrologer promise to do if the king killed the hundreth tiger?
11. Why didn’t the Maharaja agree to the suggestion of the British officer’s
secretary?
12. Why wasn’t the Maharaja informed that the hundredth tiger did not die from
his shot?
13. How and why was the hundredth tiger honoured?
14. Why didn’t the shopkeeper quote the real price of the wooden tiger?
15. How did the wooden tiger lead to the death of the Maharaja?
16. Why does the writer compare the new born prince’s claim with the bulletins
issued by war office?

119 XII – English


Long Answer Type Questions
Answer the following in about 125-150 words. (7 Marks each)
1. What danger loomed over the Maharajah’s throne? How did he succeed in
dispelling it?
2. “The king spent a lifetime trying to kill hundred tigers but his death came
due to a toy tiger.” Comment
3. Bring out the element of humour, satire and irony in the story “The Tiger
King.”
4. Write the character sketch of ‘Dewan’.

3. JOURNEY TO THE END OF THE EARTH

–By Tishani Doshi


❑ Antarctica’s Past
– 650 million years ago part of Gondwana, a giant southern subcontinent
existed
– higher temperatures
– huge landmass separated into countries
❑ Antarctica Today
– No human population
– No trees, bill boards, buildings
– Blue whales and icebergs as big as countries
– Silence everywhere
❑ Human Impact
– Man has disturbed balance in nature
– Dominated nature with villages, towns, cities megacities
– Burning of fossil fuels leads to increase in global temperature.
– Climate Change
– Activities of phytoplankton will be affected
– Marine birds and animals will be affected.

120 XII – English


❑ Students on Ice Programme
– Aims at providing students educational opportunities to foster new
understanding and respect for our planet.
– Hope for the Future
– Students can make a significant contribution in saving the earth

Short Answer Type Questions


Answer the following questions in about 30-40 words each. (3 Markseach)
1. How did the author feel after reaching Antarctica?
2. How is present day Antartica different from GONDWANA?
3. Why does one lose all earthly perspective on reaching Antarctica?
4. How have human beings contributed to an increase in average global
temperature?
5. Why does Tishani Doshi call hereself a sun worshipping South Indian?
6. Why is Antarctica still pristine?
7. Why is Antarctica a crucial element in the debate on climate change?
8. Why did Geoff Green stop organizing tours for the rich and curiosity seekers?
9. Why is the involvement of students crucial in the “Students on Ice”
Programme?
10. How have human beings created a ruckus in 12000 years of their existence
on earth?

Long Answer Type Questions


Answer the following in 125-150 words. (7 Marks each)
1. How are human beings posing a threat to the pristine purity of Antarctica?
2. By whom and with what objective was the “Students on Ice” Programme
started? How far has it achieved its goals?
3. Why is Antarctica essential to understand the earth’s past, present and
future?
4. “A lot can happen in a million years but what a difference a day makes”.
What is the relevance of this statement with reference to the lesson?

121 XII – English


4. THE ENEMY

–Pearl S Buck
❑ Character Sketch of Sadao
– Dedicated surgeon and doctor, has fellow feeling and kindness for
people in distress
– Conflict between duty as a doctor and duty as a Japanese
– prejudiced against the white man, afraid of being called a traitor
– Shelters the enemy and saves his life.
– Out of loyalty to his country, informs the General
– helps the white man escape
❑ Character Sketch of Hana
– Balanced woman, stands with her husband
– Responsible - washes prisoner, treats him respectfully
– Dignified and graceful about servants leaving the house
– helped Sadao in the operation
– administers anesthesia to the prisoner
❑ American Soldier – Tom
– Hardly 17, taken prisoner
– Escaped, but shot at the back
– suffered torture and hunger
– Wounded, bleeding was in great pain, lay unconscious
– Strong will-power
– Full of gratitude towards Dr. Sadao
❑ The General
– Selfish, thinks of only his treatment
– had faith in Sadao’s skill as a doctor
– Promises to send private assassins to get rid of the prisoner
– Careless – forgot to send assassins

122 XII – English


❑ Reaction of Servants
– Don’t agree with the idea of helping an enemy
– Yumi refused to wash the white man
– Gardener felt Sadao should have let the soldier die
– Thought sea and gods would take revenge if Sadao saved the soldier
– felt Sadao was proud of his skill and used it irresponsibly
– left the house till the time the prisoner stayed there.
❑ How Sadao helped the soldier escape
– Put food and extra clothing in the boat.
– Directed him to row to an uninhabited island
– Asked him to wait for a Korean fishing boat
– Told him to catch fish but eat it raw
– Gave him his flash light
– Told him to flash the torch twice if food ran out and one flash if he was
allright
– Gave him Japanese clothes and covered his hair with a black cloth.

Short Answer Type Questions


Answer the following questions in about 30-40 words each. (2 Markseach)
1. Why did Sadao wait before falling in love with Hana?
2. Give two reasons why Sadao was not sent abroad with the troops.
3. What do you learn about Sadao’s father from the story ‘The Enemy’?
4. In which condition did Sadao and Hana find the man? How did they establish
his identity?
5. What superstitious beliefs of the servants made them oppose Sadao’s
decision to give shelter to the injured man?
6. Why did Sadao decide to treat the injured man?
7. What did the servants think about Sadao’s sheltering an enemy?
8. What message did the messenger from the palace deliver to Dr. Sadao?
9. How did Hana react to the servants leaving the house?

123 XII – English


10. What plan did the General suggest to get rid of the prisoner?
11. Why couldn’t the General’s plan of eliminating the prisoner succeed?
12. What things did Sadao provide for the wounded prisoner on the boat?
13. What medical aid did Dr. Sadao give the man who had been washed
ashore in front of his house?

Long Answer Type Questions


Answer the following in 125-150 words. (7 Marks each)
1. Dr. Sadao and Hana rise above man made divisions to serve the nobler
cause of humanity. Discuss with reference to the story.
2. What efforts were made by Dr. Sadao and Hana to save the life of the
injured man?
3. Hana plays an instrumental role in saving the life of the injured prisoner.
Give an account of her role in the light of this statement.
4. How did the servant’s react to the efforts made by Sadao and Hana to save
the prisoner’s life?
5. What conflict of duties does Dr. Sadao face? How does he resolve them?

5. SHOULD WIZARD HIT MOMMY?

–By John Updike


❑ Jack’s Story Telling
– Began two years ago
– to tell stories in the evening for the Sunday naps
– Story telling very tiresome
– no longer takes everything as true
– growing up and questions everything.
❑ The Story
– a basic story line and a few characters
– the main character – a small creature
– usually named Roger

124 XII – English


– when in trouble Roger goes to wise owl
– owl sends him to the wizard
– wizard finally solves the problem
❑ Roger Skunk’s Story
– smelled very bad
– no one wants to play with him
– teased and called stinky skunk
– meets the owl and tells his story
– owl asks him to meet the wizard
– the wizard asks for 7 pennies
– he had 4 and took 3 from magic wall
– changed his smell like roses
– Friends were happy and want to play with
– But his mother didn’t like his smell
– became angry and took him back to wizard
– ordered to change back to his original smell
– once again smelled very bad
❑ Jo’s Opinion About the Ending
– not likes the end of the story
– wants to see her character happy
– not wants to smell bad once again
– not likes mommy’s interference
– wants her dad to change the ending
– wants dad to make the wizard hit her
– not convinced at her father’s saying that his mother loved him as he
smelled like her baby
– Jo wants his mommy to understand how his friends used to tease him
and not to play with.

125 XII – English


Short Answer Type Questions
Answer the following in about 30-40 words each. (2 Marks each)
1. What made Jack realize that the custom of telling his daughter a story, had
turned futile?
2. What was unique about the story that Jack told?
3. How did the wizard solve as well as create problem for Roger?
4. What was the cause of Roger Skunk’s sadness?
5. What change occurred in the attitude of other animals when the smell of
Roger Skunk changed.
6. How did Roger Skunk’s mother react to his newly acquired smell?
7. Did Jo accept the violence of Roger Skunk’s mother to the wizard? What
did she want?
8. Jo is no longer a passive listener to Jack’s stories. How?
9. How does Jack justify his ending of the story?
10. Why does Jo want a different ending to the story?

Long Answer Type Questions


Answer the following in 125-150 words. (7 Marks each)
1. Why does Jo want the wizard to hit mommy? Discuss with reference to the
text.
2. Adults should not impose their perspective on kids. Do you agree? Give a
reasoned answer.
3. Bring out the moral issues raised in the story.
4. Jack’s handling of the ending of his story projects him in a poor light as a
father. Do you agree? Why? Why not?
5. Write a detail account of the affinity between jack and Roger skunk

126 XII – English


6. ON THE FACE OF IT

–By Susan Hill


❑ Mr. Lamb Welcomes Derry
– in his garden in spite of jumping over the wall
– tries to make him comfortable
– tells him to pick crab apples
– talks to him, without considering his disability
– Derry thinks, pretending not to be afraid of his burnt face
– tells usually people afraid of his face
– Mr. Lamb asks but doesn’t probe
– tells Derry that he has a tin leg
❑ Mr. Lamb’s advice to Derry
– tells that in Nature shapes differ but quality is the same
– no difference among the plants whether they are weeds, flowers and
others all grow
– inside is important than outside
– accept yourself, not to fear others’ comments.
– calling Lamey-Lamb not bother him
– not to keep thinking of these things
– life offers many more things
– isolation not good
– inspires to look at thing positively
❑ Derry’s reaction to Mr. Lamb’s advice
– initially, very suspicious of Mr. Lamb
– never had anybody to talk to and understand
– not believe that he does not find him ugly
– later, fascinated by Mr. Lamb’s talk
– motivated and drawn towards him

127 XII – English


❑ Mr. Lamb
– an old man with a tin leg
– lives alone, lonely
– found ways to overcome his loneliness
– keeps his gate open
– never curtains his windows
– imagines kids and people walking all over
– spends time listening to the bees
– has a positive approach to life
❑ Derry’s Transformation
– had bitter experience in the world
– tells Mr. Lamb about sarcastic comments about his looks
– Even his mother kisses on the other side of face
– not trust people and ignore their comments about his face
– sad that he will have only half a face
– Mr. Lamb’s meeting changes his outlook towards life
– talks to Mr. Lamb about his likes, dislikes and fears
– wants to be loved and appreciated
– wants to break free from the stigma of being disabled
– his going back to Mr. Lamb’s garden shows his changing personality.

Short Answer Type Questions


Answer the following in 30-40 words each. (2 Marks each)
1. How does Derry enter the garden? What had he thought about it?
2. What had happened to Derry’s face? How do people react when they look
at it?
3. What does Mr. Lamb say about weeds and flowers?
4. What did Mr. Lamb tell Derry about his handicap?
5. Why did children call Mr. Lamb ‘Lamey Lamb’?

128 XII – English


6. How does Mr. Lamb convince Derry of the uselessness of being a recluse
(Lonely Person)?
7. How does Mr. Lamb spend his time?
8. What did Derry overhear his parents talking about his future? How did this
affect him?
9. Why does Mr. Lamb advise Derry to stop having feelings of hatred towards
the people?
10. What draws Derry to Mr. Lamb?
11. What did Derry hear the woman at the bus stop say?
12. How is Mr. Lamb’s concept of a ‘friend’ different from that of Derry’s?
13. What, according to Derry, would happen if Mr. Lamb continued his friendship
with him?
14. Why is not Mr. Lamb troubled by the people who stare at him?
15. Why were there no curtains in Mr. Lamb’s house?
16. Why did Derry want to go back to Mr. Lamb’s garden?
17. Why does Derry’s mother object to his going to Mr. Lamb’s house?
18. Give reasons to show that Derry is a changed person at the end of the
play.
19. ‘The world’s got a whole face.....”. What does Mr. Lamb mean to suggest
to derry with this remark.

Long Answer Type Questions


Answer the following in 125-150 words. (7 Marks each)
1. The play ends on a tragic note but reaffirms hope. Discuss with reference
to the text.
2. Despite his show of Bravado, Mr. Lamb is a lonely man. Do you agree?
Give a reasoned answer.
3. How is Mr. Lamb successful in infusing Derry with a zest for living?
4. Derry suffers from a sense of insecurity and alienation. Comment.

129 XII – English


5. “Derry is true successor of Mr. Lamb” Discuss.

7. EVANS TRIES AN O-LELVEL

–By Colin Dexter


❑ Evans tries an O-Level
– James Roderick Evans – a prisoner
– earlier escaped thrice from the prison
– therefore titled ‘Evans the Break’
– started night classes in O-Level German
– Govt. arranged to examine Evans for O-Level in his prison cell
– On the exam day he was given time to smarten up
– Unshaven and having a filthy looking hat
– refused to remove the cap that’s lucky
❑ Examination Arrangements
– removal of razors and nail scissors
– set tables opposite each other and placed two hard chairs in the cell
– deputed Stephens on D-Wing and newly recruited to visit Evans’ prison
– invigilator Mc Leery left his house at 8:45 am as the exam was to begin
at 9:15 am.
– Mc Lerry came, having a brown suitcase, semi-inflated rubber ring,
needed for haemorrhoids?
❑ The Exam Begins
– asked Evans to write index No., centre No, 313 and 271
– at 9:40 a.m. Mc Leery asked for correction slip
– Evans hardly understood anything, kept his pen between lips and
staring towards the door
– at 10:15 Evans requested for a blanket
– The exam got over at 11:20
– Hearing the Governor’s order at 11:22 a.m. accompanied McLeery to

130 XII – English


the prison gate
– Stephens obeyed the orders
– On the way asked Mcleery about Evans
– McLeery’s answer made him feel, his broader scots accent, to be
slimmer due to long black overcoat.
❑ Evans Escapes
– After seeing McLeery off, wanted to go for coffee, must take one last
look at Evans
– he saw McLeery sprawled in Evan’s chair slipping the blanket and
blood dripping through the beard.
– instead of calling ambulance McLeery low moaned and asked for police
to call
– he knew where Evans was
– opened the German Q. Paper and found a photocopied sheet carefully
and cleverly super imposed over the last page of Q. Paper
– instructions and the plan written in German as Make your way to
Neugraben
– Governor was furious, not made any call to see off McLeery at the
prison gate
– Governor wanted to know about the false beared, spectacles and
other things
– Later the Governor was puzzled Mcleery was not in the hospital
– The ambulance sent to Elsfield to pick, Mc Leery but just vanished
– after a quarter and half an hour they found McLeery bound and gagged
– Now they understood it was not Evans impersonating as McLeery who
had walked out but it was Evans, impersonating Mc Leery who stayed
in
❑ Final Escape
– reached his hotel at Golden Lion
– very happy for his successful plan
– able to hide his closely cropped hair due to lucky hat

131 XII – English


– as he reached the reception, found the receptionist not the same girl
– collected the keys, asked for early morning call at 6.45 am.
– as entered the room, shocked to see the Governor sitting on the
narrow bed
– finally spoke that was the correction slip which he left behind, given the
clue about
– Index No. 313, Centre No. 271 the six figures 313271 got him there
– Governor wanted to know where the blood came from, answered that
was pig’s blood in ring
– told about his German friend, helped him
– winked at receptionist and get back
– hand cuffed and clambered into the back seat of prison van, As he
turned right, unlocked hand cuffs and asked to drive fast
– on the driver’s query, suggested Newbury
– prison officer was Evans’ own man and the Governor was fooled once
again.

Short Answer Type Questions


Answer the following questions in 30-40 words each. (2 Marks each)
1. What was unusual about the request received by the examination board?
2. How is Evans not a typical criminal?
3. What is the nickname that Evans has got and why?
4. The prison authorities were not taking any chances with Evans’. What
precautions did they take?
5. How did Evans convince Jackson to allow him to wear his hat?
6. What reason did McLeery give for bringing the rubber ring with him?
7. What did the governor instruct Stephens to do after the exam got over?
8. What did Stephens discover in Evans’ cell after McLeery had left?
9. Who had impersonated whom in the plan to escape?
10. Mention two changes that Stephens notices in McLeery when he was leaving
after the exam?

132 XII – English


11. How did Evans exploit the soft corner Jackson had for him?
12. How did Evans manage to procure the things that had helped him in his
escape?
13. What was Evans actually doing when the pen was stuck in his mouth?
14. How did Evans manage to get blood? How was its clotting prevented?
15. How did the governor manage to track down Evans to the hotel where he
was hiding?
16. How is Evans able to manage his escape in the absence of visitors and
letters?
17. Why did Evans leave the question paper with German written on it in the
cell?
18. How did Jackson check McLeery’s suitcase? What two things did he find
objectionable in it?
19. How did the correction slip help in Evan’s escape?
20. What fate did the real McLeery meet?
21. How did Evans manage to give a slip to the governor at the end?
22. What reason did Evans give for asking for a blanket? What was the real
reason?

Long Answer Type Questions


Answer the following in about 125-150 words. (7 Marks each)
1. What steps were taken by the prison authorities to prevent any escape
attempt by Evans?
2. How was Evans able to turn the table on his captors?
3. “The Governor was just another, good for a giggle, gullible governor.” Do
you agree? Give a reasoned answer.
4. What is more responsible for Evans escape. His smartness or the blunders
of the prison staff? Give reason in support of your answer.

133 XII – English


8. MEMORIES OF CHILDHOOD

–Zitkala S.A. and Bama


❑ Zitkala SA
– bitter cold first day in the land of apples
– discriminated against native American
– her appearance, moccasined feet and blanket
– Jude told her about the cutting of hair
– tried to hide, dragged, tied to a chair and long hair was cut off
– suffer tremendous trauma and losing her spirit
– unskilled captured warriors, mourners and cowards had short hair in
her culture
– the cruelty makes her feel like a beast
– craves for comfort but nobody offered
2. We Too Are Human Beings
❑ Bama
– innocent childhood, used to walk back home from school, covered 10
min. of walk to ½ or 1 hour
– watched all the fun games such as street play, puppet show, a snake
charmer, performing monkey
– experienced untouchability very early in life
– one day saw an elder by person carrying food packet with the help of
strings
– her brother told the low caste of that man and that they belonged to
a low caste
– got inspiration to fight back through hard work and studies
– He told to work hard to win honour and dignity
– studied hard and stood first in her class.
❑ Common Theme
– two accounts took place in two cultures different places and in different
times but they explore a common universal theme

134 XII – English


– auto biographical accounts deal with women of marginalized communities
– Both narrate prejudices and humiliation faced from mainstream culture
– Both are brave who use their talent, education to stand up for this own
and community rights
– Both use the power of pen to fight oppression

Short Answer Type Questions


Answer the following questions in about 30-40 words each (2 Markseach)
1. Why was Zitkala-sa not feeling at home in her new surroundings?
2. Why was Zitkala-sa being keenly watched in the dining hall?
3. What embarrassing situation arose when Zitkala-sa sat on hearing the first
bell?
4. What information did Judewin give Zitkala-sa?
5. Why was Zitkala-sa resisting the cutting of her hair?
6. When and how did Zitkala-sa lose her spirit?
7. What indignities did Zitkala-sa suffer after she was separated from her
mother?
8. How did Zitkala-sa resist on being discovered under bed?
9. Why did Bama usually take long to reach home from school?
10. Why did Bama feel like shrieking with laughter at the sight of the elderly
man?
11. What was the elderly man carrying and for whom?
12. Why did Annan not feel amused when Bama told him about the elderly
man?
13. How did Bama react when she came to know why the elderly man was
carrying the packet.
14. Why did the landlord’s man ask Annan where he lived?
15. What reason did Annan give for not getting honour and dignity in society?
16. What impact did Annan’s advice have on Bama and with what effect?

135 XII – English


Long Answer Type Questions
Answer the following in 125-130 words (7 Marks each)
1. What is common in the discrimination and injustice experienced by Zitkalasa
and Bama? How do they react to their situations?
2. The cutting of my hair shows insensitivity of the mainstream culture towards
marginalized communities. Discuss.
3. How did Zitkala-sa resist the cutting of her hair and why?
4. What was Bama’s initial reaction to the incident on the street? How did her
reaction change later on?
5. Zitkala-sa and Bama are rebels” Discuss
6. Describe the various sights and scenes Bama would see on her way to
school in your own words.

136 XII – English


ENGLISH CORE
Code No. 301

CLASS XII

SAMPLE QUESTION PAPER-I

Time : 3 hrs. M.M. 100


General Instructions
1. This paper is divided into three sections : A, B and C. All the sections are
compulsory,
2. Separate instructions are given with each section and question, wherever
necessary. Read these instructions very carefully and follow them faithfully.
3. Do not exceed the prescribed word limit while answering the questions.

Section A : Reading
1. Read the passage given below and answer the questions that follow:
1. Many recent international studies suggest that single-sex may be better
than co-ed after all. But the reasons are far removed from the
sociocultural factors that have bothered Indian parents all these years.
Instead, research shows that the brains of boys and girls are wired
differently, and so have different cognitive and emotional responses to
different situations. Hence, a different learning environment makes
sense.
2. The research has been piling up over the years. It showed how students
from single-sex set-ups were more likely to proceed to college; how
girls who attend single-six schools are more likely to participate in
competitive sports than are girls at co-ed schools; and how co-ed
classrooms may be emotionally detrimental to teenaged girls. A study
of 13.000 people conducted by University of London found that those
who went to singlesex schools were more likely to study subjects not
traditionally associated with their gender. Girls from single-sex schools

137 XII – English


also went on to earn more than those from co-educational schools.
3. And thus the gender differences, which have always lurked somewhere
in the minds of most teachers anyway, have now become a subject of
scientific researches, educational policies and bestsellers. At least two
books–Michael Gurian’s ‘Boys and Girls Learn Differently!’ and Leonard
Sax’s ‘Why Gender Matters : What Parents and Teachers Need to
know about the Emerging Science of Sex Differences’–have successfully
contributed to a change of mindset and educational policy in the US.
In 2006, the Department of Education had, in fact, passed new
regulations that made it easier for districts to create single-sex
classrooms and schools.
4. In the Indian context, of course, the situation only gets more complex.
There are, after all, a lot more factors at play here, including a
widespread social preference for male children, a lesser emphasis on
women’s careers and adults are wary about the two sexes mixing. But
like elsewhere in the world, here too opinions are diverse about whether
co-ed is better or not. “Obviously, we have felt that single-sex education
has great strengths.”
5. “In India, many parents are not comfortable with the intermingling of
boys and girls. So if you want all girls to be educated, it’s better to
have some separate schools for them.” One cannot be categorical
about this issue. “There are schools that are coeducational that have
done extremely well. Having said that, there is a place in every
educational system for singlesex schools because there will always be
at least some students who do better in that environment, and they
should have the choice of going to such schools.”
6. Availability of this choice is what educationists and parents all over the
world emphasise. Even Sax, a family physician-turned-author who has
emerged as a champion of single-sex education in the US, has been
reported to have stressed that one size does not fit all. “Our movement
is about choice,” Sax, the head of the National Association for Single
Sex Public Education, told Time recently. And yet his website is full of
pages that cite how co-educational settings actually reinforce gender
stereotypes. Such a premise might actually be true in a country like
India, where society at large still perceives males as primary bread-
earners and women as bread-makers, where men are supposed to
lead and women expected to follow.
7. “One does see that girls get relegated to a secondary role in a co-ed

138 XII – English


setup.” It is not a straightforward issue about girls and boys merely
being with each other. “It’s more important to see how a school handles
the building up of gender roles in society.” In Indian government schools,
It’s not uncommon to come across boys cleaning the blackboard while
girls sweep the floor or bring water.” The focus should be on respectful
intermingling, rather than forceful coexistence. “Bajpai too admits that
teachers in India are not sensitized towards gender difference as part
of their training in education. “Given the new research, it is important
to take-stock of the findings and adjust our teaching accordingly,” he
says.
8. Educationists also say that they understand that each child learns
differently, and the differences don’t just have to do with the gender.
In such a scenario, experienced and sensitive teachers would
automatically find the best way to interact with and bring out the best
in, each student. Utopian expectations, some might say. For most Indian
parents though, expectations only extend as far as getting admission
in a good school, one that is not 15 km away from home.
A. (a) Mention the role played by the books by Michael Gurian and Leonard
Sax. 2
(b) Why is a single sex education schools preferable to co-educational
schools in India? 2
(c) What are the major roles assigned to men and women in general? 2
(d) What are the demerits of studying in a single sex education system? 2
(e) How can the teachers play a positive role in bringing out the best
qualities in both kinds of school? 1
B. Find out the words from the passage which mean the same as the following:
(a) harmful (Para - 2)
(b) cautious (Para - 4)
(c) surviving together (Para - 7) 1 × 3 = 3
2. Read the passage given below and answer the questions that follow:
Brian G Dyson, former President and CEO, Coca-Cola Enterprises, once
said, “Imagine life as a game in which you are juggling five balls in the air
namely– work, family, health, friends and spirit. You will soon understand that
work is a rubber ball. If you drops it, it will bounce back. But the other four balls–
family, health, friends and spirit–are made of glass. If you drop one of these,

139 XII – English


they will be irrevocably scuffed, marked, nicked, damaged or even shattered.
They will never be the same. You must understand that and strive for balance
in your life”. Industries have realised the importance of his words.
Organisations are setting up policies for maintaining a work life balance.
They are going in for innovative methods to keep their employees happy and
satisfied, as it makes office a better place to work and also positively impact
productivity.
The Concept of Work-life balance (WLB) is not a new one, but with the
changing pace of but with changing pace of life and increase in stress levels,
negatively affecting the quality of work, has made many organisations think
about the strategies for maintaining a work-life balance.
Experts say that there is a strong link between work-life balance policies
and reduced absenteeism, increased productivity and job satisfaction. Other
benefits include improved recruitment and retention rates with associated cost
savings, a reduction in employee stress, greater levels of job satisfaction. And
loyalty and an improved corporate image.
The work-life balance strategy offers a variety of ways to reduce stress
levels and increase job satisfaction of the employees. The fundamental principle
of designing activities around the topic of work-life. Companies are ready to do
everything possible to keep their employees happy and are pampering them like
never before with new policies like flexitimings, day care centers for kids, extended
maternity leave, health care centers, medical insurance, fun and games at work,
team outing, spas and gyms at office etc.
Employees tend to feel motivated when they feel that the organisation is
putting extra effort in providing a healthy, balance between work and life. Motivated
employees not only enhance the productivity but also help creating a positive
work environment at office.
Work life balance is a choice that an individual has to make. However, it is
the organisation that needs to take an initiative to help the employees. Industry
experts add that today, an employee is not looking at their employer just for
jobthey want the company to care for their work-life balance and their well being.
If a company can address these needs, in addition to providing great career
opportunities, they can be very successful in providing job satisfaction to the
employee.
Companies are adopting new means to ensure that their employees get
enough time to enjoy their personal life and Spend time with family. Certain
companies also support the policy of adopting a child by their employees. For

140 XII – English


this they have adoption leave policy, which allows the employees to avail eight
weeks of paid leaves to spend some quality time with their newly adopted
children.
Another recent trend is the sabbatical. Certain companies encourage
employees on completion of certain period at work to take a sabbatical for a
year to rethink and re-plan his/her career or just to take a break, relax and
rejuvenate.
Firms are going for innovative practices to keep their employees happy,
e4e a business solution providing company, promotes ROWE (Results Only
Work Environment) culture within the organisation, negating the age-old dogma
that equates physical presence with productivity and encouraging people to
focus on working smarter rather than longer. Another attempt by the company
to manage work-life balance is to encourage all employees to take a minimum
of seven days annual leave in a year. It is the responsibility of the reporting
managers or supervisors to ensure that the employees reporting to them take
this leave.
Some organisations have initiatives to provide timely assistance to the
employees in a crisis situation rising out of financial or personal problems. HEAL
(Honeywell Employee Assistance for Life issues) is the counselling service of
honeywell Technology Labs. HEAL has tied up with PPC World wide and provides
round the clock advice and counselling on all issues ranging from financial, legal
or emotional to all our employees. This service is totally confidential where the
consultant respects the privacy of the employee and treats all details and issues
as highly confidential.
At Accenture, they believe that telecommuting is one of the key tools for
attracting and retaining employees especially Women employees. Our employees
have the flexibility to exercise a need based work from home option as if helps
them more in maintaining balance between work and home. The benefits of
telecommuting include increased productivity, lower attrition, less number of
leaves taken, cost savings on infrastructure and of course an extremely happy
workforce.
Firms are not leaving it at this point. They are also evaluating the results
of these initiatives to ensure that all these initiatives have the desired impact.
Company heads talk informally with the staff to understand how they are balancing
their personal and professional lives.
(a) On the basis of reading the above passage make notes on the passage
using headings and sub headings. Use recognizable abbreviations
(minimum 4) Wherever necessary. Give suitable title to the passage. 5

141 XII – English


(b) Write a summary of the passage in about 80 words. 3

Sectrion B : Advanced Writing Skills


3. You are Rekha/Rehan, the secretary of the cultural club of Nalanda Public
School, Nagpur. Your school is organising an inter-house English Essay
Writing Competition on the topic “Use of Mobile Phones - A Boon or a Bane”
on the occasion of YUVA Cultural Week. Draft a notice for school Notice-
Board informing the students about the competition in 50 words.
OR
You are Gagan/Gitika of A-69, Yojna Vihar, Delhi. You have been invited to
attend the birthday party of your friend’s brother on 23rd Aug., 2011. Draft
in 50 words, an informal reply to the invitation regretting your inability to
attend it. 5
4. You are Jaya/Jayant, a resident of Sarita Vihar locality. The Youth Club of
your area organised a programme for spreading awareness among the
residents regarding peace and communal harmony. Write a report on the
programme for publication in a local magazine. 10
OR
You are Akash/Akshita, a reporter in India Today. You had been enjoying
holidays in Mumbai in the month of August, when one day you saw two
cargo ships colliding which resulted in an oil spill. Write a report on ship
accident and oil spill in about 125-150 words for publication in your paper.
5. You are Vishakha/Vishwanath, a resident of A-101, Khayala, Delhi. You feel
disturbed to read the cases of road rage daily. Write a letter to the editor
of a national daily on this issue.
OR
You are Rukmani/Rajat from C-101, Patparganj, Delhi. You purchased a
LCD, TV from Galaxy Electronics, Laxmi Nagar. After three weeks the working
of TV became faulty. Write a letter to the dealer complaining about the
problem and asking him to exchange it for new one as per terms and
conditions of the sale. 10
6. You are Geetika/Gandharva, you feel that inspite of starting the Metro train
service, the condition of traffic on road is still worse. You find it disturbing
that the number of vehicles on roads are increasing steadily. You want
people to contribute for tackling this problem. Write an article in about 150-

142 XII – English


200 words on the topic – “Traffic Woes – Solution”.
OR
You are Babita/Bahadur, a student of Class XII of Sarvodaya School, Jamia
Nagar. You feel that there is need of Career Counselling throughout the
year not just for two days. You are asked to deliver a speech on “Need of
Continuous Career Counselling in School”. Write a speech on this topic.
10
7. (a) Read the extract given below and answer the questions that follow:4
The stunted, unlucky heir
of twisted bones, reciting a father’s gnarled disease,
His lesson, from his desk. At back of the dim class
One unnoted, sweet and young. His eyes live in a dream,
Of squirrel’s game, in tree room, other than this.
Questions
(i) Who is the ‘unlucky heir’? What will he inherit? 1
(ii) Why the boy is called ‘the stunted’? 1
(iii) Who is sitting at the ‘back of the dim class’? 1
(iv) “His eyes live in a dream”–What dream does he have? 1
OR
Aunt Jennifer’s tigers prance across a screen.
Bright topaz denizens of a world of green
They do not fear the men beneath the tree
They pace in sleek chivalric certainty.
(i) Where do the tigers appear? What are they doing? 1
(ii) Explain ‘Bright topaz denizens of a world of green’? 1
(iii) Name the two qualities of the tiger as shown in the extract. 1
(iv) Why the tigers are not afraid of the men? 1
7. (b) Answer the following questions in 30-40 words each. 6
(i) What is the poet’s familiar ache and why does it return? (My Mother
at Sixty Six)

143 XII – English


(ii) What are the things that cause suffering and pain to human beings?
(A Thing of Beauty)
(iii) What does the poet call “An exotic Moment” and Why? (Keeping Quiet)
8. Answer the following questions in 30-40 words each 10
(i) What is Saheb looking for in the garbage dumps? Where is he and
where has he come from?
(ii) Why was the crofter so talkative and friendly with the peddler?
(iii) What does the writer mean by ‘the fiery misery’ of those subjected to
make up?
(iv) What are some of the positive views on interviews?
(v) What were the options that Sophie was dreaming of? Why does Jansie
discourage her from having such dreams?
Answer the following in 125-150 words.
9. Write a character sketch of Subbu. (Poets and Pancakes) 10
OR
Why do you think Gandhi considered the Champaran episode to be a
turning point in his life?
10. While hatred against a member of the enemy race is justifiable, especially
during wartime, what makes a human being rise above narrow prejudices.
Answer with reference to ‘The Enemy’.
OR
Why is Antarctica the place to go to understand the earth’s present, past
and future. 7
Answer the following question in 30-40 words each. (8 Marks)
(a) What do you infer from Sam’s letter to Charley?
(b) What did the Maharaja do to find the required number of tigers to kill?
(c) How is Evans not a typical criminal?
(d) How was Bama influenced by the words of her brother, Annan?

144 XII – English


SOLUTIONS

Section B : Advanced Writing Skills


3. NOTICE
Format : include the word NOTICE, Heading, Date and Writer’s name with
designation. 1
Content : Suggested value points
– day, date, time, venue
– topic
– last date for submission of names
– contact person for queries (2)
Expression : Grammatical accuracy Coherence, etc. (2)
OR
Informal Reply of Denial/Refusal
Format : Address of the writer with date salutation (1)
Content : thanks for the invitation
– Reason for refusal
– Best wishes for the occasion
– Complimentary close (2)
Expression : Grammatical accuracy, relevance and coherence. (2)

Question Paper
1. Report Writing
– Format 1
1 Title, reporter’s name, date and place
– Content 4
– Expression 5
* Grammatical accuracy appropriate words and spelling 2½
* Coherence and relevant of ideas and style 2½

145 XII – English


Suggested Value Points
– “Programme for Peace and Communal Harmony”
– When (Day, date, timings)
– Where
– activities undertaken (Exhibition, rally, etc.)
– People’s Views
– Special features
OR
– “Collision of Cargo ships causes oil spill”
– When (Day, date, Timings)
– Where
– How
– Casualties
– Damage to ships
– Oil spill harm done to marine life
– Action taken
5. Letter Writing
– Format 2
– Content 4
– Expression 4
❑ Grammatical accuracy appropriate words and spellings 2
❑ Coherence and relevance of ideas and style 2
Suggested Value Points
– Increasing cases of road rage
– Youngsters with vehicles on road
– Lack of fellow feeling
– attacks and murders
– damage to the vehicles
– traffic jams and accidents

146 XII – English


– counselling and seminars
– Role of traffic police
– Role of media and society
OR
– Regarding defective TV Set
– Model No., Bill No., Date of Purchase
– Defects in TV set
– Inability of Technician to repair it
– Request for changing it.
Article/Speech Writing
– Format (Title & Writer’s name) 1
– Content 4
– Expression 5
Suggested Value Points
– Types of vehicles on road
– Problem of traffic jam
– Problem of road rage
– Use of Public transport
– Car pooling
– 5 days week
– Advantages of reducing traffic
OR
– Greetings
– Common Problem faced due to lack of information
– Steps to gain career related information
– sources of information
– benefits of career counselling throughout the year.

147 XII – English


Section C : Text Books
7(a)
(i) The thin boy, having rat’s eyes, twisted bones from his father, old
chronic diseases.
(ii) underdeveloped, old chronic diseases
(iii) A sweet, young boy, unnoticed.
(iv) seems hopeful, dreams of better times of outdoor games of squirrel’s
game.
OR
(i) across a screen, prancing
(ii) shining, dwellers of a forest
(iii) majestic, courageous
(iv) they are created in the panel.
7(b)
(i) pain of ageing, separation and death she has entertained this fear
since childhood.
(ii) malice, disappointment, unhealthy, evil ways
(iii) unusual, enticing, when there is complete silence
8. (i) coins, notes, valuable things, Seemapuri, Dhaka
(ii) due to circumstances, no wife or child generous has confidence.
(iii) pain and trouble caused by heat produced by lights and mirrors.
(iv) common place of journalism, most serviceable medium of
communication.
(v) having a boutique, to be manager, an actress fashion designer /
earmarked for biscuit factory, economic conditions of Sophie not good,
Jansie planted to ground.
9. Kothamanglam Subbu – No. 2 at Gemini Studio
– Officially on the rolls of story department, always with boss
– cheerful, loyal, identifies himself with Principal
– turned entire creativity to advantage
– tailor-made for films

148 XII – English


– poet, novelist, actor, genuine love for all
– charitable, hospitable
OR
– Champaran incidents,
– brought economic and political solution to peasants problem.
– made Britishers surrender-part of their money/prestige
– infused self-reliance among Indians
– Britishers could not order him in his own country
– gave Gandhi self confidence
– launched freedom movement throughout the country.
10.
– Human values, sensitivity, companion important
– human beings rise above narrow prejudices
– white man-wounded, unconscious, needed medical help
– Dr. Sadao and Hana rise above difference in nations
– perform duty, have right values
– patriotic feeling important
– killing another, because he is an enemy maker
– human beings ruthless
OR
Antarctica – driest, coldest, windiest continent
– No trees, less bio diversity,
– Visual range – microscopic to mighty
– perfect place to study changes in environmental
– holds in ice-cores, half million year old carbon records
– ‘Students on Ice’ programme
– future generation of policy makers
– ready to absorb, learn, apply.

149 XII – English


11. (a) letter comes in mysterious way
in oldest first day cover
mailed to grandfather
saying Sam has found the third level – the imaginary world
World of fantasy, very interesting
Charley and Louisa should also try for third level

(b) married a girl of royal family


that state having a large tiger population

(c) not violent, pleasant person


One of the stars at Christmas concert

(d) deep impression


studied hard, stood first
made friends.

150 XII – English


ENGLISH CORE
Code No. 301

CLASS XII

SAMPLE QUESTION PAPER-2

Time : 3 hrs. M.M. 100


General Instructions
(I) This paper is divided into three sections : A, B and C. All the sections are
compulsory,
(II) Separate instructions are given with each section and question, wherever
necessary. Read these instructions very carefully and follow them faithfully.

Section A : Reading
1. Read the passage given below and answer the questions that follow:
1. India is so lively and refreshing. From here, the West looks somewhat
old, gray and struggling. Indeed, India looks all set to regain the
centrestage it occupied a few centuries ago. But while it is developing
very fast and its elite is striving to match up with the West, is it not,
at the same time, putting at stake much of its precious environment,
culture, traditions and values? Of course, the great country has
abscribed influences of many previous invaders and colonizers without
losing its identity. But presently, its “westernization” seems to be taking
place at a frightening speed.
2. Foreign trade delegations are rushing into the country even during the
hottest months of the year. Starry-eyed businessmen, dressed in crisp
black suits are hopping from one five-star hotel to another, cracking
mega deals. Market scales and huge figures make them drool. They
all want their chunk of the great Indian curry. However, the West is not
always exporting its very best to India. Because it’s not just higher
standards of safety or environmental friendliness, or valuable
technologies and know-how that are flowing from the western world.

151 XII – English


3. Fast food, tobacco, alcohol and toxic skin-whitening creams are pouring
into India. This wave is promoting a western lifestyle based on
consumerism, individualism and meaninglessness. And the pace of
consumption is indeed picking up in India. Manufacturers of goods as
different as cars, clothes or cellphones have noticed that in recent
years the life cycle of products has shortened.
4. While the older generation of Indians would buy a new product only
when the previous one’s useful life had ended, the young generation
tends to fall for novelty. People want the latest generation of gadgets
because they have more money and more access to such goods. But
mostly, they are made to believe, by ever increasing and omnipresent
publicity, that they will feel happier with the new product. To encourage
this shopping frenzy, a quiet revolution is taking place in the form of
rapidly increasing credit penetration.
5. For historical and cultural reasons, well-to-do Indians traditionally
avoided indebting themselves. This may be one less talked about
reasons why the country’s economy avoided major trouble during the
recent crisis. Only 20 million Indians possess a credit card and personal
loans represent about 10% of GDP. Whilst in most western economies,
the latter account for approximately 100% of it. But all this is about to
change with a booming credit industry.
6. These trends may be healthy ingredients for thriving capitalism. But
they may not prove soothing for the soul. Even if they all possess their
own television, car, washing machine and fridge, Westerners are not
necessarily a happier lot. Surveys show that their happiness has
declined in correlation with the development of consumerism since
World War II. Moreover, most people in the so-called developed nations
suffer from obesity, loneliness depression and addiction to prescribed
drugs. May be once upon a time they danced, sang songs and told
stories, but now, free time often means watching TV or shopping.
7. To meet the growing demand, natural resources are now being exploited
in India, displacing millions of tribal people towards urban slums fuelling
growing pockets of civil war-like conflicts across the country. Efforts
may be made to fight the old caste system, but a social stratification
based on consumption power is emerging. Along with this, India’s
comparatively good criminality track record is bound to go on the rise
in the coming years.
8. From environmental perspective, the current scenario is down-right

152 XII – English


terrifying. Mountains are being blown up, forest are being shaved, soil
and sacred rivers are being polluted to cater to the endless production
of goods that are meant to be replaced as fast as possible. And whilst
the West exports its toxic junk to places like India, where it is “recycled”
in dangerous conditions by the informal sector, for peanuts, where will
the Indians’ rapidly growing waste be dumped?
9. Maybe 50 years ago, the effects of massive consumption were largely
ignored. But today, most people acknowledge many planets would be
required if everyone picked up American consumer habits.
10. Even the western toilets have become a status symbol in Indian
middleclass homes. Millions of fancy flush toilets are added every year
to new trendy restaurants, malls, condominiums and five-star hotels in
Indian cities. Instead of adopting unsustainable imports from the West,
there are areas wherein India should inspire the latter. With its traditional
squatting toilet for instance. Not just because it is cheaper, more
hygienic and better adapted to the human anatomy, but mostly, in a
context where clean water is increasingly rare and pricy, because it
requires much less of it. Countless other sustainable concepts, values
and products from India could benefit Westerners. Strong extended
family ties, respect for elders, kriya yoga and tongue-scrapers are just
a few of them.
11. India needs to avoid repeating the West’s mistakes. Only enlightened
citizens can show the way towards a more viable economy but putting
pressure on government, stressing India’s success should not be
measured by GDP growth rates and spending habits alone. Nor should
it aspire to become like the US or China.
12. Concerned and responsible Indian citizens can encourage sensible
consumption behaviour – by shopping less and wasting even lesser.
By buying local products to keep the cottage industries, bazaar culture,
traditional crafts and wisdom alive. In essence, by valuing their rich
heritage and living simple and meaningful lives, Indians can set an
example, and show the West that Mother India is much more than a
well of business opportunities. It’s not just a matter of common sense;
it’s about our survival.
A. (a) What cost have we to pay for westernization? 2
(b) How do the salesmen try to tempt buyers for buying cell phones? 2
(c) How do the old and young consumers differ in buying goods? 2

153 XII – English


(d) What are the threats faced by the developed countries due to
consumerism? 1
(e) How can Indians inspire western countries through their traditions and
life styles? 2
B. Find out the words from the passage which mean the same as follows :
(a) attackers (para - 1) 1
(b) one who is present everywhere (para - 4) 1
(c) frightening (para - 8) 1
2. Read the passage given below and answer the questions that follows:
India’s economy is doing well and we are right to celebrate that. But what
we do not like to acknowledge, let alone address, is another fact; our economy,
and society, is still extremely biased against women. Perhaps paying attention
to such inconvenient truths would distract us as we march towards superpower
status. In the latest gender gap index report released by the World Economic
Forum (WEF), India keeps company with the worst in the world. Among the 128
countries that have been evaluated by the WEF, India is ranked 114, followed
among others by Yemen, Chad, Pakistan and Saudi Arabia, Even China,
Philippines, Sri Lanka and Botswana fare much better than us.
The survey considers the proportion of resources and opportunities made
available to women on educational, economic, political and health. Parities it is
only in the realm of political empowerment that we seem to have done somewhat
OK, ranking 21st. That’s also thanks to a long spell of prime ministership by
Indira Gandhi. It begs the question why women are so badly off in our country
if they are politically empowered.
To begin with, we are still largely a feudal and patriarchal society. In many
parts of our country–especially in UP, Bihar, Rajasthan, Haryana and Punjab–
women are often treated as if they were a piece of property. In these parts, the
sex ratio is most skewed because families often snuff out the lives of girl children
before, or immediately after, they are born, in many parts of India women are
viewed as an economic liability despite contributing in several ways to our society
and economy.
The state has not covered itself with glory either in bridging the gender
divide. Its policies and projects for women are woefully inadequate. For instance,
the literacy rate for females is a mere 48 per cent against 73 per cent for males.
Unless we put more of our girl children in schools and equip them with quality
education–as opposed to making them merely literate–we can forget about

154 XII – English


sustainable progress. Public health is another area of failure. Hundreds of women
in rural India die every year during childbirth for want of medical attention. There
are thousands more who do not even have access to a primary health centre.
Importantly, reforming property laws more rigorously so that gender parity
becomes a reality must rank among the government’s priorities. While these
changes are necessary, they will amount to nothing if we, as a society, continue
to deny our women the dignity, liberty and opportunities that are rightfully theirs.
No society will ever prosper as a whole as long as half of it is constantly created
as somehow less than the other half.
(a) On the basis of reading the passage make notes on the passage
using headings and sub-headings using recognizable abbreviations
wherever necessary. Give a suitable title to the passage. 5
(b) Write a summary of the passage in about 80 words using the notes.3
3. You have lost your wrist watch in the school play ground while playing
cricket. Write a notice in about 50 words for the ‘Lost and Found’ Notice-
Board of the school giving details of the watch, contact address and reward.
You are Anuj/Anuja of Class XI, Vanasthali Public School, Delhi. 5
OR
You want to let out the first floor of your newly constructed house with all
modern facilities. Write an advertisement to be published under the classified
column of a local daily in 50 words.
4. You are Gaytri/Ganesh a resident of Madhuban Chowk, Delhi. The Residents
Welfare Association of your locality organised a medical camp for free
check-up of women. It was organised in collaboration with Safdarjang Hospital.
Women came from getting themselves checked up in large number. Write
a report in about 125 words for publication in hospital’s journal. 10
OR
You are Disha/Daksh a resident of C-5, Kalyanpuri. You had been travelling
in a low floor DTC bus when a speeding truck from opposite side collided
with the bus. You escaped with minor bruises but many passengers were
severely injured. Write a report on it in about 125 words.
5. You are Simran/Sandeep, head of school Examination Committee of JPL
school, Motibagh. You had placed an order for supply of some articles with
ASN stationers, Karolbagh. The articles were not supplied on time so you
had to arrange for them from other sources. Write a letter to the dealer for
cancellation of the order. 10

155 XII – English


OR
You are Ankur/Ankita a resident of C-7, Bhajanpura, Delhi. You read an
advertisement given by Sports Authority of India for the recruitment of refrees
for junior girls and junior boys hockey tournaments. Write a letter to the
Chairman of the Department applying for the job giving your complete bio-
data.
6. You are Sukhmani/Sukhpal from Laxmi Nagar, Delhi. You are appalled to
read the survey reports of UNICEF which have rated India below many
Africans countries in providing Health Care Facilities to its people. You feel
that neglecting health care is going to have a very bad impact on India’s
prospects in future. Write an article in about 200 words expressing your
views. 10
OR
You are Riddhima/Ritik from Nai Sarak, Delhi. You observe lots of
developmental works are being carried out in and around Delhi. You strongly
feel that there is urgent need to change in the mindset of people so that
they feel proud and responsible for such changes. Write an article in about
15-200 words expressing your views.

Section C : Text Book


7. (a) Read the extract given below and answer the question that
follow:
1 × 1 = 4
A thing of beauty is a joy forever
Its loveliness increases, it will never
pass into nothingness, but will keep
A bower quite for us, and a sleep
full of sweet dreams, and health and
quite breathing.
Questions
(a) Why is a beautiful thing a joy forever? 1
(b) Why does it never pass into nothingness? 1
(c) How does beauty help in our sound physical and mental health? 1

156 XII – English


(d) Explain : ‘A bower quite for us? 1
OR
I looked again at her wan, Pale
as a late winter’s moon and felt that
old familiar ache, my childhoods fear
but all I said was, see you soon
Amma
All I did was smile and smile and smile.
Questions
(a) Who do ‘I’ and ‘her’ refer to? 1
(b) What is poet’s ‘childhood fear’? 1
(c) Explain the comparison – “as a late winter’s moon”. 1
(d) How were the parting words and smile of poet in contrast to her real
feelings? 1
(b) Answer the following questions in 30-40 words each. 2 × 3 = 6
(i) According to Stephen Spender, who can change the life of slum children
and how?
(ii) Why does Aunt Jennifer find it difficult to pull the needle?
(iii) What does the writer want to clear about total inactivity and death?
(Keeping Quiet).
8. Answer the following questions in 30-40 words each : 2 × 5 = 10
(i) How does M. Hamel praise the French language.
(ii) Why did Gandhi agree to a settlement of 25% refund to the farmers?
(iii) How was hierarchy maintained in the make up department?
(iv) How is Umberto Eco’s style of writing different from that of the
conventional style?
(v) How are Sophie and Geoff a contrast to each other despite being
siblings?

157 XII – English


9. Answer the following question in 125-150 words. 10
How did the instructor help the author to become a swimmer and overcome
his fear of water?
OR
The beauty of the bangles of Firozabad is in direct contrast to life of the
people who make them. Elaborate.
10. How does Mr. Lamb infuse Derry with a zest for living? 7
OR
Bring out the element of humour and irony in the story ‘The Tiger King.’
11. Answer the following questions in 30-40 words each : 8
(i) What is a first-day cover? (The Third Level).
(ii) Why is the involvement of students crucial in ‘Students on Ice’
Programme?
(iii) How did the Wizard solve as well as create problems for Roger?
(iv) Why was Zitkala sa against the cutting of her hair?

158 XII – English


BOARD PAPERS 2012 (DSSCE-2012

ENGLISH CORE
Code No. 1/1/2

CLASS XII
Time : 3 hrs. M.M. 100
General Instructions
1. This paper is divided into three sections : A, B and C. All the sections are
compulsory,
2. Separate instructions are given with each section and question, wherever
necessary. Read these instructions very carefully and follow them faithfully.
3. Do not exceed the prescribed word limit while answering the questions.

Section A : Reading
1. Read the passage given below and answer the questions that follow:
1. While there is no denying that the world loves a winner, it is important
that you recognize the signs of stress in your behaviour and be healthy
enough to enjoy your success. Stress can strike anytime, in a fashion
that may leave you unaware of its presence in your life. While a certain
amount of pressure is necessary for performance, it is important to be
able to recognise your individual limit. For instance, there are some
individuals who accept competition in a healthy fashion. There are
others who collapse into weeping wrecks before an exam or on
comparing marks-sheets and finding that their friend has scored better.
2. Stress is a body reaction to any demands or changes in its internal
and external environment. Whenever there is a change in the external
environment such as temperature, pollutants, humidity and working
conditions, it leads to stress. In these days of competition when a
person makes up his mind to surpass what has been achieved by
olhers, leading to an imbalance between demands and resources, it
causes psycho-social stress. It is a part and parcel of everyday life.

159 XII – English


3. Stress has a different meaning, depending on the stage of life you are
in. The loss of a toy or a reprim and from the parents might create a
stress :uock in a child. An adolescent who fails an examination may
feel as if everything has bepn lost and life has no further meaning. In
an adult the loss of his or her companion, job or professional failure
may appear as if there is nothing more to be achieved.
4. Such signs appear in the attitude and behaviour of the individual, as
muscle tension in various parts of the body, palpitation and high blood
pressure, indigestion and hyper-acidity. Ultimately the result is self-
destructive behaviour such as eating and drinking too much, smoking
excessively, relying on tranquilisers. There are other signs of stress
such as trembling, shaking, nervous blinking, dryness of throat and
mouth and difficulty in swallowing.
5. The professional under stress behaves as if he is a perfectionist. It
leads to depression, lethargy and weakness. Periodic mood shifts also
indicate the stress status of the students, executives and professionals.
6. In a study sponsored by World Health Organization and carried out by
Harvard School of Public Health, the global burden of diseases and
injury indicated that stress diseases and accidents are going to be the
major killers in 2020.
7. The heart disease and depression - both stress diseases - are going
to rank first and second in 2020. Road traffic accidents are going to
be the third largest killers. These accidents are also an indicator of
psycho-social stress in a fast moving society. Other stress diseases
like ulcers, hypertensions and sleeplessness have assumed epidemic
proportions in modern societies.
8. A person under stress reacts in different ways and the common ones
are flight, fight and flee depending upon the nature of the stress and
capabilities of the person. The three responses can be elegantly
chosen to cope with the stress so that stress does not damage the
system and become distress.
9. When a stress crosses; the limit, peculiar to an individual, it lowers his
performance capacity. Frequent crossings of the limit may result in
chronic fatigue in which a person feels lethargic, disinterested and is
not easily motivated to achieve anything. This may make the person
mentally undecided, ,-confused and accident prone as well. Sudden
exposure to un-nerving stress may also result in a loss of memory.
Diet, massage, food supplements, herbal medicines, hobbies, relaxation

160 XII – English


techniques and dance movements are excellent stress busters.
(a) (i) What is stress ? What factors lead to stress ? 2
(ii) What are the signs by which a person can know that he is
under stress ? 2
(iii) What are the different diseases a person gets due to
stress? 2
(iv) Give'any two examples of stress busters. 1
(v) How does a person react under stress ? 2
(b) Which words in the above passage mean the same as the
following ? 3
(i) fall down (para 1)
(ii) rebuke (para 3)
(iii) inactive (para 9)
2. Read the passage given below and answer the questions that follow:
8 marks
Research has shown that the human mind can process words at the rate
of about 500 per minute, whereas a speaker speaks at the rate of about
150 words a minute. The difference between the two at 350 is quite large.
So a speaker must make every effort to retain the attention of the audience
and the listener should also be careful not to let his mind wander. Good
communication calls for good listening skills. A good speaker must necessarily
be a good listener.
Listening starts with hearing but goes beyond. Hearing, in other words is
necessary but is not a sufficient condition for listening. Listening involves
hearing with attention. Listening is a process that calls for concentration.
While, listening, one should also be observant. In other words, listening has
to do with the ears, as well as with the eyes and the mind. Listening is to
be understood as the total process that involves hearing with attention,
being observant and making interpretations. Good communication is
essentially an interactive process. It calls for participation and involvement.
It is quite often a dialogue rather than a monologue. It is necessary to be
interested and also show or make it abundantly clear that one is interested
in knowing what the other person has to say.

161 XII – English


Good listening is an art that can be cultivated. It relates to skills that can
be developed. A good listener knows the art of getting much more than
what the speaker is trying to convey, lie knows how to prompt, persuade but
not to cut off or interrupt what the other person has to say. At times the
speaker may or may not be coherent, articulate and well organised in his
thoughts and expressions. He may have it in his mind and yet he may fail
to marshal the right words while communicating his thought. Nevertheless
a good listener puts him at ease, helps him articulate and facilitates him to
get across the message that he wants to convey. For listening to be effective,
it is also necessary that barriers to listening are removed. Such barriers
can be both physical and psychological. Physical barriers generally relate
to hindrances to proper hearing whereas psychological barriers are more
fundamental and relate; to the interpretation and evaluation of the speaker
and the message.
(a) On the basis of your reading of the above passage, make notes in
points only, using abbreviations wherever necessary. Supply a suitable
title. 5
(b) Write a summary of the above passage in about 80 words. 3

SECTION - B (Advanced Writing Skills)


35 Marks
3. Your school has planned an excursion to Lonavala near Mumbai during the
autumn holidays. Write a notice in not more than 50 words for your school
notice board, giving detailed information and inviting the names of those
who are desirous to join. Sign as Naresh/Namita, Head Boy/Head Girl, D.V.
English School, Thane, Mumbai. 5 marks
OR
CP.R. Senior Secondary School, Meerut is looking for a receptionist for the
school. Draft an advertisement in not more than 50 words to be published
in classified columns of Hindustan Times. You are Romola Vij, Principal of
the school.
4. Your school conducted a seminar on 'How to prevent cruelty towards animals',
in which 40 city CBSE sehools took part. As Co-ordinator of the programme,
write a report in 100-125 words for the school magazine. You are Vikram/
Vidhi of C.P.S. Senior Secondary School, Bangalore. 10 marks
OR

162 XII – English


You witnessed a road accident near Nalbandh Chowraha at Agra in which
a bus and a scooter were involved. Write a report for Amar Ujala in 100-
125 words. Sign as Vivek/Virnala, Special Correspondent.
OR
Your school celebrated international year of Chemistry 36 schools from
Delhi participated in various competitions. Chairman, CBSE was the chief
guest. As co-ordinator, draft a report in 100-125 words for your school
magazine. You are Ram/Ramya of GB Senior Secondary School, Delhi
(Set 1/3)
5. You are Pritam/Priti, 27, W.E.A. Karol Bagh, Delhi. You have decided to
shift your residence to Faridabad and hence decided to discontinue your
membership of Brain Trust Library, Karol Bagh. Write a letter to the Librarian,
requesting him to cancel your membership and refund your security
deposit of Rs. five thousand explaining your inability to continue your
membership. 10 marks
OR
You are Anu/Arun, 13 W.E.A. Karol Bagh, New Delhi. You feel very strongly
about the ill-treatment meted out to stray dogs at the hands of callous and
indifferent people. Write a letter to the editor of a national daily giving your
views on why some people behave in such a manner and how these dogs
should be treated.
6. Spurt of violence previously unknown in Indian schools makes it incumbent
on the educationists to introduce value education effectively in schools.
Write an article in 150-200 words expressing your views on the need of
value education. You are Anu/Arun. 10 marks
OR
Regular practice of yoga can help in maintaining good health and even in
the prevention of so many ailments. Write a speech in 150-200 words to
be delivered in the morning assembly on the usefulness of yoga.

163 XII – English


SECTION - C (Text Books)
45 Marks
7. (a) Read the Extract given below and answer the questions that
follow: 4 marks
The stunted, unlucky heir
Of twisted bones, reciting a father's gnarled disease,
His lesson, from his desk. At back of the dim class
One unnoted, sweet and young. His eyes live in a dream,
Of squirrel's game, in tree room, other than this.
(i) Who is the 'unlucky heir' and what has he inherited ? 2
(ii) What is the stunted boy reciting ? 1
(iii) Who is sitting at the back of the dim class ? 1
OR
For once on the face of the Earth
let's not speak in any language,
let's stop for one second,
and not move our amis s;o much.
(i) Why does the poet want us to keep quiet ? 2
(ii) What does he want us to do for one second ? 1
(iii) What does he mean by 'not move our amis' ? 1

(b) Answer any three of the following in 30-40 words each : 2 x 3 = 6 marks
(i) Why are the young trees described as 'sprinting' ?
(ii) How is a thing of beauty a joy forever ?
(iii) Why didn't the 'polished traffic' stop at the roadside stand ?
(iv) Why did Aunt Jennifer choose to embroider tigers on the panel ?

8. Answer the following in 30-40 words each : 2x5 = 10 marks


(a) What changes did the order from Berlin cause in the school ?
(b) Who is Mukesh ? What is his dream ?
(c) Why did the peddler decline the invitation of the ironmaster ?

164 XII – English


(d) What made the lawyer lose his job ? What does the writer find so
funny about the situation ?
(e) What did the publisher think of 'The Name of the Rose'?
– Is Saheb happy working in the tea stall? Why/Why not? (Set1/3)
– Why was the crofter so friendly and talkative with the peddler (Set 1/3)
– Why did the author appear to be doing nothing ? (Set 1/3)
9. Answer the following in 125-150 words : 10 marks
Attempt a character sketch of Sophie as a woman who lives in her dreams.
OR
How did Douglas develop an aversion to water ? ,
– How did Douglas try to save himself from drowning in YMCA pool?
(Set 1/3)
OR
– Draw a character sketch of Sophie’s father.
10. Answer the following in 125-150 words : 7 marks
How did the Tiger King stand in danger of losing his Kingdom ? How was
he able to avert the danger ?
OR
What are phytoplanktons ? How are they important to our eco-system ?
11. Answer the following in 30-40 words each : 2x4 = 8 marks
(a) Why had Hana to wash the wounded man herself ?
(b) How did the Wizard help Roger Skunk ?
(c) How does Mr. Lamb keep himself busy when it is a bit cool ?
(d) What were the contents of the small brown suitcase that McLeery
carried ?
– What help did Dr. Sadao seek from Hana while operating on the wounded
white man?
– What did the detective superintendent inform the governor about
Evans? (Set 1/3)

165 XII – English


Note : This marking scheme of English Core (Delhi-2012) gives an idea about
the tips, value points and acceptable answers. Though an equally acceptable
and relevent answers does fetch marks as per the marking scale. In case of any
doubt or clarification, students will consult their teachers.

Section A : Reading

1 1 1 Comprehension Passage

(a) (a) (a) Note : No mark(s) should be deduction for mistakes in


usages and grammer, spelling, or word limit. Full marks
may be awarded if a student has been able to identify
the core ideas. If a student literally lifts a portion of
the given passage as an answer to a question. no
mark(s) to be deducted for this as long as it is relevant

(i) (i) (i) – body reaction to any demand or changes in its


internal and external environment 1 mark
– changes in external environment such as
temperature, pollutants, humidity and working
conditions 1 mark
– imbalance between demands and resources
– unrealistic ambitions

(ii) (ii) (ii) – muscle tension in various parts of the body/


palpitation/ high blood pressure/ indigestion/ hyper-
acidity/ lethargic/ disinterested/ not easily motivated/
mentaly undecided/ confused/ accide if prone/
trembling/ shaking/ nervous blinking/ dryness of
throat and mouth/ difficulty in swallowing/ chronic
fatigue/ lowers performance capacity/ periodic
mood shifts/ self destructive behaviour such as
eating and drinking too much smoking excess ......
relying on tranquilisers (any two) 2 marks

(iii) (iii) (iii) – heart disease/ depression/ ulcers/ hypertension/


sleeplessness/ high BP/ indigestion/ hyperacidty/
chronic fatigue/ loss of memory (any two) 2
marks

(iv) (iv) (iv) – diet/ massage/ food supplements/ herbal


medicines/ hobbies/ relaxation techniques/ dance
movements (any two) 1 mark
166 XII – English
(v) (v) (v) – reacts in different ways, common ones are flight,
fight and flee depending on the nature of stress
and capabilities of the person/ a professional
behaves as if he is a perfectionist/ stress has a
different meaning depending on the stage of life/
self destructive behaviour such as eating and
drinking too much, smoking excessively, relying on
tranquilisers (any two) 2 mark

(b)(i) (b)(i) (b)(i) collapse 1 mark

(ii) (ii) (ii) reprimand 1 mark

(iii) (iii) (iii) lethargic 1 mark

2 2 2 Note:
– If a student has attempted only summary or only
notes, due credit should be given.
– I mark allotted for the title be given even if a student
has written the title either in Q2(a) or Q2(b) part
– Content must be divided into heading and sub-
headings.
– The notes provided below are only guidelines. Any
other title, main points and sub-points may be
accepted if they are indicative of the candidate’s
understanding of the given passage, and the notes
include the main points, with suitable and
recognizable abbreviations. Complete sentences
not to be accepted as notes. (In such cases ½ –
1 mark may be deducted from the marks awarded
to content)

(a) (a) (a) NOTE MAKING


Distribution of Marks
Abbreviations/ Symbols (with/without key) –
any four 1 mark
Title 1 mark
Content (Minimum 3 headings and sub-headings, with
proper indentation and notes) 3 marks

Suggested Notes

167 XII – English


Tile : Good Communication Skills/ Good Listening/
Listening skills/ Art of Listening/ Listening/ Good
Communication and Listening/ any other relevant title

1. Researve
1.1 human mind processes 500 wpm
1.2 speaker speaks 150 wpm
1.3 difference between the 2

2 A Good Speaker/ Good Communication/ Listening


2.1 must retain attention of audience
2.2 stop not to let mind wander
2.3 must be a good listner

3 Listening/ Requirement of Listening/ Listening Skills


3.1 hearing with attention
3.2 being observant
3.3 making interpretations
3.4 concentration
3.5 participation

4. A Good Listener/ Good Listening - An Art/ Traits of


Good Listening
4.1 gets much more from speaker
4.2 knows how to prompt and persuade
4.3 puts speaker at ease
4.4 helps him articulate
4.5 facilitates speaker to convey thoughts

5 Effective Listening/ Barriers To Good Listening


5.1 barriers - phy./ psychological
5.1.1 physical-hindrance to hearing
5.1.2 psychological-interpretations & evaluation

(b) (b) (b) Summary


The summary should include all the important points
given in the notes
Content 2 marks
Expression 1 mark

168 XII – English


Section B : Advanced Writing Skills
Note : The objective of the section an Advanced
Writing Skills is to test a candidate’s writing ability.
Hence, expression assumes as much importance as
the content of the answer.

3 3 3 NOTICE 2 marks

Format 1 mark

The format should include: NOTICE/ TITLE, DATE,


and WRITER’S NAME WITH DESIGNATION. The
candidate should not be penalized if he has used
capital letters for writing a notice within or without a
box.

Content 2 marks

Expression 2 marks

Suggested value points


(EXCURSION TO LONAVALA)
– what excursion to Lonavala)
– when (autumn break/ duration of trip/ date)
– mode of transport (optional)
– mean for which class/ age group
– other details (cost of trip/ accommodation)
– last date for submission of names
– whom to contact
– any other relevant details

OR

ADVERTISEMENT

Content 3 marks

Expression 2 marks
Suggested value points
(SITUATION VACANT - FOR A RECEPTIONIST)
– name of school

169 XII – English


– name of vacant post
– qualifications required, skills, personality, age
– salary/ gender (optional)
– minimum experience required
– remuneration
– last date of applying
– contact details
– any other relevant details
(due credit should be given for economy of words
used)

4 4 4 REPORT WRITING

Format
1. title reporter’s name 1 mark
2. place, date – (optional)

Content 4 marks

Expression 5 marks
grammatical accuracy, appropriate words and
spelling (2½)
coherence and relevance of ideas and style (2½)

4 – – Suggested value points


(UNPRECEDENTED PRICE RISE/RISING PRICES
CREATE A CRISIS/ any other suitable heading)
– what - seminar on the problems of price rise and
ways of dealing with it
– When - lay, date and time
– Where - venue
– by whom (School Commerce Association)
– participants (class XII)
– panel of resource persons/ speakers
– highling its of the lecture
– any other relevant details

OR

170 XII – English


Suggestee value points
INAUGURATION OF GYMNASIUM)
– what - a new indoor gymnasium constructed and
inaugurated
– size of the gym/ numer and types, of machines
available ambience/ facilities available
– when (day, date, time of inauguration)
– where (AP) International School, Goa)
– chief guest other guests/ audience
– details of the inauguration programme
– any other relevant details

– 4 – Suggested value points:


PREVENT CRUELTY TOWARDS ANIMALS/ any other
suitable heading)
– what - seminar one how to prevent cruelty towards
animals
– when - day, date and time
– where - venue
– participant’s (40 city CBSE schools)
– panel of resource persons/ chief guest/ other guests
– highlights of the seminar
– students interaction (queries, clarifications,
suggestions, response etc.)
– any other relevant details

OR

Suggested value points:


BUS-SCOOTER ACCIDENT)
– what - an accident involving a bus and a scooter
– when (day, date, time)
– where (near Nalbandh Chowraha, Agra)
– cause of the accident
– people in volved - number of causalities/ injured
– condition of the vehicles
– action by police
171 XII – English
– eyewitners account
– rescue/help provided compensation
– medical help provided
– any other relevant details

– – 4 Suggested value points:


INTERNATIONAL YEAR OF CHEMISTRY/ any other
suitable heading)
– what - competition to celebrate International Year of
Chemistry
– when-day, date and time
– where - venue
– participants (36 schools from Delhi)
– chief guest (Chairman, CBSE)/ other guests
– judges
– variety of competitions - Power Point presentations,
debates, paper reading, poster making, collage making
etc.
– highlights of the programme
– result declaration/ prize distribution
– any other relevant details

Suggested value points


(bOMB BLAST IN DELHI)
– what - a bomb blast in a Delhi market
– when (day, date, time of the blast)
– where (name of the market, exact location of where
the explosion took place)
– loss of life, property
– rescue relief/ compensation
– action by police
– any other relevant details
Note: 1st person/ 3rd person account acceptable

5 5 5 LETTER WRITING
(Note:- No marks are to be awarded if only the format
is given, Credit should be given for the conditate’s

172 XII – English


creativity in presentation of ideas. Use of both the
traditional and the new format is permitted.)

Format 2 marks

1. sender’s address, 2. date, 3. receiver’s address, 4.


subject/ heading, 5. salutation, 6. complimentary close

Content 4 marks

Expression 4 marks
grammatical accuracy, appropriate words and
spelling (2)
coherence and relevance of ideas and style (2)

CANCELLATION OF LIBRARY MEMBERSHIP)


Suggested Value Points
– purpose (cancellation of membership and refund of
Rs. 5000 security deposit.
– reason (shift of residence to Faridabad)
– mention membership number
– status of issued books (if any)
– request for cancellation of membership and refund
of Rs. 5000 as security deposit
– any other relevant details

OR

(ILL TREATMENT OF STRAY DOGS)


Suggested Value Points :
Introduction : the problem
– inhuman treatment to stray dogs
– present condition in cities (a large number of stray
dogs in streets beaten and teased by adults and
children alike killed due to accidents/ catch fatal
diseases
– callous and indifferent attitude of people
– why people behave in this manner - insensitive,
treat stray dogs as a menace no love for stray dogs,
out of fear or any other

173 XII – English


Suggestions (sensitising people/ creating awareness/
urging humane treatment/ role of municipal authorities/
NGOs etc.
– any other relevant details

6 6 6 ARICLE WRITING

Format : (Title and writer’s name) 1 mark

Content 4 marks

Expression 5 marks
grammatical accuracy, appropriate words and
spelling (2½)
coherence and relevance of ideas and style (2½)

Suggested Value Points


(VALUE EDUCATION - A MUST IN SCHOOLS/ any other
suitable title)
– present condition in-schools (increase in violence/
lack of discipline)
– reasons (overexposure to media/ lack of emotional
support/ overambitions nature/ peer pressure/ lack of
deterrence/ any other
– consequences (spoils school culture and
environment/ self destruct on/ harm to society and
country)
– suggestions for improvement (morning assemblies
inspiring talks, stories, interaction with parents and
teachers, counselling etc.)
– any other relevant details

OR

6 6 6 SPEECH

Content (Format) 5 marks

Expression 5 marks
grammatical accuracy, appropriate words and
spelling (2½)
coherence and relevance of ideas and style (2½)
174 XII – English
Suggested value points :
USEFULNESS OF YOGA/ any other suitable title)
– addressing the audience
– highlighting the problems (increase in health
problems – physical, mental, emotional, psychological,
etc)
– advantages of yoga (a way of life)
– increase in yoga centres today/ its popularity
– cures many ailments/ works on body, mind and soul/
no side effects/ slow and steady process/ gives long
lasting results
– appropriate and relevant conclusion
– any other relevant details

SECTION C : LITERATURE (TEXT BOOKS)


NOTE : The objective of the section of Literature is to
test a candidate’s ability to understand and interprer
the presecribed text through short and long answer
type questions. Hence both content and expression in
answers to the given questions deserve equal
importance while awarding marks.

7 7 7 (This question has been designed to test the students


understanding of the text and their ability to interpret,
evaluate and respond to the questions base on the
given extract. In other words. It attempts to test their
reading comprehension ONLY).

Value points

(a) (a) (a)

(i) (i) (i) – the boy with twisted bones/ the one with deformity/
a boy sitting in the slum classroom/ the stunted boy/
the boy who inherited his father’s gnarled disease
1 mark
– inherited twisted bones/ deformity/ gnarled
disease 1 mark

(ii) (ii) (ii) – his lesson/ father’s gnarled disease 1 mark

175 XII – English


(iii) (iii) (iii) – one unnoted/ sweet and young/ dreamer/ one who
dreams about a squirrels game 1 mark

(i) (i) (i) – to take stock of our mindless activities/ to be able


to count to twelve to do self analysis for mental
relaxation/ to ensure peace/ to introspect/ to save
mankind from imminent doom 2 marks

(ii) (ii) (ii) – keep quir/ not speak/ not to move our arms so
much/ stop all activities/ to do nothing - no speech. no
activity. 1 mark

(iii) (iii) (iii) – no movement/ no activity/ not to harm others/ no


wars/ no violence 1 mark

(b) (b) (b) Short answer type questions (Poetry)


Distribution of marks : 1 marks

Content : 1 mark
Expression 1 mark
(deduct ½ mark for two or more grammatical/spelling
mistakes)

Value points :

(i) (i) (i) – trees appear to be running when seen from the
moving car/ sumbolic of life/ your/ energy/ vigour/
continuity of life 2 marks

(ii) (ii) (ii) – its loveliness increases/ it is perennial/ it is constant/


sustains human spirit in all ages & stages/ never passes
into nothingness/ moves away the pall from our dark
spirits/ makes life worth living. (2 marks)

(iii) (iii) (iii) – their mind set on their destination/ are insensitive
and indifferent/ the road side stand does not matter
to them/ does not have anything of value for them
2 marks

(iv) (iv) (iv) – wanted to project her wishes on the panel/ to express
her hidden desire wanted to forget her meek, miserable
life/ wanted to be like the tiger: - fearless, proud,
unafraid, chivalrous/ to escape from the harsh realities
of her life 2 marks

176 XII – English


8 8 8 Short answer type questions (Prose)
Distribution of marks
Content : 1 mark

Expression 1 mark

(deduct ½ mark for two or more grammatical/spelling


mistakes)

Value points :

(a) (a) (a) – announcement that French would not be taught


anymore German would be taught by a new master
2 marks
– it was their last French lesson
– no bustle and commotion (quiet as a Sunday
morning)
– M. Hamel - patient, calm but inwardly emotional
– students in their seats, sitting quietly
– the teacher M. Hamel in special dress
– sad villagers sitting on last benches like other
students
– school seemed strange and solemn on that day
– the teacher explained everything very patiently
(any two)

(b) – – – his fear of water ruined his fishing trips 2 marks


– deprived him of the joy of canoeing, boating and
swimming (any one) 2 marks

– (b) – – son of a poor bangle maker of Firozabad (½ mark)


– dreams of becoming a motor mechanic and driving
a car/insists on being his own master (½ mark)

– – (b) – No (½ mark) 2 mark


– he is no more a master of his own life/ has lost his
freedom and the carefree lokk/ has to obey orders of
the tea stall owner (½ mark)

(c) – – – allowed them to stay on the grounds 2 marks

177 XII – English


– Rajendra Prasad was out of town
– the servants knew Shukla
– they thought Gandhi was another peasant
– Gandhi not permitted to draw water from the well as
servants not sure about his caste (any two)

– (c) – – was afraid as he was carrying stolen money/ knew


that the Ironmaster had mistaken him for an old
regimental comrade feared that Ironmaster would send
him to the police/ felt it was like waling into a lion’s
den

– – (c) – was a lorely man with no wife or child/ was happy


to get someone to talk to in his loneliness/ sympathised
with the peddler (2 marks)

(d) – – – went to British Council Library to get details of the


short story contest organised by a British periodical
– read editor’s name and found it was the poet who
had visited the studies

(2 marks)

– (d) – – the owner of Gemini Studios closed the story


department and the lawyer was on the attendance
rolls of the Story Department

– writer feels that it must have been the only instance


in human history when a lawyer lost his job because
poets were asked to go home
(any one) (2 marks)

– – (d) – was always tearing up newspapers (clippings) and


filling them

– so was always sitting in his cubicle/ appeared to be


doing no work
(2 marks)

(e) (e) (e) – liked the novel but did not expect to sell more than
3,000 copies

– novel dealt with mystery, medieval history,

178 XII – English


metaphysics and theology.

– assured that it was a difficult reading experience

– did not expect a good response in America as few


people have seen a cathedral and have studied latin
(any two)

Q9 & 10 (These question have been set to test the


students under standing of the text and their ability to
interpret, evaluate and respond to the isuues raised
therein. Hence no particular answer can be accepted
as the only correct answer. All presentations may be
accepted as equally correct provided they have been
duly supported by the facts drawn from the text. The
important thing is that the student should be able to
justify his or her viewpoint.)

9 9 9 Distribution of marks : 5 marks

Content

Expression 5 marks
grammatical accuracy, appropriate words and
spelling (2½)
coherence and relevance of ideas and style (2½)

Value points :

9 – – Ironmaster
– impulsive, doubting, poor judge of character, loving
father
– invites Peddler to his house, realises that he is no
acquaintance, threatens to hand him over to the Sheriff,
asks him to leave his house as fast as he can
– owner of Ramsjo Iron Mill/ devoted to his work
greatest ambition is to ship out good iron to the market
– watches the work in his mill, both night and day
makes nightly rounds of inspection
– gives in to the request of his daughter to allow the
peddler to be with them on the Christmas Eve (any
two)

179 XII – English


Daughter

– loving, caring, kind, thoughtful, generous,


magnanimous, observant, insightful, true Christian spirit

– observan – notices that the peddler is afraid,


concludes that either he has stolen something or he
has escaped from jail

– better powers of persuasion - convinces the peddler


to accompany her to the Manor house on the promise
that he can leave as freely as he comes

– even after knowing the truth, argues with her father


to be kind and generous and to allow the peddler to
stay with them

– succeeds in transforming the peddler (any two)

OR

Sophie and Jansie are classmates and friends

Sophie

– daydreamer/ escapist/ hero worshipper

– wants to have a boutique/ to be an actress/ a


designer/ an actress/ or do something sophisticated

shares her dreams only with her brother, considers


Jansie as nosey (gossip monger)

– adores Danny Casey - football player and fantasizes


meeting him

– any other point


(any two)

Jansie
– realistic and practical/ wordly wise
– knows that they are poor and earmarked for biscuit
factory
– warkms Sophie against unrealistic and unachievable
dreams
180 XII – English
– takes interest in knowing about new things/ different
in thinking and temperament
– has no unrealistic dreams
– any other point
(any two)

– 9 – Character sketch of Sophie


– a young girl of lower middle class family
– wants to open a boutique/ become an actress/ a
fashion designer/ a manager
– wants to come out of her situation
– incurable dreamer and escapist
– warned against such dreams by Jansie and her
father, but ignores
– no one telieyes her, so she shares her dreams only
with her brother who is an introvert
– also-hero worships the foodball player-Danny Casey
– fantasizes about meeting him at the market place
and later near the canal
(any four)

OR

– Beach incident – Douglas developed aversion to


water first as a child whether went to the beach in
California with his father
– happened when he was three or four years old/
went to beach in California with father/ the waves
knocked him down and swept over him was buried in
water/ breath ways gone/ was frightened/ terror in
heart at the overpowering force of the wave (any two)
– YMCA drowning incident– a big bruiser of a boy
threw him in the deeper end of the pool/ hit water in
sitting position: went to the bottom/ tried to come out
thrice but failed/ could not breathe/ head throbbing/
lungs aching/ finally gave up and fainted/ was saved

181 XII – English


later but terror stayed with him/ could not sleep or eat
for days/ did not go near water for years (any two)
– – 9 – was throw into YMCA pool by a big bruiser of a boy
– went down in sitting position to the bottom of the
pool
– not out of wits, he made a plan – to make a big
jump as his feet hit the bottom.
– hoped to move up to the surface of water like a cork
– then he would lie flat on it, paddle to the edge of the
pool and come out
– three time he tried but was always partially successful
– called out for help
– tried to catch something
– sucked instead of air
– finally lost hope and gave up/ fainted
(any four)

OR

Character sketch of Sophie’s father


– strong, heavy man of lower middle class/ travelled
on his bicycle
– hardworking (signs of labour reflected on his face)
– humble living - yearns to make his family comfortable
– feels helpless due to paucity of resources
– does not in dulge in self – pity
– attitude towards football - is a fan of football game
and watches live matches every Saturday with the
family
– comments on Danny Casey - wants him to be
balanced
– displays Irish nationalism
– very practical, hardworking/ realistic- does not

182 XII – English


humour Sophie for her concocred stories
– knows his children well - knows Sophie lives in her
dreams, reminds he of their appalling living conditions
– any other relevant point
(any four)

10 10 10 Distribution of marks :

Content 4 marks

Expression 3 marks

grammatical accuracy, appropriate words and


spelling (1½)

coherence and relevance of ideas and style (1½)

Value Points :
– the Tiger King annoyed a high ranking British officer
by refusing to allow him to kill tigers in his province
– did not even allow him to get himself photographed
with the tiger killed by the king
– prevented of British officer from fulfilling his desire
– so stood in danger of losing his kingdom
(any one)
– averted the danger by sending a gift of gift diamond
rings (to choose one or two from), to the British officer’s
wife

OR

– microscopy single celled plants (grass) of the sea


– important of our ecosystem
– nourish and sustain the entire food chain of the
southern Ocean
– they use sun’s energy to absorb carbon and
synthesize organic compounds by photosynthesis
– the will be affected with ozone depletion

183 XII – English


– the lives of all the marine animals and birds and
global carbon cycle will be afftected
– they carry a metraphor for existence – take care of
small things and the big things will fall into place
(any two) (3 marks)

11 11 11 Distribution of marks

Content : 1 mark

Expression 1 mark
(deduct ½ mark for two or more grammatical/spelling
mistakes)

Value points :

(a) – – – Yes (½ mark)


– had heard about stories of suffering of prisoners of
war/ tortued body of the American POW confirmed
her fears/ remembered General I akima who beat his
wife cruelly at home/ wondered if he could be so cruel
to his wife, he would be more cruel to any enemy
soldier (½ mark)

– (a) – – because wounded man was a prisoner of war an


American and an enemy of Japan/Yumi the servant,
refused to wash him as she consider him an enemy
(2 marks)

– – (a) – asked her to fetch towels/asked her to help him turn


him/ told her to give the anasthetic to the wonded
white man, if required/asked her to soak cotton with
the anaethetic and hold it near his nostrils(2 marks)

(b) (b) (b) – removed his foul smell with the magic spell/gave him
the beautiful smell of roses/helped him in having many
friends (2 marks)

(c) (c) (c) – gets a ladder and a stick and pulls down the crab
apples/makes jelly/enjoys the humming of bees in his
garden/site in the sun/reads books/ makes toffee with
honey (any two) (2marks)

184 XII – English


(d) – – – Detective Superintendent (½ marks)

– asked him to take McLeery with him and follow Evans


on McLerry’ direction (½ makrs)
(2 marks)

– (d) – – a sealed question paper envelops a yellow invigilation


from / a special authentication card from the
Examination Board/ a paper knife / a Bible / a copy of
The Church Times / a smallish semi inflated rubber
ring
(2 marks)

– – (d) – McLeery had spotted Evans driving off along Elsfield


Way/they had got the number of the car, had given
chase immediately/ but had lost the track/assumed
that Evans must have come back into the city
(2 marks)

185 XII – English


These questions are based on values & key messages brought out on the basis of prescribed
text. These should be answered in about 100 words. Content will be marked for three marks
and expression for two marks. Students have the liberty to go beyond the set text.

The Last Lesson

1 “When a people are enslaved, as long as they hold fast to their language it is as if they
had the key to their prison.”

After reading M. Hamel‟s statement you begin to realise the importance of mother
tongue that is losing its significance in the present times.

Write an article for a national daily on the need of reviving the status of the mother
tongue.
2 Then he turned to the black board , took a piece of chalk and bearing on with all his
might , he wrote as large as he could -
Vive La France ……………….
Abraham Lincoln , a former President of America said , “I like to see a man proud of the
place in which he lives.
I like to see a man live so that his place will be proud of him.”
After reading the lesson and the above quote of Abraham Lincoln you begin to reflect
on the loss of spirit of patriotism amongst the youth in India due to which there is no
respect for one‟s countrymen and no determination amongst the youth to lead the
country to a better future.
Write an article in about hundred words for a national magazine on the need for revival
of patriotic spirit amongst the youth in India.

3 “…….. his terrible iron ruler under his arm”. It has been said, “spare the rod and spoil
the child”.

The abolition of corporal punishment from schools in India has boosted students‟
confidence. Do you agree or not?

4 “They preferred to put you to work on a farm or at the mills, so as to have a little more
money”.

Some parents do not understand the importance of education. They want their kids to
work. The Govt. has introduced the Right to Education Act. What changes it can bring
about in the attitude of the parents. Explain.

5 The elders of the village were sitting in the classroom. The participation of the
community is necessary for a healthy and productive education system. Comment.
6 Franco Prussian war had impacted all spheres of life, even the schools. It caused an
emotional turmoil in the life of the civilians.
Can war/violence be a solution to any problem?
7 “It was so warm, so bright ……….. birds were chirping …… The Prussian soldiers were
drilling ………… It was all much more tempting”. In the present scenario / times there
are more distractions (malls, i-pads, facebook etc). The will to resist is the only solution.
Explain.

8 M. Hamel becomes an honest role model for the students on the day of the last lesson.
How does a teacher play a constructive role in the life of the students in the present
times?

Lost Spring

1. Saheb and Mukesh are representatives of the underprivileged and impoverished section
of the society. The society has its role of credit for their condition.
How as a concerned student of the society you can perform your duty to educate them?
Discuss.

2. Childhood is the spring time of one‟s life but Saheb and Mukesh are the victims of the
evil of child labour. You feel pained to see children working at factories, dhabas and tea
stalls. Empathising with them give concrete suggestions to deal with the problem.

3. In search of decent living conditions and better means of survival people are migrating
to metropolitan cities. That is not a journey to heaven. In this rat race they are losing
their identity. One should not lose one‟s identity in search of gold. Comment.

4. Saheb-e-Alam, a destitute, watches a game of tennis from a distance. He longs to be one


of the players.
Shakespeare says,” Sweet are the uses of adversity”.
Can one use his dreams to lead to brighter life in future? Elucidate.

5. Slums & Mukesh‟s lodgings pose a threat to healthy life. Do you think that better living
conditions will give a positive viewpoint to them towards life. Write an article on Health
is Wealth.

6. But promises like mine abound ….. in their bleak world”.


Saheb and others like him spend their life on unfulfilled promises. What role should the
youth play to improve their conditions through programmes like „Each one Teach one‟ .
Give your views in 100 words.
7. Cottage industry is being exploited by the middlemen and sahukars. The workers are
afraid of organizing themselves into co-operatives. The main reason is that they are not
aware and daring. Write an article on Awareness may open new avenues for the poor.

8. Being illiterates Saheb and Mukesh are unaware of their rights and are exploited by „the
vicious circle‟.
Can education be an answer to this? Comment.

9. “Can a God given lineage ever be broken” says Mukesh‟s grandmother.


Do you think that one should choose his profession by choice and not by chance (of
being born in a particular caste) Discuss.

10. “And daring is not a part of his growing up. When I sense a flash of it in Mukesh. I am
cheered….” Daring and fearlessness is the only way which can bring about a radical
change in life. Do you agree with the statement? Comment.

11. Migration has resulted in overpopulation in the big cities hence increasing the burden on
limited resources. Conservation of resources is the duty of everyone. Comment that
sustainable development is our moral duty.

12. Apathy and lack of concern have added fuel to the miseries of Saheb and Mukesh. The
bureaucrats and politicians are unconcerned to their miseries. Do you feel that concern
and compassion can drastically change the condition of the deprived.

Deep Water

1. Fear is the stumbling block to success unless challenged. Douglas proves it by


overcoming his fear of water. Grit and courage of Douglas helped him do that.
“Braving the odds is the key to success”. Comment.

2. “Practice makes a man perfect”.


Douglas tries hard to reach to the level of perfection by perseverance .
“Never, never, never give in” Churchill.
Comment.

3. Douglas was tossed into the deeper end by a big bruiser of a boy. Bullying is a common
phenomenon of present times. Even in Indian schools it is commonly witnessed. Do you
think it is the moral duty of every senior to support and protect the juniors.
Write a note on Bullying; a threat to development of a child.

4. “What though the field be lost


all is not lost, the unconquerable will
and the courage never to submit or yield” John Milton
Strong will power sustains even in the worst circumstances. Strong will power helped
Douglas to overcome his fear of water .

Write an article on “Role of strong will power in attaining success”.

5. Even after becoming a swimmer Douglas was not satisfied whether he had overcome his
fear of water. He decided to achieve it through self learning.

“The goal may be distant but awake, arise & stop not till the goal is reached.” Swami
Vivekananda. Write an article in 100 words on self learning helps in building the self
confidence and a complete personality.

6. Fear of water remained with Douglas even as years rolled by. He did not want to live
with his handicap and so tried his level best to overcome this fear. Refusal to live with a
handicap paves the way for greater achievement. Comment.

The Rattrap

1. The peddler betrayed the trust of the crafter and was caught in the trap of the world.
Temptation to bait should be resisted at any cost. Write a note on „Strength of Character‟.

2. In the forest, the peddler felt trapped. He even accepted his approaching death. This
shows that the burden of guilt of theft of money from the crafter is really heavy upon
him. Write a note on Guilt burdens you and disturbs the peace of mind.

3. The Rattrap highlights human predicament / weakness and tendency to fall prey to
temptations. In this pursuit, the real aim of life i.e. attaining spiritual heights is lost
somewhere. Comment on “Contentment – the purpose of life”.

4. Edla with her love and compassion is a true living example of humanity and brings out
the best in the rattrap peddler. Confidence, optimism and kindness may reform weak
personalities. Comment.

5. Selfless love, empathy, sympathy and compassion provide a healing touch to the
distracted souls. This is evident from the story The Rattrap. Common on the role of
human values in bringing about change in one‟s behaviour.
6. The letter of the peddler addressed to Edla is a fine example of purification of soul
through confession. He signs himself as Captain Von Stahle, achieving the self esteem.
Write an article on “Confession leads to self esteem”.

7. “But of course it is you, Nils Olof. How you do look? Now of course, you will come
home with me,” says the iron master. However he proves to be wrong later on.
“Impulsive / rash decisions are generally wrong”. Road rage is the burning example of
such behaviour. Comment.

Indigo

1. Rajkmar Shukla kept following Gandhi for weeks and finally reached his ashram.
“Fix a date” he begged.
Impressed by the Sharecropper‟s tenacity and story Gandhi agreed.
Write an article on determination works wonders.

2. “But Gandhi was not allowed to draw water from the well lest some drops from his
bucket pollute the entire sources…..”
“Untouchability is a curse”. Write an article in about 100 words.

3. “Where peasants are so crushed and fear stricken law courts are useless. The real
relief for them is to be free from fear.” Gandhi.
Though the times were difficult but fear has increased their suffering. It happens
even today. Write an article on Fear is Man‟s worst enemy.

4. “The officials felt powerless without Gandhi‟s Co-operation. He helped them


regulate the crowd”. This shows Gandhi was a great leader.
Can good leadership quality help a student in becoming a good citizen in a
democracy?

5. “Gandhi signed a receipt for the notice and write on it that he would disobey the
order”.
With these words Gandhi started a new era in Indian politics. Civil disobedience
helped him in achieving freedom from the British without any violence. Write an
article on the relevance of Non-violence in modern times.

6. “He saw the cultural and social backwardness in the Champaran villages and wanted
to do something about it immediately”. He appealed and many joined from different
parts of India to do social work in Champaran.
Do you think social service can provide solution to many of the problems faced by
Indians like illiteracy, cleanliness, health, environment etc. Comment.
7. “Gandhi in this way taught us a lesson in self-reliance”.
Write an article on value of self reliance.

8. “What about the injustice to the sharecroppers”. Gandhi demanded. These words of
Gandhi inspired the lawyers to fight for the just cause of the poor peasants.
This is a selfless trait of every good human being to fight for justice to the poor and
deprived. Write an article on “Fight for justice –need of the hour”.

Poets and Pancakes

1. The office boy joined Gemini Studios in the hope of becoming star actor, director,
screen writer or a lyric writer. But he was put at the lowest in the hierarchy in the
make up department which led to disappointment. The glittering world of films
attracts millions of youngsters to Mumbai but most end up in frustration. Write an
article on “All that glitters is not gold”.

2. The make up men could turn any decent looking person into a hideous, crimson
hued monster …. In modern times, beauty has become a business. Write an article on
“Simplicity is Beauty”.

3. The office boy in Poets and Pancakes is jealous of Subbu and so a frustrated soul. Cut
throat competition today has led to unreasonable jealously resulting in stress. Do you
agree that jealously leads to frustration. Comment.

4. Though well informed, prose writer Asokamitran admits “Prose writing is not and
cannot be the true pursuit of a genius”. What trait of character do you come across ?
Write a note on “Modesty is the basic human virtue”.

5. Moral Rearmament Army visited Madras in 1952 to instill moral values and left an
impact on the audience. Do you think that in the present scenario more MRAs should
be formed so that the youth is armed with values. Comment.

6. The make up department of Gemini Studios highlighted an example of national


integration. Today we are engaged in disputes over petty issues related to one or the
other state. Write an article on “Ways to promote national integration”.

The Interview

1. “In its highest form interview is a source of truth, and in practice, an art”.
Today we find a lot of intrusion in the private lives of personalities by interviewers.
The responsibility of the media person is to project real / true picture of the
contemporary society. Write an article on “Media – a source of truth”.

2. We have a lot of empty spaces in our lives. I call them interstices …… I work in these
empty spaces. (Umberto Eco).

That is the reason that his written output is large.

Time is precious and should be utilized to the fullest . Write an article on “Time
Management”.

3. Umberto Eco faces the questions of Mukund Padmnabhan confidently and answered
all of them honestly. Write an article on “Confidence and honesty – assets of
captivating personality”.

Going Places

1. Sophie lives in a poor locality in miserable conditions. Even in her house dirt and
squalor are all around and it affects the mindset of Sophie. Write an article on
“Cleanliness is next to godliness”.

2. Sophie always dreamt of becoming a manager, fashion designer, an actress and owner of
a boutique. Though she was earmarked / destined for biscuit factory, dreams are
required for growth but day dreams hamper it. Write an article on “Unrealistic dreams –
bane to growth”.

3. Like all other teenagers, Sophie lived in world of fantasy and worshipped her soul‟s hero
Danny Casey. She goes to the extent of imagining a date with him and feels disappointed
– when this realization dawns upon her that it was a waking dream. Write an article on
“Self deception leads to depression”.

4. “Now I have become sad, she thought. And it is a hard burden to carry this sadness.
Sitting here and waiting” Sophie‟s wild fantasies have led her to a very disappointing
situation. She is herself responsible for her frustration . You feel that one has to live in
the realistic world. Write an article on adolescent fantasies leading to disappointment.

5. “The small room was steamy from the stove and cluttered with the heavy-breathing man
in his vest at the table and the dirty washing piled up in the corner”.

This shows that Sophie‟s family belonged to the lower middle class of the society. This
has played a role in Sophie‟s fantasizing .
How does socio – economic background influence a child‟s development?

POETRY SECTION

1) Kamala Das is shattered to see her mother ageing & nearing her death. On the other
hand she feels agonized on her inability to take care of her. You feel that youth should
take care of the aged. Write an article on Taking care of the aged, our duty.

2) Stephen Spender is pained at the indifferent attitude of the society as well as the
authorities towards the poor, who are concerned to lead a life of deprivation, penury and
want. You empathize with those children & want something to be done for this section.
Comment Education and exposure can change the plight of the poor children.

3) Keeping Quiet is an expression of the poet to silence. He highlights the need of quiet
introspection and mutual understanding leading to peace. Do you think that in the
present situation keeping quiet may prove the way to peace. Comment.

4) “He prayeth well


Who loveth well, all things big & small” (Coleridge)
A thing of beauty provides a platform for all to be one with God. Appreciating beautiful
things is appreciating God. In the present times man is stressed & you feel that proximity
to values (beautiful things) will lead him to everlasting happiness. Comment .

5) The Roadside stand is a pathetic description of poor rural people who find themselves
dejected on the breach of trust. The social agencies & political parties though make
promises but do not keep them. Robert Frost is in insufferable pain & seeks suggestions
from all. As the aware & concerned citizen you feel anguished and pen down your ideas
on improving the condition of rural poor .

6) Aunt Jennifer is the representative of exploited women. She succumbs to the tortures &
ordeals. In the wake of twenty first century we boast of development but exploit women.
Write an article on woman empowerment.

Answer
VALUE POINTS
Students may cover any three of the following suggestive points. Any other relevant
point will be equally acceptable.

THE LAST LESSON

1. Value Points

i. Better understanding of the subject matter.


ii. Better expression.
iii. Easy references / allusions.
iv. Related to culture and way of living.
v. Everyone should feel proud of one‟s mother tongue.

2. Value Points

i. Country‟s youth value – starved-facing a total crisis.


ii. Violence, separations and inter & intra group conflicts, political apathy
reigning order of the day.
iii. Secured freedom but not utilized freedom in its true perspective.
iv. Need for good leadership and governance.

3. Value Points

i. Increased interest of students to come to school.


ii. Not scared of teachers.
iii. Truancy has fallen.
iv. Confidence in expressing their view points.

4. Value Points:

i. More aware of value of educating kids.


ii. Constitutionally bound.
iii. No fees- Free and compulsory education is encouraging.
iv. Various schemes: free books, uniform, mid day meal etc.

5. Value Points:

i. Helps in maintaining discipline and healthy environment.


ii. Inculcating the social values.
iii. Spreading awareness – controlling diseases, education social evils etc.
iv. Immediate Solutions – Role of V.K.S. and P.T.A.
6. Value Points:

i. No; Violence is no solution.


ii. Disturbs peace: internal & external.
iii. Hampers development.
iv. Snatches identity.

7. Value Points:

i. Helps in evading distractions.


ii. Provides focus to life‟s aim.
iii. Strength of character.
iv. Nurturing the will to resist by guidance and counselling.

8. Value Points:

i. Friend, philosopher and guide.


ii. Strength of character.
iii. Motivates.
iv. Model behavior.
v. Use of modern- technology in education.

LOST SPRING

1.
i) Each one Teach one.
ii) Spreading awareness.
iii) Motivating parents as well as kids.
iv) Working with the authorities.

2.
i) Strict and stringent laws with focus on implementation.
ii) Awareness about various programmes – R.T.E.
iii) Education-with vocational training and flexi timings, mobile-schools.

3.
i) Culture to be preserved.
ii) Development of all the regions: slums & villages.
iii) Dignity of labour.
iv) Food security.

4.
i) Dreams help in achieving goals.
ii) Motivation
iii) Hard work
iv) Consistency.

5.
i) Healthy mind in a healthy body.
ii) Healthy person a productive citizen.
iii) Not a liability but an asset.
iv) Better health better outlook.

6.
i) Can educate them.
ii) Vocational training.
iii) Making them self sufficient.
iv) Can fulfill the promises themselves.

7.
i) Knowledge is power.
ii) Understanding rights & duties.
iii) Can take maximum advantage of govt. schemes.
iv) Stops exploitation.
v) Power of co-operatives.

8.
i) Yes, Education awares.
ii) Comes to know about their rights.
iii) Confident & fearless.
iv) Rationale.
9.
i) Freedom of choice to the youth.
ii) No stress: Learning – a pleasure.
iii) Better output.
iv) Self satisfaction.

10.
i) Fear is the worst enemy of Man.
ii) Free thinking & good decisions with fearlessness.
iii) High level of confidence.
iv) Constructive & Creditable citizen.

11.
i) The earth is not our property but a legacy to be transferred.
ii) Think about future generations.
iii) Renewable sources of energy.
iv) Check on population & migration to cities.

12.
i) Added to the main stream.
ii) Eliminates the feeling alienation.
iii) Gives Confidence.
iv) Brings out the best.
Deep Water
1.
i) Acceptance to challenge.
ii) Patience & perseverance.
iii) Overcoming hurdles.
iv) Achieving goals.

2.
i) Persistent efforts.
ii) Will: Never to surrender.
iii) Braving the odds.
iv) Self satisfaction: true measure of success.

3.
i) Increases inequality.
ii) Instills fear.
iii) Aversion to schools & colleges.
iv) Drastic / fatal results.
v) No to Bullying.

4.
i) Resistance to temptations.
ii) Goal – oriented.
iii) Overcoming obstacles.
iv) Increases positive – thinking.

5.
i) You do-you learn.
ii) Self – reliance.
iii) Helps in development of personality.
iv) Trust in one‟s capabilities.

6.
i) Handicap – not a stumbling block always.
ii) Strong will power can counter handicap.
iii) Better training & proper follow up – revival of confidence.
iv) End product: a complete personality.

The Rattrap

1.
i) When character is lost – everything is lost.
ii) Develops resistance to temptations.
iii) Role model for society.
iv) Trustworthy.

2.
i) Increases stress and snatches peace.
ii) Negative thoughts develop.
iii) Loses confidence.
iv) Confession purifies and provides relief.

3.
i) Cut throat competition lessens.
ii) Brings peace of mind & minimizes wants.
iii) Enhances personality.
iv) Gives birth to diversity.

4.
i) Love to humanity is love to God.
ii) Fellow feeling is a lifeline of humanity.
iii) Reform weak personalities and society.
iv) Bring out best in man.

5.
i) Value education – need of the hour.
ii) Understanding oneself.
iii) Brings about positive changes.
iv) Achievement of self esteem.

6.
i) Confession helps in releasing burden of guilt
ii) Purification of soul.
iii) Helps in regaining confidence.
iv) Understanding oneself.

7.
i) Impulsive decisions are rash.
ii) Think before deciding or acting.
iii) Disastrous results.
iv) May lead to lifelong suffering.
Indigo

1.
i) Key to success.
ii) Seemingly unattainable targets are met.
iii) Gives strength to fight.
iv) Energies are channelized.

2.
i) Against the principle of equality.
ii) Results in hatred, segregation.
iii) Class struggle.
iv) Unconstitutional
v) Right to equality is the answer.

3.
i) Checks growth.
ii) Leads to exploitation and disappointment
iii) Low self esteem.
iv) Freedom from fear – the only answer.

4.
i) Confidence in oneself.
ii) Effective argumentation and negotiation.
iii) Fearlessness.
iv) Realisation of goals.
v) Examples from life.

5.
i) World is violence ridden.
ii) Tolerance towards others.
iii) No destruction.
iv) Guarantee for universal peace.

6.
i) Improve quality of life.
ii) Assists government agencies.
iii) Eradicated evil.
iv) Can spread awareness.
v) Influences all fields of life.

7.
i) Self help is greatest help.
ii) Leads to independence.
iii) Following one‟s instincts and ideas.
iv) Leads to free thinking.

8.
i) Not to succumb to circumstances.
ii) Better to die on your feet than to live on your knees.
iii) Ensures justice.
iv) Leads through peace.
Poets and Pancakes

1.
i) Appearances are deceptive.
ii) Films attract youth.
iii) First evaluate, and then decide.
iv) Don‟t be a copycat – no blind following.

2.
i) Mushrooming growth of beauty salons & gyms.
ii) Harmful side effects.
iii) Expensive beauty treatments.
iv) Simple living, high thinking should be motto.

3.
i) Breeds hatred and negativity.
ii) Breaks / sours relationships.
iii) Unfounded and immoral.
iv) Self destructive.

4.
i) Stops confirmation.
ii) Wins admiration.
iii) Positive environment.
iv) Leaves no scope for jealousy.

5.
i) Youth have no respect for values.
ii) Values cannot be enforced by law.
iii) Contribution and role of society.
iv) Restoration of values – top priority.
6. A
i) Unity in diversity – India having varied cultures, languages and customs.
ii) Respect and tolerance for every culture and religion.
iii) Role of education and media.
iv) Cultural festivals and meets.
Interview

1.
i) Media influence life.
ii) Should highlight truth without intruding in private lives.
iii) Reporting without bias.
iv) Responsible media can play positive role.

2.
i) Time is precious – do not waste it.
ii) More scope for better output.
iii) Time managed – time saved.
iv) Time management – key to success.

3.
i) Helps in character building.
ii) Fearless and courageous attitude.
iii) Admired by all.
iv) Solution to problem like corruption.
Going Places

1.
i) Healthy environment – good for growth.
ii) Check on spread of diseases.
iii) Leads to positive thinking.
iv) Helps in building strong nation.

2.
i) Unrealistic dreams – distract from goals.
ii) Demotivating.
iii) Develops the feeling of escapism.
iv) Self deception and distorted personality.

3.
i) Self deception leads to frustrated personality.
ii) Cause of failure.
iii) Creates complexes.
iv) Lame excuses.
v) Live in realistic world.

4.
i) Immature thinking.
ii) Lack of experience.
iii) Limited vision.
iv) Guidance and counselling can help.

5.
i) Major factor influencing child psychology.
ii) Disparity leads to escapism.
iii) Confused due to different sets of values.
iv) Social evils lead to personality disorders.
Poetry

1.
i) Pathetic condition.
ii) Soft targets for criminals.
iii) Emotional support from family.
iv) Financial security and respect.

2. A
i) Education empowers.
ii) Exposure to outside world
iii) Education Prepares for the battle of life.
iv) Brings out the best in them.

3.
i) Keeping quiet leads to introspection.
ii) Helps in understanding others.
iii) Must for world peace.
iv) Lessens arguments and disputes.

4. A
i) Appreciating beautiful things.
ii) What man has made of man.
iii) Nature has a pacifying effect.
iv) Sustenance through hardships.
v) Must be preserved.

5.
i) Role of social agencies.
ii) Empathy.
iii) Education and training.
iv) Financial support.

6.
i) Women empowerment – necessary for country‟s growth.
ii) Education for women.
iii) Gender sensitization.
iv) Women are future leaders.

You might also like